Download as pdf or txt
Download as pdf or txt
You are on page 1of 359

CA FOUNDATION

BUSINESS
MATHEMATICS
LOGICAL REASONING
& STATISTICS

Updated upto:

 Past Exam Question Paper - upto December 2020.


INDEX
PART-A: BUSINESS MATHEMATICS (40 MARKS)
Chapter No. Chapter Name Page No.
1 Ratio, Proportion, Indices, Logarithm 1
2 Equation & Matrices 30
3 Linear Inequalities 56
4 Time Value of Money 70
5 Basic Concepts of Permutation & Combination 86
Sequence and Series Arithmetic & Geometric
6 107
Progression
7 Set, Relation and Function 124
8 Differentiation & Integration 139

PART-B: LOGICAL REASONING (20 MARKS)


Chapter No. Chapter Name Page No.
Number Series, Coding & Decoding and odd one out
9 166
series
10 Direction Test 180
11 (A) Sitting Arrangement 188
(B) Sitting Arrangement 194
12 Blood Relation 202
13 (A) Syllogism 211
(B) Syllogism 215

STATISTICS (40 MARKS)


Chapter No. Chapter Name Page No.
14 Statistical Description of Data 222
15 Measures of Central Tendency and Dispersion 233
16 Correlation and Regression 254
Probability and Expected Value by Mathematical
17 271
Expectation
18 Theoretical Distribution 290
19 Index Number 303
20 Time Series 315

MAY 2019 EXAM QUESTION PAPER

May 2019 Exam Question Paper 319

NOVEMBER 2019 EXAM QUESTION PAPER

November 2019 Exam Question Paper 329

DECEMBER 2020 EXAM QUESTION PAPER

December 2020 Exam Question Paper 341


CHAPTER- 1 - RATIO
PRACTICE QUESTIONS

1. The inverse ratio of 11: 15 is

(a) 15 : 11 (b) 11: 15


(c) 121 : 225 (d) none of these

2. The ratio of two quantities is 3 : 4. If the antecedent is 15, the consequent is


(a) 16 (b) 60
(c) 22 (d) 20

3. The ratio of the quantities is 5 : 7. If the consequent of its inverse ratio is 5, the antecedent is
(a) 5 (b) 5
(c) 7 (d) none of these

4. The ratio compounded of 2 :3,9:4,5:6 and 8 : l0 is


(a) 1:1 (b) 1:5
(c) 3:8 (d) none of these

5. The duplicate ratio of 3 : 4 is


(a) 3 (b) 4:3
(c) 9 : 16 (d) none of these

6. The sub duplicate ratio of 25 : 36 is


(a) 6:5 (b) 36:25
(c) 50:72 (d) none of these

7. The triplicate ratio of 2 : 3 is


(a) 8:27 (b) 6:9
(c) 3:2 (d) none of these

8. The sub triplicate ratio of 8 : 27 is


(a) 27:8 (b) 24:81
(c) 2:3 (d) none of these

9. The ratio compounded of 4 : 9 and the duplicate ratio of 3 : 4 is


(a) 1:4 (b) 1:3
(c) 3:1 (d) none of these

10. The ratio compounded of 4:9, the duplicate ratio of 3:4, the triplicate ratio of 2:3 is
(a) 2:7 (b) 7:2
(c) 2:27 (d) none of these

11. The ratio compounded of duplicate ratio of 4 : 5, triplicate ratio of 1: 3, sub duplicate ratio of
81: 256 and sub triplicate ratio of 125 : 512 is
(a) 4:512 (b) 3:32

1|Pa g e
(c) 1:12 (d) none of these
12. If a : b = 3 :4, the value of (2a+3b) : (3a+4b) is
(a) 18:25 (b) 8:25
(c) 17:24 (d) none of these

13. Two numbers are in the ratio 2: 3. If 4 be subtracted from each, they are in the ratio 3 : 5.
The numbers are
(a) (16,24) (b) (4,6)
(c) (2,3) (d) none of these

14. The angles of a triangle are in ratio 2 : 7: 11. The angles are
(a) (20°,70°,90°) (b) (300,700,800)
(c) (18°,63°,99°) (d) none of these

15. Division of Rs. 324 between X and Y is in the ratio 11 : 7. X & Y would get Rupees
(a) (204, 120) (b) (200, 124)
(c) (180, 144) (d) none of these

16. Anarid earns Rs. 80 in 7 hours and Promode Rs. 90 in 12 hours. The ratio of their earnings is
(a) 32 : 21 (b) 23 : 12
(c) 8:9 (d) none of these

17. The ratio of two numbers is 7: 10 and their difference is 105. The numbers are
(a) (200, 305) (b) (185, 290)
(c) (245, 350) (d) none of these

18. P, Q and R are three cities. The ratio of average temperature between P and Q is 11 : 12 and
that between P and R is 9 : 8.
The ratio between the average temperature of Q and R is
(a) 22 : 27 (b) 27 : 22
(c) 32 : 33 (d) none of these

19. If x : y = 3 : 4, the value of x2y+xy2 : x3+y3 is


(a) 13 :12 (b) 12:13
(c) 21:31 (d) none of these

20. If p : q is the sub duplicate ratio of p—x2 : q—x2 then x2 is


p q
(a) (b)
pq pq
pq pq
(c) (d)
pq pq

21. If 2s : 3t is the duplicate ratio of 2s — p : 3t — p then


(a) p2 = 6st (b) p = 6st
(c) 2p = 3st (d) none of these

22. If p:q=2:3andx : y=4:5,then the value of 5px+3qy : 10px+4qy is


(a) 71: 82 (b) 27:28

2|Pa g e
(c) 17 : 28 (d) none of these

23. The number which when subtracted from each of the terms of the ratio 19 : 31 reducing it to
1 : 4 is
(a) 15 (b) 5
(c) 1 (d) none of these

24. Daily earnings of two persons are in the ratio 4:5 and their daily expenses are in the ratio 7:
9. If each saves Rs. 50 per day, their daily incomes in Rs. are
(a) (40, 50) (b) (50, 40)
(c) (400, 500) (d) none of these

25. The ratio between the speeds of two trains is 7 : 8. If the second train runs 400 Kms. in 5
hours, the speed of the first train is
(a) 10 Km/hr (b) 50 Km/hr
(c) 71 Km/hr (d) 70 Km/hr

ANSWER

1. 2. 3. 4. 5. 6. 7. 8. 9. 10.
(A) (D) (C) (A) (C) (D) (A) (C) (A) (C)
11. 12. 13. 14. 15. 16. 17. 18. 19. 20.
(D) (A) (A) (C) (D) (A) (C) (B) (B) (D)
21. 22. 23. 24. 25. 26. 27. 28. 29. 30.
(A) (C) (A) (C) (D)

3|Pa g e
CHAPTER – 1 - PROPORTION

PRACTICE QUESTIONS

1. The fourth proportional to 4, 6, 8 is


(a) 12 (b) 32
(c) 48 (d) none of these

2. The third proportional to 12, 18 is


(a) 24 (b) 27
(c) 36 (d) none of these

3. The mean proportional between 25, 81 is


(a) 40 (b) 50
(c) 45 (d) none of these

4. The number which has the same ratio to 26 that 6 has to 13 is


(a) 11 (b) 10
(c) 21 (d) none of these

5. The fourth proportional to 2a, a3 ,c is


(a) a2c/2 (b) ac
(c) 2/ac (d) none of these
6. If four numbers 1/2, 1/3, 1/5, 1/x are proportional then x is
(a) 6/5 (b) 5/6
(c) 15/2 (d) none of these
7. The mean proportional between 12x2 and 27y2 is
(a) l8xy (b) 8lxy
(c) 8xy (d) none of these
(Hint: Let z be the mean proportional and z = (12 x 2  27 y 2

8. If A=B/2=C/5,thenA:B:Cis
(a) 3:5:2 (b) 2:5:3
(c) 1:2:5 (d) none of these
abc
9. If a/3 = b/4 = c/7, then is
c
(a) 1 (b) 3
(c) 2 (d) none of these

10. If p/q = r/s = 2.5/1.5, the value of ps : qr is


(a) 3/5 (b) 1
(c) 5/3 (d) none of these

11. If x : y=z :w=2.5:1.5,thevalueof(x+z)/(y+w)is


(a) 1 (b) 3/5
(c) 5/3 (d) none of these

4|Pa g e
12. If (5x—3y)/(5y—3x) = 3/4, the value of x : y is
(a) 2:9 (b) 27:29
(c) 29:27 (d) none of these

13. If A:B=3:2 and B:C=3:5, then A:B:C is


(a) 9:6:10 (b) 6:9:10
(c) 10:9:6 (d) none of these

14. If x/2 = y/3 = z/7, then the value of (2x—5y+4z)/2y is


(a) 6/23 (b) 23/6
(c) 3/2 (d) none of these

15. If x : y=2:3,y:z=4:3then x : y : z is
(a) 2:3:4 (b) 43:2
(c) 3:2:4 (d) none of these

16. Division of Rs. 750 into 3 parts in the ratio 4: 5: 6 is


(a) (200, 250, 300) (b) (250, 250, 250)
(c) (350, 250, 150) (d) none of these

17. The sum of the ages of 3 persons is 150 years. 10 years ago their ages were in the ratio 7: 8
: 9. Their present ages are
(a) (45, 50, 55) (b) (40, 60, 50)
(c) (35, 45, 70) (d) none of these

18. The numbers 14, 16, 35, 42 are not in proportion. The fourth term for which they will be in
proportion is
(a) 45 (b) 40
(c) 32 (d) none of these

19. If x/y = z/w, implies y/x = w/z, then the process is called
(a) Dividendo (b) Componendo
(c) Altemendo (d) none of these

20. If p/q = r/s = p—r/q—s, the process is called


(a) Subtrahendo (b) Addendo
(c) Invertendo (d) none of these

21. If a/b = c/d, implies (a+b)/(a—b) = (c+d)/(c—d), the process is called


(a) Componendo (b) Dividendo
(c) Componendoand Dividendo (d) none of these

22. If u/v = w/p, then (u—v)/(u+v) = (w—p)/(w+p). The process called


(a) Invertendo (b) Alternendo
(c) Addendo (d) none of these

23. 12, 16,*, 20 are in proportion. Then * is


(a) 25 (b) 14
(c) 15 (d) none of these

5|Pa g e
24. 4 , *, 9, 13½ are in proportion. Then * is
(a) 6 (b) 8
(c) 9 (d) none of these

25. The mean proportional between 1.4 gms and 5.6 gms is
(a) 28 gms (b) 2.8 gms
(c) 3.2 gms (d) none of these

a b c abc
26. If   then is
4 5 9 c
(a) 4 (b) 2
(c) 7 (d) none of these.

27. Two numbers are in the ratio 3 : 4; if 6 be added to each terms of the ratio, then the new
ratio will be 4: 5, then the numbers are
(a) 14,20 (b) 17, 19
(c) 18, 24 (d) none of these

a b
28. If  then
4 5
a4 b5 a4 b5
(a)  (b) 
a4 b5 a 4 b5
a4 b5
(c)  (d) none of these
a4 b5

a b
29. If a : b = 4 : l then  is
b a
(a) 5/2 (b) 4
(c) 5 (d) none of these

x y z
30. If   then (b-c)x + (c-a)y +(a-b)z is
bca c  ab ab c
(a) 1 (b) 0
(c) 5 (d) none of these

31. Division of Rs. 1100 into 3 parts in the ratio of 4:5:6 is


(a) 293.33, 366.67, 440 (b) 200, 500, 400
(c) 400, 300, 400 (d) None of these.

32. If (7p+3q) : (3p-2q) = 43:2 , then p:q is __________


(a) 5:4 (b) 4:5
(c) 7:2 (d) 2:7

33. Two number are in the ratio of 5:6. If 5 is subtracted from each of them their ratio becomes
4:5. Find the numbers
(a) 25 and 30 (b) 15 and 18
(c) 60 and 72 (d) None of these

6|Pa g e
34. The number ________ has same ratio to 7/33 as that 8/21 does to 4/9.
(a) 2/11 (b) 3/11
(c) 2/21 (d) None of these

35. A man has only 20 paise coins and 25 paise coins in his purse. If he has 50 coins in all
totalling Rs. 11.25, how many coins of each does he have
(a) 15, 35 (b) 25, 25
(c) 40, 10 (d) 30, 20

36. The ratio of the number of boys and girls in a school is 2:5. If there are 280 students in the
school, find the number of girls in the school.
(a) 200 (b) 250
(c) 150 (d) None of these

37. A bag contains an equal number of one rupee, 50 paise and 25 paise coins respectively. If the
total value is Rs. 35, how many coins of each type are there.
(a) 30 (b) 20
(c) 25 (d) None of these

38. One-third of a number is greater than one-fourth of its successor by 1. Find the number.
(a) 51 (b) 21
(c) 15 (d) None of these

39. 1230 baskets of mangoes were loaded in three trucks. When unloaded, it was found that 5,
10 and 15 baskets were rotten in the trucks respectively, but the remaining baskets were in
the ratio of 3:4:5. How many baskets were loaded initially in third truck?
(a) 575 (b) 515
(c) 565 (d) None of these

40. Find the ratio x : y : z from 2x + 3y – 5z = 0 and –3x + 2y + 7z = 0


(a) 10:12:13 (b) 30:2:12
(c) 31:1:13 (d) None of these

41. The value of 1.4


̅ is
(a) 13/9 (b) 10/9
(c) 4/9 (d) None of these

42. The value of 0.356


̅̅̅̅̅ is
(a) 356/999 (b) 353/999
(c) 353/990 (d) None of these

43. The ratio of the sum and the difference of two numbers is 7:1. Find the ratio of those two
numbers.
(a) 5:3 (b) 4:3
(c) 4:5 (d) None of these

44. The compound ratio of 4: 3, 9:13, 26 : 5 and 2 : 15 is


4 16
(a) (b)
25 25
18
(c) (d) None of these
27

7|Pa g e
45. A bag contains rupee, 50 paise and 25 paise coins in the ratio 10:14:18. If the total amount
in the bag is Rs. 430, find the number of coins of each kind.
(a) 200, 280, 360 (b) 280, 200, 360
(c) 360, 280, 200 (d) None of these

46. Two vessels contain equal quantity of mixtures of milk and water in the ratio 5:2 and 6:1
respectively. Both the mixtures are now mixed thoroughly. Find the ratio of milk to water in
the new mixture so obtained.
(a) 3:11 (b) 11:3
(c) 12:13 (d) None of these

47. The ratio of the incomes of two persons is 9:7 and the ratio of their expenditures is 4:3. If
each of them saves Rs. 200 per month, then their monthly incomes.
(a) Rs. 1,800, Rs. 1,400 (b) Rs. 1,600, Rs. 1,200
(c) Rs. 1,400, Rs. 1,200 (d) None of these

48. An amount of Rs. 950 is distributed among A, B & C in the ratio of 5:11:3, what is the
difference between the share of B and A.
(a) 300 (b) 340
(c) 500 (d) None of these

49. The ratio of two numbers is 4:1. If 5 is added to both the numbers, the ratio of the new
numbers obtained becomes 3:1. Then the numbers are
(a) 30, 20 (b) 40, 10
(c) 20, 10 (d) None of these

50. The ratio of the money with A and B is 3:4 and that with B and C is 4:5. If A has Rs. 300, how
much money does C have?
(a) 400 (b) 300
(c) 500 (d) None of these

51. Two numbers are in the Ratio 5:6. If 5 is subtracted from each number, the ratio becomes
4:5. Then the number are:
(a) 20, 30 (b) 20, 25
(c) 25, 30 (d) None of these

52. If 1 add to each of the two given numbers their ratio is 1:2. If subtract 5 from each the ratio
is 5:11. Then the numbers are
(a) 71, 25 (b) 35, 71
(c) 35, 51 (d) None of these

53. What must be subtracted from each term of the ratio 27:43 to make it equal to 7:15?
(a) 13 (b) 15
(c) 17 (d) None of these

54. An employer reduces the number of employees in the ratio of 9:8 and increases their wages
in the ratio of 14:15. In what ratio is the wages bill decreased?
(a) 20:22 (b) 20:33
(c) 21:20 (d) None of these

8|Pa g e
55. Divide Rs. 680 among A, B and C such that A gets
2
of what B gets and B gets
1
th of what C
3 4
gets. What is C’s share
(a) Rs. 180 (b) Rs. 280
(c) Rs. 480 (d) None of these

56. A, B and C have to distribute Rs. 1,000 between them, A and C together have Rs. 400 and B
and C Rs. 700. How much does C have?
(a) Rs. 100 (b) Rs. 200
(c) Rs. 150 (d) None of these

57. The fourth proportional to 2/3, 3/7, 4/9,........ is


(a) 2/7 (b) 14/8
(c) 7/2 (d) None of these.

58. The numbers 2.4, 3.2, 1.5, 2 are in proportion & their product of means is 4.8, find the
product of extremes?
(a) 4.8 (b) 2.4
(c) 8.4 (d) None of these.

59. What must be added to each of the numbers 6, 15, 20 and 43 to make them proportional.
(a) 5 (b) 4
(c) 3 (d) 2

60. The fourth proportional to (𝑎2 – ab + 𝑏 2), (𝑎3 + 𝑏 3) and (a-b) is equal to _________
(a) 𝑎2 + 𝑏 2 (b) 𝑎2 – 𝑏 2
(c) 1 (d) None of these

61. The third proportional to 15 and 20 is


80
(a) (b) 80
3
80
(c) 7
(d) None of these

62. What must be added to each of the four numbers 10, 18, 22, 38. So that they become in
proportion?
(a) 2 (b) 5
(c) 7 (d) None of these

PAST EXAMINATION QUESTIONS

63. Two numbers are in the ratio 2 : 3 and the difference of their squares is 320.The numbers
are:
(a) 12, 18 (b) 16, 24
(c) 14, 21 (d) None.

64. An alloy contains copper and zinc in the ratio 9 : 4. The zinc requires to melt with 24kg of
copper is:
2 1
(a) 10 Kg (b) 10 Kg
3 3
2
(c) 9 Kg (d) 9 Kg.
3

9|Pa g e
65. Two numbers are in the ratio 7 : 8. If 3 is added to each of them, their ratio becomes 8 : 9.
The numbers are:
(a) 14, 16 (b) 24, 27
(c) 21, 24 (d) 16, 18

66. A box contains Rs. 56 in the form of coins of one rupee, 50 paisa and 25 paisa. The number of
50 paisa coins is Twice the number of 25 paisa coins and four times of the numbers of one
rupee coins. The numbers of 50 paisa coins in the box is:
(a) 64 (b) 32
(c) 16 (d) 14

67. The ratio of third proportion of 12, 30 to the mean proportion of 9, 25 is:
(a) 2:1 (b) 5:1
(c) 7:15 (d) 3:5

68. What number must be added to each of the number 10, 18, 22, 38 to make the number is
proportion?
(a) 2 (b) 4
(c) 8 (d) None of these
1 1 1
69. Rs. 407 are divided among A, Band C so that their shares are in the ratio : : .The
4 5 6
respective share of A,B,C are:
(a) Rs. 165, Rs. l32, Rs. 110 (b) Rs. 165, Rs. 110, Rs. 132
(c) Rs. 132, Rs. 110, Rs. 165 (d) Rs. 110, Rs. 132, Rs. 165.

70. The incomes of A and B are in the ratio 3 : 2 and their expenditures in the ratio 5 :3. If each
saves Rs. 1,500, then B’s income is:
(a) Rs. 6,000 (b) Rs. 4,500
(c) Rs. 3,000 (d) Rs. 7,500

71. In 40 litres mixture of glycerine and water, the ratio of glycerine and water is 3:1. What
quantity of water added in the mixture in order to make this ratio 2:1 is:
(a) 15 litres (b) 10 litres
(c) 8 litres (d) 5 litres

72. The third proportional between (a2 – b2) and (a+b)2 is:
ab ab
(a) (b)
ab ab
( a  b) 2 ( a  b) 3
(c) (d)
ab ab

73. In what ratio should tea worth Rs. 10 per kg be mixed with tea worth Rs. 14 per kg so that
the average price of the mixture may be Rs.11 per kg?
(a) 2:1 (b) 3:1
(c) 3:2 (d) 4:3

74. The age of two persons is in the ratio 5:7. Eighteen years ago their ages were in the ratio of
8:13, their present age (in years) are:
(a) 50, 70 (b) 70, 50
(c) 40, 56 (d) None

10 | P a g e
75. If A, B and C started a business by investing Rs. 1,26,000, Rs. 84,000 and Rs. 2,10,000. If at
the end of the year profit is Rs. 2,42,000 then the share of each is:
(a) Rs. 72,600, Rs. 48,400, Rs. 1,21,000
(b) Rs. 48,400, Rs. 1,21,000, Rs. 72,600
(c) Rs. 72,000, Rs. 49,000, Rs. 1,21,000
(d) Rs. 48,000, Rs. 1,21,400, Rs. 72,600

p 2 2p  q
76. If  then the value of is:
q 3 2p  q
(a) 1 (b) -1/7
(c) 1/7 (d) 7

77. Fourth proportional to x, 2x, (x+1) is:


(a) (x+2) (b) (x-2)
(c) (2x+2) (d) (2x-2)

78. What must be added to each term of the ratio 49:68, so that it becomes 3:4?
(a) 3 (b) 5
(c) 8 (d) 9

79. The students of two classes are in the ratio 5:7, if 10 students left from each class, the
remaining students are in the ratio of 4:6 then the number of students initially in each class
was:
(a) 30, 40 (b) 25, 24
(c) 40, 60 (d) 50, 70

80. If A : B = 2 : 5 then (10A + 3B) : (5A + 2B) is equal to:


(a) 7:4 (b) 7:3
(c) 6:5 (d) 7:9

81. In a film shooting, A and B received money in a certain ratio and B and C also received the
money in the same ratio. If A gets Rs. 1,60,000 and C gets Rs. 2,50,000. Find the amount
received by B?
(a) Rs. 2,00,000 (b) Rs. 2,50,000
(c) Rs. 1,00,000 (d) Rs. 1,50,000

82. The ratio compounded of 4 : 5 the sub duplicate ratio of a : 9 is 8 : 15 then a:


(a) 2 (b) 3
(c) 4 (d) 5

83. Which of the following are not in proportion


(a) 6,8,5,7 (b) 7,3,14,6
(c) 18,27,12,18 (d) 8,6,12,9

84. Find the two numbers such that the mean proportional between them is 18 and third
proportional between them is 144.
(a) 9, 36 (b) 8, 32
(c) 7, 28 (d) 6, 24

85. Triplicate ratio of 4 : 5 is :


(a) 125 : 64 (b) 16 : 25
(c) 64 : 125 (d) 120 : 46

11 | P a g e
86. The mean proportion between 24 and 54 is _____
(a) 33 (b) 34
(c) 35 (d) 36

87. Divide 80 into two parts so that their products is maximum, then the numbers are :
(a) 15, 65 (b) 25, 25
(c) 35, 45 (d) 40, 40

88. The ratio of numbers is 1 : 2 : 3 and sum of their squares is 504 then the numbers are:
(a) 6, 12, 18 (b) 3, 6, 9
(c) 4, 8, 12 (d) 5, 10, 15

89. If P is 25% less than that of Q and the salary of R is 20% higher than that of Q the Ratio of
the salary of R and P will be
(a) 5:8 (b) 8:5
(c) 5:3 (d) 3:5

90. A person has assets worth Rs. 1,48,200. He wish to divide it amongst his wife, son and
daughter in the ratio 3:2:1 respectively. From this assets the share of his son will be
(a) Rs. 74,100 (b) Rs. 37,050
(c) Rs. 49,400 (d) Rs. 24,700

91. If x:y = 2:3 then (5x+2y):(3x-y)=


(a) 19:3 (b) 16:3
(c) 7:2 (d) 7:3

92. The first, second and third month salaries of a person are in the ratio 2:4:5. The difference
between the product of the salaries of first 2 months & last 2 months in 4,80,00,000. Find the
salary of the second month.
(a) Rs. 4,000 (b) Rs. 6,000
(c) Rs. 12,000 (d) Rs. 8,000

93. If 15(2p2-q2)=7pq, where p and q are positive, then p:q will be:
(a) 5:6 (b) 5:7
(c) 3:5 (d) 8:3

12 | P a g e
ANSWER

1. 2. 3. 4. 5. 6. 7. 8. 9. 10.
(A) (B) (C) (D) (A) (C) (A) (C) (C) (B)
11. 12. 13. 14. 15. 16. 17. 18. 19. 20.
(C) (B) (A) (D) (D) (A) (A) (B) (D) (A)
21. 22. 23. 24. 25. 26. 27. 28. 29. 30.
(C) (D) (C) (A) (B) (B) (C) (B) (A) (B)
31. 32. 33. 34. 35. 36. 37. 38. 39. 40.
(A) (B) (A) (A) (B) (A) (B) (C) (B) (C)
41. 42. 43. 44. 45. 46. 47. 48. 49. 50.
(A) (A) (B) (B) (A) (B) (A) (A) (B) (C)
51. 52. 53. 54. 55. 56. 57. 58. 59. 60.
(C) (B) (A) (C) (C) (A) (A) (A) (C) (B)
61. 62. 63. 64. 65. 66. 67. 68. 69. 70.
(A) (A) (B) (A) (C) (A) (B) (A) (A) (A)
71. 72. 73. 74. 75. 76. 77. 78. 79. 80.
(D) (D) (B) (A) (A) (C) (C) (C) (D) (A)
81. 82. 83. 84. 85. 86. 87. 88. 89. 90.
(A) (C) (A) (A) (C) (D) (D) (A) (B) (C)
91. 92. 93.

(B) (D) (A)

13 | P a g e
CHAPTER – 1 - INDICES
PRACTICE QUESTIONS

1. 4x-1/4 is expressed as
(a) - 4x-1/4 (b) x1
(c) 4/x1/4 (d) none of these

2. The value of 81/3 is


(a) 3
2 (b) 4
(c) 2 (d) none of these

3. The value of 2 × (32) 1/5 is


(a) 2 (b) 10
(c) 4 (d) none of these

4. The value of 4/(32)1/5 is


(a) 8 (b) 2
(c) 4 (d) none of these

5. The value of (8/27) 1/3 is


(a) 2/3 (b) 3/2
(c) 2/9 (d) none of these

6. The value of 2(256) -1/8 is


(a) 1 (b) 2
(c) 1/2 (d) none of these

1 3
2 4
7. 2 × 4 is equal to
(a) a fraction (b) a positive integer
(c) a negative integer (d) none of these
1
 81x 4  4
8.  8  has simplified value equal to
 y 
(a) xy2 (b) x2y
(c) 9xy2 (d) none of these
a-b
9. x × x b-c× x c-a is equal to
(a) x (b) 1
(c) 0 (d) none of these
0
 2 p2q3 
10. The value of   is equal to
 3 xy 
(a) 0 (b) 2/3
(c) 1 (d) none of these

14 | P a g e
11. (33)2 x (42)3 x (53)2 / (32)3 x (43)2 x (52)3 is
(a) 3/4 (b) 4/5
(c) 4/7 (d) 1

12. Which is True?


(a) 2° > (1/2)° (b) 2° < (1/2)°
(c) 2° = (1/2)° (d) none of these

13. If x1/p = y1/q = z1/r and xyz = 1, then the value of p+q+r is
(a) 1 (b) 0
(c) 1/2 (d) none of these

14. The value of ya-b × yb-c × yc-a × y-a-b is


(a) ya+b (b) y
a+b
(c) 1 (d) l/y

15. The True option is


3
(a) x
2/3
= x2 (b) x
2/3
= x3
2/3 2/3
(c) x >3 x2 (d) x > 3 x2
-3
16. The simplified value of 16 x y2 × 8-1 x
3
y-2 is
(a) 2xy (b) x y/2
(c) 2 (d) none of these

17. The value of (8/27) -1/3 × (32/243) -1/5 is


(a) 9/4 (b) 4/9
(c) 2/3 (d) none of these

18. The value of [( x +y) 2/3 ( x -y) 3/2/ x  y  ( x  y )3 ]6 is


(a) ( x +y) 3 (b) ( x -.y)
(c) x +y (d) none of these

19. Simplified value of (125)-2/3 × 25 × 3 53 × 51/2


(a) 5 (b) 1/5
(c) 1 (d) 5

20. [{(2)1/2 . (4) 3/4 . (8)5/6. (16)7/8 . (32)9/10 }4]3/25 is


(a) A fraction (b) an integer
(c) 1 (d) none of these

21. [1 - {1 - (1 - x2) -1 -1
} ] -1/2
is equal to
(a) x (b) 1/x
(c) 1 (d) none of these

15 | P a g e
1
 n n  1n  n 1
22. ( x )  is equal to
 
(a) xn (b) x n+1
(c) x n-1 (d) none of these

l 2  lm m 2 m 2  mn  n 2 n 2  nl  l 2
 xl   xm   xn 
23.  m  n   l 
x  x  x 
(a) 0 (b) 1
(c) x (d) none of these

24. Tick the correct of these when x = p1/3 — p-1/3


(a) x3 +3x= p+1/p (b) x3 +3x= p-1/p
(c) x3 +3x= p+1 (d) none of these

25. On simplification, 1/(1+am-n + am-p )+1/(1+ an-m + an-p) +1/(1+aP-m+a p-n


) is equal to
(a) 0 (b) a
(c) 1 (d) 1/a

a b bc ca
 xa   xb   xc 
26. The value of 
 xb 
 
 xc 
 
 xa 

     
(a) 1 (b) 0
(c) 2 (d) none of these

1 1

27. If x= 3 + 3
3 3
, then3x3-9x is
(a) 15 (b) 10
(c) 12 (d) none of these

28. If ax =b, by= c, c z


= a, then xyz is
(a) 1 (b) 2
(c) 3 (d) none of these

( a 2  ab b 2 ) ( b 2  bc c 2 ) ( c 2  ca  a 2 )
 xa   xb   xc 
29. The value of 
 x b    c    a 
  x  x 
(a) 1 (b) 0
(c) -1 (d) none of these

1 1 1
30. If 2x =3y =6-z,   =
x y z
(a) 1 (b) 0
(c) 2 (d) none of these

16 | P a g e
31. 1 1
If x   2 , then the value of x 2  
x x2
(a) 2 (b) 4
(c) 8 (d) 16

32. Value of 2𝑥1/2 . 3𝑥 −1 if x = 4 is


(a) 3 (b) 4
(c) 3.5 (d) 4.5
−6 −2
33. Find the value of p from (√4) × (√2) = 2𝑃
(a) 16 (b) 8
(c) –7 (d) 4

34. 184 𝑥 −9/8 is expressed as


184
(a) −184𝑥 9/8 (b)
𝑥 −9/8
184
(c) 𝑥 9/8 (d)
𝑥 9/8

35. 𝑃
𝑃 𝑄
𝑃
−1
If 𝑝 = 𝑄 then value of
𝑄 𝑃 ( ) - 𝑃𝑄 reduces to
𝑄
(a) p (b) q
(c) 0 (d) None of these

36. 31/4 × 63/4 × 25/4 is equal to


(a) 10 (b) 12
(c) 14 (d) 0

37. If 1176 = 2𝑝 . 3𝑞 . 7𝑟 . Find the value of p, q, r.


(a) 1, 2, 3 (b) 1, 3, 2
(c) 3, 1, 2 (d) 1, 3, 5

38. If 3𝑥 = 2, 5𝑦 =3 and 2𝑧 =5, find the value of multiply of x.y.z


(a) 0 (b) 1
(c) 2 (d) None of these

39. 1 1
If x   3 , then the value of x 3  
x x3
(a) 16 (b) 14
(c) 18 (d) None of these

40. 1

1
If x  5
53 3 , then 5 x 3  15 x 
(a) 26 (b) 25
(c) 27 (d) 28

41. 216 × 310 × 54


The value of is equal to
212 × 36 × 53
(a) 2160 (b) 6480
(c) 648 (d) 3240

17 | P a g e
42. 1

1
If x  4 43 3 , then 4 x 3  12 x is equal to:
(a) 15 (b) 16
(c) 17 (d) 18

43.
, then x  y
1 1 2 2
If x  3 and y  3 is equal to :
3 3
(a) 4 (b) 6
(c) 9 (d) None of these

44. a 2  ab b 2 b 2  bc c 2 c 2  ac a 2


 xa   xb   xc 
      is equal to :
 x b   x c   xa 
     
3
 b3  c3 )
(a) 1 (b) x2 (a
x 2 ( a  b3  c 3 )
3
(c) (d) None of these

45. 6
a 4b x 6 . (a 2 / 3 x 1 )  b is equal to : x
(a) 1 (b) x1 b
(c) x1 b (d) None of these

𝑜 1 4
46. The value of (64) +(64)2 + (32)−5
1

1 3
(a) 17 8 (b) 17 8
7 1
(c) 11 8 (d) 9
16

47. 2n  2n1
is equal to:
2n1  2n
1 3
(a) (b)
2 2
(c) 1 (d) None of these

48. Evaluate √0.5173


4

(a) 0.8480 (b) 0.8210


(c) 0.6480 (d) None of theses

49. 3 0.7214 ×20.37


Evaluate √ 69.8
(a) 1.5948 (b) 0.5948
(c) 0.2348 (d) None of these

8
50. If (4)3× (√2) = 2𝑛 , then n is
(a) 10 (b) 12
(c) 13 (d) None of these

PAST EXAMINATION QUESTIONS


18 | P a g e
51. Value of (a1/ 8 + a-1/ 8) (a1/ 8 – a-1/ 8) (a1/4 + a -1/4) (a1/2+ a-1/2) is :
1 1
(a) a (b) a
a a
1 1
(c) a2  2 (d) a2  2
a a

1 1 1
52. On simplification  
ab ac bc ba ca cb reduces to:
1 z z 1 z z 1 z z
1 1
(a) (b)
z 2 a b d  z  a b  c 
(c) 1 (d) 0

53. If 4x=5y=20z then z is equal to:


1
(a) xy (c )
xy
x y xy
(b) (d )
xy x y

54. If x = ya, y =zb and z= xc then abc is:


(a) 2 (b) 1
(c) 3 (d) 4

55. If 2x -2x-1=4, then the value of Xx is:


(a) 2 (b) 1
(c) 64 (d) 27

56. If x = 31/3 + 3 -1/3


then find value of 3x3 - 9x :
(a) 3 (b) 9
(c) 12 (d) 10

57. If 2x x 3Y x 5Z = 360 Then what is the value of x, y, z.?


(a) 3,2,1 (b) 1,2,3
(c) 2,3,1 (d) 1,3,2

58. The Recurring Decimal 2.7777------ can be expressed as :


(a) 24/9 (b) 22/9
(c) 26/9 (d) 25/9
n+1 n
3 +3
59. The value of =
n+3 n+1
3 -3
(a) 1/3 (b) 1/6
(c) 1/4 (d) 1/9

19 | P a g e
60. Value of x, if X  X 1/3  ( X 1/3 ) X
(a) 3 (b) 4
(c) 2 (d) 6

x m  3 n .x 4 m 9 n
61. Simplification of is
x 6 m6 n
(a) xm (b) x-m
(c) xn (d) x-n

5/ 2
 3
7/2
 9 
62.      9 is equal to:
 9  3 3
 
3
(a) 1 (b) 3
(c) 3 (d) None of these

a  b  c
3

63. If a  3 b  3 c  0 then find the value of 


3
 
 3
1
(a) 9abc (b)
9 abc
1
(c) abc (d)
abc

64. If (25)150=(25x)50 then the value of x will be


(a) 53 (b) 54
(c) 52 (d) 5

65. The value of


a 2  ab b2 b2  bc c 2 c 2  ca  a 2
 ya   yb   yc 
     
 yb   yc   ya 
     
(a) y (b) -1
(c) 1 (d) None

66. If Px=q, qy=r and rz = p6, then the value of xyz will be:
(a) 0 (b) 1
(c) 3 (d) 6

20 | P a g e
ANSWER

1. 2. 3. 4. 5. 6. 7. 8. 9. 10.
(C) (C) (C) (B) (A) (A) (B) (D) (B) (C)
11. 12. 13. 14. 15. 16. 17. 18. 19. 20.
(D) (C) (B) (D) (A) (C) (A) (C) (D) (B)
21. 22. 23. 24. 25. 26. 27. 28. 29. 30.
(A) (C) (B) (B) (C) (A) (B) (A) (A) (B)
31. 32. 33. 34. 35. 36. 37. 38. 39. 40.
(A) (A) (C) (D) (C) (B) (C) (B) (C) (A)
41. 42. 43. 44. 45. 46. 47. 48. 49. 50.
(B) (A) (A) (B) (B) (D) (B) (A) (B) (A)
51. 52. 53. 54. 55. 56. 57. 58. 59. 60.
(B) (C) (D) (B) (D) (D) (A) (D) (B) (B)
61. 62. 63. 64. 65. 66.
(B) (A) (C) (B) (C) (D)

CHAPTER – 1 - LOGARITHM
21 | P a g e
PRACTICE QUESTIONS

1. log 6 + log 5 is expressed as


(a) log 11 (b) log 30
(c) log 5/6 (d) none of these

2. log2 8 is equal to
(a) 2 (b) 8
(c) 3 (d) none of these

3. log32/4 is equal to
(a) log 32/log 4 (b) log 32 — log 4
(c) 23 (d) none of these

4. log(1 ×2×3) ise qual to


(a) log 1 + log 2 + log 3 (b) log 3
(c) log 2 (d) none of these

5. The value of log 0.0001 to the base 0.1 is


(a) -4 (b) 4
(c) 1/4 (d) none of these

6. If 2 log x=4 1og 3,the x is equal to


(a) 3 (b) 9
(c) 2 (d) none of these

7. log 2
64 is equal to
(a) 12 (b) 6
(c) 1 (d) none of these

8. log 2 3
1728 is equal to

(a) 2 3 (b) 2
(c) 6 (d) none of these

9. log (1/81) to the base 9 is equal to


(a) 2 (b) ½
(c) -2 (d) none of these

10. log 0.0625 to the base 2 is equal to


(a) 4 (b) 5
(c) 1 (d) -4

11. Given log2 = 0.3010 and log3 = 0.4771 the value of log 6 is
(a) 0.9030 (b) 0.9542
(c) 0.7781 (d) none of these

12. The value of log2 log2 log2 16 is


(a) 0 (b) 2
22 | P a g e
(c) 1 (d) none of these

13. The value of log 0.3 to the base 9 is


(a) -½ (b) ½
(c) 1 (d) none of these

14. If logx + log y = log (x+y), y can be expressed as


x 1
(a) x—1 (b)
x
x
(c) (d) none of these
x 1

15. The value of log2 [log2 {log3 (log3 273)}]is equal to


(a) 1 (b) 2
(c) 0 (d) none of these

16. If log2x + log4x + log16 x = 21/4, these x is equal to


(a) 8 (b) 4
(c) 16 (d) none of these

17. Given that log10 2 = x and log 103= y, the value of log10 60 is expressed as
(a) x-y+1 (b) x+y+1
(c) x—y-1 (d) none of these

18. Given that log10 2= x, log10 3= y, then log10 1.2 is expressed in terms of x and y as
(a) x+2y—1 (b) x+y—1
(c) 2x+y—1 (d) none of these

19. Given that log x = m + n and log y = m — n, the value of log l0x/y2 is expressed in terms of
m and n as
(a) 1—m+3n (b) m—1+3n
(c) m+3n+1 (d) none of these

20. The simplified value of 2 log10 5+ log10 8- ½ log10 4 is


(a) ½ (b) 4
(c) 2 (d) none of these

21. log [1-{1- (1 — x2)-1}-1]-1/2 can be written as


(a) log x2 (b) log x
(c) log 1/x (d) none of these

4 13  43
22. The simplified value of log 729 9 .27 is
(a) log3 (b) log 2
(c) log1/x (d) none of these

3
23. The value of (log b a x log c b x loga c) is equal to
(a) 3 (b) 0

23 | P a g e
(c) 1 (d) none of these

24. The logarithm of 64 to the base 2 2 is


(a) 2 (b) 2
(c) ½ (d) 4

25. The value of log825 given log 2 = 0.3010 is


(a) 1 (b) 2
(c) 1.5482 (d) none of these

26. Find the logarithmic of 58,564 to the base 11√ 2


(a) 3 (b) 4
(c) 2 (d) None of these

27. The base of log613 into the common logarithmic base is


log6 10 log10 6
(a) (b)
log6 13 log10 13
log10 13
(c) (d) None of these.
log10 6

28. If 10−2 = 0.01, then the value of log10 0.01 is


(a) –2 (b) 2
(c) –1 (d) 1

29. The value of


1
+
1
is
loga (ab) logb (ab)
(a) 0 (b) 1
(c) –1 (d) None of these.

30. log √2 128 is equal to


(a) 6 (b) 12
(c) 24 (d) 14

31. log 3 log 3 27 is equal to


(a) 0 (b) -1
(c) 3 (d) 1

32. If log √3 (x + 1) = 2 then x is equal to


(a) 1 (b) 3
(c) 2 (d) 0

33. The value of log 4 log 3 81 is equal to


(a) 1 (b) 0
(c) 3 (d) None of these

34. log 64 512 is equal to


(a) 3 (b) 2
(c) 1 (d) 3/2
35. The value of log (1 + 2 + 3 + ............. +n) is equal to
(a) log 1+ log 2 + …. + log n (b) log n + log (n+1) – log 2

24 | P a g e
(c) 0 (d) 1

36. log (12 + 22 + 32 ) is equal to


(a) log 12 + log 22 + log 32 (b) log 2 + log 7
(c) log 2 – log 7 (d) None of these

37. log (3 × 5 × 7)2 is equal to _________


(a) 2(log 3 + log 5 + log 7) (b) log (2×3×5×7)
(c) 2(log 3 – log 5 – log 7) (d) None of these

38. 2 2 2
a b c
The value of log  log  log is equal to
bc ca ab
(a) 0 (b) 1
(c) -1 (d) None of these

39. The value of 𝑙𝑜𝑔3


1
is
81
(a) 4 (b) –4
(c) 2 (d) –2

40. The value of 𝑙𝑜𝑔2√2


1
256
16
(a) (b) –4
3
−16
(c) 3 (d)
3

41. If 𝑙𝑜𝑔4 [𝑙𝑜𝑔3 (𝑙𝑜𝑔2x)] = 0; then value of x is


(a) 16 (b) 32
(c) 4 (d) None of these

42. The value of 𝑙𝑜𝑔𝑥 (0.00001) = –5, then x is


(a) 10 (b) 102
(c) 10 𝑜
(d) None of these

43. The value of 𝑙𝑜𝑔𝑎 √𝐴


𝑛

1
(a) 𝑙𝑜𝑔𝑎 𝐴 (b) 𝑎 𝑙𝑜𝑔1/𝑛 𝐴
𝑛
1
(c) 𝑎 𝑙𝑜𝑔𝑎 (𝑛) (d) None of these

44. The value of


𝑙𝑜𝑔10 4
𝑙𝑜𝑔10 8
1 4
(a) (b)
3 3
2
(c) (d) None of these
3

45. If 𝑙𝑜𝑔10 12.45 = 1.0952 and 𝑙𝑜𝑔10 3.79 = 0.5786, Find the value of 𝑙𝑜𝑔10 124.5 + 𝑙𝑜𝑔10 379
(a) 5.6738 (b) 4.6738
(c) 6.6738 (d) None of these

46. If 𝑙𝑜𝑔2 x + 𝑙𝑜𝑔8 x + 𝑙𝑜𝑔32 x =


23
then the value of x is
15

25 | P a g e
(a) 8 (b) 5
(c) 2 (d) None of these

47. If log
𝑎+𝑏
=
1
(log a + log b), the value of 𝑎2 + 𝑏 2 is
2 2
(a) 2ab (b) 8ab
(c) 6ab (d) None of these

48.
x  log 10 log e then x is
log e2 log 625 16 10

(a) 7 (b) 8
(c) 5 (d) None of these

49. If log 2 = 0.301 and log 3 = 0.477, then the value of log 225 is:
(a) 2.352 (b) 3.452
(c) 7.452 (d) None of these

50. If log 2 = 0.3010, find the number of digits in 2100


(a) 36 (b) 31
(c) 38 (d) None of these

51. If log x √3 =
1
find the value of x
6
(a) 9 (b) 27
(c) 18 (d) None of these

52. The value of 𝑎𝑙𝑜𝑔𝑎 𝑥 is


(a) x (b) 𝑙𝑜𝑔 𝑎 𝑥
(c) 𝑥 2 (d) None of these

53. The value of 32−𝑙𝑜𝑔3 6 is


(a) 9/5 (b) 3/2
(c) 9/4 (d) None of these

PAST EXAMINATION QUESTIONS


loga b logb c logc d logd t
54. The value of the expression: a
(a) t (b) abcdt
(c) (a+b+c+d+t) (d) None.
1
55. If log 10000 X= then X is given by:
4
1
(a)
100
1
(b )
10
1
(c )
20
(d) None of these

56. If log (2a - 3b)=log a - log b then a:


26 | P a g e
3b 2 b2
(a) (c )
2b  1 2b  1
3b 3b 2
(b) (d )
2b  1 2b  1

1 1 1
57.   is equal to:
ab 
abc 
bc 
log log abc  logca  abc 
(a) 0 (b) 1
(c) 2 (d) -1

58. Number of digits in the numeral for 2 64. [Given log2= 0.3010]
(a) 18 digits (b) 19 digits
(c) 20 digits (d) 21 digits

log 8
3
59. The value
log 16.log 10 is :
9 4
(a) 3log102 (b) 7log103
(c) 3loge2 (d) None

 10 x 
60. If log x = a+b ; log y = a-b then log  2 
 y 
(a) 1-a+3b (b) a-1+3b
(c) a+3b+1 (d) 1-b+3a

61. log144 is equal to :


(a) 2log4+2log2 (b) 4log2+2log3
(c) 3log2+4log3 (d) 3log2-4log3

62. If log2 [log3 (log2 x)]= 1 then x equals:


(a) 128 (b) 256
(c) 512 (d) None

If log  a  b   1 (log a + log b) then the value of  is:


a b
63.
 4  2 b a
(a) 12 (b) 14
(c) 16 (d) 8

64. log (m + n) = log m + log n, m can be expressed as:


n n
(a) m= (b) m=
n 1 n 1
n 1 n 1
(c) m= (d) m=
n n 1

65. log4 (x2 +x)-log4 (x+1) = 2. Find x .

27 | P a g e
(a) 16 (b) 0
(c) -1 (d) None of these

66. Find the value of [log10 25 - log10 (23) + log10 (4)2]x


(a) x (b) 10
(c) 1 (d) None

67. If logab + loga c=0 then .


(a) b=c (b) b = -c
(c) b=c=1 (d) b and c are reciprocals

68. The Value of


2 log x + 2 log x2 + 2 log x3 + ---------+2 log xn will be: -
n( n  1) log x
(a) (b) n (n+1) log x
2
(c) n2 log x (d) None of these

 log 10x  3   11  log 10x 


69.     2 Then x =
 2   3 
(a) 10-1 (b) 102
(c) 10 (d) 103

70. If n = m! where (‘m’ is a positive integer > 2) then the value of:
1 1 1 1
n
 n
 n
 .........  n
log 2 log 3 log 4 log m
(a) 1 (b) 0
(c) –1 (d) 2

71. log 2x  log 4x  6 then x 


(a) 16 (b) 32
(c) 64 (d) 128

72. If logx y =100 and log2 x =10 then the value of y


(a) 210 (b) 2100
(c) 21000 (d) 210000

1 1 1 1
73. Which is True if   
ab bc ca abc
1 1 1
(a) log (ab+bc+ca) = abc (b) log     = abc
a b c
(c) log (abc) = 0 (d) log (a+b+c) = 0

74. If log 10 5  log 10 (5x  1)  log 10 (x  5)  1, Then x 


(a) 5 (b) 3
(c) 1 (d) None
75. If (log x
2) 2  log x 2 then x =
(a) 16 (b) 32
28 | P a g e
(c) 8 (d) 4

76. Find value of [log y x log z y .log x z ]3 


(a) 0 (b) -1
(c) 1 (d) 3

77. Find the value of Log49.Log32 is


(a) 3 (b) 9
(c) 2 (d) 1

1 1 1
78. X=1+ log p qr, y=1+logq rp, z=1+logr pq then find  
x y z
(a) 0 (b) 1
(c) -1 (d) 3

79. The value of log53 x log34 x log25.


(a) 0 (b) 1
(c) 2 (d) 1/2

ANSWER
1. 2. 3. 4. 5. 6. 7. 8. 9. 10.
(B) (C) (B) (A) (B) (B) (A) (C) (C) (D)
11. 12. 13. 14. 15. 16. 17. 18. 19. 20.
(C) (C) (A) (C) (C) (A) (B) (C) (A) (C)
21. 22. 23. 24. 25. 26. 27. 28. 29. 30.
(B) (A) (C) (D) (C) (B) (C) (A) (B) (D)
31. 32. 33. 34. 35. 36. 37. 38. 39. 40.
(D) (C) (A) (D) (B) (B) (A) (A) (B) (D)
41. 42. 43. 44. 45. 46. 47. 48. 49. 50.
(D) (A) (A) (C) (B) (C) (A) (C) (A) (B)
51. 52. 53. 54. 55. 56. 57. 58. 59. 60.
(B) (A) (B) (A) (B) (A) (C) (C) (A) (A)
61. 62. 63. 64. 65. 66. 67. 68. 69. 70.
(B) (C) (B) (A) (A) (C) (D) (B) (A) (A)
71. 72. 73. 74. 75. 76. 77. 78. 79.
(A) (C) (D) (B) (A) (C) (D) (B) (C)

29 | P a g e
CHAPTER -2 - EQUATION
PRACTICE QUESTIONS

1. The equation –7x + 1 = 5–3x will be satisfied for x equal to:


(a) 2 (b) –1
(c) 1 (d) none of these

x4 x 5
2. The root of the equation   11 is:
4 3
(a) 20 (b) 10
(c) 2 (d) none of these

x 2
3. Pick up the correct value of x for 
30 45
(a) x=5 (b) x=7
1
(c) x 1 (d) none of these
3
x  24 x
4. The solution of the equation  4 :
5 4
(a) 6 (b) 10
(c) 16 (d) none of these

5. 8 is the solution of the equation


x  4 x 5 x  4 x  10
(a)   11 (b)  8
4 3 2 9
x  24 x x  15 x  5
(c)  4 (d)  4
5 4 10 5

y  11 y 1 y 7
6. The value of y that satisfies the equation   is
6 9 4
(a) –1 (b) 7
1
(c) 1 (d) 
7
7. The solution of the equation (p+2) (p–3) + (p+3) (p–4) = p(2p–5) is:
(a) 6 (b) 7
(c) 5 (d) none of these
12x  1 15x  1 2x  5
8. The equation   is true for:
4 5 3x  1
(a) x=1 (b) x=2
(c) x=5 (d) x=7
x 1 x 1
9. Pick up the correct value x for which    0
0.5 0.05 0.005 0.0005
(a) x=0 (b) x=1
(c) x=10 (d) none of these

30 | P a g e
10. The solution of the set of equations 3x + 4y = 7, 4x – y = 3 is:
(a) (1, –1) (b) (1, 1)
(c) (2, 1) (d) (1, –2)

x y
11. The values of x and y satisfying the equations   2, x  2 y  8 are given by the pair.
2 3
(a) (3, 2) (b) (–2, –3)
(c) (2, 3) (d) none of these

x y
12.   2, x  y  p  q are satisfied by the values given by the pair.
p q
(a) (x=p, y=q) (b) (x q, y=p)
(c) (x=1, y=1) (d) none of these

1 1 9 1 1 4
13. The solution for the pair of equations   ,   is given by :
16 x 15 y 20 20 x 27 y 45
1 1 1 1
(a)  ,  (b)  , 
4 3 3 4
(c) (3 4) (d) (4 3)

4 5 xy 3
14. Solve for x and y:    and 3xy = 10 (y-x):
x y xy 10
(a) (5, 2) (b) (–2, –5)
(c) (2, –5) (d) (2, 5)

xy 5
15. The pair satisfying the equations x + 5y = 36,  is given by :
xy 3
(a) (16, 4) (b) (4, 16)
(c) (4, 8) (d) none of these.

16. Solved for x and y: x–3y = 0, x+2y = 20.


(a) x=4, y=12 (b) x=12, y=4
(c) x=5, y=4 (d) none of these

17. The simultaneous equations 7x–3y=31, 9x-5y = 41


(a) (–4, –1) (b) (–1, 4)
(c) (4, –1) (d) (3, 7)

18. 1.5x + 2.4y = 1.8, 2.5(x+1) = 7y have solutions as :


(a) (0.5, 0.4) (b) (0.4, 0.5)
1 2
(c)  ,  (d) (2, 5)
2 5

31 | P a g e
19. The values of x and y satisfying the equations:
3 2 2 3 2
  3,  3 are given by :
xy xy xy xy 3
(a) (1, 2) (b) (–1, –2)
(c) (1, ½) (d) (2, 1)

20. 1.5x + 3.6y = 2.1, 2.5 (x+1) = 6y


(a) (0.2, 0.5) (b) (0.5, 0.2)
(c) (2, 5) (d) (–2, –5)

x y x y
21.   1    28
5 6 6 5
(a) (6, 9) (b) (9, 6)
(c) (60, 90) (d) (90, 60)

x y z
22.   , 7 x  8 y  5z  62
4 3 2
(a) (4, 3, 2) (b) (2, 3, 4)
(c) (24, 9, 5) (d) (30, 60, 120)

xy yz zx
23.  20,  40,  24
xy yz zx
(a) (120, 60, 30) (b) (60, 30, 120)
(c) (30, 120, 60) (d) (30, 60, 120)

24. 2x + 3y + 4z = 0, x+2y – 5z = 0, 10x + 16y – 6z = 0


(a) (0, 0, 0) (b) (1, –1, 1)
(c) (3, 2, –1) (d) (1, 0, 2)

1 1 1
25. ( x  y )  2 z  21,3 x  ( y  z )  65, x  ( x  y  z )  38
3 2 2
(a) (4,9,5) (b) (2,9,5)
(c) (24,9,5) (d) (5,24,9)

4 5 x y 3
26.    , 3xy  10( y  x )
x y xy 10
(a) (2, 5) (b) (5, 2)
(c) (2, 7) (d) (3, 4)

x y  0.03 y x  0.03
   2
27. 0.01 0.05 0.02 0.04
(a) (1, 2) (b) (0.1, 0.2)
(c) (0.01, 0.02) (d) (0.02, 0.01)

32 | P a g e
xy yz zx 60
28.  110,  132, 
yx zy z  x 11
(a) (12, 11, 10) (b) (10, 11, 12)
(c) (11, 10, 12) (d) (12, 10, 11)

29. 3x–4y+70z=0, 2x+3y–10z=0, x+2y+3z=13


(a) (1, 3, 7) (b) (1, 7, 3)
(c) (2, 4, 3) (d) (–10, 10, 1)

30. The sum of two numbers is 52 and their difference is 2. The numbers are
(a) 17 and 15 (b) 12 and 10
(c) 27 and 25 (d) none of these

31. The diagonal of a rectangle is 5 cm and one of at sides is 4 cm. Its area is
(a) 20 sq.cm. (b) 12 sq.cm.
(c) 10 sq.cm. (d) none of these

32. Divide 56 into two parts such that three times the first part exceeds one third of the second
by 48. The parts are.
(a) (20,36) (b) (25,31)
(c) (24,32) (d) none of these

33. The sum of the digits of a two digit number is 10. If 18 be subtracted from it the digits in the
resulting number will be equal. The number is
(a) 37 (b) 73
(c) 75 (d) none of these numbers.

34. The fourth part of a number exceeds the sixth part by 4. The number is
(a) 84 (b) 44
(c) 48 (d) none of these

35. Ten years ago the age of a father was four times of his son. Ten years hence the age of the
father will be twice that of his son. The present ages of the father and the son are.
(a) (50,20) (b) (60,20)
(c) (55,25) (d) none of these

36. The product of two numbers is 3200 and the quotient when the larger number is divided by
the smaller is 2.The numbers are
(a) (16,200) (b) (160,20)
(c) (60,30) (d) (80,40)

37. The denominator of a fraction exceeds the numerator by 2. If 5 be added to the numerator
the fraction increases by unity. The fraction is.
5 1
(a) (b)
7 3
7 3
(c) (d)
9 5

33 | P a g e
38. Three persons Mr. Roy, Mr. Paul and Mr. Singh together have Rs. 51. Mr. Paul has Rs. 4 less
than Mr. Roy and Mr. Singh has got Rs. 5 less than Mr. Roy. They have the money as.
(a) (Rs. 20, Rs. 16, Rs. 15) (b) (Rs. 15, Rs. 20, Rs. 16)
(c) (Rs. 25, Rs. 11, Rs. 15) (d) none of these

39. A number consists of two digits. The digits in the ten’s place is 3 times the digit in the unit’s
place. If 54 is subtracted from the number the digits are reversed. The number is
(a) 39 (b) 92
(c) 93 (d) 94

40. One student is asked to divide a half of a number by 6 and other half by 4 and then to add
the two quantities. Instead of doing so the student divides the given number by 5. If the
answer is 4 short of the correct answer then the number was
(a) 320 (b) 400
(c) 480 (d) none of these.
1
41. If a number of which the half is greater than th of the number by 15 then the number is
5
(a) 50 (b) 40
(c) 80 (d) none of these.

42. Monthly incomes of two persons are in the ratio 4 : 5 and their monthly expenses are in the
ratio 7 : 9. If each saves Rs. 50 per month find their monthly incomes.
(a) (500, 400) (b) (400, 500)
(c) (300, 600) (d) (350, 550)

43. Find the fraction which is equal to 1/2 when both its numerator and denominator are
increased by 2. It is equal to 3/4 when both are increased by 12.
(a) 3/8 (b) 5/8
(c) 2/8 (d) 2/3

44. The age of a person is twice the sum of the ages of his two sons and five years ago his age
was thrice the sum of their ages. Find his present age.
(a) 60 years (b) 52 years
(c) 51 years (d) 50 years

45. A number between 10 and 100 is five times the sum of its digits. If 9 be added to it the digits
are reversed find the number.
(a) 54 (b) 53
(c) 45 (d) 55

46. The wages of 8 men and 6 boys amount to Rs. 33. If 4 men earn Rs. 4.50 more than 5 boys
determine the wages of each man and boy.
(a) (Rs. 1.50, Rs. 3) (b) (Rs. 3, Rs. 1.50)
(c) (Rs. 2.50, Rs. 2) (d) (Rs. 2, Rs. 2.50)

47. A number consisting of two digits is four times the sum of its digits and if 27 be added to it
the digits are reversed. The number is :
(a) 63 (b) 35
(c) 36 (d) 60

34 | P a g e
48. Of two numbers, 1/5th of the greater is equal to 1/3rd of the smaller and their sum is 16. The
numbers are:
(a) (6, 10) (b) (9, 7)
(c) (12, 4) (d) (11, 5)

49. y is older than x by 7 years 15 years back x’s age was 3/4 of y’s age. Their present ages are:
(a) (x=36, y=43) (b) (x=50, y=43)
(c) (x=43, y=50) (d) (x=40, y=47)

50. The sum of the digits in a three digit number is 12. If the digits are reversed the number is
increased by 495 but reversing only of the ten’s and unit digits in creases the number by 36.
The number is
(a) 327 (b) 372
(c) 237 (d) 273

51. Two numbers are such that twice the greater number exceeds twice the smaller one by 18
and 1/3 of the smaller and 1/5 of the greater number are together 21. The numbers are :
(a) (36, 45) (b) (40, 45)
(c) (50, 41) (d) (55, 46)

q 7 .
52. The demand and supply equations for a certain commodity are 4q + 7p = 17 and p  
3 4
respectively where p is the market price and q is the quantity then the equilibrium price and
quantity are:
3 1
(a) 2, (b) 3,
4 2
3
(c) 5, (d) None of these
5

53. A man sells 6 radios and 4 televisions for Rs. 18,480. If 14 radios and 2 televisions are sold
for the same amount, what is the price of a television?
(a) Rs. 1,848 (b) Rs. 840
(c) Rs. 1,680 (d) Rs. 3,360

54. A man starts his job with a certain monthly salary and earns a fixed increment every year. If
his salary was Rs. 1,500 after 4 years of service and Rs. 1,800 after 10 years of service, what
was his starting salary and what is the annual increment in rupees?
(a) Rs. 1,300, Rs. 50 (b) Rs. 1,100, Rs. 50
(c) Rs. 1,500, Rs. 30 (d) None

x  y 13
55. If x+5y = 33 and  then (x,y)
x y 3
(a) (4,8) (b) (8,5)
(c) (4,6) (d) (16,4)

35 | P a g e
56. A man went to the Reserve Bank of India with Rs. 1,000. he asked the cashier to give him Rs.
5 and Rs. 10 notes only in return. The man got 175 notes in all. Find how many notes of Rs. 5
and Rs. 10 did he receive?
(a) (25, 150) (b) (40, 110)
(c) (150, 25) (d) None

57. If the length of a rectangle is 5 cm more than the breadth and perimeter of the rectangle is
40 cm then the length & breadth of rectangle will be:
(a) 7.5 cm, 2.5 cm (b) 10cm, 5cm
(c) 12.5 cm, 7.5 cm (d) 15.5 cm, 10.5 cm

58. Sum of two numbers is 8 and the sum of their squares is 34. The numbers are
(a) (7, 10) (b) (4, 4)
(c) (3, 5) (d) (2, 6)

59. The difference of two positive integers is 3 and the sum of their squares is 89. The integers
are.
(a) (7, 4) (b) (5, 8)
(c) (3, 6) (d) (2, 5)

60. Five times of a positive whole number is 3 less than twice the square of the number. The
number is
(a) 3 (b) 4
(c) –3 (d) 2

61. The area of a rectangular field is 2000 sq.m and its perimeter is 180m. The length and
breadth are
(a) (205m, 80m) (b) (50m, 40m)
(c) (60m, 50m) (d) none

62. Two squares have sides p cm and (p + 5) cms. The sum of their squares is 625 sq. cm. The
sides of the squares are
(a) (10 cm, 30 cm) (b) (12 cm, 25 cm)
(c) (15 cm, 20 cm) (d) none of these

63. Divide 50 into two parts such that the sum of their reciprocals is 1/12. The numbers are
(a) (24, 26) (b) (28, 22)
(c) (27, 23) (d) (20, 30)

64. There are two consecutive numbers such that the difference of their reciprocals is 1/240. The
numbers are
(a) (15, 16) (b) (17, 18)
(c) (13, 14) (d) (12, 13)

65. The hypotenuse of a right–angled triangle is 20cm. The difference between its other two sides
be 4cm. The sides are
(a) (11cm, 15cm) (b) (12cm, 16cm)
(c) (20cm, 24cm) (d) none of these

36 | P a g e
66. The sum of two numbers is 45 and the mean proportional between them is 18. The numbers
are
(a) (15, 30) (b) (32, 13)
(c) (36, 9) (d) (25, 20)

67. The sides of an equilateral triangle are shortened by 12 units 13 units and 14 units
respectively and a right angle triangle is formed. The side of the equilateral triangle is
(a) 17 units (b) 16 units
(c) 15 units (d) 18 units

68. A distributor of apple Juice has 5000 bottle in the store that it wishes to distribute in a month.
From experience it is known that demand D (in number of bottles) is given by
D = –2000p2 + 2000p + 17000. The price per bottle that will result zero inventory is
(a) Rs. 3 (b) Rs. 5
(c) Rs. 2 (d) none of these.

69. The sum of two irrational numbers multiplied by the larger one is 70 and their difference is
multiplied by the smaller one is 12; the two numbers are
(a) 3 2, 2 3 (b) 5 2, 3 5
(c) 2 2, 5 2 (d) none of these.

70. The present age of a man is 8 years more than thrice the sum of the ages of his two
grandsons who are twins. After 8 years, his age will be 10 years more than twice the sum of
the ages of his grandsons. The age of a man when his grandsons were born was
(a) 86 years (b) 73 years
(c) 68 years (d) 63 years

71. If area and perimeter of a rectangle is 6000 cm2 and 340 cm respectively, then the length of
rectangle is:
(a) 140 (b) 120
(c) 170 (d) 200

72. If kx-4=(k-1)x which of the following is true


(a) x=-5 (b) x=-4
(c) x=-3 (d) x=4

73. The number of students in each section of a school is 36. After admitting 12 new students,
four new sections were started. If total number of students in each section now is 30, than
the number of sections initially were.
(a) 6 (b) 10
(c) 14 (d) 18

74. A person on a tour has Rs.9,600 for his expenses. If this tour is extended by 16 days, he has
to cut down his daily expenses by Rs.20, his original duration of tour had been.
(a) 48 days (b) 64 days
(c) 80 days (d) 96 days

37 | P a g e
ANSWER

1. 2. 3. 4. 5. 6. 7. 8. 9. 10.
(B) (A) (C) (C) (B) (D) (A) (D) (C) (B)
11. 12. 13. 14. 15. 16. 17. 18. 19. 20.
(C) (A) (A) (D) (A) (B) (C) (B) (D) (A)
21. 22. 23. 24. 25. 26. 27. 28. 29. 30.
(C) (A) (D) (A) (C) (A) (C) (B) (D) (C)
31. 32. 33. 34. 35. 36. 37. 38. 39. 40.
(B) (A) (B) (C) (A) (D) (D) (A) (C) (C)
41. 42. 43. 44. 45. 46. 47. 48. 49. 50.
(A) (B) (A) (D) (C) (B) (C) (A) (A) (C)
51. 52. 53. 54. 55. 56. 57. 58. 59. 60.
(A) (A) (D) (A) (B) (C) (C) (C) (B) (A)
61. 62. 63. 64. 65. 66. 67. 68. 69. 70.
(B) (C) (D) (A) (B) (C) (A) (A) (C) (B)
71. 72. 73. 74.
(B) (D) (D) (C)

38 | P a g e
CHAPTER – 2 - QUADRATIC EQUATION
PRACTICE QUESTIONS

1. If the roots of the equation 2x2 + 8x – m3 = 0 are equal then value of m is


(a) –3 (b) –1
(c) 1 (d) –2

2. If 22x +3
– 32. 2x + 1 = 0 then values of x are
(a) 0, 1 (b) 1, 2
(c) 0, 3 (d) 0, –3

1
3. The values of 4
1
4
1
4
4  ...
(a) 1 2 (b) 2 5
(c) 1 5 (d) None of these

4. If  be the roots of the equation 2x2 – 4x – 3 = 0 the value of  2  2 is


(a) 5 (b) 7
(c) 3 (d) –4

5. If     2 , and   3 ,  ,  are the roots of the equation, which is:


2
(a) x -2x-3=0 (b) x2+2x-3=0
(c) x2+2x+3=0 (d) x2-2x+3=0

6. The equation x2 –(p+4)x + 2p + 5 = 0 has equal roots the values of p will be.
(a) ±1 (b) 2
(c) ±2 (d) –2

7. The roots of the equation x2 + (2p–1)x + p2 = 0 are real if.


(a) p>1 (b) p<4
(c) p > 1/4 (d) p<¼

8. If x = m is one of the solutions of the equation 2x2 + 5x – m = 0 the possible values of m are
(a) (0, 2) (b) (0, –2)
(c) (0, 1) (d) (1, –1)

9. If p and q are the roots of x2 + 2x + 1 = 0 then the values of p3 + q3 becomes


(a) 2 (b) –2
(c) 4 (d) –4
 2 2
10. If and  are the roots of x2=x+1 then the value of  is:
 
(a) 2 5 (b) 5
(c) 3 5 (d) 2 5

39 | P a g e
11. If one root of 5x2 + 13x + p = 0 be reciprocal of the other then the value of p is
(a) –5 (b) 5
(c) 1/5 (d) –1/5

12. A solution of the quadratic equation (a+b–2c)x2 + (2a–b–c)x + (c+a–2b) = 0 is


(a) x=1 (b) x = –1
(c) x=2 (d) x=–2

13. If the root of the equation x2–8x+m = 0 exceeds the other by 4 then the value of m is
(a) m = 10 (b) m = 11
(c) m=9 (d) m = 12

14. The values of x in the equation 7(x+2p)2 + 5p2 = 35xp + 117p2 are
(a) (4p, –3p) (b) (4p, 3p)
(c) (–4p, 3p) (d) (–4p, –3p)
6 x 6( x  1)
15. The solution of the equation  13 are
x 1 x
(a) (2, 3) (b) (3, –2)
(c) (–2, –3) (d) (2, –3)

16. The values of x for the equation x2 + 9x + 18 = 6 – 4x are


(a) (1, 12) (b) (–1, –12)
(c) (1, –12) (d) (–1, 12)

17. The solution of the equation 3x2–17x + 24 = 0 are


 2
(a) (2, 3) (b)  2, 3 
 3
 2  2
(c)  3, 2  (d)  3, 
 3  3
3(3x 2  15) 2 x 2  96
18. The equation  2x2  9   6 has got the solution as :
6 7
(a) (1, 2) (b) (1/2, –1)
(c) (1, –1) (d) (2, –1)

19. Solving equation x2–(a+b) x+ab=0 are, value(s) of x


(a) a, b (b) a
(c) b (d) none

PAST EXAMINATION QUESTIONS

x 1 x 1
20. On solving   2 , we get one value of x as :
1 x x 6
4 1
(a ) (b)
13 13
2 3
(c ) (d )
13 13

40 | P a g e
21. Find the positive value of k for which the equations: x 2 + kx + 64 = 0 and x2- 8x + k = 0 will
have equal roots:
(a) 12 (b) 16
(c) 18 (d) 22

22. If one root of an equation is 2 + 5 then the quadratic equation :


(a) x2 +4x-1=0 (b) x2-4x-1=0
(c) x2 +4x+1=0 (d) x2-4x+1=0

23. The value of 6  6  6  ..... is:


(a) -3 (b) 2
(c) 3 (d) 4

24. The value of –


2+ 1 .
2+ 1 .
2+ 1 .
2+ 1 .
2+……………. 
(a )1  2
(b ) 2  5
(c ) 2  3
(d ) None
25. If (2+ 3 ) is a root of a quadratic equation x2 + px +q = 0 then find the value of p and q.
(a) (4,-1) (b) (4,1)
(c) ( - 4,1) (d) (2,3)

26. One root of the equation: x2 - 2 (5+m) x+3 (7+m) = 0 is reciprocal of the other.Find the
value of m.
(a) -20/3 (b) 7
(c) 1/7 (d) - 1/7

27. Roots of the equation 3x2-14x+k=0 will be reciprocal of each other if :


(a) k= -3 (b) k=0
(c) k=3 (d) k=14

28. Positive value of 'k' for which the roots of equation 12 x2 + k x + 5 = 0 are in ratio 3:2 is :
(a) 5/12 (b) 12/5
5 10
(c) (d) 5 10
2
29. If one Root of the equation x2 - 3x + k = 0 is 2, then value of k will be :
(a) -10 (b) 0
(c) 2 (d) 10

41 | P a g e
30. If roots of equation x 2  x  r  0 are ' ' and '  ' and 3  3  6. Find the value ‘r’ ?
5 7
(a) (b)
3 3
4
(c) (d) 1
3
2
31. If  , β are the roots of 4x - 6x + q= 0 and  ,β are in the ratio 1 : 2 then q is .
(a) 2 (b) 8
(c) 16 (d) None

32. If one root of equation px2 + qx +r = 0 is r then other root will be:
1 1
(a) (b)
q r
1 1
(c) (d)
p pq

33. If 2 + 3 is one root of X2+ px + q = 0 Then p and q are


(a) -4,-1 (b) 4,-1
(c) -4,1 (d) 4,1

34. If the A.M between the roots of a Quadratic equation is ‘8’ and G.M is ‘5’ then the equation is
____
(a) x2 +16x − 25 = 0 (b) x2 −16x + 25 = 0
2
(c) x −16x + 5 = 0 (d) None of these

35.  are the roots of the 2x2+3x+7=0. Then the value of  1   1 is
(a) 2 (b) 3/7
(c) 7/2 (d) -19/14

36. The quadratic equation x2-2kX+16=0 will have equal roots then k =
(a) 1 (b) 2
(c) 3 (d) 4

37. If  are roots of x2 + 7x + 11 = 0 then the equation whose roots are ()2 & (–)2 is :
(a) x2–54x+245 = 0 (b) x2–14x+49=0
2
(c) x –24x+144=0 (d) x2–50x+49=0

38. If b2-4ac is a perfect square but not equal to zero then the roots of the equation
ax2+bx+c=0 are
(a) Real and equal (b) Real, irrational and equal
(c) Real, rational and unequal (d) Imaginary

42 | P a g e
2 2
39. If  and  be the roots of the quadratic equation 2x2 - 4x = 1, the value of  is.........
 
(a) -11 (b) 22
(c) -22 (d) 11

40. If the roots of the equation 4x2 -12x + k =0 are equal, then the value of k is:
(a) -3 (b) 3
(c) -9 (d) 9

ANSWER

1. 2. 3. 4. 5. 6. 7. 8. 9. 10.
(D) (D) (B) (B) (B) (C) (D) (B) (B) (D)
11. 12. 13. 14. 15. 16. 17. 18. 19. 20.
(B) (B) (D) (A) (D) (B) (C) (C) (A) (A)
21. 22. 23. 24. 25. 26. 27. 28. 29. 30.
(B) (B) (C) (A) (C) (A) (C) (D) (C) (A)
31. 32. 33. 34. 35. 36. 37. 38. 39. 40.
(A) (C) (C) (B) (D) (D) (A) (C) (C) (D)

43 | P a g e
CHAPTER – 2 - CUBIC EQUATION
PRACTICE QUESTIONS

1. The solution of the cubic equation x3–6x2+11x–6 = 0 is given by:


(a) (–1, 1 –2) (b) (1, 2, 3)
(c) (–2, 2, 3) (d) (0, 4, –5)

2. The cubic equation x3 + 2x2 – x – 2 = 0 has 3 roots namely which are:


(a) (1, –1, 2) (b) (–1, 1, –2)
(c) (–1, 2, –2) (d) (1, 2, 2)

3. x, (x – 4), (x + 5) are the factors of the cubic equation. Find the equation.
(a) x3 + 2x2 – x – 2 = 0 (b) x3 + x2 – 20x = 0
3 2
(c) x – 3x – 4x + 12 = 0 (d) x3 – 6x2 + 11x – 6 = 0

4. The equation 3x3 + 5x2 = 3x + 5 has got 3 roots and hence the factors of the left–hand side
of the equation 3x3 + 5x2 – 3x – 5 = 0 are
(a) x – 1, x – 2, x – 5/3 (b) x – 1, x +1, 3x + 5
(c) x + 1, x – 1, 3x - 5 (d) x – 1, x + 1, x – 2

5. It is being given that one of the roots is half the sum of the other two solving x 3–12x2+47x–
60=0 we get the following roots:
(A) 1, 2, 3 (B) 3, 4, 5
(C) 2, 3, 4 (D) –3, –4, –5

6. Solve x3+3x2–x–3=0 given that the roots are in arithmetical progression


(A) –1, 1, 3 (B) 1, 2, 3
(C) –3, –1, 1 (D) –3, –2, –1

7. The roots of the equation X3+7X2-21X-27=0 are


(a) (-3, -9, -1) (b) (3,- 9, -1)
(c) (3, 9, 1) (d) (-3, 9, 1)

8. The roots of X3+X2-X-1 = 0


(a) (-1, -1, 1) (b) (1, 1, -1)
(c) (-1, -1, -1) (d) (1, 1, 1)

9. The satisfying value of X3+X2-20x = 0 are


(a) (1, 4, -5) (b) (2, 4, -5)
(c) (0, -4, 5) (d) (0, 4, -5)

10. If 4x3+8x2-x-2 = 0 then value of (2x+3) is given by


(a) 4, -1, 2 (b) -4, 2, 1
(c) 2, -4, -1 (d) none of these

11. The roots of equation y3+y2-y-1=0 are


(a) 1,1,-1 (b) -1,-1,1
(c) 1,1,1 (d) None

44 | P a g e
12. The roots of the cubic equation x3 - 7x + 6=0 are:
(a) 1, 2 and 3 (b) 1, -2 and 3
(c) 1, 2 and -3 (d) 1, -2 and -3

ANSWER

1. 2. 3. 4. 5. 6. 7. 8. 9. 10.
(B) (B) (B) (B) (B) (C) (B) (A) (D) (A)
11. 12.
(B) (C)

45 | P a g e
CHAPTER – 2 - UNIT-II : MATRICES

PRACTICE QUESTIONS

Chose the most appropriate option (a), (b), (c) or (d)


1. If a matrix has 16 elements; what are the possible orders it can have
(a) 2x8; 8x1; 4x4; 1x16; 16x1 (b) 2x8; 8x2; 4x4; 1x16; 16x1
(c) 2x8; 8x2; 4x1; 1x16; 16x1 (d) 2x4; 8x2; 4x4; 1x16; 16x1

2. Transpose of a rectangular matrix is a


(a) Rectangular matrix (b) Diagonal matrix
(c) Square Matrix (d) Scalar matrix

3. Transpose of row matrix is


(a) zero matrix (b) diagonal matrix
(c) Column Matrix (d) Row matrix

4. Two matrices A and B are multiplied to get AB if


(a) Both are rectangular
(b) both have same order
(c) No. of columns of A is equal to rows of B
(d) No. of rows of A is equal to no. of columns of B

5. If │A│= 0, then A is
(a) Zero Matrix (b) Singular matrix
(c) Non-singular matrix (d) 0

6. If A is symmetric matrix, then At =


(a) A (b) │A│
(c) 0 (d) Diagonal matrix

7. If the order of matrix A is m x p. And the order of B is p x n. then the order of matrix AB is?
(a) mxn (b) nxm
(c) nxp (d) mxp

8. If A and B are matrices, then which from the following is true?


(a) A+B≠B+A (b) (At)t ≠ A
(c) AB ≠ BA (d) all are true

9. What is a, if
2 3
A    is a singular matrix?
4 a
(a) 5 (b) 6
(c) 7 (d) 8

 2i 3i 
10. If A    (i2=-1) then │A│=?
 2i  i 
(a) 2 (b) 8
(c) 4 (d) 5

46 | P a g e
11. If
 a11 a12  b11 b12 b13 
a a  A  b b  then order of matrix A = ?
 21 22   21 22 b23 
a31 a32  b31 b32 b33 
(a) 2x2 (b) 2x3
(c) 3x2 (d) 3x3

 2  3 1 5   2 5
Question No. 12 – 16 let A =   B    C   
 4 5   6  7   3 4 

12. Find A + B.
3 2   3 2
(a) 10  2 (b)  10  2
   
 2 3  3  1
(c)  10  2 (d)  
  10  2

13. Find A – B.
1 2 1  8
(a)   2  2 (b)  2
   12 
1 8   1  8
(c)  2  12 (d)  12
    2

14. 3A –C
 4  14  4  14
(a) 9 (b)
 11   9  11
 
4  14 2  3
(c) 9 11  (d) 4 5 
   

15. AB
 16 31  16 31 
(a)  34  15 (b) 34 15
   
16 31 2  3
(c) 34 5  (d) 4 5 
   

16. BA
 22 22   22 22 
(a)  16  53 (b)  16  53
   
22  11 22  33
(c) 16 53  (d) 16 53 
   

47 | P a g e
a  b   a b
17. b a  ×   b a 
   
a 2  b 2 0   a 2  b 2 0 
(a)   (b)  
 0 a 2  b2   0 a2  b2 
a 2  b 2 0  a 2  b 2 0 
(c)   (d)  
 0 a 2  b2   0 a2  b2 

a 2  b 2 b 2  c 2   2ab  2bc 
18.  2  
a  c
2
a 2  b 2   2ac  2ab
a 2  b 2  2ab b 2  c 2  2bc  (a  b) 2 (b  c) 2 
(a)  2 2 or  2
 a  c  2ac a  b  2ab (a  c) (a  b) 
2 2 2

a 2  b 2  2ab b 2  c 2  2bc  (a  b) 2 (b  c) 2 


(b)  2 2 or  
 a  c  2ac a 2  b 2  2ab (a  c) 2 (a  b) 2 
a 2  b 2  2ab b 2  c 2  2bc  (a  b) 2 (b  c) 2 
(c)  2 2 or  
 a  c  2ac a 2  b 2  2ab (a  c) 2 (a  b) 2 
a 2  b 2  2ab b 2  c 2  2bc  (a  b) 2 (b  c) 2 
(d)  2 2 or  2
 a  c  2ac a  b  2ab (a  c) (a  b) 
2 2 2

 l m  p q 
19. n o    r s 
  
l  p m  q  l  p m  q 
(a) n  r (b)
 s  n  r
 s 
l  p m  q  l  p m  q 
(c) n  r (d)
 s  n  r
 s 

 a b  a b 
20.   b a   b  a 
   
a 2  b 2 0  a 2  b 2 0 
(a)  (b)
2  2
 0 a b 
2
 0 a b 
2

 a 2  b 2 0  a 2  b 2 0 
(c)   (d)  
 0 a  b2 
2
 0 a  b2 
2

48 | P a g e
 1
 
21.  2  (3 4 5 6)
 
 5
3 4 5 6 3 5 4 6
(a)
 6 8 10 12  (b)
6 8 10 12 
  
15 20 25 30 12 16 20 24
3 4 5 6 3 4 5 6
(c)
 6 8 10 12  (d)
6 8 10 12
  
12 16 20 24 24 16 16 12

 x y   1 2 3
22.     
2 3  x y z
 x  2 xy 3x  y 2 3xyz   x  xy 2 x  y 2 3x  yz 
(a)   (b)  
 2  3x 4  3y 6  3z  2  3x 4  3 y 6  3z 
 x  2 xy 2 xy  y 2 12 yz   x  xy 2 x  y 2 3x  yz 
(c)   (d)  
 2  3x 4  3y 6  3z  2  3 x 4  3 y 6  3z 

 1  2 3  1 3 5
   
23.  4 5 6   0 2 4
7 8 9  3 0 5 
  
10  1 12  10 1 28 
22 22 70  22  2 70 
(a) (b)
  
34 37 112 34  5 112
10 1 28  10 1 28 
22  2  70 22  2 70 
(c) (d)
   
34  5 112  34  5  112

 2  3
  3 1 3   
24.     1 0 
 1 0 2   3 1 
 
14  6 14  6
(a)  4  5 (b) 4 5 
   
 14  6 2 12
(c)  4 5  (d) 4 5 
   

49 | P a g e
1 2 3 0 
 3 1 2 2 3 0 1 
25. If A =   ,B=  
 2 0 4 3 0 1 2

Find AB. Does BA exist?


(a) AB exists but BA not exists (b) AB not exists BA Exists
(c) Both AB and BA not exists (d) None of these

0 3
 
 0 2 2 3 1 2
26. If A =  ; B  
 3 2 1 0 2 1
 
3 0 

(a) AB ≠ BA (b) AB = BA
(c) AB exists BA not exists (d) AB not exists BA exists

o i 
27. If A =   ; Where i2 = -1
 i o
Find A2, A3 :
 1 0 3  0  i   1 0  3  0  i
(a) A2   A  (b) A2   A 
0 1  i 0 

0  1 
 i 0 
1 0  3  0 i 1 0 3  0  i 
(c) A2   A  (d) A2   A 
0  1 
  i 0 0 1 
 i 0 
28. Find the elements C23, C32, C31 in the product C = AB.
 2 3 4  1 3 0
   
Where A   1 2 3 , B    1 2 1 
 1 1 2  0 0 2
   
 1 12 11
(a) C23  8, C32  1, C22  7, C31  5 and AB   1 7 8 
 0 5 5 
 1 12 11
(b) C23  8, C32  5, C22  7, C31  0 and AB   1 7 8 
 0 5 5 
  1 12 11
(c) C23  8, C32  1, C22  7, C31  5 and AB    1 7 8 
 2 5 5 
 1 12 11 
(d) C23  8, C32  1, C22  7, C31  5 and AB   1 7 8 
 0  5  5
50 | P a g e
29. Using matrix Cramers method
x=1, y=-1, z=1, =1, Find x, y and z values
(a) x = 1, y = –1 and z = –1 (b) X= –1, y = 1 and z = 1
(c) X = 1, y = –1 and z = 1 (d) X = –1, y = -1 and z = 1

30. Solve the following system of equations by Cramers Rule:


x+y+z = 9
2x+5y+7z = 52
2x+y–z = 0
(a) x=–1, y=–3, z=–5 (b) x=1, y=–3, z=5
(c) x=1, y=3, z=5 (d) x=2, y=3, z=4

31. X+y = -1; y+z = 1; z+x = 0


(a) X = -1; y = 0; z=1 (b) X = 1; y = 0; z = 1
(c) X = 1; y = 0; z = -1 (d) X = -1; y = 0; z = -1

 6 5
32. If A = 3 9 , Find (A’)’
 
(a) A (b) -A
(c) A2 (d) none of these

33. Chose the correct alternative;


x y    2 3
If 2 z 9  18I
 p   1 0
(a) X = 18; z = 9/2 (b) X = 0, Z = –9/2
(c) X = 0; z = 9/2 (d) None of these

 0 3  4
34.
 3 0  5 is a
 
 4 4 8 
(a) Symmetric matrix (b) Null matrix
(c) Skew – symmetric matrix (d) None of these

6 10
35. if A = 3 5 
 
(a) is a singular matrix (b) Non-singular matrix
(c) Identity matrix (d) Symmetric matrix

36. Find the value of a


2a  b a  2b   4  3
5c  d 4c  3d   11 24 
   
(a) 1 (b) 2
(c) 3 (d) 4

51 | P a g e
1 2 3  3  1 3
37. If A = 2 3 1 and B =  1 0 2 the find 2A-B
   
 1 5 3  1 5 3
(a) 5 (b)
 6 1 6
 0 5
 1 5 3  1 5 2
(c) 5 (d)
 6 0 1
 2 2
(e) None of these

8 0   2 - 2
38.
 
If A  4  2 and B  4
 2  , then find the matrix X, such that 2A+3x=5B
  
3 6  - 5 1 
  10 
  2
  2  10 3 
 14 
(a)
 4 14  (b)  4 
   3 
 31  7    31  7 
 3 3 
 2  10   1  9
   4  5
(c)
 4 5  (d)
 
 6 7   6 7 

5 2 3 6 
39. Find y if x+y = 0 9 and x–y = 0 1
   
1  2 2  1
(a) 0 5  (b) 5 0 
   
 2  2
(c) 4 3  (d) None
 

40. Find the value of x and y


x 5  3  4  7 6 
2  
7 y  3 1 2  15 14
(a) 9, 2 (b) 2, 7
(c) 2, 9 (d) 2, 4

52 | P a g e
 6  7 0
4 2  5
41. Find BC if B =   , C   1 2 5
1 0 3   1 0 3
17  2 5 18  24  5
(a) 19  7 9 (b) 9 7 9 
   
1  5 6 
17  24  5  2 8  9
(c)  9 7 9  (d)
 
  5 6 7 

1  2 3 
42.

If A-2I = 2 1  1

 3 1 0 
Find AAT
 14 7 1  14 7 1 
(a)
 7 14  5 (b)
 7 14 5 
  
 1  5 14   1 5 14
 14 7 1 
 7  14  5
(c) (d) None

 1  5 14 

1
43. If A= [aij]2×3 where aij = 2i  3 j then A is equal to
2
 1 7 1 7
 2 2  
2 2 2 2
(a)  1 (b)
5  1 5
 1   1 
 2 2  2 2
2  3  1 7
1  2  2
(c) 1 1 1 (d) 2
 1 5
 2 3 4   1 
 2 2

1 2 3
44. Find the value of A = 4 5 6
7 8 9
(a) 0 (b) –1
(c) 2 (d) –2

53 | P a g e
3 11  1
45. Find the value of A = 5 2 0
10 3 0
(a) 6 (b) –5
(c) 5 (d) 8

1 2
46. Matrix A = 3 4
 
Find A–1
 2 1   2 1 
(a) 3 1 (b) 3 1
 2    2  2 
2
2  3  2  1 
(c) 1 3 (d) 1 3
 2 2   2  2 

1 2 3
47.

Matrix A = 2 3 1

 
3 1 2
A–1 A is equal to
(a) I2 (b) I3
(c) 2I3 (d) 4I3

 2 3
48. A= 1 2
 
A2–4A+I is equal to
(a) –1 (b) 1
(c) 0 (d) –2

1  1 2 
 
49. A = 3 0 2
 
1 0 3 
adj A is equal to :
 0 3 2  0 3 2
 11 1 8 11 1 8
(a) (b)
   
 0  1 3  0  1 3
0  11 0 
3 1  1
(c) (d) None
 
2 8 3 

54 | P a g e
50. Solve the following system of equations
2x–3y=3
2x+3y=9
(a) x=1, y=3 (b) x=3, y=1
(c) x=–1, y=–3 (d) x=–3, y=–1

ANSWER

1. 2. 3. 4. 5. 6. 7. 8. 9. 10.
(B) (A) (C) (C) (B) (A) (A) (C) (B) (B)
11. 12. 13. 14. 15. 16. 17. 18. 19. 20.
(B) (A) (B) (C) (A) (A) (A) (C) (D) (A)
21. 22. 23. 24. 25. 26. 27. 28. 29. 30.
(A) (B) (A) (D) (A) (A) (B) (B) (C) (C)
31. 32. 33. 34. 35. 36. 37. 38. 39. 40.
(A) (A) (C) (D) (A) (A) (C) (B) (A) (C)
41. 42. 43. 44. 45. 46. 47. 48. 49. 50.
(C) (A) (B) (A) (C) (B) (B) (C) (A) (B)

55 | P a g e
CHAPTER – 3 - LINEAR INEQUALITIES
PRACTICE QUESTIONS

1 An employer recruits experienced (x) and fresh workmen (y) for his firm under the condition
that he cannot employ more than 9 people. x and y can be related by the inequality
(a) x+y  9 (b) x+y  9
(c) x + y 9 (d) none of these
2. On the average experienced person does 5 units of work while a fresh one 3 units of work
daily but the employer has to maintain an output of at least 30 units of work per day. This
situation can be expressed as
(a) 5x + 3y  30 (b) 5x + 3y >30
(c) 5x+3y  30 (d) none of these
3. The graph to express the inequality x + y  9 is
(a) (b)

(c) (d) none of these

4. The graph to express the inequality 5x + 3y  30 is


(a) (b)

(c) (d) None of these

56 | P a g e
1
5. The graph to express the inequality y   is indicated by
2

(a) (b)

(c) (d)

6.

L1 : 5x+3y = 30, L2: x+y =9, L3 : y = x/3, L4 : y = x/2


The common region (shaded part) shown in the diagram refers to
(a) 5x +3y  30 (b) 5x+3y  30
x+y  9 x+y  9
y  x/5 y  x/3
y  x/2 y  x/2
x  0, y  0
(c) 5x+3y  30 (d) 5x+3y > 30
x+y  9 x+y < 9
y  x/3 y 9
y  x/2 y  x/2
x  0, y  0 x  0, y  0

57 | P a g e
7. A dietitian wishes to mix together two kinds of food so that the vitamin content of the mixture
is at least 9 units of vitamin A, 7 units of vitamin B, 10 units of vitamin C and 12 units of
vitamin D. The vitamin content per Kg. of each food is shown below:

A B C D
Food I : 2 1 1 2
Food II: 1 1 2 3
Assuming x units of food I is to be mixed with y units of food II the situation can be
expressed as
(a) 2x +y  9 (b) 2x+y  30
x+y  7 x+y  7
x+2y  10 x+2y  10
2x+3y  12 x+3y  12
X > 0, y > 0
(c) 2x+y  9 (d) 2x+y  9
x+y  7 x+y  7
x+y  10 x+2y  10
x+3y  12 2x+3y  12
x  0, y  0

8. Graphs of the inequations are drawn below:

L1 : 2x +y = 9, L2 : x + y = 7, L3 : x+2y= 10, L4 : x + 3y = 12
The common region (shaded part) indicated on the diagram is expressed by the set of
inequalities
2x  y  9
2x + y  9
xy7
x+y 7
(a) (b) x  2y  10
x + 2y  10
x  3y  12
x + 3y  12
x  0, y  0
2x  y  9
xy7
(c) x  2y  10 (d) None of these
x  3y  12
x  0, y  0

58 | P a g e
9. The common region satisfied by the inequalities L1: 3x + y  6, L2:x+y  4 L3:x+3y  6, and L4:
x+y  6 is indicated by

(a) (b)

(c) (d) None of these

10. The region indicated by the shading in the graph is expressed by inequalities

x +x 2
1 2
x1  x 2  2
(a) 2x1+ 2x2 8 (b)
x1  x 2  4
X  0, x  0,
1 2
x +x  2 x1  x 2  2
1 2
(c) (d)
2x + 2x  8 2x1  2x 2  8
1 2

11. The inequalities x1  0, x2  0, are represented by one of the graphs shows below:
(a) (b)

(c) (d)

59 | P a g e
12. The region is expressed as
(a) x1-x2  1
(b) x1+x2  1
(c) x1+x2  1
(d) None of these

13. The inequality –x1+2x2 0 is indicated on the graph as

(a) (d)

(c) (d) None of these

14.

The common region indicated on the graph is expressed by the set of five inequalities
L1 : x  0
L1 : x1  0 1
L2 : x 2  0 L2 : x  0
2
(a) L3 : x1  x 2  1 (b) L3 : x + x  1
1 2
L4 : x1  x 2  1 L4 : x - x  1
1 2
L5 : -x1  2 x 2  0
L5 : -x + 2x  0
1 2
L1 : x  0
1
L2 : x  0
2
(c) L3 : x + x  1 (d) None of these
1 2
L4 : x - x  1
1 2
L5 : -x + 2x  0
1 2

60 | P a g e
15. A firm makes two types of products : Type A and Type B. The profit on product A is Rs. 20
each and that on product B is Rs. 30 each. Both types are processed on three machines M1,
M2 and M3. The time required in hours by each product and total time available in hours per
week on each machine are as follows:

Machine Product A Product B Available Time


M1 3 3 36
M2 5 2 50
M3 2 6 60

The constraints can be formulated taking x 1= number of units A & x2 = number of unit of B as

x1  x 2  12 3x1  3x 2  36
5x1  2x 2  50 5x1  2x 2  50
(a) (b)
2x1  6x 2  60 2x1  6x 2  60
x1  0 x 2  0 x1  0 x 2  0

3x + 3x  36
1 2
5x + 2x  50
1 2
(c) (d) None of these
2x + 6x  60
1 2
x  0, x  0
1 2

16. The set of inequalities L1: x1+x2  12, L2: 5x1+2x2  50, L3:x1+3x2  30, x1  0, and x2  0 is
represented by

(a) (b)

(c) (d) None of these

61 | P a g e
17. The common region satisfying the set of inequalities x  0, y  0, L1:x+y  5, L2:x+2y 8 and
L3: 4x+3y  12 is indicated by
(a) (b)

(c) (d) None of these

18. Mr. A plans to invest upto Rs.50,000 in two stocks X and Y. Stock X(x) is priced at Rs.175 and
Stock Y(y) at Rs.95 per share. This can be shown by :
(a) 175x+95y  50,000 (b) 175x+95y  50,000
(c) 175x+95y = 50,000 (d) None of these.

19. A firm plans purchase hens (x) for its canteen. There cannot be more than 20 hens. This can
be shown by “
(a) x  20 (b) x = 20
(c) x  20 (d) None of these.

20. A dealer has only Rs.5760 to invest in fans (x) and sewing machines (y). The cost per unit of
fans and sewing machine is Rs.360 and Rs.240 respectively. This can be shown by “
(a) 360x + 240y  5760 (b) 360x + 240y  5760
(c) 360x + 240y = 5760 (d) None of these.

21. In a class of boys (x) and girls (y) the maximum seating capacity is 360. This can be shown
by “
(a) x+y  360 (b) x+y  360
(c) x+y 360
 (d) None of these.

22. An employer recruits experienced and fresh workmen for his firm under the condition that he
cannot employ more than 12 people. It can be related by the inequality :
(a) x+y=12 (b) x+y  12
(c) x+y  12 (d) None of these

23. The inequalities x  0, y  0 indicates


(a) First quadrant (b) Second quadrant
(c) Third quadrant (d) Fourth quadrant

62 | P a g e
24. If relationship between two positive numbers is a > b then :
1 1 1 1
(a)  (b) 
a b a b
1 1 1
(c)  (d) b
a b a

PAST EXAMINATION QUESTIONS

25. Graphs of Inequations are drawn below:

Y
10
9 L4
8
7
6
5 L3
4
3
2
2
0 1 2 3 4 5 6 7 8 9 10 X
L1 L2

L1:5x+3y=30 L2 : x+y=9
x x
L3:y= L4 : y=
3 2

The common region (shaded part) shown in the diagram refers to the inequalities

(a )5 x  3 y  30 (b)5x  3y  30
x y 9 xy9
1
y x y  x/3
5
y  x/2 y x/2
x  0, y  0 x  0, y  0.
(c)5 x  3 y  30
(d )5 x  3 y  30
x y9
x y 9
y  x/3 y9
y  x/2 y  x/2
x  0, y  0.
x  0, y  0.

63 | P a g e
1 7
26. If x  , then :
4 4
3
(a ) x  or x.  2
2
3
(b) x  2 or x 
2
3
( c)  2  x 
2
(d) None of these

3x  4 5
27. If  , the solution set is :
4 12
 19 29 
(a )  x :  x  
 18 18 
 7 17 
( b) x :  x  
 9 9

  29  19 
( c)  x : x 
 18 18 
(d) None of these

28. A car manufacturing company manufactures cars of two types A and B. Model A requires 150
man-hours for assembling, 50 man-hours for painting and 10 man-hours for checking and
testing. Model B requires 60 man-hours for assembling, 40 man-hours for painting and 20
man-hours for checking and testing. There are available 30 thousand non-hours for
assembling, 13 thousand man-hours for painting and 5 thousand man hours for checking and
testing. Express the above situation using linear inequalities. Let the company manufacture x
units of type A model of car and y units of type B model of car. Then, the inequalities are:

(a) 5 x  2 y  1000;5 x  4 y  1300,


x  2 y  500; x  0, y  0,
(b) 5 x  2 y  1000,5 x  4 y  1300,
x  2 y  500 : x  0, y  0.
(c ) 5 x  2 y  1000,5 x  4 y  1300,
x  2 y  500; x  0, y  0.
(d ) 5 x  2 y  1000,5 x  4 y  1300
x  2 y  500; x  0, y  0.

64 | P a g e
29. The rules and regulations demand that the employer should employ not more than 5
experienced to I fresh one and this fact is represented by:
(Taking experienced person as x and fresh person as y)
x
(a) y
5
(b) 5y  x
(c ) y  5x
(d) None

30. The shaded region represents:

(a) 3 x  2 y  24, x  2 y  16, x  y  10 x, x  0, y  0,


(b) 3 x  2 y  24, x  2 y  16, x  y  10, x  0, y  0
(c ) 3 x  2 y  24, x  2 y  16, x  y  10, x  0, y  0
(d) None

31. The shaded region represents:

(a) 3 x  5 y  15,5 x  2 y  10, x, y  0


(b) 3 x  5 y  15,5 x  2 y  10, x, y  0
(c ) 3 x  5 y  15,5 x  2 y  10, x, y  0
(d) None of these

65 | P a g e
32. The shaded region represents:

(a) x + y  5, x  2, y  1
(b) x + y  5, x  2, y  1
(c) x + y  5, x  2, y  1
(d) None of these

33. The shaded region represents:

(a) x + y > 6,2x – y > 0


(b) x + y < 6,2x – y > 0
(c) x + y > 6,2x – y < 0
(d) None of these

34. If a>0 and b<0 it follows that:


1 1
(a) 
a b
1 1
(b) 
a b
1 1
(c ) 
a b
(d) None of these

35. The Linear relationship between two variables in an inequality:


(a) ax+by = c (b) ax + by < c
(c) axy+by < c (d) ax +bxy < c
66 | P a g e
(5  2 x) x
36. The solution of the inequality  -5 is :
3 6
(a) x8 (b) x 8
(c) x=8 (d) None of these

37. Solution Space of the inequalities 2x + y < 10 and x – y < 5:


(i) includes the origin.
(ii) includes the point (4,3)
(a) Only (i) (b) Only (ii)
(b) Both (i) and (ii) (d) None of the above

38. The range of real values of ‘x’ satisfying the inequality 3x–2 > 7 and 4x-13 > 15 is
(a) x>3 (b) x >7
(c) x<7 (d) x<3

39. If X  2  X  3  7 then x =
(a) 6 (b) -1
(c) 6 & -1 (d) None

40. The union forbids the employer to employ less than 2 experienced persons (x) to each fresh
person (y). This situation can be expressed as
(a) x < y/2 (b) y < x/2
(c) y > x/2 (d) none

41. The solution of the inequality 8x+6<12x+14 is


(a) (-2,2) (b) (0, -2)
(c) (2, ) (d) ( 2, )

42. The graph of linear inequalities 7x+9y  63; x+y  1; 0  x  6 and 0  y  6 has been given
below

Common region of the inequalities is :


(a) BCDB and DEFD (b) Unbounded
(c) HFGH (d) ABDFHKA

67 | P a g e
43. Which of the following graph represents the in equality x+y  6 is

y
y
(a) (b)
O
O

X
O X
X
O X

y
(c) (d) None
O

X
O X

44. The common shaded region in the graph represents the linear inequalities as:

2x-y=2

X
0 x+y=6

(a) x+y  6
2x-y-2  0
x, y  0

(b) x+y  6
2x-y-2  0
x, y  0
(c) x+y  6
2x-y-2  0
x, y  0

(d) x+y  6
2x-y-2  0
x, y  0

68 | P a g e
ANSWER

1. 2. 3. 4. 5. 6. 7. 8. 9. 10.
(B) (C) (A) (A) (D) (B) (D) (C) (A) (A)
11. 12. 13. 14. 15. 16. 17. 18. 19. 20.
(B) (C) (A) (B) (C) (B) (A) (A) (A) (B)
21. 22. 23. 24. 25. 26. 27. 28. 29. 30.
(A) (B) (A) (B) (B) (B) (B) (C) (A) (C)
31. 32. 33. 34. 35. 36. 37. 38. 39. 40.
(B) (B) (A) (A) (B) (A) (A) (B) (C) (B)
41. 42. 43. 44.

(D) (D) (B) (C)

69 | P a g e
CHAPTER – 4 - TIME VALUE OF MONEY
(SIMPLE INTEREST)
PRACTICE QUESTIONS

1. S.I on Rs. 3,500 for 3 years at 12% per annum is


(a) Rs. 1,200 (b) Rs. 1,260
(c) Rs. 2,260 (d) none of these

2. P = 5,000, R = 15, T = 4½ years, SI will be


(a) Rs. 3,375 (b) Rs. 3,300
(c) Rs. 3,735 (d) none of these

3. If P = 5,000, T = 1 year, SI = Rs. 300, R will be


(a) 5% (b) 4%
(c) 6% (d) none of these

4. If P = Rs. 4,500, A = Rs. 7,200, than Simple interest will be


(a) Rs. 2,000 (b) Rs. 3,000
(c) Rs. 2,500 (d) Rs. 2,700

5. P = Rs.12,000, A = Rs.16,500, T = 2 ½ years. Simple rate of interest will be


(a) 15% (b) 12%
(c) 10% (d) none of these

6. P = Rs. 10,000, I = Rs. 2,500, R = 12 ½% SI. The number of years T will be


(a) 1 ½ years (b) 2 years
(c) 3 years (d) none of these

1
7. P = Rs. 8,500, A = Rs. 10,200, R = 12 2 % SI, t will be.
(a) 1 yr. 7 mth. (b) 2 yrs.
(c) 1 ½ yr. (d) none of these

8. The sum required to earn a monthly interest of Rs 1,200 at 18% per annum SI is
(a) Rs. 50,000 (b) Rs. 60,000
(c) Rs. 80,000 (d) none of these

9. A sum of money amount to Rs. 6,200 in 2 years and Rs. 7,400 in 3 years. The principal and
rate of interest are
(a) Rs. 3,800, 31.57% (b) Rs. 3,000, 20%
(c) Rs. 3,500, 15% (d) none of these

10. A sum of money doubles itself in 10 years. The number of years it would triple itself is
(a) 25 years. (b) 15 years.
(c) 20 years (d) none of these

70 | P a g e
11. Sania deposited Rs. 50,000 in a bank for two years with the interest rate of 5.5% p.a. How
much interest would she earn?
(a) 5600 (b) 5500
(c) 5100 (d) 5000

12. In question 11 what will be the final value of investment?


(a) 56500 (b) 55500
(c) 57500 (d) 54500

13. Sachin deposited Rs. 1,00,000 in his bank for 2 years at simple interest rate of 6%. How
much would be the final value of deposit?
(a) 102000 (b) 112000
(c) 120000 (d) 100200

14. Find the rate of interest if the amount owed after 6 months is Rs. 1050, borrowed amount
being Rs. 1000.
(a) 11% (b) 10%
(c) 12% (d) None of these

15. Rahul invested Rs. 70,000 in a bank at the rate of 6.5% p.a. simple interest rate. He received
Rs. 85925 after the end of term. Find out the period for which sum was invested by Rahul.
(a) 5 years (b) 3 years
(c) 2 years (d) 3.5 years

16. Kapil deposited some amount in a bank for 7 1


2 years at the rate of 6% p.a. simple interest.
Kapil received Rs. 101500 at the end of the term. Compute initial deposit of Kapil.
(a) 80,000 (b) 85,000
(c) 70,000 (d) 60,000
17. A sum of Rs. 46875 was lent out at simple interest and at the end of 1 year 8 months the
total amount was Rs. 50,000. Find the rate of interest percent per annum.
(a) 3% (b) 2%
(c) 4% (d) 1%

18. What sum of money will produce Rs. 28600 as an interest in 3 years and 3 months at 2.5%
p.a. simple interest?
(a) 350000 (b) 353000
(c) 352000 (d) 354000

19. In what time will Rs. 85000 amount to Rs. 157675 at 4.5% p.a.?
(a) 18 years (b) 17 years
(c) 19 years (d) 20 years

20. A certain sum of money was invested at simple rate of interest for three years. If the sum has
been invested at a rate that was seven percent higher, the interest amount would have been
Rs. 882 more. The amount of sum invested is:
(a) Rs. 12,600 (b) Rs. 6,800
(c) Rs. 4,200 (d) Rs. 2,800

71 | P a g e
21. The rate of simple interest on a sum of money is 6% pa. for first 3 years, 8% pa. for the next
five years and 10% pa. for the period beyond 8 years. If the simple interest accrued by the
sum for a period for 10 years is Rs. 1,560. The sum is:
(a) Rs. 1,500 (b) Rs. 2,000
(c) Rs. 3,000 (d) Rs. 5,000

22. A sum of money doubles itself in 10 years. The number of years it would be triple itself is
(a) 25 years (b) 15 years
(c) 20 years (d) None

23. How much investment required to yield an annual income of Rs.420 at the rate of 7% p.a by
Simple interest.
(a) 6000 (b) 6420
(c) 5580 (d) 5000

PAST EXAMINATION QUESTIONS

24. The S.I on Rs. 2000 for 5 months at the rate of 16% per annum is_____
(a) 133.33 (b) 133.26
(c) 134.00 (d) 132.09

25. Rs. 8000 becomes Rs. 10000 in two years at simple interest. The amount that will become
Rs. 6,875 in 3 years at the same rate of interest is :
(a) Rs. 4,850 (b) Rs. 5,000
(c) Rs. 5,500 (d) Rs. 5,275

26. A certain sum of money amounts to Rs. 6,300 in two years and Rs. 7,875 in three years nine
months at simple interest find the rate of interest per annum :
(a) 20% (b) 18%
(c) 15% (d) l0%

27. If Rs. 1,000 be invested at interest rate of 5% and the interest be added to the principal
every 10 years then the number of years in which it will amount to Rs. 2,000 is:
2 1
(a ) 16 years (b)6 years
3 4
2
(c) 16 years (d )6 years
3

28. In how many years will a sum of money become four times at 12% p.a. simple interest?
(a) 18 years (b) 21 years
(c) 25 years (d) 28 years

29. A person borrows Rs. 5,000 for 2 years at 4% pa. simple interest. He immediately lends to
1
another person at 6 % p.a. for 2 years. Find his gain in this transaction per year.
4
(a) Rs. 112.50 (b) Rs. 125
(c) Rs. 225 (d) Rs. 167.50

72 | P a g e
1 1
30. Two equal sums of money were lent at simple interest at 11 p.a. for 3 years and 4 years
2 2
respectively. If the difference in interests for two periods was Rs. 412.50 then each sum is :
(a) Rs. 3,250 (b) Rs. 3,500
(c) Rs. 3,750 (d) Rs. 4250

31. In how much time would the simple interest on a certain sum be 0.125 times the principal at
10% per annum?
1 3
(a )1 years (b)1 years
4 4
1 3
(c) 2 years (d) 2 years
4 4

32. Find the numbers of years in which a sum doubles itself at the simple interest rate of 8% per
annum.
1 1
(a) 11 (b) 12
2 2
1 1
(c) 9 (d) 13
2 2

33. What is the rate of simple interest if a sum of money amounts to Rs. 2,784 in 4 years and Rs.
2,688 in 3 years?
(a) 1%p.a. (b) 4% p.a.
(c) 5% p.a. (d) 8%p.a.

34. If a simple interest on a sum of money at 6% for 7 years is equal to twice of simple interest
on another sum for 9 years at 5% p.a.. The ratio of sums will be :
(a) 2 : 15 (b) 7 : 15
(c) 15 : 7 (d) 1:7

35. By mistake a clerk, calculated the simple interest on principal for 5 months at 6.5 % p.a.
instead of 6 months at 5.5 % p.a. If the error in calculation was Rs. 25.40. The original sum
of principal was .
(a) Rs. 60,690 (b) Rs. 60,960
(c) Rs. 90,660 (d) Rs. 90,690

36. If the S.I on Rs.1,400 at certain rate of interest for 3 yrs is less than that on Rs.1,800 for the
same period and same rate of interest is Rs.80, then the rate of interest is:
(a) 6.67% (b) 7.67%
(c) 5.67% (d) None

37. The S.I on a sum of money is 4/9 of the principal and the no. of years is equal to the rate of
interest per annum. Find the rate of interest per annum
(a) 5% (b) 6%
(c) 20/3% (d) 22/7%

73 | P a g e
38. The principle of 90,500 with rate of interest 7.5% will be calculated at simple interest when it
is calculated the interest wrongly taken as 5.7%. Then the difference will be 9, 774. At what
time the amount will be cleared?
(a) 7yrs (b) 4.5Yrs
(c) 6yrs (d) 8yrs

39. How much interest will be earned on Rs. 2000 at 6% simple interest for 2 years?
(a) 250 (b) 260
(c) 240 (d) 230

40. In what time will a sum of money double itself at 6.25% p.a. simple interest?
(a) 5 years (b) 8 years
(c) 12 years (d) 16 years

41. What principal will amount to Rs. 370 in 6 years at 8% p.a. at simple interest?
(a) Rs. 210 (b) Rs. 250
(c) Rs. 250 (d) Rs. 350

42. If a sum triples in 15 years at simple rate of interest, the rate of interest per annum will be:
(a) 13.0% (b) 13.3%
(c) 13.5% (d) 18.0%

43. A sum of money doubles itself in 8 years at simple interest. The number of years it would
triple itself is _______.
(a) 20 years (b) 12 years
(c) 16 years (d) None of these

44. A sum of Rs.44,000 is divided into three parts such that the corresponding interest earned
after 2 years, 3 years and 6 years may be equal. If the rates of simple interest are 6% p.a.,
8% p.a. and 6% p.a. respectively, then the smallest part of the sum will be
(a) Rs.4,000 (b) Rs.8,000
(c) Rs.10,000 (d) Rs.12,000

ANSWER
1. 2. 3. 4. 5. 6. 7. 8. 9. 10.
(B) (A) (C) (D) (A) (B) (A) (C) (A) (C)
11. 12. 13. 14. 15. 16. 17. 18. 19. 20.
(B) (B) (B) (B) (D) (C) (C) (C) (C) (C)
21. 22. 23. 24. 25. 26. 27. 28. 29. 30.
(B) (C) (A) (A) (B) (A) (A) (C) (A) (C)
31. 32. 33. 34. 35. 36. 37. 38. 39. 40.
(A) (B) (B) (C) (B) (A) (C) (C) (C) (D)
41. 42. 43. 44.
(B) (B) (C) (B)

74 | P a g e
CHAPTER – 4 - COMPOUND INTEREST
PRACTICE QUESTIONS

1. If P = Rs. 1,000, R = 5% p.a, n = 4; What is Amount and C.I. ?


(a) Rs. 1,215.50, Rs. 215.50 (b) Rs. 1,125, Rs. 125
(c) Rs. 2,115, Rs. 115 (d) none of these

2. Rs. 100 will become after 20 years at 5% p.a compound interest amount of
(a) Rs. 250 (b) Rs. 205
(c) Rs. 265.50 (d) none of these

3. A machine is depreciated at the rate of 20% on reducing balance. The original cost of the
machine was Rs. 1,00,000 and its ultimate scrap value was Rs. 30,000. The effective life of
the machine is
(a) 4.5 years (appx.) (b) 5.4 years (appx.)
(c) 5 years (appx.) (d) none of these

4. If A = Rs. 1,000, n = 2 years, R = 6% p.a compound interest payable half-yearly, then


principal ( P ) is
(a) Rs. 888.80 (b) Rs. 885
(c) Rs. 800 (d) none of these

5. The population of a town increases every year by 2% of the population at the beginning of
that year. The number of years by which the total increase of population be 40% is
(a) 7 years (b) 10 years
(c) 17 years (app) (d) none of these

6. The difference between C.I and S.I on a certain sum of money invested for 3 years at 6% p.a
is Rs. 110.16. The sum is
(a) Rs. 3,000 (b) Rs. 3,700
(c) Rs. 12,000 (d) Rs. 10,000

7. The useful life of a machine is estimated to be 10 years and cost Rs. 10,000. Rate of
depreciation is 10% p.a. The scrap value at the end of its life is
(a) Rs. 3,486.78 (b) Rs. 4,383
(c) Rs. 3,400 (d) none of these

1
8. The C.I on Rs. 16000 for 1 years at 10% p.a payable half -yearly is
2
(a) Rs. 2,222 (b) Rs. 2,522
(c) Rs. 2,500 (d) none of these

9. The C.I on Rs. 40000 at 10% p.a for 1 year when the interest is payable quarterly is
(a) Rs. 4,000 (b) Rs. 4,100
(c) Rs. 4,152.51 (d) none of these

10. The difference between the S.I and the C.I on Rs. 2,400 for 2 years at 5% p.a is
(a) Rs. 5 (b) Rs. 10
(c) Rs. 16 (d) Rs. 6

75 | P a g e
11. The C.I on Rs. 4,000 for 6 months at 12% p.a payable quarterly is
(a) Rs. 243.60 (b) Rs. 240
(c) Rs. 243 (d) none of these

12. Determine the compound interest on Rs. 1000 at 6% compounded semi-annually for 6 years.
(a) Rs. 425.76 (b) Rs. 525.46
(c) Rs. 600 (d) None

13. Compute the compound interest on Rs. 4000 fro 1 1 2 years at 10% per annum compounded
half-yearly.
(a) Rs. 740.40 (b) Rs. 630.50
(c) Rs. 580.70 (d) None

14. On what sum will the compound interest at 5% per annum for two years compounded
annually be Rs. 1640?
(a) Rs. 18000 (b) Rs. 20000
(c) Rs. 16000 (d) None

15. What annual rate of interest compounded annually doubles an investment in 7 years? Given
that 21/7=1.104090
(a) 10% (b) 9.41%
(c) 12% (d) 10.41%

16. In what time will Rs. 8000 amount to Rs. 8820 at 10% per annum interest compounded half-
yearly?
(a) 1 year (b) 2 years
(c) 3 years (d) 4 years
17. Find the rate percent per annum if Rs. 2,00,000 amount to Rs. 2,31,525 in 1 1 2 year interest
1
being compounded half-yearly. Given that (1.157625) 3 =1.05
(a) 9% (b) 11%
(c) 10% (d) 12%

18. A certain sum invested at 4% per annum compounded semi-annually amounts to Rs. 78030
at the end of one year. Find the sum.
(a) Rs. 80,000 (b) Rs. 90,000
(c) Rs. 85,000 (d) Rs. 75,000

19. Rs. 16000 invested at 10% p.a. compounded semi-annually amounts to Rs. 18522. Find the
time period of investment.
(a) 1 1 2 years (b) 3 years
(c) 2 years (d) 1 years

PAST EXAMINATION QUESTIONS

20. In what time will Rs. 3,90,625 amount to Rs. 4,56,976 at 8% per annum. When the interest
is compounded semi-annually?
(a) 2 years (b) 4 years
(c) 5 years (d) 7 years

76 | P a g e
21. How long will Rs. 12,000 take to amount to Rs. 14,000 at 5% converted quarterly?
[(1.0125)12.4 = 1.1666]
(a) 3 years (b) 3.1 years
(c) 13.5 years (d) 12.4 years.

22. Anshul’s father wishes to have Rs. 75,000 in a bank account when his first college expenses
begin. How much amount his father should deposit now at 6.5% compounded annually if
Anshul is to start college in 8 years hence from now?
(a) Rs. 45,317 (b) Rs. 46,360
(c) Rs. 55,360 (d) Rs. 48360

23. The compound interest for a certain sum 5% p.a. for first year is Rs. 25. The S-I for the same
money @ 5% p.a. for 2 years will be.
(a) Rs. 40 (b) Rs. 50
(c) Rs. 60 (d) Rs. 70

24. A sum amount to Rs. 1,331 at a principal of Rs. 1,000 at 10% compounded annually. Find the
time.
(a) 3.31 years (b) 4 years
(c) 3 years (d) 2 years

25. The cost of machinery is Rs. 1,25,000/- if its useful life is estimated to be 20 years and the
rate of depreciation of its cost is 10% p.a., then the scrap value of the machinery is
(a) 15,197 (b) 15,400
(c) 15,300 (d) 15,250

26. The difference between compound interest and simple interest on a certain sum for 2 years at
10% pa. is Rs. 10. Find the sum :
(a) Rs. 1,010 (b) Rs. 1,095
(c) Rs. 1,000 (d) Rs. 990

27. A sum of money invested of compound interest doubles itself in four years. It becomes 32
times of itself at the same rate of compound interest in
(a) 12 years (b) 16 years
(c) 20 years (d) 24 years

28. The difference between the simple and compound interest on a certain sum for 3 year at 5%
pa. is Rs. 228.75. The compound interest on the sum for 2 years at 5% pa. is :
(a) Rs. 3175.0 (b) Rs. 3,075
(c) Rs. 3275.0 (d) Rs.2975.0

29. A person deposited Rs. 5,000 in a bank. The deposit was left to accumulate at 6%
compounded quarterly for the first five years and at 8% compounded semi-annually for the
next eight years. The compound amount at the end of 13 years is :
(a) Rs. 12621.50 (b) Rs. 12613.10
(c) Rs. 13613.10 (d) None.

30. If the difference between simple interest and compound interest is Rs. 11 at the rate of 10%
for two years then find the sum.
(a) Rs. 1,200 (b) Rs. 1,100
(c) Rs. 1,000 (d) None of these

77 | P a g e
31. The time by which a sum of money is 8 times of itself if it doubles itself in 15 years by
compound interest.
(a) 42 years (b) 43 years
(c) 45 years (d) 46 years

32. If the simple interest on a sum of money at 12% p.a. for two years is Rs. 3600. The
compound interest on the same sum for two years at the same rate is:
(a) Rs. 3,816 (b) Rs. 3,806
(c) Rs. 3,861 (d) Rs. 3,860

33. Mr. X Invests 'P' amount at simple interest rate 10% and Mr. Y invests 'Q' amount at rate 5%
compounded annually. At the end of two years both get the same amount of interest, then
the relation between two amounts P and Q is given by:
41Q 41Q
(a) P= (b) P=
80 40
41Q 41Q
(c) P= (d) P=
100 200
34. If the difference of S.I and C.I is Rs. 72 at 12 % for 2 years. Calculate the amount.
(a) 8,000 (b) 6,000
(c) 5,000 (d) 7,750

35. At what % rate of compound interest (C.I.) will a sum of money become 16 times in four
years, if interest is being calculated compounding annually:
(a) r=100% (b) r=10%
(c) r=200% (d) r=20%

36. The difference between compound and simple interest on a certain sum of money for 2 years
at 4% p.a. is Rs. 1. The sum (in Rs) is:
(a) 625 (b) 630
(c) 640 (d) 635

37. A sum of money compounded annually becomes Rs. 1,140 in two years and Rs. 1,710 in
three years. Find the rate of interest per annum.
(a) 30% (b) 40%
(c) 50% (d) 60%

38. On what sum difference between compound interest and simple interest for two years at 7%
p.a., interest is Rs. 29.4.
(a) Rs. 5000 (b) Rs. 5500
(c) Rs. 6000 (d) Rs. 6500

39. The partners A and B together lent Rs. 3,903 at 4% per annum interest compounded
annually. After a span of 7 years, A gets the same amount as B gets after 9 years. The share
of A in the sum of Rs. 3,903 would have been:
(a) Rs. 1,875 (b) Rs. 2,280
(c) Rs. 2,028 (d) Rs. 2,820

40. The simple interest for a certain sum for 2 years at 10% per annum is Rs.90. The
corresponding compound interest in (in Rs.):
(a) 99 (b) 95.60
(c) 94.50 (d) 108

78 | P a g e
41. The annual birth and death rate per 1000 are 39.4 and 19.4 respectively. The number of
years in which the population will be double assuming there is no immigration or emigration
is:
(a) 35 years (b) 30 years
(c) 25 years (d) None of these.

42. The effective rate of interest corresponding to a nominal rate 3% p.a payable half yearly is
(a) 3.2% p.a (b) 3.25% p.a
(c) 3.0225% p.a (d) none of these

43. The effective rate of interest corresponding a nominal rate of 7% p.a convertible quarterly is
(a) 7% (b) 7.5%
(c) 5% (d) 7.18%

44. The effective rate equivalent to nominal rate of 6% compounded monthly is:
(a) 6.05 (b) 6.16
(c) 6.26 (d) 6.07

45. The effective annual rate of interest corresponding to nominal rate 6% p.a. payable half
yearly is :
(a) 6.06% (b) 6.07%
(c) 6.08% (d) 6.09%

46. Find the effective rate of interest of 9.9% p.a. calculated monthly :
(a) 9.9% (b) 11.36%
(c) 9.36% (d) 10.36%

ANSWER

1. 2. 3. 4. 5. 6. 7. 8. 9. 10.
(A) (C) (B) (A) (C) (D) (A) (B) (C) (D)
11. 12. 13. 14. 15. 16. 17. 18. 19. 20.
(A) (A) (B) (C) (D) (A) (C) (D) (A) (A)
21. 22. 23. 24. 25. 26. 27. 28. 29. 30.
(B) (A) (B) (C) (A) (C) (C) (B) (B) (B)
31. 32. 33. 34. 35. 36. 37. 38. 39. 40.
(C) (A) (A) (C) (A) (A) (C) (C) (C) (C)
41. 42. 43. 44. 45. 46.
(B) (C) (D) (B) (D) (D)

79 | P a g e
CHAPTER – 4- ANNUITY
PRACTICE QUESTIONS

1. Find the future value of an annuity of Rs.500 made annually for 7 years at interest rate of
14% compounded annually. Given that (1.14)7 = 2.5023.
(a) Rs. 5365.35 (b) Rs. 6365.35
(c) Rs. 7839.43 (d) Rs. 8369.21

2. Rs. 200 is invested at the end of each month in an account paying interest 6% per year
compounded monthly. What is the future value of this annuity after 10th payment? Given that
(1.005)10 = 1.0511
(a) Rs. 3044 (b) Rs. 2044
(c) Rs. 4024 (d) Rs. 2135

3. Z invests Rs. 10,000 every year starting from today for next 10 years. Suppose interest rate
is 8% per annum compounded annually. Calculate future value of the annuity.
Given that (1 + 0.08)10 = 2.15892500.
(a) Rs. 156454.875 (b) Rs. 156544.675
(c) Rs. 154654.375 (d) None of these

4. S borrows Rs. 5,00,000 to buy a house. If he pays equal instalments for 20 years and 10%
interest on outstanding balance what will be the equal annual instalment?
(a) Rs. 48792.72 (b) Rs. 58729.84
(c) Rs. 57829.61 (d) None of these

5. Rs. 5,000 is paid every year for ten years to pay off a loan. What is the loan amount if
interest rate be 14% per annum compounded annually?
(a) Rs. 26080.55 (b) Rs. 25805.25
(c) Rs. 26808.45 (d) None of these

6. Y bought a TV costing Rs. 13,000 by making a down payment of Rs. 3000 and agreeing to
make equal annual payment for four years. How much would be each payment if the interest
on unpaid amount be 14% compounded annually?
(a) Rs. 3423.45 (b) Rs. 4332.05
(c) Rs. 3432.05 (d) None of these

7. The present value of an annuity of Rs. 3000 for 15 years at 4.5% p.a CI is
(a) Rs. 23,809.41 (b) Rs. 32,218.63
(c) Rs. 32,908.41 (d) none of these

8. The amount of an annuity certain of Rs. 150 for 12 years at 3.5% p.a C.I is
(a) Rs. 2,190.28 (b) Rs. 1,290.28
(c) Rs. 2,180.28 (d) none of these

9. A loan of Rs. 10,000 is to be paid back in 30 equal installments. The amount of each
installment to cover the principal and at 4% p.a CI is
(a) Rs. 587.87 (b) Rs. 587
(c) Rs. 578.87 (d) none of these

80 | P a g e
10. If the amount of an annuity after 25 years at 5% p.a. CI. Is Rs. 50,000 the annuity will be
(a) Rs. 1,406.90 (b) Rs. 1,146.90
(c) Rs. 1,047.62 (d) None of these

11. Given annuity of Rs. 100 amounts to Rs. 3137.12 at 4.5% p.a C. I. The number of years will
be
(a) 25 years (appx.) (b) 20 years (appx.)
(c) 22 years (d) none of these

12. A company borrows Rs. 10,000 on condition to repay it with compound interest at 5% p.a by
annual installments of Rs. 1000 each. The number of years by which the debt will be clear is
(a) 14.2 years (b) 10 years
(c) 12 years (d) 15 years

13. A person invests Rs. 500 at the end of each year with a bank which pays interest at 10% p.a
C.I. annually. The amount standing to his credit one year after he has made his yearly
investment for the 12th time is.
(a) Rs. 11,761 (b) Rs. 10,000
(c) Rs. 12,000 (d) none of these

14. The present value of annuity of Rs. 5,000 per annum for 12 years at 4% p.a C.I. annually is
(a) Rs. 46,000 (b) Rs. 46,925
(c) Rs. 15,000 (d) none of these

15. The present value of an annuity of Rs. 80 for 20 years at 5% p.a is


(a) Rs. 997 (appx.) (b) Rs. 900
(c) Rs. 1,000 (d) none of these

16. A person bought a house paying Rs. 20,000 cash down and Rs. 4,000 at the end of each year
for 25 yrs. at 5% p.a. C.I. The cash down price is
(a) Rs. 75,000 (b) Rs. 76,000
(c) Rs. 76,392 (d) Rs. 76375.77

17. A man purchased a house valued at Rs. 3,00,000. He paid Rs. 2,00,000 at the time of
purchase and agreed to pay the balance with interest at 12% per annum compounded half
yearly in 20 equal half yearly instalments. If the first instalment is paid after six months from
the date of purchase then the amount of each instalment is
[Given log 10.6 = 1.0253 and log 31.19 = 1.494]
(a) Rs. 8,718.45 (b) Rs. 8,769.21
(c) Rs. 7,893.13 (d) none of these.

18. A machine depreciates at 10% of its value at the beginning of a year. The cost and scrap
value realized at the time of sale being Rs. 23,240 and Rs. 9,000 respectively. For how many
years the machine was put to use?
(a) 7 years (b) 8 years
(c) 9 years (d) 10 years

81 | P a g e
19. Alibaba borrows Rs. 6 lakhs Housing Loan at 6% repayable in 20 annual installments
commencing at the end of the first year. How much annual payment is necessary.
(A) 52,420 (B) 52,419
(C) 52,310 (D) 52,320

PAST EXAMINATION QUESTIONS

20. Mr. X Invests Rs. 10,000 every year starting from today for next 10 years suppose interest
rate is 8% per annum compounded annually. Calculate future value of the annuity: [Given
that (1+0.08)10=2.15892500]
(a) Rs. 156454.88 (b) Rs. 144865.625
(c) Rs. 156554.88 (d) None of these

21. How much amount is required to be invested every year so as to accumulate Rs. 3,00,000 at
the end of 10 years, if interest is compounded annually at 10%?
(a) Rs. 18,823.62 (b) Rs. 18,828.65
(c) Rs.18,832.65 (d) Rs. 18,882.65

22. Vipul purchases a car for Rs. 5,50,000. He gets a loan of Rs. 5,00,000 at 15% pa. from a
Bank and balance Rs. 50,000 he pays at the time of purchase. He has to pay the whole
amount of loan in 12 equal monthly installments with interest starting from the end of the
first month. The money he has to pay at the end of every month is: [Given that
(1.0125)12=1.16075452]
(a) Rs. 45,130.43 (b) Rs. 45,230.43
(c) Rs, 45,330.43 (d) None of these

23. Raja aged 40 years wishes his wife Rani to have Rs. 40 lakhs at his death. if his expectation
of life is another 30 years and he starts making equal annual investments commencing now at
3% compound interest p.a. how much should he invest annually?
(a) Rs. 84,077 (b) Rs. 81,628
(c) Rs. 84,449 (d) Rs. 84.247

24. Future value of an ordinary annuity:


 (1  i) n  1  (1  i) n  1
(a) A (n,i) = A   (b) A (n,i) = A  
 i   i 

1  (1  i ) n   (1  i) n  1
(c) A (n,i) = A   (d) A (n,i) = A  n 
 i   i(1  i) 
25. Find the present value of an annuity of Rs.1,000 payable at the end of each year for 10 years.
If rate of interest is 6% compounding per annum (given (1.06) -10=0.5584):
(a) Rs. 7,360 (b) Rs. 8,360
(c) Rs. 12,000 (d) None of these

26. The future value of an annuity of Rs. 5,000 is made annually for 8 years at interest rate of
9% compounded annually [Given that (1.09)8 = 1.99256] is .
(a) Rs. 55,142.22 (b) Rs. 55,412.22
(c) Rs. 65,532.22 (d) Rs. 57,425.22

82 | P a g e
27. How much amount is required to be invested every year as to accumulate Rs. 6,00,000 at the
end of 10 years, if interest is compounded annually at 10% rate of interest
[Given: (1.1)10 = 2.59374].
(a) Rs. 37,467 (b) Rs. 37,476
(c) Rs. 37,647 (d) Rs. 37,674

28. The future value of an annuity of Rs. 1,000 made annually for 5 years at the interest of 14%
compounded annually is: [Given that (1.14)5=1.92541]
(a) Rs. 5,610 (b) Rs. 6,610
(c) Rs. 6,160 (d) Rs. 5,160

29. Suppose your parent decides to open a PPF (Public Provident Fund) account in a bank towards
your name with Rs.10,000 every year starting from today for next 15 years. When you
receive and get 8.5% per annum interest rate compounded annually. What is the present
value of this annuity? (Give answer in Rs. without any fraction.)
(Give P(15,0.085)=8.304236576)
(a) 83,042 (b) 1,66,084
(c) 93,042 (d) 8,30,423

ANSWER
1. 2. 3. 4. 5. 6. 7. 8. 9. 10.
(A) (B) (A) (B) (A) (C) (B) (A) (C) (C)
11. 12. 13. 14. 15. 16. 17. 18. 19. 20.
(B) (A) (A) (B) (A) (D) (A) (C) (C) (A)
21. 22. 23. 24. 25. 26. 27. 28. 29.
(A) (A) (B) (A) (A) (A) (C) (B) (C)

Leasing: is a financial arrangement under which the owner of the asset (lessor) allows the user of
the asset (lessee) to use the asset for a defined period of time(lease period) for a consideration
(lease rental) payable over a given period of time. This is a kind of taking an asset on rent.

PRACTICE QUESTIONS

1. A machine can be purchased for Rs. 50,000. Machine will contribute Rs. 12,000 per year for
the next five years. Assume borrowing cost is 10% per annum. Determine whether machine
should be purchased or not :
(a) Should be purchased (b) Should not be purchased
(c) Can’t say about purchase (d) None of the above

2. A company is considering proposal of purchasing a machine either by making full payment of


Rs. 4,000 or by leasing it for four years at an annual rate of Rs. 1,250. Which course of action
is preferable, if the company can borrow money at 14% compounded annually?
(a) Leasing is preferable (b) Should be purchased
(c) No difference (d) None of these

3. A company may obtain a machine either by leasing it for 5 years (useful life) at annual rent of
Rs. 2,000 or by purchasing the machine for Rs. 8,100. If the company can borrow money at
18% per annum, which alternative is preferable ?
(a) Leasing (b) Purchasing
(c) Can’t say (d) None of these

83 | P a g e
ANSWER

1. 2. 3.
(B) (A) (A)

SINKING FUND

It is the fund credited for a specified purpose by way of sequence of periodic payments over a
time period at a specified interest rate. Interest is compounded at the end of every period. Size
of the sinking fund deposit is computed from A = P.A(n, i) where A is the amount to be saved,
P the periodic payment, n the payment period.

1. A machine costs Rs. 5,20,000 with an estimated life of 25 years. A sinking fund is created to
replace it by a new model at 25% higher cost after 25 years with a scrap value realization of
Rs. 25000. what amount should be set aside every year if the sinking fund investments
accumulate at 3.5% compound interest p.a.?
(A) 16,000 (B) 16,500
(C) 16,046 (D) 16,005

2. A sinking fund is created for redeming debentures worth Rs. 5 lakhs at the end of 25 years.
How much provision needs to be made out of profits each year provided sinking fund
investments can earn interest at 4% p.a.?
(A) 12,006 (B) 12,040
(C) 12,039 (D) 12,035

3. A company establishes a sinking fund to provide for the payment of Rs. 2,00,000 debt
maturing in 20 years. Contributions to the hind are to be made at the end of every year. Find
the amount of each annual deposit if interest is 5% per annum :
(a) Rs. 6,142 (b) Rs. 6,049
(c) Rs. 6,052 (d) Rs. 6,159

ANSWER

1. 2. 3.
(C) (A) (B)

84 | P a g e
CALCULATION OF MULTI PERIOD PERPETUITY

1. Ramesh wants to retire and receive Rs. 3,000 a month. He wants to pass this monthly
payment to future generations after his death. He can earn an interest of 8% compounded
annually. How much will he need to set aside to achieve his perpetuity goal?
(a) 4,50,000 (b) 4,39,780
(c) 4,99,775 (d) 5,99,775

CALCULATION OF GROWING PERPETUITY

2. Assuming that the discount rate is 7% per annum, how much would you pay to receive Rs.
50, growing at 5%, annually, forever?
(a) 3,000 (b) 2,500
(c) 3,500 (d) 4,500

NET PRESENT VALUE

3. Compute the net present value for a project with a net investment of Rs. 1,00,000 and net
cash flows year one is Rs. 55,000; for year two is Rs. 80,000 and for year three is Rs.
15,000. Further, the company’s cost of capital is 10%?
(PVIF @ 10% for three years are 0.909, 0.826 and 0.751)
(a) 25,880 (b) 27,960
(c) 27,384 (d) 25,390

COMPOUND ANNUAL GROWTH RATE (CAGR)

4. Suppose the revenues of a company for four years, V (t) in the above formula, have been
Year 2013 2014 2015 2016
Revenues 100 120 160 210
1
 210  3
Calculate Compound annual Growth Rate. Give that  100   1.2774
(a) 27.74% (b) 25.54%
(c) 28.41% (d) 33.00%

ANSWER

1. 2. 3. 4.
(A) (B) (C) (A)

85 | P a g e
CHAPTER – 5 - BASIC CONCEPTS OF PERMUTATION & COMBINATION

PRACTICE QUESTIONS

4
1. P 3 is evaluated as
(a) 43 (b) 34
(c) 24 (d) None of these.

2.
5
P2 6 P3 is equal to
(a) 120 (b) 140
(c) 160 (d) None of these

3. 7 ! is equal to
(a) 5040 (b) 4050
(c) 5050 (d) None of these

4. 0 ! is a symbol equal to
(a) 0 (b) 1
(c) Infinity (d) None of these

n
5. In P r , n is always
(a) an integer (b) a fraction
(c) a positive integer (d) none of these

n
6. In P r , the restriction is
(a) n>r (b) nr
(c) nr (d) none of these

n
7. In P r = n (n—1) (n—2) ---------------- (n—r—1), the number of factors is
(a) n (b) r—1
(c) n—r (d) r

n
8. Pr can also written as
n! n!
(a) (b)
n  r! r! n  r!
r!
(c) (d) none of these
n  r!

n n
9. If P 4 = 12 x P 2 the n is equal to
(a) —1 (b) 6
(c) 5 (d) none of these

86 | P a g e
n
10. If.
n
P3 : P2 = 3:1 then n is equal to
(a) 7 (b) 4
(c) 5 (d) none of these

m n
11. P 2  56, mn P 2  30 then
(a) m=6,n=2 (b) m = 7, n=1
(c) m=4,n=4 (d) None of these

5
12. if Pr = 60, then the value of r is
(a) 3 (b) 2
(c) 4 (d) none of these

n1  n2
13. If P2  132, n1 n2 P2  30 then,

(a) n 1 = 6, n 2 =6 (b) n 1 = 10, n 2 = 2

(c) n1  9,n 2  3 (d) none of these

14. 4.np3=5.n-1p3 the value of n is


(a) 12 (b) 13
(c) 14 (d) 15

15. If nP3=60 then value of n is__________


(a) 5 (b) 2
(c) 1 (d) 3

16. If nP5 : nP3 = 2:1, then the value of n is


(a) 4 (b) 5
(c) 10 (d) None of these

17. How many six digits numbers can be formed with the digits 9,5,3,1,7,0?
(a) 600 (b) 720
(c) 120 (d) None

18. In how many ways can 5 people occupy 8 vacant chairs?


(a) 5720 (b) 6720
(c) 7720 (d) None

19. How many words can be formed beginning with ‘n’ and ending in ‘a’ with the letters of the
word “Sunday”?
(a) 6! (b) 5!
(c) 4! (d) None

20. The number of ways the letters of the word COMPUTER can be rearranged is
(a) 40320 (b) 40319
(c) 40318 (d) none of these

87 | P a g e
21. The number of arrangements of the letters in the word FAILURE, so that vowels are always
coming together is
(a) 576 (b) 575
(c) 570 (d) none of these

22. 10 examination papers are arranged in such a way that the best and worst papers never
come together. The number of arrangements is
(a) 9 x 8! (b) 10!
(c) 8 x 9! (d) none of these

23. A articles are arranged in such a way that 2 particular articles never come together. The
number of such arrangements is
(a) (n—2) (n-1)! (b) (n—1) (n—2)!
(c) n! (d) none of these

24. The sum of all 4 digit number containing the digits 2, 4, 6, 8, without repetitions is
(a) 133330 (b) 122220
(c) 213330 (d) 133320

25. The number of ways the letters of the word “Triangle” to be arranged so that the word ‘angle’
will be always present is
(a) 20 (b) 60
(c) 24 (d) 32

26. If the letters word ‘Daughter’ are to be arranged so that vowels occupy the odd places, then
number of different words are
(a) 576 (b) 676
(c) 625 (d) 2880

27. If 12 school teams are participating in a quiz contest, then the number of ways the first,
second and third positions may be won is
(a) 1230 (b) 1320
(c) 3210 (d) none of these

28. The number of 4 digit numbers greater than 5000 can be formed out of the digits 3,4,5,6 and
7 (no digit is repeated). The number of such is
(a) 72 (b) 27
(c) 70 (d) none of these

29. 4 digit numbers to be formed out of the figures 0, 1, 2, 3, 4 (no digit is repeated) then
number of such numbers is
(a) 120 (b) 20
(c) 96. (d) none of these

30. The number of ways in which the letters of the word DOGMATIC can be arranged is
(a) 40319 (b) 40320
(c) 40321 (d) none of these

31. Mr. X and Mr. Y enter into a railway compartment having six vacant seats. The number of
ways in which they can occupy the seats is
(a) 25 (b) 31
(c) 32 (d) 30

88 | P a g e
32. The number of numbers lying between 100 and 1000 can be formed with the digits 1, 2, 3, 4,
5, 6, 7 is
(a) 210 (b) 200
(c) 110 (d) none of these

33. The number of numbers lying between 10 and 1000 can be formed with the digits 2,3,4,0,8,9
is
(a) 124 (b) 120
(c) 125 (d) none of these

34. In a group of boys the number of arrangement of 4 boys is 12 times the number of
arrangements of 2 boys. The number of boys in the group is
(a) 10 (b) 8
(c) 6 (d) none of these

35. The total number of 9 digit numbers of different digits is


(a) 10x9! (b) 8x9!
(c) 9x9! (d) none of these

36. There are 10 trains plying between Calcutta and Delhi. The number of ways in which a person
can go from Calcutta to Delhi and return by a different train is
(a) 99 (b) 90
(c) 80 (d) none of these

37. The number of ways in which 8 T-shirts of different sizes can be distributed among 8 persons
of different ages so that the largest T-shirts always goes to be younger assuming that each
one of then gets a T-shirt is
(a) 8! (b) 5040
(c) 5039 (d) none of these

38. The number of arrangement of the letters of the word `COMMERCE’ is


(a) 8! (b) 8!/(2!2!2!)
(c) 7! (d) Both (b) & (c)

39. 5 letters are written and there are five letter-boxes. The number of ways the letters can be
dropped into the boxes, are in each
(a) 119 (b) 120
(c) 121 (d) none of these

40. In how many different ways 3 letters can be formed using the letters of the word SPECIAL?
(a) 210 (b) 6
(c) 840 (d) 450

41. How many numbers greater than 2000 can be formed with the digits 1, 2,3,4,5 with each
digit distinct?
(a) 216 (b) 120
(c) 24 (d) 240
42. In how many ways can 4 single seated rooms in a hostel be occupied by 3 students?
(a) 24 (b) 12
(c) 4 (d) 6

89 | P a g e
43. In how many ways the word “arrange” be arranged such that the 2 ‘r’ do not come together?
(a) 1000 (b) 900
(c) 800 (d) None
44. In how many ways of the word “mathematics” be arranged so that the vowels occur together
(a) (8! x 4!)÷(2!)3 (b) 11! ÷ (2!)3
3
(c) 12! ÷ (2!) (d) None of these
45. There are 11 trains plying between Delhi & Kanpur. The number of ways in which a person
can go from Delhi to Kanpur and return by a different train
(a) 121 (b) 100
(c) 110 (d) None of these
46. How many three digit numbers are there, with distinct digits, with each digits odd
(a) 120 (b) 60
(c) 30 (d) 15
47. How many numbers greater than 3000 can be formed with the digits 1, 2,3,4,5?
(a) 216 (b) 192
(c) 24 (d) 240
48. There are 7 routes from station X to station Y. In how many ways one may go from X to Y
and return, if for returning one can make a choice of any of the routes?
(a) 49 (b) 17
(c) 42 (d) 35
49. How many permutations can be formed from the leters of the word “PARALLEL” in which 3L’s
do not come together?
(a) 3000 (b) 4436
(c) 3360 (d) 360

50. How many five digit numbers can be formed out of digits 1, 2, 4, 5, 6, 7, 8, if no digit is
repeated in any number?
(a) 2520 (b) 840
(c) 1680 (d) None of these

51. Find the number of even numbers greater than 100 that can be formed with the digits 0, 1,
2, 3?
(a) 10 (b) 15
(c) 20 (d) None of these

52. How many numbers between 100 and 1000 can be formed with the digits. 2, 3, 4, 0, 8, 9?
(a) 100 (b) 105
(c) 200 (d) None of these

53. A room has 10 doors. In how many ways can a man enter the room by one door and come
out by a different door.
(a) 90 (b) 100
(c) 50 (d) None of these

54. How many four digits numbers greater than 1000 can be formed with the digits of the
number 2,3,4,1,6 if the digits are not repeated in the same number.
(a) 120 (b) 200
(c) 240 (d) None of these

90 | P a g e
55. How many numbers can be formed with the digits of the number 1,1,2,3,2,1 that are greater
than one lakh?
(a) 60 (b) 80
(c) 70 (d) None of these

56. The number of arrangements of 10 different things taken 4 at a time in which one particular
thing always occurs is
(a)2015 (b) 2016
(c) 2014 (d) none of these

57. The number of permutations of 10 different things taken 4 at a time in which one particular
thing never occurs is
(a) 3020 (b) 3025
(c) 3024 (d) none of these

58. The number of ways in which 6 men can be arranged in a row so that the particular 3 men sit
together, is
4
(a) P4 (b) 4
P 4 3 P 3
2
(c) (!3) (d) none of these

59. There are 5 speakers A, B, C, D and E. The number of ways in which A will speak always just
before B is
(a) 24 (b) 4! 2!
(c) 5! (d) none of these

60. The number of arrangements in which the letters of the word MONDAY be arranged so that
the words thus formed begin with M and do not end with N is
(a) 720 (b) 120
(c) 96 (d) none of these

61. The total number of ways in which six ‘+’ and four ‘—‘ signs can be arranged in a line such
that no two ‘—‘ signs occur together is
(a) 7!/3! (b) 6! x 7! / 3!
(c) 35 (d) none of these

62. The number of ways in which the letters of the word MOBILE be arranged so that consonants
always occupy the odd places is
(a) 36 (b) 63
(c) 30 (d) none of these.

63. The number of words that can be made by rearranging the letters of the word APURNA so that
vowels and consonants appear alternate is
(a) 18 (b) 35
(c) 36 (d) none of these

64. The number of even numbers greater than 300 can be formed with the digits 1, 2, 3, 4, 5
without repetion is
(a) 110 (b) 112
(c) 111 (d) none of these

91 | P a g e
65. The letters of the words CALCUTTA and AMERICA are arranged in all possible ways. The ratio
of the number of these arrangement is _______.
(a) 1: 2 (b) 2:1
(c) 1:1 (d) 1.5:1

66. Eleven students are participating in a race. In how many ways the first 5 prizes can be won?
(a) 44550 (b) 55440
(c) 120 (d) 90

67. In how many ways can the letters of words “ACCOUNTANT” be arranged if vowels always
occur together?
(a) 7560 (b) 7650
(c) 7660 (d) 7550

68. The letters of the words ALLAHABAD and INDIA are arranged in all possible ways. The ratio
of the number of these arrangement is _________________
(a) 9:5 (b) 126:1
(c) 1:1 (d) 2:5

69. In how many ways 5 physics, 3 chemistry and 3 maths books be arranged keeping the
books of the same subject together.
(a) 5! X 3! X 3! (b) 5 P3
(c) 5! X 3! (d) 5! X 3! X 3! X 3!

70. In how many ways can the letters of the word PENCIL be arranged so that N is always next
to E
(a) 60 (b) 40
(c) 720 (d) 120

71. A letter lock has three rings each marked with 10 different letters. In how many ways it is
possible to make an unsuccessful attempt to open the lock.
(a) 1000 (b) 999
(c) 5040 (d) None of these

72. The no. of words can be made out the letters of word “GLOOMY” so that 2 "O" never come
together is ________
(a) 240 (b) 360
(c) 480 (d) None of these

73. How many numbers can be formed using the digits 2, 3, 4, 1, 6 that are greater than 1000
is _____
(a) 120 (b) 188
(c) 240 (d) 144

74. There are 5 books on Physics, 3 on Chemistry and 2 on Mathematics. In how many ways can
these be placed on a shelf if the books on the same subject are to be together?
(a) 8640 (b) 1440
(c) 4320 (d) None of these

75. How many different arrangements are possible from the letters of the word CALCULATOR?
(a) 453600 (b) 50400
(c) 45360 (d) None of these

92 | P a g e
76. In how many ways can the letters of the word “ALGEBRA” be arranged without changing the
relative order of the vowels?
(a) 82 (b) 70
(c) 72 (d) None of these

77. How many words can be formed with the letters of the word “UNIVERSITY”, the vowels
always remaining together?
(a) 60480 (b) 60482
(c) 60000 (d) None of these

78. A family of 4 brothers and three sisters is to be arranged for a photograph in one row. In
how many ways can they be seated if all the sisters sit together,
(a) 600 (b) 720
(c) 900 (d) 1440

79. Six boys and five girls are to be seated in a row such that no two girls and no two boys sit
together. Find the number of ways in which this can be done.
(a) 86,400 (b) 85,000
(c) 85,400 (d) None of these

80. A family of 4 brothers and three sisters is to be arranged for a photograph in one row. In
how many ways can they be seated if no two sisters sit together,
(a) 600 (b) 720
(c) 900 (d) 1440

81. Find the sum of four digit numbers made by the given digits 1, 3, 3, 0?
(a) 22554 (b) 22550
(c) 22,000 (d) None of these

82. There are 6 students of whom 2 are Indians, 2 Americans, and the remaining 2 are
Russians. They have to stand in a row for a photograph so that the two Indians are together,
the two Americans are together and so also the two Russians,. Find the number of ways in
which they can do so.
(a) 40 (b) 42
(c) 48 (d) None of these

83. 5 letters are written and there are 5 letter boxes. The number of ways the letters can be
dropped into the boxes, are in each
(a) 119 (b) 120
(c) 121 (d) None of these

93 | P a g e
ANSWER

1. 2. 3. 4. 5. 6. 7. 8. 9. 10.
(C) (B) (A) (B) (C) (B) (D) (D) (B) (C)
11. 12. 13. 14. 15. 16. 17. 18. 19. 20.
(B) (A) (C) (D) (A) (B) (A) (B) (C) (B)
21. 22. 23. 24. 25. 26. 27. 28. 29. 30.
(A) (C) (A) (D) (C) (D) (B) (A) (C) (B)
31. 32. 33. 34. 35. 36. 37. 38. 39. 40.
(D) (A) (C) (C) (C) (B) (B) (D) (B) (A)
41. 42. 43. 44. 45. 46. 47. 48. 49. 50.
(A) (A) (B) (A) (C) (B) (B) (A) (A) (A)
51. 52. 53. 54. 55. 56. 57. 58. 59. 60.
(C) (A) (A) (A) (A) (B) (C) (B) (A) (C)
61. 62. 63. 64. 65. 66. 67. 68. 69. 70.
(C) (A) (C) (C) (B) (B) (A) (B) (D) (D)
71. 72. 73. 74. 75. 76. 77. 78. 79. 80.
(B) (A) (C) (A) (A) (C) (A) (B) (A) (D)
81. 82. 83.
(A) (C) (B)

CIRCULAR PERMUTATION

If we arrange the objects alongs a closed curve (circle), the permutations are known as circular
permutations.
(i) The number of circular permutations of n different things chosen at time is given by (n-1)!
1
(ii) The number of necklaces formed with n beads of different colours = ( n  1)!
2
(iii) The number of circular permutations of n different things when clock wise and anticlockwise
( n  1)!
arrangements are not different are given by
2
Example 1. Find the number of ways in which 5 different beads can be arranged to form a
necklace.
( n  1)!
Solution: Required number of ways =
2
(5  1)! 4! 24
    12
2 2 2

94 | P a g e
PRACTICE QUESTIONS

1. The number of ways in which 7 girls form a ring is


(a) 700 (b) 710
(c) 720 (d) none of these

2. The number of ways in which 7 boys sit in a round table so that two particular boys may sit
together is
(a) 240 (b) 200
(c) 120 (d) none of these

3. If 50 different jewels can be set to form a necklace then the number of ways is
1 1
(a) 50 ! (b) 49 !
2 2
(c) 49! (d) none of these

4. 5 persons are sitting in a round table in such way that Tallest Person is always on the right–
side of the shortest person; the number of such arrangements is
(a) 6 (b) 8
(c) 24 (d) none of these

5. In how many ways that 8 members including the President & vice President can be arranged
at a round table so that the president and Vice President do not come together is
(a) 3600 (b) 40320
(c) 4320 (d) None

6. In ______ ways can 4 Americans and 4 English men be seated at a round table so that no 2
Americans may be together.
(a) 4! × 3! (b) 4𝑝 4
(c) 3 × 4𝑝 (d) 4𝑐 4
4

7. The chief ministers of 17 states meet to discuss the hike in oil price at a round table. In how
many ways they seat themselves if the Kerala and Bengal chief ministers choose to sit
together?
(a) 15! x 2! (b) 17! x 2!
(c) 16! x 2! (d) None of these

8. In how many ways can 5 persons be seated at a round table if 2 particular persons sit
together
(a) 24 (b) 6
(c) 12 (d) None of these

9. In how many ways 5 gents and 5 ladies sit at a round table; if no two ladies are to sit
together.
(a) 720 (b) 120
(c) 2,880 (d) 34,600

95 | P a g e
10. A gentlemen invites 6 of his friends to a party. In how many different arrangements they
along with the wife of the gentleman can sit at a round table for a dinner if the host and his
wife always sit side by side?
(a) 1440 (b) 144
(c) 1445 (d) None of these

11. The number of ways in which 8 different beads be strung on a neckace is


(a) 2500 (b) 2520
(c) 2250 (d) None of these

ANSWER

1. 2. 3. 4. 5. 6. 7. 8. 9. 10.
(C) (A) (B) (A) (A) (A) (A) (C) (C) (A)
11.
(B)

96 | P a g e
CHAPTER – 5 - COMBINATION
PRACTICE QUESTIONS

12 12
1. The value of C4 + C3 is
(a) 715 (b) 710
(C) 716 (d) none of these

2. If npr = 336 and nCr = 56, then n and r will be


(a) (3, 2) (b) (8, 3)
(c) (7, 4) (d) none of these

3. If 18Cr = 18Cr+2, the value of rC5 is


(a) 55 (b) 50
(c) 56 (d) none of these

4. If c(n, 8) = c (n, 6), find c(n, 2)


(a) 14 (b) 91
(c) 19 (d) 41

5. A person has 8 friends. The number of ways in which he may invite one or more of them to a
dinner is.
(a) 250 (b) 255
(c) 200 (d) none of these

6. The number of ways in which a person can chose one or more of the four electrical appliances
: T.V, Refrigerator, Washing Machine and a cooler is
(a) 15 (b) 25
(c) 24 (d) none of these

7. If nc10 = nc14, then 25


cn is
(a) 24 (b) 25
(c) 1 (d) none of these

8. Out of 7 gents and 4 ladies a committee of 5 is to be formed. The number of committees such
that each committee includes at least one lady is
(a) 400 (b) 440
(c) 441 (d) none of these

9. If 500
C 92  499 C 92  n C 91 then n is
(a) 501 (b) 500
(c) 502 (d) 499
51
10. C31 is equal to
51
(a) C 20 (b) 2.50C20
(c) 2.45C15 (d) none of these

n
11. C1 + nC2 + nC3 + nC4 + …..+nCn is equal to
(a) 2n –1 (b) 2n
(c) 2n +1 (d) none of these

97 | P a g e
15
12. C3+15C2 is equal to
16 17
(a) C3 (b) C3
15 15
(c) C16 (d) C15
5
13. c1+5c2+ 5c3 +5c4+ 5c5 is equal to ________
(a) 30 (b) 31
(c) 32 (d) 25

14. If 12C5+2 12C4+ 12


C3=14Cx then the value of X is:
(a) 5 (b) 9
(c) 5 or 9 (d) None of these
3
15.
Evaluate : 47
C4 + 
j 0
50  j
c3

(a) 249900 (b) 24990


(c) 249000 (d) None of these

16. The number of diagonals in a decagon is


(a) 30 (b) 35
(c) 45 (d) none of these

17. There are 12 points in a plane of which 5 are collinear. The number of triangles is
(a) 200 (b) 211
(c) 210 (d) none of these

18. The number of straight lines obtained by joining 16 points on a plane, no twice of them being
on the same line is
(a) 120 (b) 110
(c) 210 (d) none of these

19. At an election there are 5 candidates and 3 members are to be elected. A voter is entitled to
vote for any number of candidates not greater than the number to be elected. The number of
ways a voter choose to vote is
(a) 20 (b) 22
(c) 25 (d) none of these

20. Every two persons shakes hands with each other in a party and the total number of hand
shakes is 66. The number of guests in the party is
(a) 11 (b) 12
(c) 13 (d) 14

21. 8 points are marked on the circumference of a circle. The number of chords obtained by
joining these in pairs is
(a) 25 (b) 27
(c) 28 (d) none of these

22. The Supreme Court has given a 6 to 3 decision upholding a lower court; the number of ways
it can give a majority decision reversing the lower court is
(a) 256 (b) 276
(c) 245 (d) 226.

98 | P a g e
23. Five bulbs of which three are defective are to be fried in two bulb points in a dark room.
Number of trials the room shall be lighted is
(a) 6 (b) 8
(c) 5 (d) 7.

24. The number of different words that can be formed with 12 consonants and 5 vowels by
taking 4 consonants and 3 vowels in each word is
(a) C4  5C3
12
(b)
17
C7
(c) 4950  7 (d) None of these

25. A question paper contains 6 questions, each having an alternative. The number of ways an
examine can answer one or more questions is
(a) 720 (b) 728
(c) 729 (d) none of these

26. A candidate is required to answer 6 out of 12 questions which are divided into two groups
containing 6 questions in each group. He is not permitted to attempt not more than four from
any group. The number of choices are.
(a) 750 (b) 850
(c) 800 (d) none of these

27. The number of ways a person can contribute to a fund out of 1 ten-rupee note, 1 five-rupee
note, 1 two-rupee and 1 one rupee note is
(a) 15 (b) 25
(c) 10 (d) none of these

28. How many ways a team of 11 player can be made out of 15 players if one particular player is
not to be selected in the team.
(a) 364 (b) 728
(c) 1,001 (d) 1,234

29. A box contains 7 red, 6 white and 4 blue balls. How many selections of three balls can be
made so that none is red?
(a) 90 (b) 120
(c) 48 (d) 24

30. How many diagonals can be drawn in a plane figure of 16 sides.


(a) 100 (b) 50
(c) 104 (d) 54

31. A person has 12 friends of whom 8 are relatives. In how many ways can he invite 7 guests
such that 5 of them are relatives?
(a) 330 (b) 336
(c) 363 (d) 633

32. 6 seats of articled clerks are vacant in a ‘Chartered Accountant firm’. How many different
batches of candidates can be chosen out of 10 candidates if one particular candidate is
always selected.
(a) 124 (b) 125
(c) 126 (d) None of these

99 | P a g e
33. 9 distinct things are to be divided in 3 groups, consisting of 2, 3 and 4 things respectively,
number of ways this can be done is equal to ___________
(a) 1250 (b) 1260
(c) 1200 (d) None of these

34. Out of 6 teachers and four boys, a committee of eight is to be formed. In how many ways
can this be done when there should not be less than four teachers in the committee.
(a) 45 (b) 55
(c) 30 (d) 50

35. A Committee of 7 persons is to be formed out of 11. The number of ways of forming such a
committee is _________
(a) 660 (b) 330
(c) 300 (d) None of these

36. A man has 7 friends, in how many ways can he invite one or more of his friends?
(a) 127 (b) 256
(c) 255 (d) None of these

37. In how many ways can a consonant and a vowel be chosen out of the letters of the word
“logarithm”?
(a) 18 (b) 15
(c) 10 (d) None of these

38. There are 7 men and 3 ladies. Find the number of ways in which a committee of 6 can be
formed of them if the committee is to include atleast two ladies?
(a) 135 (b) 140
(c) 160 (d) None of these

39. In a party of 40 people, each shakes hand with others. How many handshakes took place in
the party?
(a) 780 (b) 700
(c) 880 (d) None of these

40. How many different triangles can be formed joining the angular points of a polygon of m
sides?
m(m  1) (m  2) m(m  1)
(a) (b)
6 2
(c) m (d) None of these

41. How many different cricket teams of 11 players can be selected from 14 cricket players of
which only two can play as wicketkeeper? Given each team must have exactly one
wicketkeeper?
(a) 130 (b) 132
(c) 140 (d) None of these

100 | P a g e
ANSWER
1. 2. 3. 4. 5. 6. 7. 8. 9. 10.
(A) (B) (C) (B) (B) (A) (B) (C) (D) (A)
11. 12. 13. 14. 15. 16. 17. 18. 19. 20.
(A) (A) (B) (C) (A) (B) (C) (A) (C) (B)
21. 22. 23. 24. 25. 26. 27. 28. 29. 30.
(C) (A) (D) (C) (B) (B) (A) (A) (B) (C)
31. 32. 33. 34. 35. 36. 37. 38. 39. 40.
(B) (C) (B) (A) (B) (A) (A) (B) (A) (A)
41.
(B)

PAST EXAMINATION QUESTIONS (Permutation and combination):


1. The number of triangles that can be formed by choosing the vertices from a set of 12 points,
seven of which lie on the same straight line is:
(a) 185 (b) 175
(c) 115 (d) 105

2. A codeword is consist two distinct English alphabets followed by two distinct numbers from 1
to 9. How many such code words are possible?
(a) 6,15,800 (b) 46,800
(c) 7,19,500 (d) 4,10,800

3. An examination paper consists of 12 questions divided into two parts A and B. Part A contains
7 questions and part B contains 5 questions. A candidate is required to attempt 8 questions
selecting at least 3 from each part. In how many maximum ways can the candidate select the
questions?
(a) 35 (b) 175
(c) 210 (d) 420

4. A Supreme Court Bench consists of 5 judges. In how many ways the bench can give a
majority decision?
(a) 10 (b) 5
(c) 15 (d) 16

5. Given: P(7, k)=60 P(7, k-3) Then:


(a) k=9 (b) k= 8
(c) k= 5 (d) k= 0

6. The number of ways in which n books can be arranged on a shelf so that two particular books
are not come together is :
(a) (n -2) x (n-1)! (b) (n -2) x (n +1)!
(c) (n-1) x (n+l)! (d) (n -2) x (n +2)!

7. In how many ways can the letters of the word "FAILURE" be arranged so that the consonants
may occupy only odd positions?
(a) 576 (b) 476
(c) 376 (d) 276

101 | P a g e
8. Out of Five bulbs three are defective are to be tried in two lights - points in a dark-room. In
how many trials the room shall be lighted?
(a) 10 (b) 7
(c) 3 (d) None of these

9. In how many ways can a party of 4 men and 4 women be seated at a circular table so that no
two woman are adjacent?
(a) 164 (b) 174
(c) 144 (d) 154

5
5
10. The value of  Cr is:
r 1
(a) 29 (b) 31
(c) 35 (d) 26

11. If 6Pr =24 6C r, then find r:


(a) 4 (b) 6
(c) 10 (d) 120

12. If
n
Pr  720 and n
Cr  120 , then value of r is
(a) 4 (b) 5
(c) 6 (d) 3

13. How many words can be formed with the letters of the word ‘ORIENTAL’ so that A and E
always occupy odd places:
(a) 540 (b) 8640
(c) 8460 (d) 8450

14. If 1000C98 = 999


C97+ XC901, find x .
(a) 999 (b) 998
(c) 997 (d) 1000

15. How many numbers greater than a million can be formed with the digits 4, 5, 5, 0, 4, 5, 3 ?
(a) 260 (b) 360
(c) 280 (d) 380

16. A building contractor needs three helpers and ten men apply. In how many ways can these
selections take place?
(a) 36 (b) 15
(c) 150 (d) 120

17. There are three blue balls, four red balls and five green balls. In how many ways can they be
arranged in a row?
(a) 26720 (b) 27720
(c) 27820 (d) 26,620

102 | P a g e
18. C(n, r) : C(n, r +1) = 1 : 2 and C (n, r + 1): C (n, r + 2) = 2:3, determine the value of n and
r:
(a) (14,4) (b) (12,4)
(c) (14,6) (d) None.

19. Six seats of articled clerks are vacant in a ‘Chartered Accountant Firm’. How many different
ways of candidates can be chosen out of ten candidates?
(a) 216 (b) 210
(c) 220 (d) None

20. There are 6 men and 4 women in a group, then the number of ways in which a committee of
5 persons can be formed of them, if the committee is to include at least 2 women are:
(a) 180 (b) 186
(c) 120 (d) 105

21. In how many ways a committee of 6 members can be formed from a group of 7 boys and 4
girls having at least 2 girls in the committee.
(a) 731 (b) 137
(c) 371 (d) 351

22. If 18Cr = 18 Cr + 2, find the value of rc5.


(a) 55 (b) 50
(c) 56 (d) None of these

23. 7 books are to be arranged in such a way so that two particular books are always at first and
last place. Find the number of arrangements.
(a) 60 (b) 120
(c) 240 (d) 480

24. Find the number of arrangements in which the letters of the word ‘MONDAY’ be arranged so
that the words formed begin with ‘M’ and do not end with ‘N’.
(a) 720 (b) 120
(c) 96 (d) None.

25. In how many ways can 17 billiard balls be arranged if 7 of them are black, 6 red and 4 white?
(a) 4084080 (b) 1
(c) 8048040 (d) None of these

26. (n+1)! =20 (n-1)!, find n .


(a) 6 (b) 5
(c) 4 (d) 10

27. Out of 4 gents and 6 ladies, a committee is to be formed find the number of ways the
committee can be formed such that it comprises at least 2 gents and at least the number of
ladies should be double of gents.
(a) 94 (b) 132
(c) 136 (d) 104

103 | P a g e
28. In how many ways can the letters of 'REGULATION’ be arranged so that the vowels come at
odd places?
(a) 14400 (b) 14444
(c) 144252 (d) None of these

29. Six points are on a circle. The number of quadrilaterals that can be formed are :
(a) 30 (b) 360
(c) 15 (d) None of the above

30. The number of ways of arranging 6 boys and 4 girls in a row so that all 4 girls are together is:
(a) 6!.4! (b) 2 (7!.4!)
(c) 7!.4! (d) 2. (6!4!)

31. How many numbers not exceeding 1000 can be made from the digits 1,2,3,4,5,6,7,8,9 if
repetition is not allowed.
(a) 364 (b) 585
(c) 728 (d) 819

32. A garden having 6 tall trees in a row. In how many ways 5 children's stand one in a gap
between the trees in order to pose for a photograph?
(a) 24 (b) 120
(c) 720 (d) 30

15 15
33. C3 + C2 is equal to:
(a) 16
C13 (b) 30 C1 6
(c) 15C 16 (d) 15C1 5

34. How many ways a team of 11 player can be made out of 15 players if one particular player is
not to be selected in the team.
(a) 364 (b) 728
(c) 1,001 (d) 1,234

35. Find the number of arrangements of 5 things taken out of 12 things, in which one particular
thing must always be included.
(a) 39,000 (b) 37,600
(c) 39,600 (d) 36,000

36. There are 12 questions to be answered in yes or no In how many ways can these be
answered
(a) 1024 (b) 2048
(c) 4096 (d) None

37. If nP4=20 nP2 then n =


(a) -2 (b) 7
(c) both (a) & (b) (d) None of these

38. The letters of the word “VIOLENT” are arranged so that vowels occupy even places only. The
number of permutations is
(a) 144 (b) 120
(c) 24 (d) 72

104 | P a g e
39. A man has 3 sons and 6 schools within his reach. How many ways can his sons go to school,
if no two of them are in same school.
6 6
(a) p2 (b) p3
3
(c) 6 (d) 36

40. If 13c6+213c5+13c4 = 15
cX, then x = _____
(a) 6 (b) 7
(c) 8 (d) 9

41. Number of permutations can be formed from the letters of the word “DRAUGHT”, if no two
vowels are separable.
(a) 720 (b) 1440
(c) 140 (d) 2880

42. The total number of shake hands in a group of 10 persons to each other are ______
(a) 45 (b) 54
(c) 90 (d) 10

43. A polygon has 44 diagonals then the no. of sides are _____
(a) 8 (b) 9
(c) 10 (d) 11

44. In how many ways the word "ARTICLE" can be arranged in a row so that vowels occupy even
places ?
(a) 132 (b) 144
(c) 72 (d) 160

45. How many different words can be formed with the letters of the word "LIBERTY"
(a) 4050 (b) 5040
(c) 5400 (d) 4500

46. In how many ways can a family consist of 3 children have different birthdays in a leap year :
366
(a) 366 × 365 × 364 (b) C3
365 365
(c) C3 (d) C3 -3

15
47. If C3r 15Cr3 , then r =
(a) 2 (b) 3
(c) 4 (d) 5

48. If 6 times the no. of permutations of n items taken 3 at a time is equal to 7 times the no. of
permutations of (n-1) items taken 3 at a time then the value of n will be
(a) 21 (b) 9
(c) 13 (d) 7

6
49. Pr= 360 then find r
(a) 4 (b) 5
(c) 6 (d) None

105 | P a g e
50. If 5 books of English, 4 books of Tamil and 3 books of Hindi are to be arranged in a single row
so that books of same language come together
(a) 1,80,630 (b) 1,60,830
(c) 1,03,680 (d) 1,30,680

51. 5 Boys and 4 girls are to be seated in row. If the girls occupy even places then the no. of such
arrangements
(a) 288 (b) 2808
(c) 2008 (d) 2880

52. The four digit numbers that can be formed out of the seven digits 1,2,3,5,7,8,9 such that no
digit is repeated in any number and are greater than 3000 are
(a) 120 (b) 480
(c) 600 (d) 840

53. A person has ten friends of whom six are relatives. If he invites five guests such that three of
them are his relatives, then the total number of ways in which he can invite them are:
(a) 30 (b) 60
(c) 120 (d) 75

54. A student has three books on computer, three books on Economics and five books on
Commerce. If these books are to be arranged subject wise, then these can be placed on a
shelf in the number of ways
(a) 25290 (b) 25920
(c) 4230 (d) 4320

55. An examination paper with 10 question consist of 6 questions in mathematics and 4 questions
in statistic part. At least one question from each part is to be attempted in how many ways
can this be done?
(a) 1024 (b) 945
(c) 1005 (d) 1022

ANSWER

1. 2. 3. 4. 5. 6. 7. 8. 9. 10.
(A) (B) (D) (D) (C) (A) (A) (B) (C) (B)
11. 12. 13. 14. 15. 16. 17. 18. 19. 20.
(A) (D) (B) (A) (B) (D) (B) (A) (B) (B)
21. 22. 23. 24. 25. 26. 27. 28. 29. 30.
(C) (C) (C) (C) (A) (C) (C) (A) (C) (C)
31. 32. 33. 34. 35. 36. 37. 38. 39. 40.
(B) (B) (A) (A) (C) (C) (B) (A) (B) (A)
41. 42. 43. 44. 45. 46. 47. 48. 49. 50.
(B) (A) (D) (B) (B) (A) (B) (A) (A) (C)
51. 52. 53. 54. 55.
(D) (C) (C) (B) (B)

106 | P a g e
CHAPTER – 6 - SEQUENCE AND SERIES ARITHMETIC
& GEOMETRIC PROGRESSIONS
PRACTICE QUESTIONS

1. The 20th term of the progression 1, 4, 7, 10.................is


(a) 58 (b) 52
(c) 50 (d) none of these

2. The last term of the series 5, 7, 9,….. to 21 terms is


(a) 44 (b) 43
(c) 45 (d) none of these

3. The last term of the A.P. 0.6, 1.2, 1.8,… to 13 terms is


(a) 8.7 (b) 7.8
(c) 7.7 (d) none of these

4. The nth element of the sequence 1, 3, 5, 7,….…..Is


(a) n (b) 2n – 1
(c) 2n +1 (d) none of these

5. The first three terms of sequence when nth term tn is n2 – 2n are


(a) –1, 0, 3 (b) 1, 0, 2
(c) –1, 0, –3 (d) none of these

6. Which term of the progression –1, –3, –5, …. is –39


(a) 21st (b) 20th
(c) 19th (d) none of these

7. The value of x such that 8x + 4, 6x – 2, 2x + 7 will form an AP is


(a) 15 (b) 2
(c) 15/2 (d) none of these

7
8. 
i4
2i  1 can be written as

(a) 7  9  11  13 (b) 2 7  2 9  2 11  2 13
(c) 2 7  2 9  2 11  2 13 (d) none of these

9. The sum to of the series –5, 25, –125 , 625, ….. can be written as
 
(a)  (5) k (b) 5
k 1
k

k 1

(c) 5
k 1
k
(d) none of these

107 | P a g e
10. The sum of three integers in AP is 15 and their product is 80. The integers are
(a) 2, 8, 5 (b) 8, 2, 5
(c) 2, 5, 8 (d) 8, 5, 2

11. The arithmetic mean between 33 and 77 is


(a) 50 (b) 45
(c) 55 (d) none of these

12. The 4 arithmetic means between –2 and 23 are


(a) 3, 13, 8, 18 (b) 18, 3, 8, 13
(c) 3, 8, 13, 18 (d) none of these

13. The two arithmetic means between –6 and 14 is


1
(a) 2/3, 1/3 (b) 2/3, 7
3
1
(c) – 2/3,  7 (d) none of these
3

14. The sum of the series 9, 5, 1,…. to 100 terms is


(a) –18,900 (b) 18,900
(c) 19,900 (d) none of these

15. The sum of the series 3 ½ + 7 + 10 ½ + 14 + …. to 17 terms is


(a) 530 (b) 535
1
(c) 535 (d) none of these
2

16. The first and the last term of an AP are –4 and 146. The sum of the terms is 7171. The
number of terms is
(a) 101 (b) 100
(c) 99 (d) none of these

17. The sum of a certain number of terms of an AP series –8, –6, –4, …… is 52. The number of
terms is
(a) 12 (b) 13
(c) 11 (d) none of these

2 1
18. The number of the terms of the series 10  9  9  9  ............. will amount to 155 is
3 3
(a) 30 (b) 31
(c) 32 (d) both a & b

19. The nth term of the series whose sum to n terms is 5n 2 + 2n is


(a) 3n – 10 (b) 10n – 2
(c) 10n – 3 (d) none of these

108 | P a g e
20. The sum of n terms of an AP is 3n 2 + 5n. The series is
(a) 8, 14, 20, 26 (b) 8, 22, 42, 68
(c) 22, 68, 114, .... (d) none of these

21. The pth term of an AP is (3p – 1)/6. The sum of the first n terms of the AP is
(a) n (3n + 1) (b) n/12 (3n + 1)
(c) n/12 (3n – 1) (d) none of these

22. The number of numbers between 74 and 25,556 divisible by 5 is


(a) 5,090 (b) 5,097
(c) 5,095 (d) none of these

23. The first term of an A.P is 14 and the sums of the first five terms and the first ten terms are
equal is magnitude but opposite in sign. The 3 rd term of the AP is
4
(a) 6 (b) 6
11
(c) 4/11 (d) none of these

24. The mth term of an A. P. is n and nth term is m. The r th term of it is


(a) m + n +r (b) n + m – 2r
(c) m + n + r/2 (d) m+n–r

25. If the terms 2x, (x+10) and (3x+2) be in A.P., the value of x is
(a) 7 (b) 10
(c) 6 (d) none of these

26. A sum of Rs. 6240 is paid off in 30 installments such that each installment is Rs. 10 more
than the proceeding installment. The value of the 1 st installment is
(a) Rs. 36 (b) Rs. 30
(c) Rs. 60 (d) none of these

27. The sum of all odd numbers between 200 and 300 is
(a) 11,600 (b) 12,490
(c) 12,500 (d) 24,750

28. The sum of all natural numbers between 500 and 1000 which are divisible by 13, is
(a) 28,405 (b) 24,805
(c) 28,540 (d) none of these

29. If unity is added to the sum of any number of terms of the A.P. 3, 5, 7, 9,…... the resulting
sum is
(a) ‘a’ perfect cube (b) ‘a’ perfect square
(c) ‘a’ number (d) none of these

30. The sum of all natural numbers from 100 to 300 which are exactly divisible by 4 or 5 is
(a) 10,200 (b) 15,200
(c) 16,200 (d) none of these

109 | P a g e
31. The sum of all natural numbers from 100 to 300 which are exactly divisible by 4 and 5 is
(a) 2,200 (b) 2,000
(c) 2,220 (d) none of these

32. A man earns 10,000 per month His salary increases by 200 per month. What is his total
salary in 5 years?
(a) 3,12,000 (b) 6,24,000
(c) 6,14,000 (d) 90,000

33. A person saved Rs. 16,500 in ten years. In each year after the first year he saved Rs. 100
more than he did in the preceding year. The amount of money he saved in the 1 st year was
(a) Rs. 1000 (b) Rs. 1500
(c) Rs. 1200 (d) none of these

34. The sum of first n natural number


(a) (n/2) (n + 1) (b) (n/6) (n + 1) (2n + 1)
(c) [(n/2) (n + 1)]² (d) None of these.

35. The sum of natural numbers upto 200 excluding those divisible by 5 is:
(a) 20100 (b) 4100
(c) 16000 (d) None of these.

36. The sum of all natural numbers between 200 and 400 which are divisible by 7 is ______.
(a) 7730 (b) 8729
(c) 7729 (d) 8730

37. The sum of square of first n natural number is ________.


(a) (n/2) (n+1) (b) (n/6) (n+1) (2n+1)
(c) [(n/2) (n+1)]2 (d) None of above

38. The sum of a series in A.P. is 72 the first term being 17 and the common difference –2. The
number of terms is
(a) 6 (b) 12
(c) 6 or 12 (d) 10

39. The sum of n terms of the series 2 + 6 + 10 + ……. is


(a) 2n² (b) n²
(c) n²/2 (d) 4n²

40. The three numbers in A.P. whose sum is 27 and the sum of their squares is 341, are
(a) 2, 9, 16 (b) 16, 9, 2
(c) Both (a) and (b) (d) –2, 9, 16

41. Find the 10th term of an A.P with first term as 4 and common difference being 2
(a) 22 (b) 25
(c) 20 (d) 13

42. Determine the common difference of progression 16, 13,10 .............. 25 terms
(a) 2 (b) –2
(c) 3 (d) –3

110 | P a g e
43. Find the arithmetic mean between 4 and 10
(a) 14 (b) 7
(c) 3 (d) 6

44. Determine the first term of an A.P. with common difference 3 & 7th term being 11
(a) –7 (b) 7
(c) 6 (d) 5

45. If third tem and seventh term of an A.P are eighteen and thirty respectively, then sum of first
twenty terms will be:
(a) 540 (b) 610
(c) 740 (d) 810

46. If the sum of 'n' terms of an Arithmetic Progression (A.P) is 3n2+5n and its mth term is 164,
then the value of m is:
(a) 27 (b) 28
(c) 24 (d) 26

47. Find the value of 1 + 2 + 3 + …………….+ 105


(a) 5000 (b) 5560
(c) 5565 (d) None of these

48. If a, b, c are in Arithmetic Progression (A.P), then the value of a-b+c is


(a) a (b) -b
(c) b (d) c

49. The value of 33+43+53+ …………….. + 113


(a) 4356 (b) 4348
(c) 4347 (d) 4374

PAST EXAMINATION QUESTIONS

50. The sum of all natural numbers between 100 and 1000 which are multiple of 5 is:
(a) 98,450 (b) 96.450
(c) 97.450 (d) 95,450

51. Divide 30 into five parts in A.P., such that the first and last parts are in the ratio 2 :3 :
24 27 33 36 36 33 24 27
(a) , ,6, , (b) 6, , , ,
5 5 5 5 5 5 5 5
27 24 36 33 24 27 33 36
(c) , , , ,6 (d) 6, , , ,
5 4 5 5 5 5 5 5

52. Find the sum of all natural numbers between 250 and 1,000 which are exactly divisible by 3 .
(a) 1,56,375 (b) 1,56,357
(c) 1,65,375 (d) 1,65,357

111 | P a g e
53. A person pays Rs. 975 in monthly installments, each installments is less than former by Rs. 5.
The amount of first installments is Rs.100. In what time will the entire amount be paid?
(a) 26 months (b) 15 months
(c) Both (a) & (b) (d) 18 months

54. If the sum of n terms of an A.P. is (3n 2 - n) then its first term is :
(a) 3 (b) 2
(c) 4 (d) 1

55. Find the sum of the series: 2+7+12+………..+297.


(a) 8970 (b) 8870
(c) 7630 (d) 9875

56. A contractor who fails to complete a building in a certain specified time is compelled to forfeit
Rs. 200 for the first day of extra time required and thereafter forfeited amount is increased by
Rs.25 for every day. If he loses Rs. 9,450 for how many days did he over-run the contract
time?
(a) 19 days (b) 21 days
(c) 23 days (d) 25 days

57. A man employed in a company is promised a salary of Rs. 3.000 every month for the first
year and an increment of Rs. 1,000 in his monthly salary every succeeding year. How much
does the man earn from the company in 20 years?
(a) Rs. 30,00,000 (b) Rs. 27.50,000
(c) Rs. 19,10,000 (d) Rs. 7,90,000

58. Insert 4 A.M. between 3 and 18:


(a) 12, 15, 9, 6 (b) 6, 9, 12, 15
(c) 9, 6, 12, 15 (d) 15, 12, 9, 6

59. On 1st January every year a person buys National Saving Certificates of value exceeding that
of his last years purchase by Rs. 100. After 10 years, he finds that the total value of the
certificates purchased by him is Rs. 54,500. Find the value of certificates purchased by him in
the first year:
(a) Rs. 6,000 (b) Rs. 4,000
(c) Rs. 5,000 (d) Rs. 5,500

60. Find the 9th term of the series:


2 , 5 2, 9 2, .......... ....
(a) 25 2 (b) 31 2
(c) 33 2 (d) 52 2

61. (x + 1), 3x, (4x + 2) are in A.P. Find the value of x


(a) 2 (b) 3
(c) 4 (d) 5

112 | P a g e
62. 2 defines:
n
n(n  1)( 2n  1) n( n  1)
(a) (b)
6 2
 n(n  1) 
2

(c)  2  (d) None of these

63. If in an A.P., tn represents nth term.


If t7 :t10 = 5:7 then t8 : t11 =
(a) 13:16 (b) 17:23
(c) 14:17 (d) 15:19

64. The sum of an A P. whose first term is - 4 and last term is 146 is 7171 Find the value of n.
(a) 99 (b) 100
(c) 101 (d) 102

65. If each months Rs. 100 increases in any sum then find out the total sum after 10 months if
the sum of 1st months is Rs. 2000
(a) 24500 (b) 24000
(c) 50000 (d) 60,000

66. Divide 144 into three parts which are in AP and such that the largest in twice the smallest, the
smallest of three number will be :
(a) 48 (b) 36
(c) 13 (d) 32

67. If sum  Sn  of ' n ' terms of an Arithmetic Progression is  2n 2


 n  . What is the difference of
its 10th and 1st term?
(a) 207 (b) 36
(c) 90 (d) 63.

68. Insert two Arithmetic means between 68 and 260.


(a) 132, 196 (b) 130, 194
(c) 70, 258 (d) None of the above.

69. The sum of the all two digit odd numbers :


(a) 2475 (b) 2575
(c) 4950 (d) 5049

70. The sum of 3 A.M’s between a & 22 is 42 then a :


(a) 14 (b) 11
(c) 10 (d) 6

71. In an A.P. if a = 2000, n = 10, d = 100 then find S n .


(a) 24,500 (b) 15,400
(c) 24,400 (d) 55,300

113 | P a g e
72. In an A.P if the 4th term is 3 times the first term and 7th term exceeds twice the 3rd term by
1 then the values of a and d are
(a) 3,2 (b) 4,3
(c) 5,4 (d) 6,5

73. If the 8th term of an A.P is 15 then the sum of first 15 terms is
(a) 15 (b) 0
(c) 225 (d) 225/2

74. In an A.P the common difference is 2. Sum of n terms is 49. If 7th term is 13. Find n.
(a) 0 (b) 5
(c) 7 (d) 13

75. If the sum of n terms is 2n2+5n then its nth term is


(a) 4n – 3 (b) 3n – 4
(c) 4n + 3 (d) 3n + 4

76. The nth term of series whose sum to n terms is 5n 2 +2n is :


(a) 5n+3 (b) 10n–2
(c) 5n+2 (d) 10n–3

77. In an A.P. if the sum of 4th & 12th terms is '8' then sum of first 15 terms is _________
(a) 60 (b) 120
(c) 110 (d) 150

78. There are 'n' AMs between 7 & 71 and 5th AM is 27 then 'n' = __________
(a) 15 (b) 16
(c) 17 (d) 18

79. An AP has 13 terms whose sum is 143. The third term is 5, then first term is
(a) 4 (b) 7
(c) 9 (d) 2

80. Sum of first n terms of an A.P is 6n 2+6n. Then find 4th term of series.
(a) 120 (b) 72
(c) 48 (d) 24

81. In an A.P. If Sn = n2p and Sm = m2p, (m#n) then Sp =


(a) P2 (b) P3
3
(c) 2p (d) p4

114 | P a g e
ANSWER

1. 2. 3. 4. 5. 6. 7. 8. 9. 10.
(A) (C) (B) (B) (A) (B) (C) (A) (A) (C, D)
11. 12. 13. 14. 15. 16. 17. 18. 19. 20.
(C) (C) (B) (A) (C) (A) (B) (D) (C) (A)
21. 22. 23. 24. 25. 26. 27. 28. 29. 30.
(B) (B) (A) (D) (C) (D) (C) (A) (B) (C)
31. 32. 33. 34. 35. 36. 37. 38. 39. 40.
(A) (B) (C) (A) (C) (B) (B) (C) (A) (C)
41. 42. 43. 44. 45. 46. 47. 48. 49. 50.
(A) (D) (B) (A) (D) (A) (C) (C) (C) (A)
51. 52. 53. 54. 55. 56. 57. 58. 59. 60.
(A) (A) (B) (B) (A) (B) (A) (B) (C) (C)
61. 62. 63. 64. 65. 66. 67. 68. 69. 70.
(B) (A) (B) (C) (A) (D) (B) (A) (A) (D)
71. 72. 73. 74. 75. 76. 77. 78. 79. 80.
(A) (A) (C) (C) (C) (D) (A) (A) (D) (C)
81.
(B)

115 | P a g e
CHAPTER – 6 - GEOMETRIC PROGRESSION
PRACTICE QUESTIONS

1. The 7th term of the series 6, 12, 24,……is


(a) 384 (b) 834
(c) 438 (d) none of these

2. t8 of the series 6, 12, 24,…is


(a) 786 (b) 768
(c) 867 (c) none of these

3. t12 of the series –128, 64, –32, ….is


(a) – 1/16 (b) 16
(c) 1/16 (d) none of these

4. The 4th term of the series 0.04, 0.2, 1, … is


(a) 0.5 (b) 1/2
(c) 5 (d) none of these

5. The last term of the series 1, 2, 4,…. to 10 terms is


(a) 512 (b) 256
(c) 1024 (d) none of these

6. The last term of the series 1, –3, 9, –27............ up to 7 terms is


(a) 297 (b) 729
(c) 927 (d) none of these

7. The last term of the series x2, x, 1, …. to 31 terms is


(a) x28 (b) 1/x
(c) 1/x28 (d) none of these

8. Four geometric means between 4 and 972 are


(a) 12, 36, 108, 324 (b) 12, 24, 108, 320
(c) 10, 36, 108, 320 (d) none of these

9. The second term of a G P is 24 and the fifth term is 81. The series is
(a) 16, 36, 24, 54,.. (b) 24, 36, 53,…
(c) 16, 24, 36, 54,.. (d) none of these

10. The sum of 3 numbers of a G P is 39 and their product is 729. The numbers are
(a) 3, 27, 9 (b) 9, 3, 27
(c) 3, 9, 27 (d) none of these

11. In a G. P, the product of the first three terms 27/8. The middle term is
(a) 3/2 (b) 2/3
(c) 2/5 (d) none of these

12. If you save 1 paise today, 2 paise the next day 4 paise the succeeding day and so on, then
your total savings in two weeks will be
(a) Rs. 163 (b) Rs. 183
(c) Rs. 163.83 (d) none of these

116 | P a g e
13. The product of 3 numbers in G P is 729 and the sum of squares is 819. The numbers are
(a) 9, 3, 27 (b) 27, 3, 9
(c) 3, 9, 27 (d) none of these

1 3
14. The sum of the series  1   ......... to 18 terms is
3 3
(1  3)
(a) 9841 (b) 9841
3
9841
(c) (d) none of these
3

15. The sum of the series –2, 6, –18, …. to 7 terms is


(a) –1094 (b) 1094
(c) – 1049 (d) none of these

16. The sum of the series 1 + 2 + 4 + 8 + .. to n term


(a) 2n –1 (b) 2n – 1
(c) 1/2n – 1 (d) none of these

17. Sum of the series 1 + 3 + 9 + 27 +….is 364. The number of terms is


(a) 5 (b) 6
(c) 11 (d) none of these

18. The number of terms to be taken so that 1 + 2 + 4 + 8 + will be 8191 is


(a) 10 (b) 13
(c) 12 (d) none of these

19. The sum of the first 20 terms of a G. P is 244 times the sum of its first 10 terms. The
common ratio is
(a)  3 (b) 3
(c) 3 (d) none of these

20. The sum of the infinite GP 14 – 2 + 2/7 – 2/49 + … is


1 1
(a) 4 (b) 12
12 4
(c) 12 (d) none of these

21. The sum of the infinite G. P. 1 - 1/3 + 1/9 - 1/27 +... is


(a) 0.33 (b) 0.57
(c) 0.75 (d) none of these

22. Sum of n terms of the series 4 + 44 + 444 + … is


(a) 4/9 { 10/9 ( 10n –1 ) –n } (b) 10/9 ( 10n –1 ) –n
n
(c) 4/9 ( 10 –1 ) –n (d) none of these

117 | P a g e
23. Sum of n terms of the series 0.1 + 0.11 + 0.111 + … is
 (1  (0,1) n 
(a) 1/9 {n – ( 1– ( 0.1 )n )} (b) 1 / 9 n  
 9 
(c) n– 1 – (0.1)n/9 (d) none of these

24. The nth element of the sequence –1, 2, –4, 8 ….. is


(a) ( –1 )n2 n–1 (b) 2 n–1
n
(c) 2 (d) none of these

25. The sum of 1 + 1/3 + 1/32 + 1/33 + … +  is


(a) 2/3 (b) 3/2
(c) 4/5 (d) none of these

26. The sum of the infinite series 1 + 2/3 + 4/9 + .. is


(a) 1/3 (b) 3
(c) 2/3 (d) none of these

27. The sum of the first two terms of a G.P. is 5/3 and the sum to infinity of the series is 3. The
common ratio is
(a) 1/3 (b) 2/3
(c) – 2/3 (d) Both (b) & (c)

28. If p, q and r are in A.P. and x, y, z are in G.P. then x q–r. y r–p. zp–q is equal to
(a) 0 (b) –1
(c) 1 (d) Both (b) & (c)

29. The sum of three numbers in G.P. is 70. If the two extremes by multiplied each by 4 and the
mean by 5, the products are in AP. The numbers are
(a) 12, 18, 40 (b) 10, 20, 40
(c) 40, 20, 10 (d) Both (b) & (c)

30. The sum of 3 numbers in A.P. is 15. If 1, 4 and 19 be added to them respectively, the results
are is G. P. The numbers are
(a) 26, 5, –16 (b) 2, 5, 8
(c) 5, 8, 2 (d) Both (a) & (b)

31. Given x, y, z are in G.P. and xp = yq = zr , then 1/p , 1/q, 1/r are in
(a) A.P. (b) G.P.
(c) Both A.P. and G.P. (d) none of these

32. Three numbers are in A.P. and their sum is 15. If 8, 6, 4 be added to them respectively, the
numbers are in G.P. The numbers are
(a) 2, 6, 7 (b) 4, 6, 5
(c) 3, 5, 7 (d) none of these

33. The sum of four numbers in G. P. is 60 and the A.M. of the first and the last is 18. The
numbers are
(a) 4, 8, 16, 32 (b) 4, 16, 8, 32
(c) 16, 8, 4, 20 (d) none of these

118 | P a g e
2 3
34. The sum of 1.03 + ( 1.03 ) + ( 1.03 ) + …. to n terms is
(a) 103 {(1.03)n – 1} (b) 103/3 {(1.03 )n – 1}
(c) (1.03)n –1 (d) none of these

35. If x, y, z are in A.P. and x, y, (z + 1) are in G.P. then


(a) (x – z)2 = 4x (b) z2 = (x – y)
(c) z=x–y (d) none of these

36. The numbers x, 8, y are in G.P. and the numbers x, y, –8 are in A.P. The value of x and y are
(a) (–8, –8) (b) (16, 4)
(c) (8, 8) (d) Both (a) & (b)

37. The nth term of the series 16, 8, 4,…. is 1/2 17. The value of n is
(a) 20 (b) 21
(c) 22 (d) none of these

38. The sum of n terms of a G.P. whose first terms 1 and the common ratio is 1/2, is equal to
127
1 . The value of n is
128
(a) 7 (b) 8
(c) 6 (d) none of these

39. t4 of a G.P. in x, t10 = y and t16 = z. Then


(a) x2 = yz (b) z2 = xy
(c) y2 = zx (d) none of these

40. If x, y, z are in G.P., then


(a) y2 = xz (b) y ( z 2 + x2 ) = x ( z 2 + y2 )
(c) 2y = x+z (d) none of these

41. Three numbers are in AP and their sum is 21. If 1, 5, 15 are added to them respectively, they
form a G. P. The numbers are
(a) 5, 7, 9 (b) 9, 5, 7
(c) 7, 5, 9 (d) none of these

42. If A be the A.M. of two positive unequal quantities x and y and G be their G. M, then
(a) A<G (b) A>G
(c) A G (d) A G

43. The A.M. of two positive numbers is 40 and their G. M. is 24. The numbers are
(a) (72, 8) (b) (70, 10)
(c) (60, 20) (d) none of these

1 1
44. If a, b, c are in G,P a, x, b and b,y, c are both in A,P the value of + Is
𝑥 𝑦
1 2
(a) (b)
𝑏 𝑏
2
(c) −𝑏 (d) None of these

119 | P a g e
45. Find the four numbers in A.P. with the sum of second and third being 22 and the product of
the first and fourth being 85.
(a) 3, 5, 7, 9 (b) 2, 4, 6, 8
(c) 5 ,9, 13, 17 (d) None of these

46. Find the sum of 10 terms G. P with first term and common ratio being 8 and 3 respectively?
(a) 2,63,291 (b) – 2,36,192
(c) 2,19,631 (d) 2,36,192

47. Find the sum of progression 1,1 , 1 , 1,…………., 10 terms.


2 4 8
(a) 1.9 (b) 1.989
(c) 1.998 (d) 1.89

48. The sixth term of a G.P. with common ratio as 2 and first term being 5 is_______.
(a) 160 (b) 32
(c) 800 (d) 64

49. Which term of the series 0.004 + 0.02 + 0.1 + ……………… is 12.5
(a) 5 (b) 10
(c) 6 (d) None of these

50. The 6th term of the G.P. 1, 2, 4, 8, …………………………., is


(a) 32 (b) 1
(c) 64 (d) None of these

PAST EXAMINATION QUESTIONS

a n 1  b n 1
51. Find n such that may be the geometric mean between a and b :
an  bn
(a) ½ (b) 1
(c) -1/2 (d) 0

52. If the first term of a G.P exceeds the second term by 2 and the sum to infinity is 50, the
series is:
32 5
(a) 10, 8, ,......... . (b) 10, 8, ,......... .
5 2
10 10
(c) 10, , ,......... (d) None
3 9
53. If a1/x = b1/y =c1/Z and a, b, c are in G.P; the x, y, z are in .
(a) A.P. (b) G.P.
(c) Both (a) & (b) (d) None

54. Find the sum to n terms of the series : 7+77+777+………to n terms:


7 n 1 7n 7 n 1 7n
(a) (10  10)  (b) (10  10) 
9 9 9 9
7 7n 7 7n
(c) (10 n 1  10)  (d) (10 n 1  10 ) 
81 9 81 9

120 | P a g e
55. Find the two numbers whose geometric mean is 5 and arithmetic mean in 7.5.
(a) 10 and 5 (b) 13.09 and 1.91
(c) 12 and 3 (d) None of the above

56. The sum of m terms of the series is 1+11+111+ ………is equal to


1 m 1 1 m 1
(a) [0  9m  10] (b) [0  9m  10]
81 27
(c) [10m1  9m  10] (d) None of these

57. The sum of the series: 0.5+0.55+0.555+……… to n terms is:


5n 5 5n 5
(a)  [1  (0.1) n ] (b)  [1  (0.1) n ]
9 9 9 81
5n 5 5n 5
(c)  [1  (0.1) n ] (d)  [1  (0.1) n ]
9 81 9 81

58. The first, second and seventh term of A.P. are in G.P. and the common difference is 2, the
2nd term of A.P. is:
(a) 5/2 (b) 2
(c) 3/2 (d) 1/2

59. If a, b, c are in A.P. and x, y, z are in G.P. then the value of x (b-c) .y(c-a). z(a-b) is:
(a) 1 (b) 0
(c) b(c - a) (d) None

1 1 1 1
60. If x = 1    .......... ... y = 1   2  .......... ... Find xy.
3 32 4 4
(a) 2 (b) 1
(c) 8/9 (d) ½

61. Find three numbers in G.P. such that their sum is 21, and the sum of their squares is 189
(a) 5, 7, 9 (b) 3, 7, 1
(c) 3,6,12 (d) 4, 8, 9

62. The sum of how many terms of the sequence 256, 128, 64, .......... is 511.
(a) 8 (b) 9.
(c) 7 (d) None of these.

63. Find two numbers whose A.M is 10 and G.M. is 8.


(a) [10, 10] (b) [16, 4]
(c) [18, 2] (d) [14, 6]

1 1
64. The sum of the infinite GP 1    ......... is equal to
3 9
(a) 1.95 (b) 1.5
(c) 1.75 (d) None

121 | P a g e
65. Find the sum to infinity of the following series:
1-1+1-1+1-1+ ................ 
(a) 1 (b) 
(c) 1/2 (d) can not determine

66. If the numbers x,y,z are in G.P then the numbers x 2+y2, xy+yz, y2+z2 are in ___
(a) A.P (b) G.P
(c) H.P (d) None

67. If G be geometric means between two number a and b, then the value of
1 1
+
G A
2 2
G  B2 .
2

(a) G2 (b) 3G2


(c) 1/G2 (d) 2/G2

 243 ,  243 ,  243


1/ 6 1/36
68. Find the product of: ,.......
(a) 1,024 (b) 27
(c) 729 (d) 246

69. Geometric Mean of P,p2 ,P3............., P n will be:


n 1
n 1 2
(a) P (b) P
n  n 1

(c) P 2
(d) None of the above.

70. If A.M = 12.5, G.M = 10 then the numbers are :


(a) 20,5 (b) 10,5
(c) 5,4 (d) None

4 8 16
71. Find the sum of the series 2, , , , ...........∞ y > 2 is
Y Y 2 y3
2y 4y
(a) (b)
y2 3y  2
3y
(c) (d) None of these
y2

72. The sum of the series 1-1+1-1+1-1+ ...... to 100 terms is equal to :
(a) 1 (b) –1
(c) 0 (d) 50

73. In a G.P. the 6th term is 729 and the common ratio is 3 then 1st term is ________ :
(a) 2 (b) 3
(c) 4 (d) 7

122 | P a g e
74. The value of 13+23+33+…………+m3 is equal to
 m(m  1)  m(m  1)( 2m  1)
3

(a)   (b)
 2  6
 m(m  1) 
2

(c)   (d) None


 2 

ANSWER

1. 2. 3. 4. 5. 6. 7. 8. 9. 10.
(A) (B) (C) (C) (A) (B) (C) (A) (C) (C)
11. 12. 13. 14. 15. 16. 17. 18. 19. 20.
(A) (C) (C) (A) (A) (A) (B) (B) (A) (B)
21. 22. 23. 24. 25. 26. 27. 28. 29. 30.
(C) (A) (B) (A) (B) (B) (D) (C) (D) (D)
31. 32. 33. 34. 35. 36. 37. 38. 39. 40.
(A) (C) (A) (B) (A) (D) (C) (B) (C) (A)
41. 42. 43. 44. 45. 46. 47. 48. 49. 50.
(A) (B) (A) (B) (C) (D) (C) (A) (C) (A)
51. 52. 53. 54. 55. 56. 57. 58. 59. 60.
(C) (A) (A) (C) (B) (A) (B) (A) (A) (A)
61. 62. 63. 64. 65. 66. 67. 68. 69. 70.
(C) (B) (B) (B) (D) (B) (C) (C) (B) (A)
71. 72. 73. 74.
(A) (C) (B) (C)

123 | P a g e
CHAPTER – 7 - SETS, RELATIONS AND FUNCTIONS
PRACTICE QUESTIONS

1. The number of subsets of the set {2, 3, 5} is


(a) 3 (b) 8
(c) 6 (d) none of these,

2. The number of subsets of a set containing n elements is


(a) 2n (b) 2–n
(c) n (d) none of these
3. The null set is represented by
(a) {} (b) {0}
(c)  (d) none of these

4. A = {2, 3, 5, 7} , B= { 4, 6, 8, 10} then A  B can be written as


(a) {} (b) { }
(c) (AUB)' (d) None of these

5. The set {x|0<x<5} represents the set when x may take integral values only
(a) {0,1, 2, 3, 4, 5 } (b) {1, 2, 3, 4 }
(c) {1,2, 3, 4, 5 } (d) none of these

6. The set {0, 2, 4, 6, 8, 10} can be written as


(a) {2x : 0<x <5} (b) {x : 0<x<5}
(c) {2x : 0<x<5} (d) none of these

Q.7 to Q10 If P = {1, 2, 3, 5, 7}, Q = {1, 3, 6, 10, 15},Universal Set S = {1, 2, 3, 4, 5, 6, 7,


8, 9, 10, 11, 12, 13, 14, 15}

7. The cardinal number of P  Q is


(a) 3 (b) 2
(c) 0 (d) none of these

8. The cardinal number of P  Q is


(a) 10 (b) 9
(c) 8 (d) none of these

9. n(P-Q) is
(a) 10 (b) 5
(c) 6 (d) 3

10. n(Q-P) is
(a) 3 (b) 10
(c) 4 (d) none of these

The set of cubes of the natural number is


11.
(a) a finite set (b) an infinite set
(c) a null set (d) none of these

124 | P a g e
12. The set {2x|x is any positive rational number } is
(a) an infinite set (b) a null set
(c) a finite set, (d) none of these

13. {1– (–1)x} for all integral x is the set


(a) {0} (b) {2}
(c) {0,2} (d) none of these

14. E is a set of positive even number and O is a set of positive odd numbers, then E  O is a
(a) set of whole numbers (b) N
(c) a set of rational number (d) none of these

15. If R is the set of positive rational number and E is the set of real numbers then
(a) R=E (b) R E
(c) E R (d) none of these

16. If N is the set of natural numbers and I is the set of positive integers, then
(a) N=I (b) N I
(c) IN (d) none of these

17. If I is the set of isosceles triangles and E is the set of equilateral triangles, then
(a) IE (b) E I
(c) E=I (d) none of these

18. If R is the set of isosceles right angled triangles and I is set of isosceles triangles, then
(a) R=I (b) R I
(c) RI (d) none of these

19. {n(n+1)/2 : n is a positive integer} is


(a) a finite set (b) an infinite set
(c) is an empty set (d) none of these

20. If A = {1, 2, 3, 5, 7}, and B = {x2 : x  A}


(a) n(A)>n(B) (b) n(B) > n(A)
(c) n(A)= n(B) (D) None of these

21. A  A is equal to
(a) A (b) E
(c)  (d) none of these

22. A  A is equal to
(a)  (b) A
(c) E (d) none of these

23. (A  B)' is equal to


(a) (A  B)' (b) A  B'
(c) A'  B' (d) none of these

24. (A  B)' is equal to


(a) (A'  B)' (b) A'  B'
(c) A'  B' (d) none of these

125 | P a g e
25. A  E is equal to (E is a superset of A)(Q.25 to Q.30)
(a) A (b) E
(c)  (d) none of these

26. A  E is equal to
(a) A (b) E
(c)  (d) none of these

27. E  E is equal to
(a) E (b) 
(c) 2E (d) none of these

28. A  E’ is equal to
(a) E (b) 
(c) A (d) none of these

29. A  A equal to
(a) A (b) E
(c)  (d) none of these

30. A  A' is equal to


(a) E (b) 
(c) A (d) none of these

31. If E = {1, 2, 3, 4, 5, 6, 7, 8, 9}, the subset of E satisfying 5 + x > 10 is


(a) {5,6,7,8,9} (b) {6, 7, 8, 9}
(c) {7, 8, 9} (d) none of these

32. If A  B = (A–B)  (B–A) and A = {1, 2, 3, 4}, B = {3,5,7} than A  B is


(a) {1,2,4,5,7} (b) {3}
(c) {1,2,3,4,5,7} (d) none of these

33. Given A = {2, 3}, B = {4, 5}, C = {5, 6} then A × (B  C) is


(a) {(2,5),(3,5)} (b) {(5, 2), (5, 3)}
(c) {(2, 3), (5, 5)} (d) none of these

34. If A = { 1, 2, 3, 5, 7} and B = {1, 3, 6, 10, 15}. Cardinal number of A-B is


(a) 3 (b) 4
(c) 6 (d) none of these

35. If A has 32 elements, B has 42 elements and A  B has 62 elements, the number of elements
in A  B is
(a) 12 (b) 74
(c) 10 (d) none of these

36. Let A = {a, b}. Set of subsets of A is called power set of A denoted by P(A). n[P(A)] is equal
to
(a) 2 (b) 4
(c) 3 (d) none of these

126 | P a g e
37. If the set P has 3 elements, Q has 4 elements and R has 2 elements then the set P×Q×R
contains
(a) 9 elements (b) 20 elements
(c) 24 elements (d) none of these

38. In a group of 20 children, 8 drink tea but not coffee and 13 like tea. The number of children
drinking coffee but not tea is
(a) 6 (b) 7
(c) 1 (d) none of these

39 In a class of 30 students, 20 students like maths, 18 like science and 12 like both the
subjects.
Find the number of students who like no subject.
(a) 4 (b) 5
(c) 8 (d) None of these

40. A town has a total population of 50,000. Out of it 28,000 read the newspaper X and 23,000
read Y while 4,000 read both the papers. The number of persons not reading X and Y both is
(a) 2,000 (b) 3,000
(c) 2,500 (d) none of these

41. At a certain conference of 100 people there are 29 Indian women and 23 Indian men. Out of
these Indian people 4 are doctors and 24 are either men or doctors. There are no foreign
doctors. The number of women doctors attending the conference is
(a) 2 (b) 4
(c) 1 (d) none of these

42. Out of 2000 employees in an office 48% preferred Coffee (c), 54% liked (T), 64% used to
smoke (S). Out of the total 28% used C and T, 32% used T and S and 30% preferred C and
S, only 6% did none of these. The number having all the three is
(a) 360 (b) 300
(c) 380 (d) none of these

43. Referred to the data of Q. 42 the number of employees having T and S but not C is
(a) 200 (b) 280
(c) 300 (d) none of these

44. Referred to the data of Q. 42. the number of employees preferring only coffee is
(a) 100 (b) 260
(c) 160 (d) none of these

45. A survey shows that 68% of women like apples, 74% of women like orange. What
percentage like both
(a) 12% (b) 6%
(c) 21% (d) 42%

46. In a group of 40 children 16 like cricket but not movie and 26 like cricket. The number of
children like movie but not cricket are :
(a) 12 (b) 14
(c) 2 (d) None of these

127 | P a g e
47. Complaints about works canteen had been about Mess (M), Food (F) and Service (S). Total
complaints 173 were received as follows:-
n(M) = 110, n(F)=55, n(S)=67, n(M ∩ F ∩ S′)=20, n(M ∩ S ∩ F′)=11
and n(F ∩ S ∩ M′) = 16. Determine the complaints about all the three.
(a) 6 (b) 53
(c) 35 (d) None

48. As per question No. (47) determine the complaints about two or more than two.
(a) 6 (b) 53
(c) 35 (d) None

49. After qualifying out of 400 professionals, 112 joined service, 120 strarted practice and 160
joined assistantship. There were 32, who were in both practice and service, 40 in both
practice and assistantship and 20 in both service and assistantship. There were 12 who did all
the three. Find how many could not get any of these.
(a) 88 (b) 244
(c) 122 (d) None

50. Identify the elements of P if set set Q = {1, 2, 3} and PxQ =


{(4, 1) (4, 2) (4, 3) (5, 1) (5, 2)(5, 3)(6, 1)(6, 2)(6, 3)}
(a) {3, 4, 5} (b) {4, 5, 6}
(c) {5, 6, 7} (d) None

51. if A = {2, 3}, B = {4, 5}, C = {5, 6} then A x (B ∪ C) is


(a) {(2, 4) (2, 5) (2, 6) (3, 4) (3, 5)(3, 6)} (b) {(2, 5) (3, 5)}
(c) {(2, 4) (2, 5) (3, 4) (3, 5) (4, 5)(4, 6)(5, 5)(5, 6)} (d) None

52. The sets V =  x | x + 2 = 0 ,R = x | x 2


  
+ 2x = 0 and S= x : x 2 + x - 2 = 0 are equal to one
another if x is equal to
(a) -2 (b) 2
(c) 1/2 (d) None

53. U = { x / x is a positive integer less than 15}. A = {2, 8, 10, 14}, B = {3, 6, 8, 14} then
(a) (A  B)' = A'  B' (b) (A  B)' = A'  B'
(c) (A'  B)' =  (d) (A  B')' = 

PAST EXAMINATION QUESTIONS


54. If A = {p,q,r,s}
B = {q,s,t}
C = {m, q, n}
Find C - (A  B)
(a) {m,n} (b) {p, q}
(c) {r,s) (d) {p, r}

55. If A={x: x2-3x+2=0},


B={x: x2+4x-12=0}, then
B-A is equal to
(a) {-6} (b) {1}
(c) {1,2} (d) {2, -6}
128 | P a g e
56. For any two sets A and B, A  ( A  B) = _________, where A' represent the compliment of
the set A .
(a) A B (b) A B
(c) A'  B (d) None of these

57. If A  B, then which one of the following is true :


(a) A B = B (b) A B = B
(c) A  B = A1 (d) A B = 

58. There are 40 students, 30 of them passed in English, 25 of them passed in Maths and 15 of
them passed in both. Assuming that every student has passed at least in one subject. How
many student’s passed in English only but not in Math’s.
(a) 15 (b) 20
(c) 10 (d) 25

59. If A  1, 2,3, 4,5 B  2, 4 C  1,3,5 then  A  C  B


(a) {(4,4), (2,2)} (b) 2,2 ,2,4 ,4,2 ,4,4 
(c) 1,2 (d) 2,4
60.  A  B'' is equal to (where ’ denotes compliment of the set)
(a) B–A (b) A–B
(c) B’ – A’ (d) None of these

61. The number of proper sub sets of the set {3, 4, 5, 6, 7} is


(a) 32 (b) 31
(c) 30 (d) 25

62. Given A = {2, 3}, B = {4, 5}, C = {5, 6} then A × (B  C) is :


(a) {(2, 5), (3, 5)} (b) {(5, 2), (5, 3)}
(c) {(2, 3), (5, 5)} (d) None of these

63. Out of 20 members in a family, 11 like to take tea and 14 like coffee. Assume that each one
likes at least one of the two drinks. Find how many like both coffee and tea
(a) 2 (b) 3
(c) 4 (d) 5

64. In a group of 70 people, 45 speak Hindi and 33 speak English 10 speak neither Hindi nor
English. Find how many can speak both English as well as Hindi:
(a) 13 (b) 19
(c) 18 (d) 28

65. In a survey of 300 companies, the number of companies using different media - newspapers
(N), radio (R) and Television (T) are as follows:
n(N)=200, n(R) = 100, n(T) = 40, n(N  R) = 50, n(R  T) = 20, n(N  T)=25 and n(N  R  T) =
5.
Find the numbers of companies using none of these media:
(a) 20 companies (b) 250 companies
(c) 30 companies (d) 50 companies.

129 | P a g e
66. In a town of 20,000 families it was found that 40°/o families buy newspaper A, 20% families
buy newspaper B and 10% families buy newspaper C, 5% families buy A and B, 3% buy B
and C and 4% buy A and C. If 2% families buy all the three newspapers, then the number of
families which buy A only is:
(a) 6600 (b) 6300
(c) 5600 (d) 600.

67. For a group of 200 persons, 100 are interested in music, 70 in photography and 40 in
swimming, Further more 40 are interested in both music and photography, 30 in both music
and swimming, 20 in photography and swimming and 10 in all the three. How many are
interested in photography but not in music and swimming?
(a) 30 (b) 15
(c) 25 (d) 20

68. Of the 200 candidates who were interviewed for a position at call center, 100 had a two
wheeler, 70 had a credit card and 140 had a mobile phone. 40 of them had both a two
wheeler and a credit card, 30 had both a credit card and mobile phone, 60 had both a two
wheeler and a mobile phone and 10 had all the three. How many candidates had none of
them?
(a) 0 (b) 20
(c) 10 (d) 18

69. In a class of 50 students, 35 opted for maths, 37 opted for commerce. The number of such
students who opted for both maths and commerce is
(a) 13 (b) 15
(c) 22 (d) 28

70. In a class of 80 students, 35% students can play only cricket, 45% students can play only
table tennis and the remaining students can play both the games. In all how many students
can play cricket?
(a) 55 (b) 44
(c) 36 (d) 28

ANSWER

1. 2. 3. 4. 5. 6. 7. 8. 9. 10.
(B) (A) (C) (A) (B) (C) (B) (C) (D) (A)
11. 12. 13. 14. 15. 16. 17. 18. 19. 20.
(B) (A) (C) (B) (B) (A) (B) (C) (B) (C)
21. 22. 23. 24. 25. 26. 27. 28. 29. 30.
(A) (B) (C) (B) (B) (A) (A) (B) (A) (B)
31. 32. 33. 34. 35. 36. 37. 38. 39. 40.
(B) (A) (A) (A) (A) (B) (C) (B) (A) (B)
41. 42. 43. 44. 45. 46. 47. 48. 49. 50.
(C) (A) (B) (C) (D) (B) (A) (B) (A) (B)
51. 52. 53. 54. 55. 56. 57. 58. 59. 60.
(A) (A) (A) (A) (A) (A) (B) (A) (B) (A)
61. 62. 63. 64. 65. 66. 67. 68. 69. 70.
(B) (A) (D) (C) (D) (A) (D) (C) (C) (B)

130 | P a g e
CHAPTER – 7 - RELATIONS & FUNCTIONS
PRACTICE QUESTIONS

1. If A = {x, y, z}, B = {p, q, r, s} Which of the relation on A to B are function.


(a) {n, p), (x, q), (y, r), (z, s)}, (b) {( x, s), (y, s), (z, s)}
(c) {(y, p), (y, q), (y, r),(z, s), (d) {(x, p), (y, r), (z, s)}

2. {(x , y), y=x2} where x, y ∈ R is


(a) not a function (b) a function
(c) inverse mapping (d) none of these

3. {(x, y)|x<y} where x, y ∈ R is


(a) not a function (b) a function
(c) one-one mapping (d) none of these

4. The domain of {(1,7), (2,6)} is


(a) (1, 6) (b) (7, 6)
(c) (1, 2) (d) {6, 7}

5. The range of {(3,0), (2,0), (1,0), (0,0)} is


(a) {0, 0} (b) {0}
(c) {0, 0, 0, 0} (d) none of these

6. The domain and range of {(x,y) : y = x2 } where x, y ∈ R is


(a) (reals, natural numbers) (b) (reals, positive reals)
(c) (reals, reals) (d) none of these

7. If f(x) = 1/1–x, f(–1) is


(a) 0 (b) ½
(c) 2 (d) none of these

8. If g(x) = (x–1)/x, g(–½) is


(a) 1 (b) 2
(c) 3/2 (d) 3

9. If f(x) = 1/1–x and g(x) = (x–1)/x, than fog(x) is


(a) x (b) 1/x
(c) –x (d) none of these

10. If f(x) = 1/1–x and g(x) = (x–1)/x, then g of(x) is


(a) x–1 (b) x
(c) 1/x (d) none of these

11. The function f(x) = 2x is


(a) one-one mapping (b) one-many
(c) many-one (d) none of these

131 | P a g e
12. The range of the function f(x) = log10(1 + x) for the domain of real values of x when 0  x  9 is
(a) [0,3] (b) [0,2]
(c) [0,1] (d) none of these

13. The Inverse function f–1 of f(x) = 2x is


x
(a) 1/2x (b)
2
(c) 1/x (d) none of these

14. If f(x) = x+3, g(x) = x2, then fog(x) is


(a) x2 + 3 (b) x2 + x +3
(c) (x+3)2 (d) none of these

15. If f(x) = x+3, g(x) = x2 , then f(x).g(x) is


(a) (x+3)2 (b) x2+3
(c) x3+3x2 (d) none of these

16. The Inverse h –1(x) when h(x) = log10x is


(a) log10 x (b) 10x
(c) log10(1/x) (d) none of these

17. For the function h(x) = 101+x the domain of real values of x where 0 < x < 9, the range is
(a) 10 < h(x) < 1010 (b) 0 < h(x) < 1010
(c) 0 < h(x) < 10 (d) none of these

18. "Is smaller than" over the set of eggs in a box is


(a) Transitive (T) (b) Symmetric (S)
(c) Reflexive (R) (d) Equivalence (E)

19. "Is equal to" over the set of all rational numbers is
(a) T (b) S
(c) R (d) E

20. "is perpendicular to" over the set of straight lines in a given plane is
(a) R (b) S
(c) T (d) E

21. “has the same father as”........... over the set of children.
(a) R (b) S
(c) T (d) E

22. “is the reciprocal of” ......... over the set of non-zero real numbers is
(a) S (b) R
(c) T (d) E

23. “is the square of” over set of real numbers is


(a) S (b) R
(c) T (d) None of these

132 | P a g e
24. “Is greater than” over the set of all natural number if known as
(a) Transitive (b) Symmetric
(c) Reflexive (d) Equivalence

25. Find fog for the functions f (x) = x , g (x) = 2x 2 + 1


(a) 2x2 + 1 (b) 2x + 1
(c) (2x2 + 1) ( x ) (d) x

26. Find gof for the functions f (x) = x , g (x) = 2x 2 + 1


(a) 2x2 + 1 (b) 2x + 1
2
(c) (2x + 1) ( x ) (d) x

27. If f(x) = x2+ 3x then f(2) – f(4) is equal to


(a) –15 (b) –18
(c) 18 (d) 12

28. If f (x) = 2x+5 and g(x) = x2+1, then the fog(2) is


(a) 15 (b) 16
(c) 18 (d) None of these

29. If f(x+1) = 2x + 7 then f(0) is equal to


(a) 5 (b) 4
(c) 3 (d) 0

30. 𝟏
If f (x) = x3 + , then value of f(x) – f (1/x) is equal to
𝒙𝟑
(a) 0 (b) 1
𝟏
(c) x3 + (d) None of these
𝒙𝟑

𝐱
31. If a function in x is defined by f(x) = ,x ∈ R then f(1/x) =____________
𝐱𝟐 +𝟏
(a) f(x) (b) f(-x)
(c) -f(x) (d) 0

32. If f(x) = (x+1) and g(x) = 3x2-2, find the value of gof(x) =?
(a) 3x2+6x+1 (b) 2x2-6x+3
2
(c) 3x -5 (d) x-5

33. Given the function f(x)= x  5, f ( 5 ) is equal to


2

(a) 0 (b) 5
(c) 10 (d) none of these

5x  1
34. If f(x)= then f(x) is
5x 1
(a) an even function (b) an odd function
(c) a composite (d) none of these

133 | P a g e
2
35. If g(x) =3−x then g(x) is
(a) an odd function (b) a periodic function
(c) an even function (d) none of these

q  ( x  p) p  ( x  q)
36. If f(x) =  then f(p) + f(q) is equal to
(q  p) ( p  q)
(a) f(p+q) (b) f(pq)
(c) f(p−q) (d) none of these

37. If f(x)=2x −5x+4 then 2f(x)=f(2x) for


2

(a) x=1 (b) x = −1


(c) x = 1 (d) none of these

38. If f (x) =logx (x>0) then f(p)+f(q)+ f(r) is


(a) f(pqr) (b) f(p) f(q)f(r)
(c) f(1/pqr) (d) none of these

f (4  h)  f (4)
39. If f(x) = 2x −5x+2 then the value of is
2
h
(a) 11−2h (b) 11+2h
(c) 2h−11 (d) none of these

px  q
40. If y = h (x) = then x is equal to
qx  p
(a) h(1/y) (b) h(−y)
(c) h(y) (d) none of these

2
41. If f(x) = x −x then f (h+1) is equal to
(a) f(h) (b) f(−h)
(c) f(−h+1) (d) none of these

1 x
42. If f(x)= then f [f(1/x)] is equal to
1 x
(a) 1/x (b) x
(c) −1/x (d) none of these

PAST EXAMINATION QUESTIONS

43. On the set of lines in a plane the Relation “ is perpendicular to” is _______
(a) Reflexive (b) Symmetric
(c) Transitive (d) None of these

44. The range of the relation {(1, 0) (2, 0) (3, 0) (4, 0) (0, 0)} is :
(a) {1, 2, 3, 4, 0} (b) {0}
(c) {1, 2, 3,4} (d) None

134 | P a g e
45. If A = {1, 2, 3} and B = {2,4,6} then the relation R = {(1,2) (2,4) (3,6)} is :
(a) A function from A to B (b) A function from B to A
(c) Both (a) and (b) (d) Not a function

46. Let R is the set of real numbers, such that the function f: R  R and g : R  Rare defined by
f(x)= x2 + 3x +1 and g(x)= 2x - 3. Find fog(x) :
(a) 4x2+6x+1 (b) x2 +6x+1
2
(c) 4x - 6x+1 (d) x2 – 6x+1.

47. If R is the set of real numbers such that the function f: R  R is defined by f(x) = (x + 1)2
then find fof(x) :
(a) (x+l)2 +1 (b) x2 +1
(c) {(x+1) +1 
2 2
(d) None

48. If f: R  R, f(x) = 2x + 7 then the inverse of f is:


(a) f -1(x) = (x – 7)/2 (b) f -1(x) = (x +7)/2
(c) f -1 (x) =(x -3)1/2 (d) None

49. Let f: R  R be such that f(x) = 2x then f(x + y) equals:


(a) f(x)+f(y) (b) f(x) , f(y)
(c) f(x)  f(y) (d) None of these

2 x -1
50. If f(x) = , then f (x) is :
2 x
2( x  1) 2( x  1)
(a) (b)
( x  1) ( x  1)
( x  1) ( x  1)
(c) (d)
( x  1) ( x  1)

51. If A= {l,2, 3,4}


B= {2,4, 6, 8,}
f(1)=2, f(2) = 4, f(3) = 6 and
f(4) = 8, And f : A  B then f-1 is:
(a) {(2,1), (4, 2), (6, 3), (8, 4)} (b) {(1, 2), (2, 4), (3, 6), (4, 8)}
(c) {(1, 4), (2, 2), (3, 6), (4, 8)} (d) None of these

52. If f(x)=x2 + x- 1 and 4f (x)=f (2x) then find ‘x’.


(a) 4/3 (b) 3/2
(c) -3/4 (d) None of these

53. X = {x,y,w,z} ,y = {1,2,3,4}


H = {(x, 1), (y, 2), (y, 3), (z, 4), (x, 4)}
(a) H is a function from X to Y (b) H is not a function from X to Y
(c) H is a relation from Y to X (d) None of the above

54. Given the function f(x) = (2x + 3) then the value of f(2x) -2 f(x) +3 will be:
(a) 3 (b) 2
(c) 1 (d) 0

135 | P a g e
55. If f(x) = 2x + h then find f(x + h) - 2f(x) .
(a) h-2x (b) 2x-h
(c) 2x+h (d) None of these

1
56. If F: A  R is a real valued functioned defined by f(x) = , then A = .
x
(a) R (b) R-{1}
(c) R- {0} (d) R-N

57. If F : R  R, f(x) = x+1,


G : R  R g(x) = x2+1
then fog(-2) equals to
(a) 6 (b) 5
(c) -2 (d) None

58. If f(x-1) = x2-4x+8, then f(x+1) =


(a) x2+8 (b) x2+7
(c) x2+4 (d) x2-4.
x x
59. If f x   and g  x   fog(x) =
1  x2 1  x2
1
(a) x (b)
x
x
(c) (d) x 1  x2
1x 2

60. The range of the function f :N R defined by f(x) = (−1)x−1 is


(a) {0 , -1} (b) { 1 , -1}
(c) {1 , 0} (d) {1, 0, -1}

61. If f :R  R is a function defined by f(x) = 10x –7. If g(x) = f-1(x) then g(x) is equal to
1 1
(a) (b)
10 X  7 10 X  7
X 7 X 7
(c) (d)
10 10

62. The number of elements in range of constant function


(a) one (b) zero
(c) infinite (d) indetermined

63. If f(x) = x+2, g(x) = 7x then gof (x) =


(a) 7x.x+2.7x (b) 7x+2
(c) 49(7x) (d) none

136 | P a g e
 2x 
64. If f(x) = log  1  x  then f  2 =
1  x  1 x 
(a) f(x) (b) 2 f(x)
(c) 3f(x) (d) -f(x)

1
65. f(x) = (a  x ) , a>0 and n is positive integer then f[f(x)] =
n n

(a) x (b) a
1 1
(c) x n
(d) an

x
66. f(x)= , then f(x / y)
x 1 f(y / x)
x y
(a) (b)
y x
x
(c)  (d) 
y
y x

67. If N is the set of all natural numbers, E is the set of all even natural numbers and f: N E
defined by f(x)=2x x N then f is
(a) One–one, into (b) One-one, onto
(c) Many to one (d) One to many

68. If A = {1, 2, 3} then the relation R = {(1, 1), (2, 3), (2, 2), (3, 3), (1, 2)} on A is :
(a) Reflexive (b) Symmetric
(c) Transitive (d) Equi-valence

69. If a relation R = {(1,1), (2,2), (1,2), (2,1)} is symmetric on A = {1,2,3} then R is


(a) Reflexive but not Transitive (b) Transitive but not Reflexive
(c) Reflexive and Transitive (d) Neither Reflexive nor Transitive

70. If A={x,y,z}, B={a,b,c,d}, then which of the following relation from the set A to set B is a
function?
(a) {(x,a), (x,b), (y,c), (z,d)} (b) {(x,a), (y,b), (z,d)}
(c) {(x,c), (z,b), (z,c)} (d) {(a,z), (b,y),(c,z),(d,x)}

2
71. If f(x)  2x  2 and g(x)  x , then the value of fog (4) is:
(a) 18 (b) 22
(c) 34 (d) 128

137 | P a g e
ANSWER

1. 2. 3. 4. 5. 6. 7. 8. 9. 10.
(B,D) (B) (A) (C) (B) (B) (B) (D) (A) (B)
11. 12. 13. 14. 15. 16. 17. 18. 19. 20.
(A) (C) (B) (A) (C) (B) (A) (A) (D) (B)
21. 22. 23. 24. 25. 26. 27. 28. 29. 30.
(D) (A) (D) (A) (A) (A) (B) (A) (A) (A)
31. 32. 33. 34. 35. 36. 37. 38. 39. 40.
(A) (A) (A) (B) (C) (A) (C) (A) (B) (C)
41. 42. 43. 44. 45. 46. 47. 48. 49. 50.
(B) (A) (B) (B) (A) (C) (C) (A) (B) (A)
51. 52. 53. 54. 55. 56. 57. 58. 59. 60.
(A) (B) (B) (D) (A) (C) (A) (C) (A) (B)
61. 62. 63. 64. 65. 66. 67. 68. 69. 70.
(C) (A) (C) (B) (A) (C) (B) (A) (B) (B)
71.
(C)

138 | P a g e
CHAPTER – 8 - DIFFERENTIATION
PRACTICE QUESTIONS

1. The gradient of the curve y=2x3-3x2-12x+8 at x = 0 is


(a) -12 (b) 12
(c) 0 (d) none of these

2. The gradient of the curve y=2x3-5x2-3x at x = 0 is


(a) 3 (b) -3
(c) 1/3 (d) none of these

3. The derivative of y = √x + 1 is
(a) 1/√x + 1 (b) −1/√x + 1
(c) 1/2 √x + 1 (d) none of these

x2 +1
4. If f(x) = x2−1 then f’(x) is
(a) −4x/(x 2 − 1)2 (b) 4x/(x 2 − 1)2
(c) x/(x 2 − 1)2 (d) none of these

dy
5. If y = x(x − 1) (x − 2) then dx
is
(a) 2
3x − 6x + 2 (b) −6x + 2
(c) 3x 2 + 2 (d) none of these

1 dy
6. If y = then is equal to
√x dx
1 −1
(a) 2x√x
(b) x√x
1
(c) − 2x√x (d) none of these

7. The derivative of x 2 log x is


(a) 1 + 2 log x (b) x(1 + 2 log x)
(c) 2 log x (d) none of these

3−5x
8. The derivative of 3+5x
is
(a) 30/(3 + 5x) 2
(b) 1/(3 + 5x)2
(c) −30/(3 + 5x)2 (d) none of these

ex +1 dy
9. If y = ex−1 then dx
is equal to
−2ex 2ex
(a) (ex −1)2
(b) (ex −1)2
−2
(c) (ex −1)2
(d) none of these

1 2
10. Let f(x) = (√x + ) then f ′ (2) is equal to
√x
(a) 3/4 (b) 1/2
(c) 0 (d) none of these

139 | P a g e
11. If f(x) = x 2 − 6x + 8 then f ′ (5) − f ′ (8) is equal to
(a) f ′ (2) (b) 3f ′ (2)
′ (2)
(c) 2f (d) none of these

12. If f(x) = x k and f ′ (1) = 10 then value of k is


(a) 10 (b) −10
(c) 1/10 (d) none of these

13. The gradient of the curve y = 4x2-2x at x = 1 is


(a) 4 (b) 6
(c) 8 (d) none of these

14. The derivative of the function (x 2 + 1)/x is


(a) 1 + 1/x 2 (b) 1 − 1/x 2
(c) 1/x 2 (d) none of these
2
15. If f(x) = eax +bx+c the f ′ (x) is
2 2
(a) eax +bx+c (b) eax +bx+c (2ax + b)
(c) 2ax + b (d) none of these

16. The derivative of the function √x + √x is


1 1
(a) (b) 1 + 2√x
2√x+√x
1 1
(c) (1 + ) (d) none of these
2√x+ √x 2√x

17. If xy = 1 then y 2 + dy/dx is equal to


(a) 1 (b) 0
(c) −1 (d) none of these

dy
18. Given x = at 2 , y = 2at, is calculated as
dx
(a) t (b) −1/t
(c) 1/t (d) none of these
dy
19. Given x = 2t + 5, y = t 2 − 2, is calculated as
dx
(a) t (b) −1/t
(c) 1/t (d) none of these

dy
20. If x = 3t 2 − 1, y = t 3 − t, then is equal to
dx
3t2 −1
(a) (b) 3t 2 − 1
6t
3t−1
(c) 6t
(d) none of these
dy
21. Given x = t + t −1 and y = t − t −1 the value of at t = 2 is
dx
(a) 3/5 (b) −3/5
(c) 5/3 (d) none of these

dy
22. Let y = √2x + 32x then dx
is equal to
(a) (1/√2x) + 2. 3 log. 3 2x
(b) 1/√2x
(c) 2. 32x log. 3 (d) none of these

140 | P a g e
2
23. The derivative of e3x −6x+2 is
(a) 30(1 − 5x)5 (b) (1 − 5x)5
2
(c) 6(x − 1)e3x −6x+2 (d) none of these

dy
24. If x = at 2, y = 2at then [dx] is equal to
t=2
(a) 1/2 (b) −2
(c) −1/2 (d) none of these

25. If y = √x 2 + m2 then y y1 (where y1 = dy/dx) is equal to


(a) −x (b) x
(c) 1/x (d) none of these

dy
26. If y = ex + e−x then dx
− √y 2 − 4 is equal to
(a) 1 (b) −1
(c) 0 (d) none of these

dy
27. If y = e√2x then is equal to ___________.
dx
e√2x
(a) (b) e√2x
√2x
e√2x
(c) (d) none of these
√2x

28. If x = (1 − t 2 )/(1 + t 2 ), y = 2t/(1 + t 2 ) then dy/dx at t = 1 is __________.


(a) 1/2 (b) 1
(c) 0 (d) none of these

29. f(x) = x 2 /ex then f ′ (1) is equal to ____________.


(a) −1/e (b) 1/e
(c) e (d) none of these

dy
30. The curve y 2 = ux 3 + v passes through the point P(2, 3) and dx
= 4 at P. The value of u and v
are.
(a) (u = 2, v = 7) (b) u = 2, v = −7
(c) u = −2, v = −7 (d) (0, −)

dy
31. Given e−xy − 4xy = 0, dx
can be proved to be
(a) −y/x (b) y/x
(c) x/y (d) none of these

x2 y2 dy
32. If a2
− a2 = 1, dx
can be expressed as
x x
(a) (b)
a √x2 −a2
1
(c) (d) none of these
2
√x2 −1
a

141 | P a g e
dy
33. If log(x/y) = x + y, may be found to be
dx
y(1−x) y
(a) x(1+y)
(b) x
1−x
(c) (d) none of these
1+y

dy
34. If f(x, y) = x 3 + y 3 − 3axy = 0, dx can be found out as
ay−x2 ay−x2
(a) y2 +ax
(b) y2 −ax
ay+x2
(c) y2 +ax
(d) none of these

35. The slope of the tangent to the curve y = √4 − x 2 at the point, where the ordinate and the
abscissa are equal, is
(a) −1 (b) 1
(c) 0 (d) none of these

36. The slope of the tangent to the curve y = x 2 − x at the point, where the line y = 2 cuts the
curve in the 1st quadrant, is
(a) 2 (b) 3
(c) −3 (d) none of these
dy
37. For the curve x 2 + y 2 + 2gx + 2hy = 0, the value of at (0, 0) is
dx
g g
(a)  (b)
h h
h
(c) (d) none of these
g
e3x −e2x dy
38. If y = e3x+e2x , then is equal to
dx
(a) 2e 5x
(b) 1/(e5x + e2x )2
2e5 x
(c) e5x /(e5x + e2x ) (d)
(e 5 x  e 2 x ) 2
dy
39. If x 3 − 2x 2 y 2 + 5x + y − 5 = 0 then dx
at x = 1, y = 1 is equal to
(a) 4/3 (b) −4/3
(c) 3/4 (d) none of these
n
40. If y = (x + √x 2 + m2 ) then dy/dx is equal to
(a) ny (b) ny/√x 2 + m2
(c) −ny/√x2 + m2 (d) none of these

41. If y = √x/m + √m/x then 2xy dy/dx − x/m + m/x is equal to


(a) 0 (b) 1
(c) −1 (d) none of these
x2 x3 xn dy
42. If y = 1 + x + 2! + 3!
+.........+ n! then dx
− y is proved to be
(a) 1 (b) −1
(c) 0 (d) none of these

142 | P a g e
√x……∞ dy
43. If y = √x then is equal to __________.
dx
y2 y2
(a) (b)
2−y log x x(2−y log x)
y2
(c) (d) none of these
log x

2
 dy 
44. If y  ( x  x  1) then (x -1)    m y is proved to be:-
2
2 m 2 2

 
dx
(a) -1 (b) 1
(c) 0 (d) none of these

dy
45. If x y . y x = M, Where M is constant then dx
is equal to
−y −y(y+x log y)
(a) (b)
x x(y log x+x)
y+x log y
© (d) none of these
y log x+x

x−2 ¾
46. The derivative of log [ex {x+2} ] is
x2 +1 x2 −1
(a) x2 +4
(b) x2 −4
1
(c) (d) none of these
x2 −4

(a+x) a+1+2x
47. If (x) = {(1+x)} the value of f ′ (0) is
1−a2
(a) aa+1 (b) aa+1 [ + 2 log a]
a
(c) 2 log a (d) none of these

48. f(x) = x 2 /ex then f ′ (−1) is


(a) −l/e (b) I/e
(c) −3e (d) None of these

49. If y = x 2x then
dy
is ___________
dx
(a) 2x
2x (1 + logx) (b) 2(1 + logx)
(c) x 2x (1 + logx) (d) None of these

50. The derivative of log x. e x is ___________.


ex 1
(a) + ex (logx) (b) ex ( − log x)
x x
(c) x
e (1 + log x) (d) None of these

51. 3
Let f(y) = x x then f ′ (y)_______________ is
3
(a) x 3 [x 2 + 3x. log x] (b) x x [x 2 + 3x 2 . log x]
(c) x x 3 [x2 − 3x. log x] (d) None of these

52. Differentiate y w.r.t. x wheny = (x 2 − 2x)(x 2 + 1)


(a) 4x 3 + 6x 2 − 2x + 2 (b) 4x 2 − 6x + 2
(c) 4x 3 − 6x 2 + 2x − 2 (d) None of these

143 | P a g e
53. The curve 4y = ux 2 + v passes through the point p at (2,3) and
dy
= 4 this point ′p′. So the
dx
value of u and v are
(a) u = 2, v = 2 (b) u = −4, v = −4
(c) u = 4, v = −4 (d) None of these

54. dy x2 y2
Find dx
of (a2 + b2 = 1)
(a) −b2 x/a2y (b) −b2 y/a2 x
(c) −b2 /a2 (d) o

55. Find the gradient of curve y = 3x 2 − 5x + 4 at the point (1,2)


(a) 1 (b) 3
(c) 4 (d) 5

56. d
√log x
dx
1
(a) 2x√log x
(b) 2x. √log x
1 1
(c) (d)
√log 𝑥 𝑥

57. The derivative of y = log x x is


(a) 1/x (b) log x
(c) 1/x. log x (d) 1 + log x

58. Find the second differential coefficient of y = x 2 log x


(a) x + 2x log x (b) 3 + 2 log x
(c) 3 log 𝑥 (d) 2x log x

59. If f(x) = 5x a + 10ax + 3aa then


dy
is equal to
dx
(a) 3ax a−1
+ 10xa + 3a. a x−1 a−1
(b) 5ax a−1 + 10ax log a
(c) 5x a log x + 10xax−1 (d) None of these

60. If y = eax
3 +bx2 +cx+d
, when
dy
is ________.
dx
(a) 2
(3ax + 2bx + c)y (b) 3ax 2 + 2bx + c
3+ 2
(c) eax bx +cx+d (d) None of these

61. If y = x 5 + e2x + log 3x when


dy
is
dx
1 1
(a) 4
5x + 2e 2x
+x (b) x 5 + e2x + 3x
1
(c) 5x 4 + e2x + 3x (d) None of these

62. If y = x x , then
dy
is
dx
(a) x x (2
+ log x) (b) x x log(ex)
e
(c) x x log (x) (d) None of these

63. d2 y
If y = aemx + be−mx then equal to
dx2
(a) my (b) m2 y 2
(c) m2 y (d) None of these

144 | P a g e
64. dy ex −e−x
Evaluate if 𝑦 =
dx ex +e−x
−4 4
(a) (ex +e−x )2
(b) (ex +e−x )2
1
(c) (ex +e−x )2
(d) None of these

65. Find
dy
if y = √x + √x
dx
2√x+1 2√x−1
(a) (b)
4√x√x+√x 4−√x√x+√x
2√𝑥+1
(c) (d) None of these
2√𝑥−1

66. Evaluate
dy
If y = 7x
2 +2x
dx
2 2
(a) (2x + 1). 7𝑥 +2𝑥 . log 7 (b) 2(x + 1). 7x +2x . log 7
2
(c) 7𝑥 +2𝑥 . log 7 (d) None of these

67. If y = log(x + √x 2 + a2 ) then


dy
the value of
dx
1 −1
(a) (b)
√x2 +a2 √x2 +a2
x
(c) (d) None of these
√x2 +a2

68. If x y = ex−y ; then


dy
is
dx
log x log x
(a) (b)
(1+log x)2 (1−log x)2
1
(c) (1−log x)2
(d) None of these

69. if y 3 . x 5 = (x + y)8 , then


dy
is
dx
y −y
(a) (b)
x x
y5
(c) (d) None of these
x3

70. If 𝑦 = 𝑥 𝑥
𝑥−∞
; then
dy
is
dx
y2 y2
(a) 1+ylog x
(b) X(1−ylog x)
−y2
(c) (d) None of these
1−y log x

71. If y = x log(log x) ; then


dy
is
dx
y x
(a) x
[log(log x) + 1] (b) y
[log(log x) + 1]
x
(c) − y [log(log x) + 1] (d) None of these

72. if y = x + 1
1
then
dy
is
x+ dx
x
4 2
x +x +2 x4 +x2 +2
(a) (x2 +1)2
(b) x2 +1
2
(x4 +x2 +2)
(c) x2 +1
(d) None of these

145 | P a g e
73. y x dy
If √ + √ = 6, then is
x y dx
x+17y x−17y
(a) (b)
17x+y 17x+y
x−17y
(c) 17x−y
(d) None of these

PAST EXAMINATION QUESTIONS

74. The slope of the tangent at the point (2, -2) to the curve x2 + xy + y 2
- 4= 0 is given by:
(a) 0 (b) 1
(c) -1 (d) None

75. The derivative of x2 log x is


(a) 1+2logx (b) 2logx
(c) x(1+2 log x) (d) None of these

dy
76. If x = y log (xy), then is equal to:
dx
x y x y
(a) (b)
x(1  log xy) x(1  log xy)
x y x y
(c) (d)
x(log x  log x) x(log x  log x)

2
4 2 d y dy
77. If y = 2x+ , then x  x y yields to
x dx 2 dx
(a) 3 (b) 1
(c) 0 (d) 4

78. If f(x) = xk and f '(1) = 10, then the value of k is :


(a) 10 (b) -10
(c) 1/10 (d) None

d2y
79. If Y=a. e mx
+ be -mx
then 
dx 2
(a) Y.m2 (b) Y
(c) Y/m2 (d) Y2m

80. If xy = yx , then dy gives:


dx
x (x logy - y) x (y logx - x)
(a) (b)
x (y logx - x) y (x logy - y)
y (x logy - y)
(c) (d) None of these
x (y logx - x)

146 | P a g e
81. If x 3  2x 2 y 2  5x  y  5 then dy
at x  1 and y=1 is :
dx

(a)
4 (b)
5
3 4
(c)
4 (d)
4
5 3
dx
82. If y  ( x  x 2  m 2 ) n then =:
dy
ny
(a) (b) xy
x 2  m2
- xy
(c) (d) none
x 2  m2
dy
83. If xy(x  y)  0 , find
dx
y(2x  y) x(2x  y)
(a) (b)
x(2y  x) y(2y  x)
y(2y  x)
(c) (d) none
x(2x  y)

x 
x
84. If y x then
dy
is equal to :
dx
y2 y2
(a) (b)
log x 2 - ylogx
y2
(c) (d) none
x(2 - ylogx)
x2 x3 xn dy
85. If y = 1 + x + + +.........+ then the value of − y is
2! 3! n! dx
(a) 1 (b) −1
(c) 0 (d) none of these

86. The slope of the tangent to the curve y  4  x 2 at the point. Where the ordinate and the
abscissa are equal is :
(a) -1 (b) 1
(c) 0 (d) none
x
(x )
87. Differentiate e
(a) (1+logx) (b) xx(1+logx)
x x
(c) ex (1 + logx)xx (d) ex (1 + logx)

147 | P a g e
dy
88. If xm yn = (x+y)m+n then find
dx
x y
(a) (b)
y x
(c) xy (d) None

89. If f(x) = ax xa then find f '(x).


(a) f(x) [a + log a] (b) f(x)  a  log a 
x 

(c) f(x)  a  log a  (d) f(x) [a + x log a]


x 

dy
90. If x3 y2 = (x-y)5 Find at (1,2).
dx
(a) - 7/9 (b) 7/9
(c) 9/7 (d) - 9/7

x dy
91. If Y = Then X3 =
dx
x 1
2

(a) Y3 (b) Y
(c) Y2 (d) Y5

dy
92. If the curve Y2=AX4+B Passes through the point P(1, 2). The value of at P is 4. Then
dx
(a) A=4, B=0 (b) A=0, B=4
(c) A=2, B=0 (d) A=1, B=2

93. Find the second derivative of y = x 1


(a) 1/2 (x+1)-1/2 (b) -1/4 (x+1) -3/2
(c) 1/4 (x+1) -1/2 (d) None of these.

94. If x2+y2=4 then


2 2
d 2 y  dy  d 2 y  dy 
(a) y   2  1  0 (b) y    1  0
dx 2  dx  dx 2  dx 
2 2
d 2 y  dy  d2y  dy 
(c) y    1  0 (d) y  2   1  0
dx 2  dx  dx 2
 dx 

95. The cost function for the production of x unit of a commodity is given by C(x) = 2x 3 - 15x2 +
36x + 15 the cost will be minimum when 'x' is equal to
(a) 3 (b) 2
(c) 1 (d) 4

148 | P a g e
96. If f ( x)  x C3 then f' 1  ?
1 1
(a)  (b)
6 6
5 5
(c) (d)
6 6

97.
dx
2  
d log2x

(a) 1 (b) 0
(c) 1/x (d) 2x log2

d2 y
98. If y = xx then
dx2
d dy d dy
(a) y 1  log x   1  log x  (b) (1  log x)  (1  log x)
dx dx dx dx
d d
(c) 1  log x   y 1  log x  (d) None of these
dx dx

dy
99. If y = logy x then =
dx
1 1
(a) (b)
x  log y x  x log y
1
(c) (d) None of these
1  x log y

dy
100. If X = ct ; y = c/t then =
dx
(a) 1/t (b) t.et
1
(c) (d) None
t2

dy
101. If Y = ea.logx+ ex.loga then =
dx
(a) Xa + ax (b) a.xa−1+ ax.loga
(c) a.Xa−1+ X.ax−1 (d) xx+ aa

d2y dy
102. For the function y = x3-3x then the value of 2
at which = 0 is
dX dX
(a) 1 (b) 3
(c) 6 (d) None

149 | P a g e
5  4X 2 dy
103. If y = log than
3  5X 2
dX
8 10
(a)  (b) (4x2−5) −(3 +5x2)
4X  5 3  5X 2
2

8X 10 X
(c)  (d) 8x-10
4X  5 3  5X 2
2

dy
104. x=logt, y=et =?
dx
1
(a) (b) t.et
t
1
(c)  (d) None of these
t2

p q pq dy
105. If x y  (x  y) , then is equal to __________
dx
q x
(a) p (b) y
y p
(c) x (d) q

106. If e
xy
 4xy  4 then dy __________
dx
y -y
(a) x (b) x
x -x
(c) y (d) y

du
107. If u=3t4 + 5t3 + 2t2 + t + 4, then the value of at t =-1 is:
dt
(a) 0 (b) 1
(c) 2 (d) 5

x 1
108. The slope of the tangent to the curve y  at x=2 is:
x2
3 3

(a) 16 (b) 16
1 1

(c) 4 (d) 4

ANSWER

150 | P a g e
1. 2. 3. 4. 5. 6. 7. 8. 9. 10.
(A) (B) (C) (A) (A) (C) (B) (C) (A) (A)
11. 12. 13. 14. 15. 16. 17. 18. 19. 20.
(B) (A) (B) (B) (B) (C) (B) (C) (A) (A)
21. 22. 23. 24. 25. 26. 27. 28. 29. 30.
(C) (A) (C) (A) (B) (C) (A) (C) (B) (B)
31. 32. 33. 34. 35. 36. 37. 38. 39. 40.
(A) (B) (A) (B) (A) (B) (A) (D) (A) (B)
41. 42. 43. 44. 45. 46. 47. 48. 49. 50.
(A) (C) (B) (C) (B) (B) (B) (C) (A) (A)
51. 52. 53. 54. 55. 56. 57. 58. 59. 60.
(B) (C) (C) (A) (A) (A) (D) (B) (B) (A)
61. 62. 63. 64. 65. 66. 67. 68. 69. 70.
(A) (B) (C) (B) (A) (B) (A) (A) (A) (B)
71. 72. 73. 74. 75. 76. 77. 78. 79. 80.
(A) (A) (C) (B) (C) (B) (C) (A) (A) (C)
81. 82. 83. 84. 85. 86. 87. 88. 89. 90.
(A) (A) (A) (C) (C) (C) (C) (B) (C) (A)
91. 92. 93. 94. 95. 96. 97. 98. 99. 100.
(A) (A) (B) (B) (A) (A) (A) (A) (B) (C)
101. 102. 103. 104. 105. 106. 107. 108.
(B) (C) (C) (B) (C) (B) (A) (A)

APPLICATIONS OF DIFFERENTIAL CALCULUS

1. The cost in Rs. to produce x lamps for a certain factory is c(x) = 300 + 23x + 0.4x 2. What is
the marginal cost at 250 lamps?
(a) 223
(b) 265
(c) 255
(d) 225

x3
2. If the cost of function of a commodity is given by c(x) = 150x – 5x2 + where C stands
6
for cost and x stands for output. If the average cost is equal to the marginal cost then the
output x is:
(a) 5
(b) 10
(c) 15
(d) 20

3. The cost function of a company is given by:


x3
C(x) = 100x – 8x2 + ,
3
Where x denotes the output. Find the level of output at which:
(i) Marginal cost is minimum
(ii) Average cost is minimum

(a) Marginal cost is minimum at X = 2, Average cost is minimum at X=6

151 | P a g e
(b) Marginal cost is minimum at X = 4, Average cost is minimum at X=6
(c) Marginal cost is minimum at X = 6, Average cost is minimum at X=12
(d) Marginal cost is minimum at X = 8, Average cost is minimum at X=12

4. A computer software company wishes to start the production of floppy disks. It was
observed that the company had to spend Rs. 2 lakhs for the technical information. The cost
of setting up the machine is Rs. 88,000 and the cost of producing each unit is Rs. 30 while
each floppy could be sold at Rs. 45. Find:
(i) The total cost function for producing x floppies : and
(ii) The break-even point.
(a) C(x) = 30x, break-even point = 19,200
(b) C(x) = 30x + 2,88,000, break-even point = 19,200
(c) C(x) = 30x – 2,88,000, break-even point = 19,200
(d) None of these

5. A company decided to set up a small production plant for manufacturing electronic clocks.
The total cost for initial set up (fixed cost) is Rs. 9 lakhs. The additional cost for producing
each clock is Rs. 300. Each clock is sold at Rs. 750. During the first months, 1,500 clocks
are produced and sold.
(i) What profit or loss the company incurs during the first month, when all the 1,500
clocks are sold?
(ii) Determine the break-even point.
(iii) Total cost of producing 20 items of a commodity is Rs. 205, while total cost of
producing 10 items is Rs. 135. Assuming that the cost function is a linear function and
the cost function and marginal cost function.
(a) Loss of Rs. 2,25,000, break-even point = 2000, cost function = 7x + 65, marginal
cost function = 7
(b) Profit of Rs. 2,25,000, break-even point = 2000, cost function = 7x + 65, marginal
cost function = 7
(c) Loss of Rs. 2,00,000, break-even point = 2000, cost function = 7x + 65, marginal
cost function = 7
(d) Profit of Rs. 2,25,000, break-even point = 2000, cost function = 7x + 65, marginal
cost function = 7x

ANSWER

(1) (2) (3) (4) (5)


A C D B A

152 | P a g e
CHAPTER – 8 - INTEGRATION
PRACTICE QUESTIONS

1. Evaluate ∫ 5x 2 dx:
4 3 5x3
(a) x k (b) 3
+k
3
(c) 5x 3 (d) none of these

2. Integration of 3 − 2x − x 4 will become


x5
(a) −x 2 − x 5 /5 (b) 3x − x 2 − 5
+k
x5
(c) 3x − x + 2
5
+k (d) none of these

3. Given f(x) = 4x 3 + 3x 2 − 2x + 5 and ∫ f(x)dx is


(a) x4 + x3 − x2 + 5 + k (b) x 4 + x 3 − x 2 + 5x + k
(c) 12x 2 + 6x − 2x 2 (d) none of these

4. Evaluate  (x  1) 2 dx
2

x3
(a) x 5 /5 − 2/3x 3 + x + k (b) −x+k
3
(c) 2x (d) none of these

5. ∫(1 − 3x)(1 + x)dx is equal to


(a) x − x2 − x3 (b) x3 − x2 + x
(c) x − x2 − x3 + k (d) none of these

 1 
6.   x  dx is equal to :
x
2 3/ 2 2
(a) x  2 x1/ 2  k (b) x 2 x k
3 3
1 1
(c)  k (d) none of these
2 x 2x x
7. ∫ e3x+5 dx is equal to
e3x+5 e3x
(a) ∫ 3
+c (b) 5
+c
−e3x+5
(c) 3
+c (d) None of these

8. Integrate the function f(x) = (4x + 5)6


(a) 1/28 (4x + 5)7 + k (b) (4x + 5)7/7 + k
(c) (4x + 5)7/7 (d) none of these

 x(x  4)5 dx
2
9. Evaluate
(a) (x2 + 4)6 + k (b) 1/12 (x2 + 4)6 + k
(c) 1/12 (x2 + 4)6 / + k (d) none of these

153 | P a g e
10. Integrate (x+a)n
( x  a) n1 ( x  a) n1
(a) k (b)
n 1 n 1
(c) (x+a)n+1 (d) none of these

 8x / ( x3  2)3 dx is equal to
2
11.
4
(a) -4/3(x3+2)3 dx (b)  k
3( x  2) 2
3

4
(c) k (d) none of these
3( x  2) 2
3

1
12. Integration of f(x) when f(x)
x 2  a2
a
(a) log x  k (b) log (x–a) – log (x+a)+k
xa
1 xa
(c) log k (d) none of these
2a xa

13. Evaluate x
2
e 3 x dx

(a) x 2e3x / 3  2 x e3x / 9  2 / 27e3x  K (b) x(log x)2 – 2x + k

(c) e3x / 3  x e3x / 9  2e3x  k (d) none of these

14.  log x dx is equal to :


(a) x logx + k (b) x logx–x2+k
(c) xlogx1  k (d) x(logx1) k

15.  xe
x
dx is :
(a) (x–1) ex + k (b) (x–1) ex
(c) x ex + k (d) none of these

 (log x)
2
16. dx
(a) x (logx)2 – 2 x logx + 2x + k (b) x (logx)2 – 2x + k
(c) 2x logx – 2x + k (d) none of these

 ( x  5) dx / ( x  1) ( x  2)
2
17. Evaluate we get
(a) 4 log (x+1) – 4 log (x+2) + 3/x + 2 + k
(b) 4 log (x + 2) –3/x + 2 + k
(c) 4 log (x + 1) – 4 log (x + 2)
(d) none of these

18. Evaluate l
(a) 4/3 + k (b) 5/12
(c) –4/3 (d) none of these

154 | P a g e
4
19. Evaluate  (3x  2) dx dx
2

2
(a) 104 (b) 100
(c) 10 (d) none of these
1
20. Evaluate
 xe dx
x

0
(a) -1 (b) 10
(c) 10/9 (d) 1

21. x (1+logx) dx is equal to


x

(a) x log x  k (b) xx  k


xx
(c) k (d) none of these
2

22.  1  x 2 dx
2 x 1
(a) x(1  x 2 ) 3 / 2  k (b) 1  x 2  log ( x  x 2  1
3 2 2
2
(c) x(1  x 2 ) 3 / 2  k (d) none of these
3

 (x  1) / x 2  2
2
23. dx is equal to
x 2
(a) x 2 k (b) k
2
(c) 1 /( x 2  2)3 / 2  k (d) none of these

 (e  e x ) 2 (e x  e x ) dx is
x
24.
1 x 1 x
(a) (e  e  x ) 3  k (b) (e  e  x ) 3  k
3 2
(c) ex  k (d) none of these
1 2
25. The value of the integral will be for ∫ (X − X) dx
x3 1 x3 1
(a) 3
− 2x + x + c (b) 3
+ 2x + x + c
x3 1
(c) − 2x − + c (d) None of these
3 x

 xe / ( x  1) 2 dx is equal to
x
26.

(a) e x /( x  1)  k (b) ex / x  k
(c) ex  k (d) none of these

155 | P a g e
 3
27.
  x   dx is equal to
4
x
(a) x 5 / 5  3 log x  k (b) 1/ 5x 5  3log x  k
(c) 1/ 5x 5  k (d) none of these

 (1  x)
3
28. Evaluate the integral / xdx

(a) log x  3x 3  3 / 2 x 2  k (b) log x  2  3x 2  k


x 3 3x 2
(c) log x  3x  k2
(d) log x    3x  k
3 2
29. The equation of the curve in the from y = f(x) if the curve passes through the point (1,0) and
f'(x) = 2x-1 is
(a) y  x2  x (b) x  y2  y
(c) y  x2 (d) none of these
4
30. Evaluate  (2 x  5) dx
1
(a) 3 (b) 10
(c) 30 (d) none of these

2
2x
31. 1 x
1
2
dx is equal to

(a) log e (5 / 2) (b) log e 5  log e 2  k


(c) log e (2 / 5) (d) none of these

4
32.

0
3x  4 dx is equal to

(a) 9/112 (b) 112/9


(c) 11/9 (d) none of these

x2
2
33.
 x 1
0
dx

(a) 2  log e 2 (b) 2  log e 3


(c) log e 3 (d) none of these

e2
dx
34. Evaluate  x(1  log x )
1
2

(a) 3/2 (b) 1/3


1
(c) 26/3 (d) log 5e
2

156 | P a g e
3
35. Evaluate ∫−3(x 3 + x) dx
(a) 0 (b) 3
(c) 3 (d) 1

36. The equation of the curve which passes through the point (1,3) and has the slope 4x-3 at any
point (x,y) is
(a) y  2 x 3  3x  4 (b) y  2 x 2  3x  4
(c) x  2 y 2  3y  4 (d) none of these
3 3
37. The value of  f (5  x)dx   f ( x)
2 2
dx is

(a) 1 (b) 0
(c) -1 (d) none of these

 ( x  1)e
x
38. / x 2 dx is equal to
(a) ex / x  k (b) ex / x  k
(c)  ex / x  k (d) none of these

e x ( x log x  1)
39.  x
dx is equal to

(a) e x log x  k (b) ex  k


(c) logx + k (d) none of these

 log x
2
40. dx is equal to
(a) x(log x-1) + k (b) 2x(log x-1) + k
(c) 2(log x-1) + k (d) none of these

2
41. x
1
log x dx is equal to

(a) 2 log 2 (b) -3/4


(c) 2 log 2 - 3 4 (d) none of these

( x 2  1) x  x
2 1
42. Evaluate  e dx
1
x2
(a) e 2 ( e  1) (b) e 2 [ e  1]  k
(c) e2 e (d) none of these

2
43.
 3x dx is
2

(a) 7 (b) -8
(c) 8 (d) none of these

157 | P a g e
( 2  x )e x
44. Evaluate  (1  x)2 dx
ex
(a) k (b) ex  k
1 x
(c) 1/ 1  x  k (d) none of these

45. Integrate x3 log x


(a) x 4 / 16  k (b) x4/16(4logx-1)+k
(c) 4 log x-1 + k (d) none of these

46.  log (log x) /x dx is


(a) log ( logx-1)+k (b) log x-1 + k
(c) [log(log x-1)]logx + k (d) none of these

 x(log x)
2
47. dx is equal to

x2 1
(a) [(log x) 2  log x  ]  k (b) (log x) 2  log x 
1
k
2 2 2
x2 1
(c) [(log x) 2  ]  k (d) none of these
2 2

 e x  ex 
48. Evaluate
  e x  e  x  dx

(a) log e e x  e  x (b) log e e x  e  x  k
(c) loge x (d) none of these

 3x /( x  x  2) dx
2
49. Evaluate

(a) 2 log e x  2  log e x  1  k (b) 2 log e x  2  log e x  1  k


(c) log e x  2  log e x  1  k (d) none of these

50. If f'(x) = x - 1, the equation of a curve y = f(x) passing through the point (1,0) is given by
(a) y  x 2  2x  1 (b) y  x2 / 2  x 1
(c) y  x 2 / 2  x  1/ 2 (d) none of these

51. Evaluate ∫1
2 xdx
x2 +2
(a) log√2 (b) log√3
1 1
(c) log (d) log
√2 √3

158 | P a g e
52. Evaluate ∫1
2 dx
(x+1)(x+2)
9 8
(a) log (8) (b) log (9)
6 5
(c) log (8) (d) log (4)

PAST EXAMINATION QUESTIONS


1
53.
 (e  e  x )dx is
x

(a) e – e-1 (b) e-1-e


(c) e+e-1 (d) None

8x2 dx
54.  3
( x  2)3 is equal to:
4 3 4 3
(a) - ( x  2) 2  c (b) - ( x  2) 2  c
3 3
4 3
(c) ( x  2) 2  c (d) None of these
3

dx
55. Evaluate:  x2  a2
:

1
(a) log( x  x2  a2 )  c (b) log( x  x 2  a 2 )  c
2
1
(c) log( x x 2  a 2 )  c (d) log( x x 2  a 2 )  c
2
2
x
56. The value of

0 x  2 x
dx is :

(a) 0 (b) 3
(c) 2 (d) 1

(e 3x  e -3x )
57. The integral of  e x dx is:
e 2 x e 4 x e 2 x e 4 x
(a)  c (b)  c
2 4 2 4
(c) e2x-e-4x+c (d) None of these

2
58. x e 3 x dx is:

e 3 x x.e 3 x
(a) 2 3x
x .e -2xe +2e +c 3x 3x
(b)   2e 3 x  c
3 9
x 2 .e 3 x 2 x.e 3 x 2 3 x
(c)   e c (d) None of these
3 9 27

159 | P a g e
2
2x
59. 1 x
1
2
dx :

5
(a) loge (b) loge5-loge2+1
2
(c) loge 2 (d) None of these
5

(1  logx)
e
60. The value of 
1
x
dx is :

(a) 1 (b) 3
2 2
(c) 1 (d) 5
2

x3
61. Find  2 dx :
( x  1) 3
1  2x 2  1  1  2x 2  1 
(a) (b) 
4  (x 2  1) 2  4  (x 2  1) 2 
1  2x 2  1  1  2x 2  1 
(c) (d) 
2  (x 2  1) 2  2  (x 2  1) 2 

1
62. x 2
 a2
dx is
a
(a) log (x-a)-log (x+a) + c (b) log x- c
xa
1  x a 
(c) log  c (d) none
2a xa
1

dx
63. The value of is :
0 (1  x)(2  x)
3 4
(a) log (b) log
4 3
(c) log 12 (d) none
3 3
64. The value of  f(5  x) dx   f(x)dx is :
2 2
(a) 1 (b) 0
(c) -1 (d) none

x
e loge
65.  x dx is:

(a) x-1 + c (b) x +c


(c) x2 + c (d) none

160 | P a g e
1
66. Evaluate  (x -1) (x - 2) dx :
 x2
(a) log  c (b) log [(x-2)(x-1)]+c
 x 1 
 x 1 
(c) log  c (d) None
 x2
4
67.  (2x  5) dx
1
(a) 10 (b) 3
(c) 30 (d) None

68. 1
 x( x 5
 1)
dx

 x5  1  x5 
(a) log  5   C (b) log  C
 x 1  5  x 5 1 
   
1  x5  1  x 5 1 
(c) log  C (d) log  C
3  x 5 1  3  x5 
   

3
69. Find the value of 
3
x 8  x 2 dx
(a) 1 (b) -1
(c) 0 (d) None of these

 x.e dx
x
70. Evaluate
(a) ex (x+1)+c (b) ex (x-1) + c
(c) ex+c (d) x - ex + c

71. 1  x  x2
 x2(1  x)
dx 

1
(a)  log(1  x)  c
x
-log(1  x) 1
(b)  c
x x
-1
(c)  log(1  x)  c
x
(d) None of these

161 | P a g e
 1 
72.   x dx

x
1
1/2  1/2  1 
(a) 2x  x 1 (b) 2x  x  1
3  3 
1 
(c) 2  x  x1 / 2  (d) None of these
3 

1 1 x 
73.   1  x dx
0

(a) 2 log 2-1 (b) 4 log 2-1


(c) 2 log 2 (d) None of these


dx
74. Equal to.
3x  4  3x  1
2 2
(a) [(3 x  4) 3 / 2  (3 x  1) 3 / 2 ]  c (b) [3 x  4) 3 / 2  (3 x  1) 3 / 2 ]c
27 27
2
(c) [(3 x  4)3 / 2  (3 x  1) 3 / 2 ]  c (d) None of these
3
2
xdx
75.  
1 x2  2
(a) log 2 (b) log 3
1 1
(c) log (d) log
2 3
6x  4
76.
 ( x  2)( x  3)
dx is equal to

(a) 22 log (x-3) - 16 (x-2) (b) 11 log (x-3) -8 (x-2)


(c) 22 log (x-3) -16 log (x-2) (d) 22 log (x-3) +16 log (x-2)

1
77.
 x(1  log ) 2
dx is equal to

1 1
(a)  c (b) c
2(1  log x) 2 (1  log x)
1
(c)  c (d) None of these
1  log x
1

 e  e  x  dx
x
78. Solve:
1
(a) 0 (b) 1
(c) 12 (d) None of the above.

162 | P a g e
logx  x 2

79. Solve :  x3
.dx
3 1
(a) log x 3  C (b) log x 3  C
2 3
1 3
(c) log x 3  C (d) log x 3  C
6 7

 e  exloga  dx; then dy


aloxx
80. Given, y =
dx
(a) x aax (b) xa  ax
(c) axx1  ax loga (d) None of the above.

2
81. If f ' ( x)  3x 2  , f(1) = 0 and f(x) =
x3
x3
(a)  x2  2 (b) x 3  x2  2
3
(c) x3  x2  2 (d) None of these.

ex ex
82.
 1  x  3
.dx  
2 1  x 
2
.dx

ex
(a) 0 (b) c
2 1  x 
2

ex ex
(c) c (d) c
2 1  x  1  x 
2 2

1
x
83. 
1
x
dx

(a) -1 (b) 0
(c) 1 (d) 2

1 dx
84.  2 

0 ax  b 1  X
 
(a) a/b (b) b/a
(c) ab (d) 1/ab

85. 2  32 X  5 X dX 
3X

3X 2 X X 3X 2 X X
2 3 5 2 3 5
(a) +c (b) +c
log 720 log 360
3X 2 X X 3X 2 X X
2 3 5 2 3 5
(c) +c (d) +c
log180 log 90

163 | P a g e
7
x 3 / 2 dx
86. 
2
(9  x ) 3/ 2  x 3/ 2
equals to –

7 5
(a) (b)
2 2
9
(c) 5 (d)
2

87. a
2x
dx 

a2x log a 2.a2 x


(a) (b)
2 log a
a2x
(c) (d) None
2 log a

5
x2
88. 0 x2  (5  x)2 is equal to
(a) 5 (b) 5/2
(c) 1 (d) None
2
[log e (ex)]n
89. 1 x dx(n#1) is =
[log e (2e)]n 1  1
(a) (b) [log e (2e) n 1  1
n 1
[log e (2e)]n 1  1
(c) (d) None
n
90. The points on the curve y =x3–x2–x+1. Where the tangent is parallel to x-axis are :
  1 32 
(a) (1, 0),  ,  (b) (1, 0), (1, 1)
 3 27 
(c) (0, 1), (1, 1) (d) (0, 0), (1, 0)

91.  1  x dx 
0
3 1
(a) (b)
2 2
(c) 0 (d) -1
1/ 2

dx
is
92. The value of 0 3  2x

(a) 1 (b) 3/ 2

(c) 3 2 (d) 2 3

164 | P a g e
2
 xe
x2
dx
93. The value of 0 is
(a) 1 (b) e 1
1 4
(e 1)
(c) (e/2)1 (d) 2

2 1
 1  x dx is equal to:
x
94. The value of 1

3 3
log  1 2 log  1
(a) 2 (b) 2
1 3 1 2
log  1 log  1
(c) 2 2 (d) 2 3

ANSWER
1. 2. 3. 4. 5. 6. 7. 8. 9. 10.
(B) (B) (B) (A) (C) (A) (A) (A) (B) (A)
11. 12. 13. 14. 15. 16. 17. 18. 19. 20.
(B) (C) (A) (D) (A) (A) (A) (B) (A) (D)
21. 22. 23. 24. 25. 26. 27. 28. 29. 30.
(B) (B) (A) (A) (C) (A) (B) (D) (A) (C)
31. 32. 33. 34. 35. 36. 37. 38. 39. 40.
(A) (B) (B) (D) (A) (B) (B) (A) (A) (B)
41. 42. 43. 44. 45. 46. 47. 48. 49. 50.
(C) (A) (C) (A) (B) (C) (A) (B) (A) (C)
51. 52. 53. 54. 55. 56. 57. 58. 59. 60.
(A) (A) (A) (B) (B) (D) (B) (C) (A) (B)
61. 62. 63. 64. 65. 66. 67. 68. 69. 70.
(B) (C) (B) (B) (B) (A) (C) (B) (C) (B)
71. 72. 73. 74. 75. 76. 77. 78. 79. 80.
(C) (B) (A) (B) (A) (C) (C) (A) (B) (B)
81. 82. 83. 84. 85. 86. 87. 88. 89. 90.
(C) (C) (B) (D) (B) (B) (C) (B) (A) (A)
91. 92. 93. 94.
(D) (C) (D) (B)

165 | P a g e
CHAPTER-9 - NUMBER SERIES, CODING, DE-CODING AND
ODD ONE OUT SERIES
Number Series (Q.1 to Q.30):

1. 6, 11, 21, 36, 56 ?


(a) 42 (b) 51
(c) 81 (d) 91

2. 10, 100, 200, 310 ?


(a) 400 (b) 410
(c) 420 (d) 430

3. 11, 13, 17, 19, 23, 25 ?


(a) 33 (b) 27
(c) 29 (d) 49

4. 6, 12, 21, 33 ?
(a) 33 (b) 38
(c) 40 (d) 48

5. 2, 5, 9, 14, ?, 27
(a) 20 (b) 16
(c) 18 (d) 24

6. 6, 11, 21, ?, 56, 81


(a) 42 (b) 36
(c) 91 (d) 51

7. 10, 18, 28, 40, 54, ?, 88


(a) 70 (b) 86
(c) 87 (d) 98

8. 120, 99, ?, 63, 48, 35


(a) 80 (b) 36
(c) 45 (d) 40

9. 22, 24, 28, 36, ?, 84


(a) 44 (b) 52
(c) 38 (d) 54

10. 4832, 5840, 6848, 7856 ?


(a) 8864 (b) 8815
(c) 8846 (d) 8887

11. 10, 100, 200, 310, 430 ?


(a) 560 (b) 540
(c) 550 (d) 590

166 | P a g e
12. 28, 33, 31, 36, 34 ?
(a) 38 (b) 39
(c) 40 (d) 42

13. 120, 80, 48, 24, ?, 0


(a) 15 (b) 8
(c) 25 (d) 30

14. 2, 15, 41, 80, 132 ?


(a) 184 (b) 144
(c) 186 (d) 197

15. 6, 17, 39, ?, 116


(a) 72 (b) 75
(c) 85 (d) 80

16. 1, 4, 10, 22, ?, 94


(a) 46 (b) 48
(c) 49 (d) 47

17. 4, 9, 25, 49, ?, 169, 289, 361


(a) 120 (b) 121
(c) 122 (d) 164

18. 4, 12, 36, ?, 324


(a) 107 (b) 109
(c) 108 (d) 110

19. 1, 1, 4, 8, 9, ?, 16, 64
(a) 27 (b) 28
(c) 32 (d) 40

20. 5760, 960, 192, ?, 16, 8


(a) 47 (b) 48
(c) 52 (d) 50

21. 1, 2, 6, 7, 21, 22, 66, ?, 201


(a) 69 (b) 68
(c) 67 (d) 71

22. 48, 24, 96, ?, 192


(a) 48 (b) 47
(c) 44 (d) 54

23. 165, 195, 255, 285, ?, 375


(a) 345 (b) 390
(c) 335 (d) 395

24. 2, 3, 3, 5, 10, 13, 39, ?, 172, 177


(a) 42 (b) 44
(c) 43 (d) 40

167 | P a g e
25. 7, 26, 63, 124, 215, ?, 511
(a) 342 (b) 343
(c) 441 (d) 421

26. 3, 7, 15, 31, ? 127


(a) 62 (b) 63
(c) 64 (d) 65

27. 8, 28, 116, 584, ?


(a) 1752 (b) 3502
(c) 3504 (d) 3508

28. 6, 13, 28, 59, ?


(a) 122 (b) 114
(c) 113 (d) 112

29. 2, 7, 27, 107, 427, ?


(a) 1707 (b) 4027
(c) 4207 (d) 1207

30. 5, 2, 7, 9, 16, 25, 41, ?


(a) 65 (b) 66
(c) 67 (d) 68

31. In a certain language, MADRAS is coded NBESBT, how DELHI is coded in that code ?
(a) EMMJI (b) EFMIJ
(c) EMFIJ (d) JIFEM

32. If RAMAN is written as 12325 and DINESH as 675489 how HAMAM is written ?
(a) 92323 (b) 92233
(c) 93233 (d) 93292

33. If RED is coded as 6720 then GREEN would be coded as


(a) 9207716 (b) 167129
(c) 1677209 (d) 1972091

34. If A = 1, FAT = 27, FAITH = ?


(a) 44 (b) 45
(c) 46 (d) 36

35. If BROTHER is coded 2456784, SISTER coded as 919684, what is coded for BORBERS ?
(a) 2542849 (b) 2542898
(c) 2454889 (d) 2524889

36. If DELHI is coded 73541 and CALCUTTA as 82589662. How can CALICUT be coded?
(a) 5279431 (b) 5978213
(c) 8251896 (d) 8543962

37. If CLOCK is coded 34235 and TIME is 8679, what will be code of MOTEL ?
(a) 72894 (b) 77684
(c) 72964 (d) 27894

168 | P a g e
38. If PALE is coded as 2134 and EARTH is coded as 41590, how is PEARL coded ?
(a) 29530 (b) 24153
(c) 25430 (d) 25431

39. If LOSE is coded as 1357 and GAIN is coded as 2468, what do figure 82146 stands for ?
(a) NGLAI (b) NGLIA
(c) GNLIA (d) GNLAI

40. If MEKLF is coded as 91782 and LLLJK as 88867, how can IHJED is coded as ?
(a) 97854 (b) 64512
(c) 54610 (d) 75632

41. If in a certain code language NAME is written as 4258 then what is coded as MEAN ?
(a) 2458 (b) 5842
(c) 8524 (d) 5824

42. If GOLD is written as IQNF, how WIND can be written as code ?


(a) YKPF (b) VHCM
(c) XJOE (d) DNIW

43. If ROSE is written as TQUG, how BISCUIT can be written in that code?
(a) DKUEWKV (b) CJTDVJU
(c) DKVEWKV (d) DKUEWKY

LETTER : CZNVRSWFD
CODE DIGIT 8 6 4 7 2 9 3 5 1

(Q. No. 44–46) In each of the following questions find out the correctly coded alternative from
amongst the given four alternatives (a), (b), (c), (d).

44. ZDRCVF
(a) 612875 (b) 619875
(c) 612845 (d) 612835

45. WNCSZV
(a) 348267 (b) 318267
(c) 348957 (d) 348967

46. RDNFVS
(a) 21679 (b) 216549
(c) 214579 (d) 218579

47. If DELHI is coded as CCIDD, how would you encode BOMBAY ?


(a) AJMTVT (b) AMJXVS
(c) MJXVSU (d) WXYZAX

48. In a certain code, RIPPLE is written as 613382 and LIFE is written as 8192. How is PILLER
written in that code ?
(a) 318826 (b) 318286
(c) 618826 (d) 338816

169 | P a g e
49. If PALAM could be given the code number 43, what code number can be given to
SANTACRUZ ?
(a) 123 (b) 85
(c) 120 (d) 125

50. Directions: The number in each question below is to be codified in the following code:
Digit 7 2 1 5 3 9 8 6 4
Letter W L M S I N D J B

50. 184632
(a) MDJBSI (b) MDJBIL
(c) MDJBWL (d) MDBJIL

51. In a certain code '256' means 'you are good', '637' means 'we are bad' and '358' means
'good and bad'. Which of the following represents 'and' in that code ?
(a) 2 (b) 5
(c) 8 (d) 3

Directions: Find odd One out of the following (52 - 76):


52. 3, 5, 7, 15, 17, 19
(a) 15 (b) 17
(c) 19 (d) 7

53. 10, 14, 16, 18, 23, 24, 26


(a) 26 (b) 23
(c) 24 (d) 18

54. 1, 4, 9, 16, 24, 25, 36


(a) 9 (b) 24
(c) 25 (d) 36

55. 41, 43, 47, 53, 61, 71, 73, 75


(a) 75 (b) 73
(c) 71 (d) 53

56. 16, 25, 36, 73, 144, 196, 225


(a) 36 (b) 73
(c) 196 (d) 225

57. 1, 4, 9, 16, 19, 36, 49


(a) 19 (b) 9
(c) 49 (d) 16

58. 1, 5, 14, 30, 49, 55, 91


(a) 49 (b) 30
(c) 55 (d) 91

59. 835, 734, 642, 751, 853, 981, 532


(a) 751 (b) 853
(c) 981 (d) 532

170 | P a g e
60. 4, 5, 7, 10, 14, 18, 25, 32
(a) 7 (b) 14
(c) 18 (d) 33

61. 52, 51, 48, 43, 31, 27, 16


(a) 27 (b) 31
(c) 43 (d) 48

62. 7, 28, 63, 124, 215, 342, 511


(a) 7 (b) 28
(c) 124 (d) 215

63. 156, 468, 780, 1094, 1404, 1760


(a) 468 (b) 780
(c) 1094 (d) 1716

64. 56, 72, 90, 108, 132


(a) 72 (b) 108
(c) 132 (d) 90

65. 5, 8, 15, 20, 29, 40, 53


(a) 15 (b) 20
(c) 29 (d) 40

66. 17, 22, 32, 45, 67, 92


(a) 22 (b) 32
(c) 45 (d) 67

67. 113, 130, 164, 215, 293, 368


(a) 130 (b) 164
(c) 215 (d) 293

68. 10, 15, 24, 35, 54, 75, 100


(a) 15 (b) 24
(c) 35 (d) 54

69. 2, 9, 28, 63, 126


(a) 2 (b) 9
(c) 28 (d) 63

70. 4, 9, 16, 25, 34, 49, 64


(a) 4 (b) 16
(c) 34 (d) 64

71. 3, 4, 12, 38, 103, 228


(a) 12 (b) 38
(c) 103 (d) 228

72. 1, 5, 14, 30, 55, 97


(a) 5 (b) 14
(c) 30 (d) 97

171 | P a g e
73. 3, 4, 9, 17, 33, 58, 94
(a) 4 (b) 9
(c) 17 (d) 33

74. 50, 51, 47, 56, 42, 65, 29


(a) 42 (b) 47
(c) 51 (d) 56

75. 2, 7, 17, 37, 67, 157, 317


(a) 7 (b) 17
(c) 37 (d) 67

76. 8, 14, 26, 48, 98, 194, 386


(a) 14 (b) 48
(c) 98 (d) 194

ANSWERS

1. 2. 3. 4. 5. 6. 7. 8. 9. 10.
(C) (D) (C) (D) (A) (B) (A) (A) (B) (A)
11. 12. 13. 14. 15. 16. 17. 18. 19. 20.
(A) (B) (B) (D) (A) (A) (B) (C) (A) (B)
21. 22. 23. 24. 25. 26. 27. 28. 29. 30.
(C) (A) (A) (C) (A) (B) (D) (A) (A) (B)
31. 32. 33. 34. 35. 36. 37. 38. 39. 40.
(B) (A) (C) (A) (A) (C) (A) (B) (A) (C)
41. 42. 43. 44. 45. 46. 47. 48. 49. 50.
(D) (A) (A) (A) (D) (C) (B) (A) (A) (D)
51. 52. 53. 54. 55. 56. 57. 58. 59. 60.
(C) (A) (B) (B) (A) (B) (A) (A) (A) (C)
61. 62. 63. 64. 65. 66. 67. 68. 69. 70.
(B) (B) (C) (B) (A) (C) (D) (C) (D) (C)
71. 72. 73. 74. 75. 76.
(B) (D) (B) (A) (D) (B)

PRACTICE QUESTIONS

1. 10, 20, 31 ? 56, 70


(a) 42 (b) 43
(c) 44 (d) 45

2. 102, 99, 104, 97, 106, ?


(a) 94 (b) 95
(c) 96 (d) 100

3. 14, 70, 350, ?, 8750, 43750 ?


(a) 785 (b) 875
(c) 1570 (d) 1750

172 | P a g e
4. 7776, 1296, 216, ?
(a) 16 (b) 21
(c) 36 (d) 108

5. 12, 6, 18, 9, 26, 13, 36, 18 ?


(a) 46 (b) 48
(c) 50 (d) None of these

6. 1, 4, 27, 256, 3125, 46656 ?


(a) 16807 (b) 117649
(c) 823543 (d) 705894

1 3 5 7
7. , , , ,?
2 4 8 16
9 10
(a) (b)
32 17
11 12
(c) (d)
36 35

8. 3, 5, 8, 12, 17, ?, 30
(a) 22 (b) 24
(c) 25 (d) 23

9. 5, 16, 29, 46, 65, 88, ?


(a) 117 (b) 115
(c) 121 (d) 134

10. 235, 346, 457, ?


(a) 468 (b) 558
(c) 568 (d) 578

11. 5690, 5121, 4552, 3983, 3414, 2845, ?


(a) 2276 (b) 2356
(c) 2516 (d) 2746

12. 2, 6, ?, 20, 30
(a) 10 (b) 12
(c) 15 (d) 22

13. 30, 46, 78, 126, 190, 270, ?


(a) 414 (b) 398
(c) 382 (d) 366

14. 3, 7, 15, 31, 63, ?


(a) 92 (b) 115
(c) 127 (d) 131

173 | P a g e
15. 13, 10, 4, –5, –17, ?
(a) –34 (b) –32
(c) –22 (d) 34

16. 6, 11, 21, 36, 56, ?


(a) 42 (b) 51
(c) 81 (d) 91

17. 104, 109, 99, 114, 94, ?


(a) 69 (b) 78
(c) 120 (d) None of these

18. 4, 16, 36, 64, 100, ?


(a) 120 (b) 136
(c) 144 (d) 180

1 3 5 7
19. , , , ,?
2 4 8 16
9 10
(a) (b)
24 24
9 11
(c) (d)
32 32

20. 3, 6, 9, 18, 27, ?, 81.


(a) 48 (b) 54
(c) 63 (d) 69

21. 1, 0.5, 0.25, 0.125, ?


(a) 0.625 (b) 0.0625
(c) 6.25 (d) 0.00625

22. 1, 2, 9, 28, 65, ?


(a) 126 (b) 182
(c) 196 (d) 245

2 3 4 5
23. , , , ,?
5 5 5 5 25
7 6
(a) (b)
25 125
6 6
(c) (d)
5 5 25 5

24. 0, 4, 18, 48, 100, ?


(a) 144 (b) 156
(c) 180 (d) 196

174 | P a g e
25. 36, 157, 301, 470, ?, 891
(a) 639 (b) 646
(c) 669 (d) 666

26. 7, 8, ?, 43, 107, 232


(a) 12 (b) 16
(c) 20 (d) 24

27. 2, 6, 33, 49, 174, 210, ?


(a) 259 (b) 274
(c) 426 (d) 553

28. 625, 5, 125, 25, 25, ?, 5


(a) 5 (b) 25
(c) 125 (d) 625

29. 15, 29, 57, 113, ? 449


(a) 215 (b) 224
(c) 226 (d) 225

30. 13, 30, 66, 140, ?, 592


(a) 210 (b) 290
(c) 428 (d) 430

31. In a certain code DESIGN is written as FCUGIL, how is REPORT written in that code?
(a) TCRMPR (b) TCRMTR
(c) TCTMPR (d) TCTNTR

32. In a certain code LOCK is written MPBJ and BLOW is written as CMNV. How is WINE written
in that code?
(a) VHOF (b) XJMD
(c) XJOF (d) VHMD

33. In a certain code HOUSE is written as FTVPI, how is CHAIR written in that code?
(a) DIBJS (b) SBJID
(c) SHBGD (d) SJBID

34. In a certain code language, the work IMAGINE written as ENIGAMI. How will the word
FLOWERS be written in that code?
(a) RESWFOL (b) SREWOLF
(c) SRFWOLE (d) WOLFSRE

35. In a certain code language the word SIMPLE is written as LEMPSI. How will the word
JUNGLE be written in that code?
(a) LENGJU (b) LEGNJU
(c) LEJUGN (d) LENGUJ

36. In a certain code language CROWD is written as DQPVE. How will BLEND be written in that
code language?
(a) CMFOE (b) CKFME
(c) AKDMC (d) AMDOC

175 | P a g e
37. In a certain code, MIGHT is written as LHFGS. How is BELOW written in that code?
(a) CFMPX (b) ADJNU
(c) ADKMV (d) ADKNV

38. If DO is written as FQ and IN is written as KP then how would AT be written?


(a) CV (b) BS
(c) CU (d) DV

39. In a code language, PINK is written as QHOJ and BOLT is written as CNMS. How would
MUST be written in that code?
(a) NVTS (b) NTTS
(c) NTRS (d) NITU

40. In a certain code CHITON is written as IHCNOT. How will DILATE be written in that code?
(a) ETALID (b) LIDATE
(c) LIDETA (d) ETADIL

41. In a certain code BROWN is written as CSPXO, how is PART written in that code?
(a) OBQU (b) QBSU
(c) OZQS (d) RBTU

42. In a certain code language LABOUR, is written as KBAPTS. How is CANDID written in that
code language?
(a) DBOEJE (b) DZDCJC
(c) BBMCHC (d) BBMEHE

43. In a certain code DROWN is written as MXNSC. How is BREAK written in that code?
(a) LBFSC (b) JBDSA
(c) JZDQA (d) LZFQC

44. In a certain code SEAL is written as $75@ and DOSE is written as #8$7. How is SOLD
written in that code?
(a) 58@# (b) #87$
(c) $8@# (d) $5@#

45. In a certain code ROPE is written as $3%6 and RITE is written as $4#6. How is PORT
written in that code?
(a) %4$# (b) $3%#
(c) $64% (d) %3$#

46. In a certain code DATE is written as #%$@ and STYLE is written as *$©↑@. How is DELAY
written in that code?
(a) #@↑%© (b) #©$%@
(c) #@$%© (d) #$↑%©

47. In GROW is written as =@%# and WITHIN is written #÷+×÷∆ in a certain code language.
How would WING be written in that code?
(a) #÷∆= (b) #%∆=
(c) %÷∆= (d) #÷©=

176 | P a g e
48. In a certain code BOND is written as 1543 and DEAN is written as 3864. How is BED written
in that code?
(a) 153 (b) 183
(c) 138 (d) 143

49. In a certain code language BREAK is written as 51342 and KITE is written as 2796. How will
RIB be written in that code language?
(a) 175 (b) 176
(c) 185 (d) 135

50. In a certain code DEAL is written as 3524 and LIE is written as 475. How is IDLE written in
that code?
(a) 7345 (b) 3745
(c) 7342 (d) 7245

51. In a certain code RAIL is written as 5796 and TAPE is written as 3748. How is PAIR written
in that code?
(a) 4795 (b) 4785
(c) 3795 (d) 8795

52. If ‘table’ is called ‘chair’, ‘chair’ is called, ‘cupboard’, ‘cupboard’ is called ‘chalk’, ‘chalk’ is
called ‘book’, ‘book’ is called ‘duster’ and ‘duster’ is called ‘table’ what does the teacher use
to write on the black board?
(a) book (b) cupboard
(c) table (d) duster

53. If ‘road’ is called ‘car’, ‘car’ is called ‘train’, ‘train’ is called ‘school’, ‘school’ is called ‘house’
and ‘house’ is called ‘office’ then where do children go for study?
(a) Car (b) School
(c) Train (d) House

54. If ‘see’ means ‘hear’, ‘hear’ means ‘feel’, ‘feel’ means ‘smell’ and ‘smell’ means ‘see’ than
what are ears used for?
(a) hear (b) See
(c) feel (d) smell

55. In a certain code language ‘pi na hu’ means ‘tea is good’ and ‘sa ni hu’ means ‘coffee is
bad’. What is the code of is?
(a) pi (b) na
(c) sa (d) hu

56. In a certain code language ‘do re me’ means ‘he is late’, ‘fa me la’ means ‘she is early’ and
‘so ti do’ means ‘he leaves soon’. Which word in that language means late’?
(a) la (b) do
(c) me (d) None of these

57 In a certain code language ‘la ke ta’ means ‘go and swim’ and ‘ne la se’ means ‘you swim
here’ and ‘pe ke ne ta’ means ‘he and you go’. Which of the following is the code for ‘here’
in that code language?
(a) Cannot be determined (b) La
(c) Ne (d) Se

177 | P a g e
58. In a certain code ‘go home’ is written as ‘ta na’ and ‘nice little home’ is written as ‘na ja pa’.
how is ‘go’ written in that code?
(a) ta (b) na
(c) ja (d) na or ta

59. In a certain code language, ‘go and come’ is written as ‘na ta ka’ and ‘black and white’ is
written as ‘pa ma ta’. How is ‘go’ written in that code language?
(a) na (b) ka
(c) pa (d) na or ka

60. In a certain code language ‘in ba pe’ means ‘he has won’, ‘le ki ba’ means ‘she has lost’ and
‘in se pe’ means ‘he always won’. Which word in that language means he’?
(a) in (b) pe
(c) se (d) Data inadequate

61. If ‘+’ means ‘÷’, ‘÷’ means ‘x’, ‘x’ means ‘–‘ and ‘–’ means ‘+’ what will be the value of the
following expression ?
15 ÷ 5 x 9 + 3 – 6 = ?
(a) 78 (b) 72
(c) 28 (d) 30

62. If ‘-‘ means ‘added to’, ‘x’ means ‘subtracted from’, ‘÷’ means ‘multiplied by’, and ‘+’ means
‘divided by’ , then
20 x 12 + 4 – 16 ÷ 5 =
(a) 17 (b) 80
(c) 63 (d) 97

63. If ‘x’ means ‘÷’, ‘÷’ means ‘+’, ‘+’ means ‘-‘ and ‘-‘ means ‘x’, then
2 – 50 + 40 x 10 ÷ 96 = ?
(a) 104 (b) 98
(c) 182 (d) 192

64. If ‘M’ means ‘÷’ ‘R’ means ‘+’, ‘T’ means ‘-‘ and ‘K’ means ‘x’ then –
20 R 16 K 5 M 10 T 8 = ?
(a) 36 (b) 20
(c) 36.5 (d) 12

65. If ‘+’ means ‘x’, ‘–‘ means ‘÷’, ‘÷’ means ‘+’ and ‘x’ means ‘-‘ then what is the value of –
540 – 36 + 12 ÷ 75 x 55 = ?
(a) 255 (b) 512
(c) 180 (d) None of these

66. If ‘M’ denotes ‘x’, ‘R’ denotes ‘-‘, ‘k’ denotes ‘+’ and ‘B’ denotes ‘÷’, then –
24 B 4 M 8 K 6 R 4 ?
4 1
(a) −3 (b) −3
7 4
(c) 52 (d) 50

67. ACF: GIL:: CEH: ?


(a) ILN (b) IKN
(c) IKM (d) ILM

178 | P a g e
68. JM : LO:: GJ: ?
(a) HK (b) HL
(c) IL (d) IK

ANSWERS

1. 2. 3. 4. 5. 6. 7. 8. 9. 10.
(B) (B) (D) (C) (B) (C) (A) (D) (A) (C)
11. 12. 13. 14. 15. 16. 17. 18. 19. 20.
(A) (B) (D) (C) (B) (C) (D) (C) (C) (B)
21. 22. 23. 24. 25. 26. 27. 28. 29. 30.
(B) (A) (D) (C) (D) (B) (D) (C) (D) (B)
31. 32. 33. 34. 35. 36. 37. 38. 39. 40.
(B) (B) (C) (B) (A) (B) (D) (A) (B) (C)
41. 42. 43. 44. 45. 46. 47. 48. 49. 50.
(B) (D) (B) (C) (D) (A) (A) (B) (A) (A)
51. 52. 53. 54. 55. 56. 57. 58. 59. 60.
(A) (A) (D) (C) (D) (D) (D) (A) (D) (D)
61. 62. 63. 64. 65. 66. 67. 68.
(A) (D) (D) (B) (D) (D) (B) (C)

179 | P a g e
CHAPTER-10 - DIRECTION SENSE TEST
Choose the appropriate answer (a) or (b) or (c) or (d)

1. Mohan starts from point A and walks 1 km towards south, turns left and walks 1 km. Then
he turns left again and walks 1 km. Now he is facing
(a) East (b) West
(c) North (d) South-west

2. Suresh starts from a point, walks 2 miles towards south, turns right and walks 1½ miles,
turns left and walks ½ miles and then he turns back. What is the direction he is facing
now ?
(a) East (b) West
(c) South (d) North

3. A man starts from a point, walks 4 miles towards north and turns left and walks 6 miles,
turns right and walks for 3 miles and again turns right and walks 4 miles and takes rest for
30 minutes. He gets up and walks straight 2 miles in the same direction and turns right and
walks one mile. What is the direction he is facing ?
(a) North (b) South
(c) Sought-east (d) West

4. Arun started from point A and walked 10 km East to point B, then turned to North and
walked 3 km to point C and then turned West and walked 12 kms to point D, then again
turned South and walked 3 kms to point E. In which direction is he from his straight point ?
(a) East (b) South
(c) West (d) North

5. A starts from a point and walks 5 kms north, then turns left and walks 3 kms. Then again
turns left and walks 5 km. Point out the direction in which he is going now.
(a) North (b) South
(c) East (d) West

6. A rat runs 20 m towards East and turns to right runs 10 m and turns to right runs 9 m and
again turns to left runs 5 m and then turns to left runs 12 m and finally turns to left and
turns 6 m. Now what direction is the rat facing ?
(a) East (b) North
(c) West (d) South

7. A driver left his village and drove North for 20 km, after which he stopped for breakfast.
Then he turned left and drove another 30 km, when he stopped for lunch. After some rest,
he again turned left and drove 20 kms before stopping for evening tea. Once more he
turned left and drove 30 kms to reach the town where he had supper. After evening tea in
which direction did he drive ?
(a) West (b) East
(c) North (d) South

180 | P a g e
8. A man is facing East, then he turns left and goes 10m, then turns right and goes 5m then
goes 5 m to the South and from there 5 m to West. In which direction is he from his original
place ?
(a) East (b) West
(c) North (d) South

9. From her home Prerna wishes to go to school. From home she goes towards North and then
turns left and then turns right, and finally she turns left and reaches school. In which
direction her school is situated with respect to her home?
(a) North-East (b) North-West
(c) South-East (d) South-West

10. A child walks 25 feet towards North, turns right and walks 40 feet, turns right again and
walks 45 feet. He then turns left and walks 20 feet. He turns left again walks 20 feet.
Finally, he turns to his left to walk another 20 feet. In which direction is the child from his
starting point ?
(a) North (b) South
(c) West (d) East

11. Raju facing North and moves 20 km, then he turns to his right and moves 20 km and then
he moves 10 km in North-East, then he turned to his right and moves 20 km and then he
turned to his right and moves 20 km and again he turned to his left and moves 20 km. Now
in which direction Raju is facing ?
(a) South-East (b) North-East
(c) South-West (d) North-West

12. K is a place which is located 2 km away in the north-west direction from the capital P. R is
another place that is located 2 km away in the south-west direction from K. M is another
place and that is located 2 km away in the North-west direction from R. T is yet another
place that is located 2 km away in the south-west direction from M. In which direction is T
located in relation to P ?
(a) South-West (b) North-West
(c) West (d) North

13. Babu is Rahim's neighbour and his house is 200 metres away in the north-west direciton.
Joseph is Rahim's neighbour and his house is located 200 meter away in the south-west
direction. Gopal is Joseph's neighbour and he stays 200 metres away in the south-east
direction. Roy is Gopal's neighbour and his house is located 200 metres away in the north-
east direction. Then where is the position of Roys' house in relation to Babu's ?
(a) South-east (b) South-west
(c) North (d) North-east

14. A tourist drives 10 km towards west and turns to left and takes a drive of another 4 km. He
then drives towards east another 4 km. and then turns to his right and drives 5 km.
Afterwards he turns to his left and travels 6 km. In which direction is tourist from the
starting point ?
(a) North (b) East
(c) West (d) South

181 | P a g e
15. A man started walking West. He turned right, then right again and finally turned left.
Towards which direction was he walking now ?
(a) North (b) South
(c) West (d) East

16. One evening, Raja started to walk toward the Sun. After walking a while, he turned to his
right and again to his right. After walking a while, he again turned right. In which direction
is he facing ?
(a) South (b) East
(c) West (d) North

17. Five boys A, B, C, D, E, are sitting in a park in a circle. A is facing South-West, D is facing
South-East, B and E are right opposite A and D respectively and C is equidistant between D
and B. Which direction is C facing ?
(a) West (b) South
(c) North (d) East

18. If a man on a moped starts from a point and rides 4 km South then turns left and rides 2
km and turn again to the right to ride to go more towards which direction is he moving ?
(a) North (b) West
(c) East (d) South

19. A man starts from a point, walk 8 km towards North, turns right and walks 12 km, turns left
and walks 7 km turns and walks 20 towards South, turns right and walks 12 km. In which
direction is he from the starting point ?
(a) North (b) South
(c) West (d) East

20. Daily in the morning the shadow of Gol Gumbaz falls on Bara Kaman and in the evening the
shadow of Bara Kaman falls on Gol Gumbaz exactly. So in which direction is Gol Gumbaz to
Bara Kaman ?
(a) Eastern side (b) Western side
(c) Northern side (d) Southern side

21. Ashok went 8 km. South and turned West and walked 3 km again he turned North and
walked 5 kms. He took a final turn to East and walked 3 kms. In which direction was Ashok
from the starting point ?
(a) East (b) North
(c) West (d) South

22. If X stands on his head with his face towards south, to which direction will his left hand
point ?
(a) East (b) West
(c) North (d) South

23. I drove East for 5 miles then drove North 3 miles, then turned to my left and drove for 2
miles and again turned to my left. Which direction am I going now ?
(a) South (b) North
(c) West (d) North-west

182 | P a g e
24. If A stands on his head with his face towards north. in which direction will his left hand
point?
(a) North-East (b) North
(c) East (d) North-West

25. A car travelling from south covers a distance of 8 km, then turns right and runs another 9
kms ad again turns to the right and was stopped. Which direction does it face now ?
(a) South (b) North
(c) West (d) East

26. A taxi driver commenced his journey from a point and drove 10 km toward north and turned
to his left and drove another 5km. After waiting to meet a friend here, he turned to his right
and continued to drive another 10 km. He has covered a distance of 25 km so far, but in
which direction would he be now ?
(a) South (b) North
(c) East (d) South-east

27. A walks 3 kms northward and then he turns left and goes 2 km. He again turns left and
goes 3 km. He turns right and walks straight. In which direction is he walking now ?
(a) East (b) West
(c) North (d) South

28. A walks south-east, then turns right, then left and then right. In which direction is he from
the starting point ?
(a) South (b) East
(c) West (d) North

29. A man starts from a point, walks 15 metres towards east, turns left and walks 10 metres,
turns right again and walks. Towards which direction is he now walking ?
(a) North (b) East
(c) West (d) South

30. A boy starts walking towards West, he turns right and again he turns right and then turns
left at last. Towards which direction is he walking now ?
(a) West (b) North
(c) South (d) East

31. I stand with my right hand extended side-ways towards South. Towards which direction will
my back be ?
(a) North (b) West
(c) East (d) South

32. If a person moves 4 km towards west, then turns right and moves 3 m and then turns right
and moves 6 km, which is the directions in which he is now moving ?
(a) East (b) West
(c) North (d) South

33. If Mohan sees the rising sun behind the temple and the setting sun behind the railway
station from his house, which is the direction of the temple from the railway station ?
(a) South (b) North
(c) East (d) West

183 | P a g e
34. Laxman went 15 km to North then he turned West and covered 10 kms. Then he turned
south and covered 5 kms. Finally turning to East he covered 10 kms. In which direction he
is from his house ?
(a) East (b) West
(c) North (d) South

35. A man starts form a point, walks 4 miles North, turns to his right and walks 2 miles, again
turns to his right and walks 2 miles, again turns to his right and walks 2 miles. In which
direction would he be now from his starting point?
(a) North (b) South
(c) East (d) West

36. I started walking down a road in the morning facing the Sun. After walking for sometime I
turned to my left. Then I turned to my right. In which direction was I going then ?
(a) East (b) West
(c) North (d) South

37. Lakshmi walked 2 furlongs north from her house and took a turn to left and continued to
walk another one kilometre and finally she turned left and reached the school. Which
direction is she facing now ?
(a) West (b) East
(c) South (d) North

38. You are going straight, first eastwards, then turn to the right, then right again, then left. In
which direction would you be going now ?
(a) East (b) West
(c) South (d) North

39. If Ahmed travels towards North from his house, then to left, then to south covering equal
distances in each direction to reach Sohan's house, in which direction is Ahmed's house
now?
(a) East (b) South
(c) North (d) West

40. You go North, turn right, then right again and then go to the left. In which direction are you
now ?
(a) South (b) East
(c) West (d) North

41. Roopa starts from a point and walks 15 metre towards west, turns left and walks 12 metre,
turns right again and walks. What is the direction she is now facing?
(a) South (b) West
(c) East (d) North

42. A man starts his journey facing the sun early morning. He then turns right and walks 2 km.
He then walks 3 km after turning right again. Which is the direction he is facing now?
(a) North-East (b) North
(c) West (d) South

184 | P a g e
43. Roy walks 2 km to East, then turns North-West and walks 3 km. Then he turns south and
walks 5 km. Then again he turns West and walks 2 km. Finally he turns North and walks 6
km. In which direction, is he from the starting point ?
(a) South-West (b) South-East
(c) North -West (d) North-East

44. Seeta starts from a point, walks 2 km towards north, turns towards her right and walks 2
km, turns right again and walks. What is the direction she is facing now ?
(a) East (b) West
(c) South (d) North

45. Shyam was facing East. He walked 5 km forward and then after turning to his right walked
3 km. Again he turned to his right and walked 4 km. After this he turned back. Which
direction was he facing at that time ?
(a) East (b) West
(c) North (d) South

46. Raju is standing facing north. He goes 30 metres ahead and turns left and goes for 15
metres. Now he turns right and goes for 50 metres and finally turns to his right and walks.
In which direction is he heading ?
(a) North (b) East
(c) South (d) West

47. Sanmitra starts from his house and walks 3 km towards north. Then he turns right and
walks 2 km and then turns right and walks 5 km, then turns right and walks 2 km and then
again turns right and walks 2 km. Which direction is he facing now ?
(a) North (b) South
(c) West (d) East

48. Raju is Ramu's neighnour and he stays 100 metres away towards southeast. Venu is Raju's
neighbour and he stays 100 metres away towards southwest. Khader is Venu's neighbour
and he stays 100 metres away towards, north-west. Then where is the position of Khader's
home in relation to Ramu's ?
(a) South-East (b) South-West
(c) North-West (d) East

49. Ramesh walked 3 km, towards West and turned to this left and walked 2 km. He, then
turned to his right and walked 3 km. Finally, he turned to his right again and walked
another 2 km. In which direction is Ramesh from his starting point now ?
(a) East (b) West
(c) North (d) South

50. Deepa starts walking towards North and after a while she turns to her right. After walking
some distance, she turns to his left and walks a distance of 1 km. She then turns to her left
again. In which direction she moving now ?
(a) North (b) West
(c) East (d) South

185 | P a g e
51. Raman starts walking in the morning facing the Sun. After sometime, he turned to the left
later again he turned to his left. At what direction is Raman moving now ?
(a) East (b) West
(c) South (d) North

52. A starts walking towards North turns left, again turns left, turns right, again turns right,
once again turns left. In which direction is A walking now ?
(a) East (b) South
(c) West (d) South-east

53. X walks southwards and then turns right, then left and then right. In which direction is he
moving now ?
(a) South (b) North
(c) West (d) South-west

54. A man started to walks East. After moving a distance, he turned to his right. After moving a
distance, he turned to his right again. After moving a little he turned in the end to his left.
In which direction was he going now ?
(a) North (b) South
(c) East (d) West

55. A man starts form a point and walks 2 km towards North, turns towards his right and walks
2 km, turns right again and walks. What is the direction now he is facing ?
(a) South (b) South-East
(c) North (d) West

56. Ramu walks 5 kms starting from her house towards west then turns right and walks 3 km.
Thereafter she takes left turn and walks 2 km. Further, she turns left and walks 3 km.
Finally, she turns right and walks 3 kms. in what direction she is now from her house ?
(a) West (b) North
(c) South (d) East

57. Gopal started walking 2 km straight from his school. Then he turned right and walked 1 km.
Again he turned right and walked 1 km to reach his house. If his house is south-east form
his school, then in which direction did Gopal start walking from the school ?
(a) East (b) West
(c) South (d) North

58. A man starts from a point, walks 2 km towards north turns towards his right and walks 2
km, turns right again and walks. What is the direction now he is facing ?
(a) South (b) East
(c) North (d) West

59. Janki started from her house and walked 2 km towards North. Then she took a right turn
and covered one kilometre. Then she took again a right turn and walked for 2 kms. In what
direction is she going ?
(a) North (b) East
(c) South (d) West

186 | P a g e
ANSWERS

1. 2. 3. 4. 5. 6. 7. 8. 9. 10.
(C) (D) (B) (C) (B) (B) (B) (C) (B) (D)
11. 12. 13. 14. 15. 16. 17. 18. 19. 20.
(A) (C) (A) (D) (A) (A) (D) (D) (B) (A)
21. 22. 23. 24. 25. 26. 27. 28. 29. 30.
(D) (B) (A) (C) (A) (B) (B) (A) (B) (B)
31. 32. 33. 34. 35. 36. 37. 38. 39. 40.
(B) (A) (C) (C) (A) (A) (C) (C) (A) (B)
41. 42. 43. 44. 45. 46. 47. 48. 49. 50.
(B) (C) (C) (C) (A) (B) (A) (B) (B) (B)
51. 52. 53. 54. 55. 56. 57. 58. 59.
(B) (C) (C) (B) (A) (A) (A) (A) (C)

187 | P a g e
CHAPTER – 11 (A) - SITTING ARRANGEMENTS
Exercise 11 (A):

Choose the appropriate answer (a) or (b) or (c) or (d)


1. Five boys A, B, C, D and E are sitting in a row. A is to the right of B and E is to the left of B
but to the right of C. A is to the left of D. Who is second from the left end? (U.P.B. Ed 2013)
(a) D (b) A
(c) E (d) B

2. There are five different house, A to E, in a row. A is to the right of B and E is to the left of C
and right of A; B is to the right of D. Which of the houses in the middle? (IB CA (IQ) 2013)
(a) A (b) B
(c) C (d) D

3. Five Friends P, Q, R, S and T are sitting in a row facing North. Here S is between T and Q
and Q is to the immediate left of R. P is to the immediate left of T. What is in the middle?
(SSC(Multi Task) 2014).
(a) S (b) T
(c) Q (d) R

4. Six, children A, B, C, D, E and F are standing in a row. B is between F and D. E is between A


and C. A does not stand next to F or D. C does not stand next to D. F is between which of
the following pairs of children? (SSC (FCI) 2012)
(a) B and E (b) B and C
(c) B and D (d) B and A

5. There are eight books kept one over the other. Two books are on Organisation Behaviour,
two books on TQM, three books on Industrial Relations and one book is on Economics
Counting. From the top, the second, fifth and sixth books are on Industrial Relations. Two
books on Industrial Relations are between two books on TQM. One book of Industrial
Relations is between two books on Organizational Behaviour while the books above the book
of Economics is a book of TQM., Which book is the last book from the top ? (MAT 2011)
(a) Economics (b) TQM
(c) Industrial Relations (d) Organizational Behaviour

6. Five boys are standing in a row facing East. Pavan is left of Tavan, Vipin and Chavan to the
left of Nakul. Chavan is between Tavan and Vipin. Vipin is fourth from the left, then how far
is Tavan to the right ? (CLAT 2014)
(a) First (b) Second
(c) Third (d) Fourth

7. Six persons M, N, O, P, Q and R are sitting in two row with three persons in each row, Both
the row are in front of each other. Q is not at the end of any row. P is second the left of R. O
is the neigbbour of Q and diagonally opposite to P. N is the neigbour of R. Who is in front N
? (UPSC (CSAT) 2011)
(a) R (b) Q
(c) P (d) M

188 | P a g e
8. Six person A, B, C, D, E and F are sitting in two row, three in each row. (MAT 2011)
(i) E is not at the end of any row
(ii) D is second to the left of F
(iii) C, the neighbour of E, is sitting diagonally opposite of D.
(iv)B is the neighbour of F.
Which of the following are in one of the two rows ?
(a) D, B and F (b) C, E and B
(c) A, E and F (d) F, B

Direction (Q.No. 9): Read the following information's carefully and answer that question
that follows:
Five boys A1, A2, A3, A4 and A5 are sitting in a stair in the following way. (RRB (TC/CC) 2010)
(i) A5 is above A1
(ii) A4 is under A2
(iii) A2 is under A1
(iv)A4 is between A2 and A3.

9. Who is at the lowest position of the stair ?


(a) A1 (b) A3
(c) A5 (d) A2

10. Five children are sitting in a row. S is sitting next to P but not T, K is sitting next to R, who
is sitting on the extreme left and T is not sitting next to K. Who is/are adjacent to S ? (NIFT
(UG) 2014)
(a) K and P (b) R and P
(c) Only P (d) P and T

11. P, T, V, R, M, D, K and W are sitting around a cricular table facing the centre. V is second to
the left of T. T is fourth to the right of M. D and P are not immediate neighbours of T. D is
third to the right of P. W is not an immediate neighbuor of P. P is to the immediate left of K.
What is R's position with respect to V ?
(a) Third to the right (b) Fifth to the right
(c) Third to the left (d) Second to the left

12. In a gathering seven members are sitting in a row. 'C' is sitting left to 'B' but on the right to
'D'. 'A' is sitting right to 'B', 'F is sitting right to 'E' but left to 'D'. 'H' is sitting left to 'E'. Find
the person sitting in the middle (SSC (10+2)2013)
(a) C (b) D
(c) E (d) F

Directions (No.: 13-17) : Study the following information carefully to answer the given
questions.
A to H are seated in straight line facing North. C sits fourth left of G. D sits second to right of G.
Only two people sit between D and A. B and F are immediate neigbours of each other. B is not an
immediate neighbour of A. H is not neighbour of D. (GIC 2012)

13. Who amongst the following sits exactly in the middle of the persons who sit fifth from the
left and the person who sit sixth from the right ?
(a) C (b) H
(c) E (d) F

189 | P a g e
14. Who amongst the following sits third to the right of C ?
(a) B (b) F
(c) A (d) E

15. Which of the following represents persons seated at the two extreme ends of the line ?
(a) C, D (b) A, B
(c) B, G (d) D, H

16. What is the position of H with respect to F ?


(a) Third to the left (b) Immediate right
(c) Second to right (d) Fourth to left

17. How many persons are seated between A and E?


(a) One (b) Two
(c) Three (d) Four

Directions (Q.No. 18-22)


Study the following information carefully to answer the given questions:
M, D, P, K, R, T and W are sitting around a circle facing the centre. D is second to the right of P,
who is third to the right of K. T is third to the right of W, who is not an immediate neighbour of D.
M is third to the left of R.

18. Who is second to the right of T ?


(a) D (b) K
(c) M (d) None of these

19. In which of the following pairs is the second person sitting to the immediate right of the first
person ?
(a) DT (b) TP
(c) PR (d) KW

20. Who is on the immediate left of R ?


(a) W (b) P
(c) K (d) T

21. Who is on the immediate left of M ?


(a) K (b) W
(c) D (d) T

22. Who is third to the left of D ?


(a) W (b) P
(c) K (d) None of these

Directions (Q. Nos. 23-24) Read the following information carefully and then answer the
questions that follow.
Five plays A, B, C, D and E were organised in a week from Monday to Saturday with one play each
day and no play was organised on one of these days. Play D was organised before Thursday but
after Monday. Play E was organised on Saturday. Play C was not organised on the first day. Play B
was organised on the next day on which play C was organise. Play A was organised on Tuesday.

190 | P a g e
23. On which day was play B organised ?
(a) Thursday (b) Friday
(c) Wednesday (d) None of these

24. On which day was no play organised ?


(a) Monday (b) Wednesday
(c) Thursday (d) Friday

Directions (Q. Nos. 25-27) Read the following information carefully and then answer the
questions that follow.
Eight persons T, R, M, Q, U, P, V and W are sitting in front of one another in two rows. Each row
has four persons. P is between U and V and facing North. Q, who is to the immediate left of M is
facing W. R is between T and M, W is to the immediate right of V.
25. Who is sitting in front of R ?
(a) U (b) Q
(c) V (d) P

26. Who is to the immediate right of R ?


(a) M (b) U
(c) V (d) None of these

27. In which of the following pairs, persons are sitting in front of each other?
(a) MV (b) RV
(c) TV (d) UR

28. Four girls A, B, C, D are sitting around a circle facing the centre. B and C infront of each
other, which of the following is definitely true ? (MAT 2009)
(a) A and D in front of each other (b) A is not between B and C
(c) D is left of C (d) A is left of C

29. Four children are sitting in a row. A is occupying seat next to B but not next to C. If C is not
sitting next to D, who is occupying seat next to adjacent to D?
(a) B (b) B and A
(c) Impossible to tell (d) A

30. There are Five houses P, Q, R, S, T. P is right of Q and T is left of R and right of P. Q is right
of S. Which house in the middle.
(a) P (b) Q
(c) R (d) T

31. Friends are sitting on a bench. A is to the left of B but on the right of C, D is to the right of B
but one the left of E. Who are at the extremes ?
(a) A, B (b) A, D
(c) C, E (d) B, D

32. In a college party, 5 girls are sitting in a row. P is to the left of M and to the right of O. R is
sitting to the right of N but to the left of O. Who is sitting in the middle?
(a) O (b) R
(c) P (d) M

191 | P a g e
33. Five boys A, B, C, D and E are standing in a row. D is on the right of E, B is on the left of E
but on the right of A. D is one the left of C, who is standing on the extreme right. Who is
standing in the middle ?
(a) B (b) C
(c) D (d) E

(Q. Nos. 34 to 36) Study the following Question carefully and answer the given
questions.
Four ladies A, B, C and D and Four Gentlemen E, F, G and H are sitting in a circle around a table
facing each other.
(i) No two ladies or gentlemen are sitting side by side.
(ii) C, who is sitting between G and E, is facing D.
(iii) F is between D and A and facing G.
(iv) H is to the right of B.

34. Who is sitting left of A ?


(a) E (b) F
(c) G (d) H

35. E is facing whom ?


(a) F (b) B
(c) G (d) H

36. Who is immediate neighbour of B ?


(a) G and H (b) E and F
(c) E and H (d) F and H

37. A, B, C, X, Y, Z are seated in a straight line facing North. C is third to the right of Z and B
sits second to the right of C. X sits to the immediate right of A. How many persons are
seated between A and C ?
(a) One (b) Two
(c) Three (d) Four

(Q.38) Five People A, B, C, D and E are seated about a round table. Every chair is spaced
equidistant from adjacent chairs. (UPSC (CSAT) 2013)
(i) C is seated next to A.
(ii) A is seated two seats from D.
(iii) B is not seated next to A.
Which of the following must be true ?
(i) D is seated next to B.
(ii) E is seated next to A.

38. Select the correct from the options given below:


(a) Only (i) (b) Only (ii)
(c) Both (i) and (ii) (d) Neither (i) nor (ii)

192 | P a g e
Question 39 to 43 : Study the following Question carefully and answer the given
questions.
Eight friends A, B, C, D, E, F, G and H are sitting in a circle facing centre, not necessarily in the
same order. D sits third to the left of A. E sits to the immediate right of A. B is third to left of D. G
is second to the right of B. C is neigbour of B. C is third to left of H. (GIC 2012)
39. Who amongst the following is sitting exactly between F and D ?
(a) C (b) E
(c) H (d) A

40. Three of the following four are alike in a certain way based on the information given above
and so form a group. Which is does not belong to that group.
(a) DC (b) AH
(c) EF (d) DF

41. Who amongst the following second to the left of H ?


(a) E (b) B
(c) A (d) None of these

42. Who amongst the following are immediate neighbours of G ?


(a) CA (b) AF
(c) DC (d) DF

43. Who amongst the following is sitting third to the right of A ?


(a) F (b) B
(c) H (d) C

ANSWERS

Seating Arrangements ANSWER 11 (A):


1. 2. 3. 4. 5. 6. 7. 8. 9. 10.
(C) (A) (A) (B) (A) (D) (B) (A) (B) (A)
11. 12. 13. 14. 15. 16. 17. 18. 19. 20.
(A) (B) (D) (C) (D) (A) (A) (C) (D) (A)
21. 22. 23. 24. 25. 26. 27. 28. 29. 30.
(C) (D) (B) (A) (D) (D) (A) (A) (D) (A)
31. 32. 33. 34. 35. 36. 37. 38. 39. 40.
(C) (A) (D) (B) (D) (A) (A) (C) (C) (D)
41. 42. 43.
(D) (C) (D)

193 | P a g e
CHAPTER 11 (B) - SITTING ARRANGEMENTS

Directions (Q 1–5): Study the following carefully and answer the questions given below:
A, B, C, D, E, F, G, H and K are sitting around a circle facing the centre. B is fourth to the
left of G, who is second to the right of C. F is fourth to the right of C and is second to the left of K.
A is fourth to the right of K. D is not an immediate neighbour of either K or B. H is third to the right
of E.

1. In which of the following combinations is the third person sitting between the first and the
second persons?
(a) EKB (b) CHB
(c) AGC (d) FGD
(e) None of these

2. who is fourth to the left of E ?


(a) A (b) C
(c) G (d) Data inadequate
(e) None of these

3. Who is the second to the right of K ?


(a) C (b) H
(c) F (d) E
(e) Data inadequate

4. Who is third to the right of H ?


(a) A (b) D
(c) G (d) F
(e) None of these

5. Who is fourth to the right of D ?


(a) K (b) H
(c) E (d) B
(e) None of these

Directions (Q. 6–10): Study the following carefully and answer the questions given
below:
A, B, C, D, E, F, G, H and K are sitting around a circle facing the centre. F is fourth to the
right of A, who is third to the right of B. K is fourth to the left of B and third to the right of D. C is
third to the right of H. E is second to the left of G.

6. Who is fourth to the left of G ?


(a) C (b) A
(c) D (d) K
(e) Data inadequate

194 | P a g e
7. What is E’s position with respect to B ?
(a) Second to the left (b) Third to the right
(c) Fourth to the right (d) Third to the left
(e) Fifth to the right

8. Who is third to the right of K ?


(a) F (b) E
(c) G (d) Data inadequate
(e) None of these

9. Who is on the immediate right of F ?


(a) B (b) G
(c) E (d) Data inadequate
(e) None of these

10. In which of the following combinations is the third person sitting between the first and the
second person ?
(a) GFB (b) BGH
(c) ADC (d) KEC
(e) EGF

Directions (Q 11–15): Study the following carefully and answer the questions given
below:
Eight friends P, Q, R, S, T, U, V and W are sitting around a circle facing the centre. V is third
to the right of Q and is second to the left of R. Q is second to the left of T and on the immediate
right of S. U is between Q and T. P is not on the left of R.

11. Who is on the immediate left of S ?


(a) Q (b) P
(c) U (d) V
(e) None of these

12. Who is second to the right of T ?


(a) R (b) P
(c) W (d) V
(e) None of these

13. Which of the following pairs of persons has the first person sitting on the immediate right to
the second person ?
(a) UT (b) WR
(c) WT (d) VT
(e) None of these

14. Which of the following is the correct position of W ?


(a) On the immediate left to V (b) On the immediate right to V
(c) Between U & V (d) On the immediate right to R
(e) None of these

195 | P a g e
15. Who is second to the right of S ?
(a) U (b) R
(c) T (d) P
(e) None of these

Directions (Q 16–21): Study the following carefully and answer the questions given
below:
Ten people are sitting in two parallel rows containing five people each, in such a way that
there is an equal distance between adjacent persons. In row – 1 P, Q, R, S and T are seated and all
of them are facing South. In rwo-2 A, B, C, D and E are seated and all of them are facing North.
Therefore, in the given seating arrangement each member seated in row faces another member of
the other row.
D sits third to the left of A, P faces immediate neighbour of D. R sits second to the right of
P. only one person sits between Q and S. B and E are immediate neighbours of each other. E does
not face P and Q.

16. How many persons are seated between Q and T ?


(a) None (b) One
(c) Two (d) Three
(e) Cannot be determined

17. Four of the following five are alike in a certain way and thus form a group. Which is the one
that does not belong to that group ?
(a) R (b) S
(c) C (d) T
(e) A

18. Who amongst the following represent the people sitting exactly in the middle of the rows ?
(a) P, E (b) S, D
(c) S, A (d) A, R
(e) P, B

19. Which of the following is true regarding B ?


(a) A and C are immediate neighbour of B
(b) B sits at one of the extreme ends of the line
(c) Q faces B
(d) T is an immediate neighbour of the person facing B
(e) D sits to the immediate left of B

20. Four of the following five are alike in a certain way and thus from a group. Which is the one
that does not belong to that group ?
(a) T-E (b) Q-C
(c) S-B (d) R-A
(e) P-D

196 | P a g e
21. Who amongst the following faces S ?
(a) A (b) B
(c) C (d) D
(e) E

Directions (Q 22–25): Study the following carefully and answer the questions given
below:
Seven friends T, U, V, W, X, Y and Z are sitting in a straight line facing north. W sits fifth to the
right of T. W does not sit at any of extreme ends. Two people sit between Z and X. Y sits third to
the left of U. Y sits exactly in the middle. Z is not an immediate neighbour of Y.

22. What is Z’s position with respect to W ?


(a) Second to the left (b) Third to the right
(c) Fourth to the left (d) Third to the left
(e) Fourth to the right

23. Who is second to the right of T ?


(a) Y (b) X
(c) U (d) V
(e) None of these

24. Four of the following five are alike in a certain way based on their seating positions in the
above line and so form a group. Which is the one that does not belong to the group ?
(a) UW (b) XV
(c) ZT (d) YV
(e) WX

25. If all the seven friends are made to sit alphabetically from right to left, positions of how
many will remain unchanged ?
(a) None (b) One
(c) Two (d) Three
(e) Four

Directions (Q 26–30): Study the following carefully and answer the questions given
below:
Seven friends – L, M, N, O, P, Q and R are sitting in a straight line facing North, not
necessarily in the same order. M sits fifth to the right of O. P sits third to the right of L. Both L and
P do not sit at the extreme ends of the line. Q and R are immediate neighbours of each other. N
sits third to the left of Q.

26. What is O’s position with respect of R ?


(a) Second to the right (b) Third to the left
(c) Second to the left (d) Third to the right
(e) None of these

27. Which of the following represents the friends sitting at the extreme ends of the line ?
(a) O, M (b) Q, O
(c) N, M (d) O, N
(e) None of these

197 | P a g e
28. If all the seven friends are made to sit in alphabetical order from left to right, the positions
of how many will remain unchanged ?
(a) Four (b) Three
(c) One (d) Two
(e) None

29. Who sits exactly in the middle of the row ?


(a) P (b) L
(c) Q (d) R
(e) None of these

30. Four of the following five are alike in a certain way based on their seating positions in the
above arrangement seating positions in the above arrangement and so form a group. Which
is the one that does not belong to the group ?
(a) MP (b) RQ
(c) ON (d) LN
(e) QL

Directions (Q 31–35): Study the following information carefully and answer the
questions given below:
A building has eight floors numbered one to eight, in such a manner that the ground floor is
numbered one, the floor above it, numbered two and so on such that the topmost floor is
numbered eight. One of the eight persons, viz, P, Q, R, S, T, U, V and W lives on each floor. R lives
on third numbered floor. Only two persons live between the floors of R and V. W lives on the floor
immediately above the floor of Q. only one person lives between the floors of T & U. T lives above
U. only one person lives between the floors of R and S. S lives on any floor below the floor of T.

31. Who among the following lives on the fifth numbered floor ?
(a) S (b) Q
(c) W (d) P
(e) V

32. Who among the following lives exactly between the floors of R and S ?
(a) U (b) P
(c) V (d) T
(e) W

33. Who among the following lives on the top most floor ?
(a) T (b) Q
(c) W (d) R
(e) V

34. Four of the following five are alike in a certain way and hence they form a group. Which one
of the following does not belong to that group ?
(a) R (b) V
(c) P (d) S
(e) Q

198 | P a g e
35. How many persons live between the floors of Q and U ?
(a) None (b) One
(c) Two (d) Three
(e) Four
Directions (Q 36–40): Study the following information carefully and answer the given
questions:
A, B, C, D, E and F are sitting in straight line facing North.
(1) A sits third to the right of D.
(2) Neither A nor D sits at any of the extreme ends of the line.
(3) C sits second to left of E.
(4) B sits second to right of F.
(5) B does not sits at an extreme end of the line.

36. If all the persons are made to sit in alphabetical order from right to left, the positions of how
many will remain unchanged as compared to the original seating positions ?
(a) None (b) One
(c) Two (d) Three
(e) Four

37. Who sits at the extreme right hand corner of the line ?
(a) C (b) E
(c) F (d) Either C or E
(e) None of these

38. What is the position of B with respect to E ?


(a) Third to the left (b) Third to the right
(c) Immediately to the right (d) Second to the left
(e) Immediately to the left

39. Four of the following five are alike in a certain way based on their seating positions in the
above arrangement and so form a group. Which is the one that does not belong to that
group ?
(a) CB (b) EA
(c) DF (d) BC
(e) AC

40. How many persons sit between C and F ?


(a) None (b) One
(c) Two (d) Three
(e) Four

Directions (Q 41–47): Study the following information carefully and answer the given
questions:
Eight friends – A, B, C, D, E, F, G and H are sitting around a circle facing the centre, but not
necessarily in the same order. D sits third to left of A. A is an immediate neighbour of both F and
H. only one person sits between C and F. B is not an immediate neighbour of D. only one person
sits between B and G.

199 | P a g e
41. A is related to G in a certain way. Similarly, C is related to H, according to the given seating
arrangement. Who among the following is related to F, following the same pattern?
(a) A (b) B
(c) C (d) D
(e) E
42. Who among the following sits second to the right of C ?
(a) F (b) A
(c) D (d) G
(e) H
43. What is the position of C with respect to the position of E ?
(a) Third to the left (b) Second to the left
(c) Immediate right (d) Third to the right
(e) Second to the right
44. Which of the following is true with respect to the given seating arrangement ?
(a) A sits on the immediate left of H
(b) B sits exactly between C and G.
(c) F sits second to the right of C
(d) E is an immediate neighbour of C.
(e) None of these
45. Which of the following pairs represents the immediate neighbours of G ?
(a) A, C (b) C, D
(c) D, H (d) D, E
(e) C, F
46. Who among the following sits exactly between C and F ?
(a) A (b) D
(c) G (d) H
(e) B
47. Starting from A, if all the friends are made to sit in alphabetical order in clock wise direction,
the position of how many (Excluding A) will remain unchanged ?
(a) None (b) One
(c) Two (d) Three
(e) Four

Directions (Q 48–52): Study the following information carefully and answer the given
questions:
A, B, C, D, E, F, G and H are sitting around a circular table facing the centre but not
necessarily in the same order.
(a) G sits third to the right of F.
(b) G sits second to the left of H.
(c) H is not an immediate neighbour of E and C.
(d) F and A are immediate neighbours of each other.
(e) Only one person sits between F and B.
(f) E and G sit opposite each other.

48. Four of the following five are similar in a certain way based on their position in the seating
arrangement. Which of the following does not belong to that group ?
(a) EH (b) CF
(c) AG (d) BD
(e) HG

200 | P a g e
49. Who amongst the following represent immediate neighbours of D ?
(a) E, H (b) C, G
(c) F, B (d) H, G
(e) C, B

50. What is the position of A with respect to G in the above arrangement ?


(a) Third to the right (b) Second to the left
(c) Fifth to the right (d) Immediate right
(e) Third to the left

51. What will come in place of the question mark (?) according to the above seating
arrangement?
EF CG HB FA ?
(a) GD (b) CG
(c) GH (d) BH
(e) CD

52. Who sits to the immediate right of A ?


(a) E (b) C
(c) B (d) F
(e) D

ANSWERS

Seating Arrangements ANSWER 11(b):


1. 2. 3. 4. 5. 6. 7. 8. 9. 10.
(D) (A) (B) (C) (D) (A) (D) (C) (B) (E)
11. 12. 13. 14. 15. 16. 17. 18. 19. 20.
(B) (C) (D) (B) (A) (C) (B) (E) (E) (D)
21. 22. 23. 24. 25. 26. 27. 28. 29. 30.
(E) (C) (D) (B) (A) (B) (C) (E) (C) (D)
31. 32. 33. 34. 35. 36. 37. 38. 39. 40.
(D) (A) (C) (B) (E) (C) (B) (A) (D) (C)
41. 42. 43. 44. 45. 46. 47. 48. 49. 50.
(D) (C) (A) (E) (B) (E) (B) (C) (D) (B)
51. 52.
(A) (B)

201 | P a g e
CHAPTER – 12 - BLOOD RELATIONS
Choose the appropriate answer (a) or (b) or (c) or (d):

1. A is B's brother. C is A's mother. D is C's father, E is B's son. How is D related to A?
(a) Son (b) Grandson
(c) Grandfather (d) Great Grandfather

2. As is B's brother. C is A's father. D is C's sister and E is D's mother. How is B related to E ?
(a) Grand-daughter (b) Great Grand daughter
(c) Grandaunt (d) Daughter

3. A is B's Sister. C is B's Mother. D is C's Father. E is D's Mother. Then how is A related to D ?
(a) Grandmother (b) Grandfather
(c) Daughter (d) Grand-daughter

4. A is the father of B. C is the daughter of B. D is brother of B. E is the son of A. What is the


relationship between C and E ?
(a) Brother and sister (b) Cousins
(c) Niece and uncle (d) Uncle and Aunt

5. If P is the husband of Q and R is the mother of S and Q. What is R to P ?


(a) Mother (b) Sister
(c) Aunt (d) Mother-in-law

6. P and Q are brother R and S are sister. P's son is S's brother. How is Q related to R ?
(a) Uncle (b) Brother
(c) Father (d) Gandfather

7. X is the husband of Y. W is the daughter of X. Z is husband of W. N is the daughter of Z.


What is the relationship of N to Y ?
(a) Cousin (b) Niece
(c) Daughter (d) Grand-daughter

8. A reads a book and find the name of the author familiar. The author 'B' is the paternal uncle
of C. C is the daughter of A. How is B related to A ?
(a) Brother (b) Sister
(c) Father (d) Uncle

9. A's mother is sister of B and she has a daughter C who is 21 years old. How is B related to
C?
(a) Uncle (b) Maternal Uncle
(c) Niece (d) Daughter

10. A is B's brother. C is A's mother. D is C's father. F is A's son. How is F related to D?
(a) Son (b) Grandson
(c) Great-grandson (d) Grand-daughter

202 | P a g e
11. A is B's brother. C is A's mother. D is C's father. E is B's son. How is B related to D ?
(a) Son (b) Grand-daughter
(c) Grandfather (d) Great grandfather

12. A is B's brother. C is A's mother. D is C's father. F a is A's son. How is B related to F's
child ?
(a) Aunt (b) Cousin
(c) Nephew (d) Grandfather

13. A is B's daughter. B is C's mother. D is C's brother. How is D related to A ?


(a) Father (b) Grandfather
(c) Brother (d) Son

14. A is D's brother. D is B's father. B and C are sisters. How is C related to A ?
(a) Cousin (b) Niece
(c) Aunt (d) Nephew

15. A is B's brother. C is A's mother, D is C's father. E is B's son. How is D related to E ?
(a) Grandson (b) Great Grandson
(c) Great Grandfather (d) Grandfather

16. X and Y are the children of A. A is the father of X but Y is not his son. How is Y related to A?
(a) Sister (b) Brother
(c) Son (d) Daughter

17. A is B's brother. C is A's mother. D is C's father E is B's son. How is E related to A?
(a) Cousin (b) Nephew
(c) Uncle (d) Grandson

18. Based on the statements given below, find out who is the uncle of P ?
(i) K is the brother of J
(ii) M is the sister of K
(iii) P is the brother of N
(iv)N is the daughter of J
(a) K (b) J
(c) N (d) M

19. A and B are sisters. A is mother of D. D has a daughter C who is married to F. G is the
husband of A. How is B related to G ?
(a) Cousin (b) Niece
(c) Aunt (d) Sister-in-law

20. R and S are brothers. X is the sister of Y and X is mother of R. What is Y to S ?


(a) Uncle (b) Brother
(c) Father (d) Mother

21. A is B's brother. C is A's mother. D is C's father. B is D's grand-daughter. How is B related
to C ?
(a) Daughter (b) Cousin
(c) Niece (d) Grandaunt

203 | P a g e
22. A is the son of B while B and C are sister to one another. E is the mother of C. If D is the
son of E, which of the following statements is correct?
(a) D is the maternal uncle of A (b) E is the brother of B
(c) D is the cousin of A (d) B and D are brothers.

23. P is the father of T. T is the daughter of M. M is the daughter of K. What is P to K?


(a) Father (b) Father-in-law
(c) Brother (d) Son-in-law

24. A and B are brothers. E is the daughter of F. F is the wife of B. What is the relation of E to
A?
(a) Sister (b) Daughter
(c) Niece (d) Daughter-in-law

25. M and F are a married couple. A and B are sisters. A is the sister of F. Who is B to M ?
(a) Sister (b) Sister-in-law
(c) Niece (d) Daughter

26. If A is the mother of D. B is not the son of C. C is the father of D, D is the sister of B, then
how is A related to B ?
(a) Mother (b) Brother
(c) Step Son (d) Sister

27. A and B are brother and sister respectively. C is A's father. D is C's sister and E is D's
mother. How is B related to E ?
(a) Grand-daughter (b) Great grand-daughter
(c) Aunt (d) Daughter

28. Q is the son of P. X is the daughter of Q. R is the aunty (Bua) of X and L is the son of R,
then what is L to P ?
(a) Grandson (b) Grand-daughter
(c) Daughter (d) Nephew

29. P and Q are brothers. R and S are sisters. P's son is S's brother. How is Q related to R ?
(a) Uncle (b) Brother
(c) Father (d) Grandfather

30. A and B are the young ones of C. If C is the mother of B but A is not the daughter of C, then
what is the relationship between C and A ?
(a) Nephew and Aunty (b) Brother and Sister
(c) Mother and son (d) Niece and Aunty

31. A is the mother of D and sister of B. B has a daughter C who is married to F. G is the
husband of A. How is G related to D ?
(a) Uncle (b) Husband
(c) Son (d) Father

32. Pointing towards A, B said "your mother is the younger sister of my mother". How is A
related to B ?
(a) Uncle (b) Cousin
(c) Nephew (d) Father

204 | P a g e
33. A is B's wife's husband's brother. C and D are sisters of B. How is A related to C ?
(a) Brother (b) Sister-in-law
(c) Wife (d) Sister

34. A and B are brothers. C and D are sisters. A's son is D's brother. How is B related to C?
(a) Father (b) Brother
(c) Uncle (d) Son

35. A is B's sister. C is B's mother. D is C's father. E is D's mother. Then how is A related to D ?
(a) Grandmother (b) Grandfather
(c) Daughter (d) Grand-daughter

36. P, Q, R, S, T, U are 6 members of a family in which there are two married couples. T, a
teacher is married to a doctor who is mother of R and U. Q the lawyer is married to P. P has
one son and one grandson of the two married ladies one is housewife. There is also one
student and one male engineer in the family. Which of the following is true about the grand-
daughter of the family?
(a) She is a lawyer (b) She is an engineer
(c) She is a student (d) She is a doctor

37. Six members of a family namely A, B, C, D, E and F are travelling together. 'B' is the son of
C but C is not the mother of B. A and C are married couple. E is the brother of C, D is the
daughter of A. F is the brother of B. How many male members are there in the family ?
(a) 3 (b) 2
(c) 4 (d) 1

38. A's mother is sister of B and has a daughter C. How can A be related to B from among the
following ?
(a) Niece (b) Uncle
(c) Daughter (d) Father

39. Rajiv is the brother of Atul. Sonia is the sister of Sunil. Atul is the son of Sonia. How is
Rajiv related to Sonia ?
(a) Nephew (b) Son
(c) Brother (d) Father

40. Sita is the niece of Ashok. Ashok's mother is Lakshmi. Kalyani is Lakshmi's mother.
Kalyani's husband is Gopal. Parvathi is the mother-in-law of Gopal. How is Sita related to
Gopal ?
(a) Great grandson's daughter
(b) Gopal is Sita's father
(c) Sita is Gopal's great grand-daughter
(d) Grand niece

41. Seema is the daughter-in-law of Sudhir and sister-in-law of Ramesh. Mohan is the son of
Sudhir and only brother of Ramesh. Find the relation between Seema and Mohan.
(a) Sister-in-law (b) Aunt
(c) Cousin (d) Wife

205 | P a g e
42. Suresh introduces a man as "He is the son of the woman who is the mother of the husband
of my mother". How is Suresh related to the man ?
(a) Uncle (b) Son
(c) Cousin (d) Grandson

43. Pointing to a lady in a photograph. Meera said. "her father's only son's wife is my mother-
in-law "How is Meera's husband related to that lady in the photo ?
(a) Nephew (b) Uncle
(c) Son (d) Father

44. Pointing to a photograph Vikas said "She is the daughter of my grandfather's only son". How
is the lady in the photograph related to Vikas ?
(a) Father (b) Brother
(c) Sister (d) Mother

45. Suresh's sister is the wife of Ram. Ram is Rani's brother. Ram's father is Madhur. Sheetal is
Ram's grandmother. Rema is Sheetal's daughter-in-law. Rohit is Rani's brother's son. Who
is Rohit to Suresh ?
(a) Brother-in-law (b) Son
(c) Brother (d) Nephew

46. Vinod introduces Vishal as the son of the only brother of his father's wife. How is Vinod
related to Vishal ?
(a) Cousin (b) Brother
(c) Son (d) Uncle

47. Among her children, Ganga's favorites are Ram and Rekha. Rekha is the mother of Sharat,
who is loved most by his uncle Mithun. The head of the family is Ram Lal, who is succeeded
by his sons Gopal and Mohan. Gopal and Ganga have been married for 35 years and have 3
children. What is the relation between Mithun and Mohan ?
(a) Uncle (b) Son
(c) Brother (d) No relation

48. Rahul and Robin are brothers. Promod is Robin's father. Sheela is Pramod's sister. Prema is
Promod's niece. Shubha is Sheela's grand-daughter. How is Rahul related to Shubha?
(a) Brother (b) Cousin
(c) Uncle (d) Nephew

49. Preeti has a son, named Arun. Ram is Preeti's brother. Neeta too has a daughter named
Reema. Neeta is Ram's sister. What is Arun's relationship to Reema?
(a) Brother (b) Nephew
(c) Cousin (d) Uncle

50. There are 2 film stars. One is the father of the other's son. What is the relationship of the
two will each other?
(a) Grandfather and Grandson (b) Grandfather and son
(c) Husband and wife (d) Father and son

206 | P a g e
51. Ramu's mother said to Ramu, "My mother has a son whose son is Achyut". How is Achyut
related to Ramu ?
(a) Uncle (b) Cousin
(c) Brother (d) Nephew

52. Ravi's father has a son Rohit who has an aunt Laxmi who has a husband Rao whose father-
in-law is Mohan. What is the relation of Mohan to Ravi ?
(a) Nephew (b) Grandfather
(c) Son (d) Uncle

53. Vijay says, Ananda's mother is the only daughter of my mother". How is Ananda relation to
Vijay ?
(a) Brother (b) Father
(c) Nephew (d) Grandfather

54. Introducing a man, a woman said, "His wife is the only daughter of my mother." How is the
woman related with the man ?
(a) Sister-in-law (b) Wife
(c) Aunt (d) Mother-in-law

55. A prisoner introduced a boy who came to visit him to the jailor as "Brothers and sisters I
have none, he is my father's son's son". Who is the boy ?
(a) Nephew (b) Son
(c) Cousin (d) Uncle

56. A is B's daughter, B is C's mother. D is C's brother. How is D related to B ?


(a) Father (b) Grandfather
(c) Brother (d) Son

57. P is Q's brother. R is Q's mother. S is R's father. T is S's mother. How is P related to T ?
(a) Grand-daughter (b) Great grandson
(c) Grandson (d) Grandmother

58. A is B's brother. C is D's father. E is B's mother. A and D are brothers. How is E related to
C?
(a) Sister (b) Sister-in-law
(c) Niece (d) Wife

59. A is the sister of B. B is the brother of C. C is the son of D. How is D related to A?


(a) Mother (b) Daughter
(c) Son (d) Uncle

60. B is the brother of A, whose only sister is mother of C. D is maternal grandmother of C. How
is A related to D ?
(a) Daughter-in-law (b) Daughter
(c) Aunt (d) Nephew

207 | P a g e
61. A and B are sisters. R and S are brothers. A's daughter is R's sister. What is B's relation to
S?
(a) Mother (b) Grandmother
(c) Sister (d) Aunt

62. E is the sister of B. A is the father of C. B is the son of C. How is A related to E?


(a) Grandfather (b) Grand-daughter
(c) Father (d) Great-grandfather

63. Given that:


A is the mother of B.
C is the son of A.
D is the brother of E.
E is the daughter of B.
Who is grandmother of D ?
(a) E (b) B
(c) C (d) A

64. A is D's brother. D is B's father. B and C are sisters. How is A related to C ?
(a) Son (b) Grandson
(c) Father (d) Uncle

65. A is B's sister. C is B's mother. D is C's father. E is D's mother, then how A is related to D ?
(a) Grandfather (b) Daughter
(c) Grandmother (d) Granddaughter

66.
(i) F is the brother of A.
(ii) G is the son of A.
(iii) K is the sister of F.
(iv)G is the brother of C.
Who is the uncle of G ?
(a) A (b) C
(c) K (d) F

67. A is father of C and D is son of B. E is brother of A. If C is sister of D how is B related to E ?


(a) Sister-in-law (b) Sister
(c) Brother (d) Brother-in-law

68. C is wife of B. E is the son of C. A is the brother of B and father of D. What is the
relationship of E to D ?
(a) Mother (b) Sister
(c) Brother (d) Cousin

69. M is the son of P. Q is the grand-daughter of O, who is the husband of P. How is M related
to O?
(a) Son (b) Daughter
(c) Mother (d) Father

208 | P a g e
70. X and Y are brothers. R is the father of Y. S is the brother of T and maternal uncle of X.
What is T to R ?
(a) Mother (b) Wife
(c) Sister (d) Brother

Directions (Q. 71-75): Following questions are based on the information provided
below:
(i) ‘A x B’ means ‘A is Mother of B’.
(ii) ‘A - B’ means ‘A is Brother of B’.
(iii) ‘A + B’ means ‘A is Sister of B’.
(iv) ‘A ÷ B’ means ‘A is father of B’.

71. Which of the following means ‘R’ is maternal uncle of ‘T’ ?


(a) R–MxT (b) R+MxT
(c) TxM-R (d) T+M÷R

72. Which of the following means ‘F’ is paternal grandfather of ‘H’ ?


(a) F–J÷H (b) F÷J-H
(c) F÷J÷H (d) H÷J÷F

73 How is K related to M in R ÷ M – K ?
(a) Son (b) Brother
(c) Sister (d) cannot be determined

74. How will ‘M is daughter of N’ be written ?


(a) M+DxN (b) N÷M+W
(c) N÷M (d) NxM

75. How is ‘H related to D’ in ‘D ÷ R – M x H’ ?


(a) Grandson (b) Granddaughter
(c) Grandson or Granddaughter (d) None of these

Directions (Q. 76-78): Following questions are based on the information provided
below:
(i) ‘P x Q’ means ‘P is brother of Q.’
(ii) ‘P ÷ Q’ means ‘P is sister of Q.’
(iii) ‘P + Q’ means ‘P is mother of Q.’
(iv) ‘P – Q’ means ‘P is father of Q.’

76. Which of the following means ‘M is nephew of R’ ?


(a) MxT+J÷R (b) RxK–MxT
(c) RxK-M (d) R–K÷M

77. Which of the following means ‘D is maternal uncle of T’ ?


(a) DxJ+T (b) DxJ-T
(c) D÷J+T (d) D÷J-T

78. Which of the following means ‘R is granddaughter of B’ ?


(a) R÷M+K-B (b) K+M÷R-B
(c) B–T+R÷D (d) B–T+R

209 | P a g e
ANSWERS

1. 2. 3. 4. 5. 6. 7. 8. 9. 10.
(C) (A) (D) (C) (D) (A) (D) (A) (B) (C)
11. 12. 13. 14. 15. 16. 17. 18. 19. 20.
(B) (D) (C) (B) (C) (D) (B) (A) (D) (A)
21. 22. 23. 24. 25. 26. 27. 28. 29. 30.
(A) (A) (D) (C) (B) (A) (A) (A) (A) (C)
31. 32. 33. 34. 35. 36. 37. 38. 39. 40.
(D) (B) (A) (C) (D) (C) (C) (A) (B) (C)
41. 42. 43. 44. 45. 46. 47. 48. 49. 50.
(D) (B) (A) (C) (D) (A) (D) (C) (C) (C)
51. 52. 53. 54. 55. 56. 57. 58. 59. 60.
(B) (B) (C) (B) (B) (D) (B) (D) (A) (B)
61. 62. 63. 64. 65. 66. 67. 68. 69. 70.
(D) (A) (D) (D) (D) (D) (A) (D) (A) (B)
71. 72. 73. 74. 75. 76. 77. 78.
(A) (C) (D) (B) (C) (B) (A) (C)

210 | P a g e
CHAPTER – 13 (A) - SYLLOGISM
Exercise 13(A):
Directions (Qs. 1 - 25): Each of the following questions contains two statements followed by
conclusions numbered I and II. You have to consider the two statements to be true, even if they to
be at variance at the commonly known facts. You have to decide which of the given conclusion
definitely follows from the given statements.

Given answer (a) if only I follows; (b) if only conclusion II follows; (c) if either I or II follows; (d)
neither I nor II follows and (e) if both I and II follows.

1. Statement: Some Chairs are glasses.


All tree are Chairs
Conclusions: I. Some trees are glasses.
II. Some glasses are trees.

2. Statement: No man is a lion


Ram is a man.
Conclusions: I. Ram is not a lion.
II. All men are not Ram.

3. Statement: All boys are Fathers.


All Fathers are Mothers.
Conclusions: I. All Fathers are boys.
II. All boys are Mothers.

4. Statement: All pens are cups.


All cups are bowls.
Conclusions: I. All pens are bowls.
II. All cups are pens.

5. Statement: All students are boys.


No boy is dull
Conclusions: I. There are no girls in the class
II. No student is dull.

6. Statement: Some cats are kittens.


All Rats are kittens.
Conclusions: I. Some cats are Rats.
II. Some rats are cats.

7. Statement: All names are dogs.


No dogs are foxes.
Conclusions: I. All names are foxes.
II. No dogs are names.

8. Statement: All pens are dogs.


Some pens are lights.
Conclusions: I. Some dogs are lights.
II. Some lights are not dogs.

211 | P a g e
9. Statement: Some animals are clouds.
Horse is a animal.
Conclusions: I. Some clouds are animal.
II. Horse is not a cloud.

10. Statement: All tables are rats


Some Rats are chairs.
Conclusions: I. All rats are tables
II. Some chairs are not rats.

11. Statement: All tigers are birds.


Some birds are cows.
Conclusions: I. Some cows are birds.
II. Some tigers are cows.

12. Statement: All papers are pens.


All pens are erasers.
Conclusions: I. Some erases are papers.
II. Some pens are no papers.

13. Statement: Some trees are monkeys.


Some ships are trees.
Conclusions: I. Some Monkeys are ships.
II. Some trees are neither ships nor monkeys.

14. Statement: All glasses are mirrors.


Some mirrors are Black.
Conclusions: I. All mirrors are glasses.
II. Some glasses are black.

15. Statement: Some Dogs are monkeys.


No monkey is black.
Conclusions: I. Some dogs are black.
II. Some monkeys are dogs.

16. Statement: All roads are poles


No poles are Bungalows.
Conclusions: I. Some roads are Bungalows.
II. Some Bungalows are poles.

17. Statement: Many actors are directors.


All directors are dancers.
Conclusions: I. Some actors are dancers.
II. No directors is an actor.

18. Statement: Only dogs are animals.


No historian is an animal.
Conclusions: I. Some dogs are not historians.
II. Some historians are not dogs.

212 | P a g e
19. Statement: Some chairs are caps.
No cap is red.
Conclusions: I. Some caps are chairs.
II. Some No Chair is red.

20. Statement: Some cups are belts.


No belt is black.
Conclusions: I. Some cups are black.
II. Some cups are not black.

21. Statement: Some girls are flowers


Some flowers are books.
Conclusions: I. Some girls are books.
II. No books are girls.

22. Statement: Some files are rats.


All animals are rats
Conclusions: I. All files are rats.
II. Some rats are animals.

23. Statement: All cricketers are tall.


Rajesh is tall.
Conclusions: I. Rajesh is a cricketer.
II. Rajesh is not cricketer.

24. Statement: Some cats are cows.


All cows are horses.
Conclusions: I. Some horses are cats.
II. Some cats are horses.

25. Statement: All scientists are hard working.


No scientists are superstitious.
Conclusions: I. No scientists are superstitious.
II. All superstitious are not scientists.

26. Statement: I. Some boys are student.


II. All students are Engineers.
Conclusions: I. All Engineers are students.
II. Some boys are Engineers.
(a) Only I follows (b) Only II follows
(c) Both I and II follows (d) Neither I nor II follows

27. Statement: I. All Lotus are flowers.


II. No Lily is a Lotus
Conclusions: I. No Lily is a flower.
II. Some Lilies are flowers.
(a) Only I follows (b) Only II follows
(c) Either I or II follows (d) Neither I nor II follows

213 | P a g e
28. Statement: I. All A's are C's
II. All D's are C's
Conclusions: I. All A's are C's
II. Some D's are not A's
(a) Only I follows (b) Only II follows
(c) Both I and II follows (d) None follows

ANSWERS

Syllogism Part-A:
1. 2. 3. 4. 5. 6. 7. 8. 9. 10.
(D) (A) (B) (A) (E) (D) (D) (A) (A) (D)
11. 12. 13. 14. 15. 16. 17. 18. 19. 20.
(A) (A) (D) (D) (B) (D) (A) (A) (E) (C)
21. 22. 23. 24. 25. 26. 27. 28.
(C) (B) (C) (E) (E) (B) (C) (A)

214 | P a g e
CHAPTER – 13 (B) - SYLLOGISM
Directions (Q 1–31): In each of the questions below are given some statements followed by two
conclusions numbered I and II. You have to take the given statements to be true even if they seem
to be at variance with commonly known facts. Read all the conclusions and then decide which of
the given conclusions logically follows from the given statements disregarding commonly known
facts.
Give answer
(1) if only conclusion I follows.
(2) if only conclusion II follows.
(3) if either conclusion I or II follows.
(4) if neither conclusion I nor II follows.
(5) if both conclusion I and II follow.
1. Statements: All rivers are ponds.
All pens are roads. All roads are Conclusions:
houses. I. Some ponds are tents.
Conclusions: II. Some ponds are desks.
I. All houses are pens. 7. Statements:
II. Some houses are pens. All chairs are pens.
2. Statements: Some pens are knives.
Some kites are threads. No thread is All knives are rats.
needle. Conclusions:
Conclusions: I. Some rats are chairs.
I. Some kites are needles. II. Some rats are pens.
II. No needle is thread. 8. Statements:
3. Statements: Some forests are huts.
Some doctors are nurses. All nurses Some huts are walls.
are patients. Some walls are nets.
Conclusions: Conclusions:
I. All doctors are patients. I. Some nets are forests.
II. Some patients are doctors. II. Some nets are huts.
4. Statements: 9. Statements:
Some shoes are coats. Some coats are All tables are windows.
buttons. All windows are rooms.
Conclusions: All rooms are buses.
I. No button is shoe. Conclusions:
II. Some shoes are buttons. I. Some buses are tables.
5. Statements: II. Some rooms are tables.
Some ovens are refrigerators 10. Statements:
Some refrigerators are A.Cs. Some trees are boxes.
Conclusions: All boxes are bricks.
I. Some A.Cs are ovens. All bricks are dogs.
II. No A.C. is oven. Conclusions:
6. Statements: I. Some dogs are trees.
Some desks are tents. II. Some bricks are trees.
Some tents are rivers.

215 | P a g e
11. Statements: Conclusions:
All goats are flowers. I. Some tables are sofas.
No flowers is branch. II. No table is sofa.
Some branches are roots. 18. Statements:
Conclusions: All rats are bats.
I. Some roots are goats. Some bats are desks.
II. No root is goat. All desks are chairs.
12. Statements: Conclusions:
All pots are rings. I. Some desks are rats.
All bangles are rings. II. Some chairs are rats.
All rings are paints. 19. Statements:
Conclusions: Some roads are ponds.
I. Some paints are pots All ponds are stores.
II. Some bangles are paints. Some stores are bags.
13. Statements: Conclusions:
Some buses are trains. I. Some bags are ponds.
Some trains are boats. II. Some stores are roads.
Conclusions: 20. Statements:
I. Some trains are buses. No table is wood.
II. Some boats are buses. Some woods are chairs.
14. Statements: All chairs are stones.
Some desks are mirrors. Conclusions:
Some mirrors are combs. I. No stone is table.
Some combs are pins. II. Some stones are woods.
Conclusions: 21. Statements:
I. Some pins are desks. All letters are black.
II. Some combs are desks. All black are blue.
15. Statements: No blue is green.
All benches are cots. Conclusions:
No cot is lamp. I. No letter is green.
Some lamps are candles. II. Some blue are black.
Conclusions: 22. Statements:
I. Some cots are benches. Some fruits are mangoes.
II. Some candles are cots. Some mangoes are red.
16. Statements: All red are vegetables.
Some cats are dogs. Conclusions:
All dogs are goats I. No fruit is red.
All goats are walls. II. Some fruits are red.
Conclusions: 23. Statements:
I. Some walls are dogs. Some eyes are ears.
II. Some walls are cats. Some ears are hands.
17. Statements: Conclusions:
Some buildings are sofas. I. No hand is an eye.
Some sofas are benches. II. Some eyes are hands.
Some benches are tables.

216 | P a g e
24. Statements: All cars are trucks.
Some books are pens. Conclusions:
Some pens are pencils. I. Some ships are not cars.
Some pencils are buttons. II. All cars are aeroplanes.
Conclusions: 31. Statements:
I. Some buttons are pens. Some clouds are ashes.
II. Some pencils are books. Some ashes are particles.
25. Statements: Conclusions:
Some caps are shirts. I. No particle is a cloud.
All shirts are papers. II. Some elements are ashes.
Conclusions:
I. All shirts which are caps are Directions (Q 32-58): In each of the questions
papers. below are given three statements flowed by
II. Some shirts are caps. three conclusions numbered I, II and III. You
26. Statements: have to take the given statements to be true
All A are Z. even if they seem to be at variance with
All Z are X. commonly known facts. Read all the
All Y are A. conclusions and then decide which of the
Conclusions: given conclusions logically follows from the
I. All A are Y. given statements disregarding commonly
II. All Y are X. known facts.
27. Statements: 32. Statements:
All keys are locks. All halls are tyres.
No locks is a door. Some tyres are wheels.
All doors are windows. All wheels are cars.
Conclusions: Conclusions:
I. No key is a door I. Some cars are wheels.
II. Some windows are locks. II. Some cars are tyres.
28. Statements: III. Some wheels are halls.
All districts are cities. (1) None follows
All states are cities. (2) Only I &II follows
Some cities are countries. (3) Only II follows
Conclusions: (4) Only III follows
I. Some states are districts. (5) Only II and III follows
II. Some countries are states. 33. Statements:
29. Statements: Some blades are hammers.
All books are pages. Some hammers are knives.
All libraries are books. Some knives are axes.
All words are pages. Conclusions:
Conclusions: I. Some axes are Hammers.
I. All words are books. II. Some knives are blades
II. All libraries are pages. III. Some axes are blades.
30. Statements: (1) None follows
All ships are aeroplanes. (2) Only I follows
All trucks are ships (3) Only II follows

217 | P a g e
(4) Only III follows Conclusions:
(5) None of these I. Some drawers are desks.
34. Statements: II. Some drawers are chairs
All dolls are windows. III. No drawer is chairs.
All bottles are windows. (1) None follows
All cars are bottles. (2) Only II follows
Conclusions: (3) Only III follows
I. All cars are windows. (4) Only either II or III follows
II. Some cars are dolls (5) Only I & either II or III follows
III. Some windows are cars. 38. Statements:
(1) Only I and II follows All flowers are trees.
(2) Only II and III follows Some trees are houses.
(3) Only I and III follows All houses are wheels.
(4) All follows Conclusions:
(5) None of these I. Some wheels are trees.
35. Statements: II. Some trees are flowers
Some benches are beads. III. Some wheels are Flowers.
All beads are flowers (1) Only I and II follow
No tree is a flower. (2) Only I and III follow
Conclusions: (3) Only II and III follow
I. Some trees are benches. (4) All I, II and III follow
II. Some trees are beads. (5) None of these
III. No tree is a bead. 39. Statements:
(1) Only I follows All windows are doors
(2) Only either I or II follows All buildings are doors.
(3) Only either II or III follows All doors are boats.
(4) Only III follows Conclusions:
(5) None of these I. All windows are boats
36. Statements: II. All buildings are boats.
All breads are tables. III. Some boats are doors.
Some tables are brushes. (1) Only I and II follow
All brushes are paints. (2) Only I and III follow
Conclusions: (3) Only II and III follow
I. Some paints are breads. (4) All follow
II. Some brushes are breads. (5) None of these
III. Some paints are tables. 40. Statements:
(1) None follows Some radios are telephones.
(2) Only II follows All telephones are mirrors.
(3) Only III follows All mirrors are desks.
(4) Only I and II follows Conclusions:
(5) Only II and III follows I. Some radios are desks.
37. Statements: II. Some radios are mirrors.
Some desks are chairs. III. Some desks are telephones.
Some chairs are pens (1) Only I and II follow
Some pens are drawers. (2) Only I and III follow

218 | P a g e
(3) Only II and III follow Some rooms are hails.
(4) All follow All halls are leaves.
(5) None of these Conclusions:
41. Statements: I. Some leaves are desks.
All furniture are jungles. II. Some halls are desks.
No jungle is road. III. Some leaves are rooms
Some roads are hills. (1) None follows
Conclusions: (2) Only I follows
I. Some roads are furniture. (3) Only II follows
II. Some jungles are furniture. (4) Only III follows
III. Some hills are jungles. (5) Only II and III follows
(1) Only I follows 45. Statements:
(2) Only II follows All buildings are mirrors.
(3) Only III follows Some mirrors are pens.
(4) Only I and II follows No pen is paper.
(5) None of these Conclusions:
42. Statements: I. Some papers are buildings
All bricks are stones. II. Some pens are buildings
Some stones are rocks. III. Some papers are mirrors.
All rocks are mountains. (1) None follows
Conclusions: (2) Only I follows
I. Some mountains are stones. (3) Only II follows
II. Some mountains are bricks. (4) Only III follows
III. Some stones are bricks. (5) Only II and III follows
(1) Only I follows 46. Statements:
(2) Only III follows Some books are trees.
(3) Only I and III follows Some trees are roads.
(4) All follows All roads are wheels.
(5) None of these Conclusions:
43. Statements: I. Some wheels are books.
Some bags are plates. II. Some roads are books.
Some plates are chairs. III. Some wheels are trees.
All chairs are tables. (1) Only I and II follow
Conclusions: (2) Only II &III follow
I. Some tables are plates. (3) Only I and III follow
II. Some chairs are bags. (4) All follow
III. No chairs is bag. (5) None of these
(1) Only I follows 47. Statements:
(2) Only either II or III follows All stones are rivers.
(3) Only I and either II or III All rivers are cars.
follows Some cars are trains.
(4) Only III follows Conclusions:
(5) None of these I. Some trains are stones.
44. Statements: II. Some cars are stones
All desks are rooms III. Some trains are rivers.

219 | P a g e
(1) None follows Some rivers are hills.
(2) Only I follows No hills is taxi.
(3) Only II follows All taxis are buses.
(4) Only III follows Conclusions:
(5) Only II and III follow I. Some buses are rivers.
48. Statements: II. Some taxis are rivers.
All stamps are packets. III. No bus is river.
Some packets are buckets (1) None follows
All buckets are tubes. (2) Only I follows
Conclusions: (3) Only III follows
I. Some tubes are stamps (4) Only II follows
II. Some buckets are stamps (5) Only either I or III follows
III. Some tubes are packets. 52. Statements:
(1) None follows All machines are crowns.
(2) Only I follows All crowns are tablets.
(3) Only II follows Some tablets are bottles.
(4) Only III follows Conclusions:
(5) Only II and III follows I. Some bottles are crowns
49. Statements: II. Some tablets are machines.
Some doors are windows III. Some bottles are machines.
Some windows are lamps. (1) Only I follows
All lamps are candles. (2) Only II follows
Conclusions: (3) Only III follows
I. Some candles are doors. (4) Only II and III follows
II. Some candles are windows (5) None of these
III. Some lamps are doors 53. Statements:
(1) Only I follows All rooms are hotels.
(2) Only II follows All hotels are buildings.
(3) Only III follows All buildings are mountains.
(4) Only I and II follows Conclusions:
(5) None of these I. Some mountains are hotels.
50. Statements: II. Some buildings are rooms.
Some towns are villages. III. Some mountains are rooms.
Some villages are lanes. (1) Only I and II follow
Some lanes are hamlets. (2) Only I and III follow
Conclusions: (3) Only II and III follow
I. Some hamlets are villages. (4) All I, II and III follow
II. Some lanes are towns (5) None of these
III. Some hamlets are towns. 54. Statements:
(1) None follows Some carrots are brinjals.
(2) Only I follows Some brinjals are apples.
(3) Only II follows All apples are bananas.
(4) Only III follows Conclusions:
(5) Only I and II follows I. Some apples are carrots.
51. Statements: II. Some bananas are brinjals.

220 | P a g e
III. Some bananas are carrots. (3) Only III follows
(1) Only I follows (4) Only either I or III follows
(2) Only II follows (5) None follows
(3) Only III follows 57. Statements:
(4) Only either II or III follow All tables are mirrors.
(5) None of these Some mirrors are chairs.
55. Statements: All chairs are glasses.
All keys are locks. Conclusions:
All locks are bangles. I. Some glasses are mirrors.
All bangles are cars. II. Some chairs are tables.
Conclusions: III. Some mirrors are tables.
I. Some cars are locks. (1) Only I and II follow
II. Some bangles are keys. (2) Only II and III follow
III. Some cars are keys. (3) Only I and III follow
(1) Only I follows (4) All I, II and III follow
(2) Only I and II follow (5) None of these
(3) Only I and III follow 58. Statements:
(4) Only II and III follow All calculators are boxes.
(5) All I, II and III follow All boxes are taps.
56. Statements: Some taps are machines.
All fruits are leaves. Conclusions:
Some leaves are trees. I. some machines are boxes.
No tree is house. II. Some taps are calculators.
Conclusions: III. Some boxes are calculators.
I. Some houses are fruits. (1) Only I and II follow
II. Some trees are fruits. (2) Only I and III follow
III. No house is a fruit (3) Only II and III follow
(1) Only I follows (4) All I, II and III follow
(2) Only II follows (5) None of these

ANSWERS
Syllogism Part-II:
1. 2. 3. 4. 5. 6. 7. 8. 9. 10.
(2) (2) (2) (3) (3) (1) (2) (4) (5) (5)
11. 12. 13. 14. 15. 16. 17. 18. 19. 20.
(3) (5) (1) (4) (1) (5) (3) (4) (2) (2)
21. 22. 23. 24. 25. 26. 27. 28. 29. 30.
(5) (3) (3) (4) (5) (2) (1) (4) (2) (2)
31. 32. 33. 34. 35. 36. 37. 38. 39. 40.
(2) (2) (1) (3) (4) (3) (4) (1) (4) (4)
41. 42. 43. 44. 45. 46. 47. 48. 49. 50.
(2) (3) (3) (4) (1) (4) (3) (4) (2) (1)
51. 52. 53. 54. 55. 56. 57. 58.
(5) (2) (4) (2) (5) (4) (3) (3)

221 | P a g e
CHAPTER – 14 - STATISTICAL DESCRIPTION OF DATA
PRACTICE QUESTIONS

1. Which of the following statements is false?


(a) Statistics is derived from the Latin word ‘Status’
(b) Statistics is derived from the Italian word ‘Statista’
(c) Statistics is derived from the French word ‘Statistik’
(d) None of these

2. Statistics is defined in terms of numerical data in the


(a) Singular sense (b) Plural sense
(c) Either (a) or (b) (d) Both (a) and (b)

3. Statistics is applied in
(a) Economics (b) Business management
(c) Commerce and industry (d) All these.

4. Statistics is concerned with


(a) Qualitative information (b) Quantitative information
(c) (a) or (b) (d) Both (a) and (b)

5. An attribute is
(a) A qualitative characteristic (b) A quantitative characteristic
(c) A measurable characteristic (d) All these

6. Which of the following graph is used to calculate the partition values


(a) Lorenz Curve (b) Ogive Curve
(c) Histogram (d) None

7. Marks of a student is an example of


(a) An attribute (b) A discrete variable
(c) A continuous variable (d) None of these

8. Nationality of a student is
(a) An attribute (b) A continuous variable
(c) A discrete variable (d) (a) or (c)

9. Drinking habit of a person is


(a) An attribute (b) A variable
(c) A discrete variable (d) A continuous variable

10. Age of a person is


(a) An attribute (b) A discrete variable
(c) A continuous variable (d) A variable

11. Data collected on religion from the census reports are


(a) Primary data (b) Secondary data
(c) Sample data (d) (a) or (b)

222 | P a g e
12. The data collected on the height of a group of students after recording their heights with a
measuring tape are
(a) Primary data (b) Secondary data
(c) Discrete data (d) Continuous data

13. The primary data are collected by


(a) Interview method (b) Observation method
(c) Questionnaire method (d) All these

14. The quickest method to collect primary data is


(a) Personal interview (b) Indirect interview
(c) Telephone interview (d) By observation

15. The best method to collect data, in case of a natural calamity, is


(a) Personal interview (b) Indirect interview
(c) Questionnaire method (d) Direct observation method

16. In case of a rail accident, the appropriate method of data collection is by


(a) Personal interview (b) Direct interview
(c) Indirect interview (d) All these

17. Which method of data collection covers the widest area?


(a) Telephone interview method (b) Mailed questionnaire method
(c) Direct interview method (d) All these

18. The amount of non-responses is maximum in


(a) Mailed questionnaire method (b) Interview method
(c) Observation method (d) All these

19. Some important sources of secondary data are


(a) International and Government sources
(b) International and primary sources
(c) Private and primary sources
(d) Government sources

20. The Choronological classification of data are classified on the basis of :


(a) Attributes (b) Area
(c) Time (d) Class Interval

21. Arrange the following dimension wise : pie – diagram, bar – diagram and cubic diagram.
(a) 1,2,3 (b) 3,1,2
(c) 3,2,1 (d) 2,1,3

22. The mode of presentation of data are


(a) Textual, tabulation and diagrammatic(b) Tabular, internal and external
(c) Textual, tabular and internal (d) Tabular, textual and external

23. The best method of presentation of data is


(a) Textual (b) Tabular
(c) Diagrammatic (d) (b) and (c)

223 | P a g e
24. The most attractive method of data presentation is
(a) Tabular (b) Textual
(c) Diagrammatic (d) (a) or (b)

25. For tabulation, ‘caption’ is


(a) The upper part of the table (b) The lower part of the table
(c) The main part of the table
(d) The upper part of a table that describes the column and sub-column

26. ‘Stub’ of a table is the


(a) Left part of the table describing the columns
(b) Right part of the table describing the columns
(c) Right part of the table describing the rows
(d) Left part of the table describing the rows

27. The entire upper part of a table is known as


(a) Caption (b) Stub
(c) Box head (d) Body

28. The unit of measurement in tabulation is shown in


(a) Box head (b) Body
(c) Caption (d) Stub

29. In tabulation source of the data, if any, is shown in the


(a) Footnote (b) Body
(c) Stub (d) Caption

30. Which of the following statements is untrue for tabulation?


(a) Statistical analysis of data requires tabulation
(b) It facilitates comparison between rows and not columns
(c) Complicated data can be presented
(d) Diagrammatic representation of data requires tabulation

31. Hidden trend, if any, in the data can be noticed in


(a) Textual presentation (b) Tabulation
(c) Diagrammatic representation (d) All these

32. The most accurate mode of data presentation is


(a) Diagrammatic method (b) Tabulation
(c) Textual presentation (d) None of these

33. The chart that uses logarithm of the variable is known as


(a) Line chart (b) Ratio chart
(c) Multiple line chart (d) Component line chart

34. Multiple line chart is applied for


(a) Showing multiple charts
(b) Two or more related time series when the variables are expressed in the same unit
(c) Two or more related time series when the variables are expressed in different unit
(d) Multiple variations in the time series

224 | P a g e
35. Pie-diagram is used for
(a) Comparing different components and their relation to the total
(b) Nepresenting qualitative data in a circle
(c) Representing quantitative data in circle
(d) (b) or (c)

36. The distribution of shares is an example of the frequency distribution of


(a) A discrete variable (b) A continuous variable
(c) An attribute (d) (a) or (c)

37. Mutually exclusive classification


(a) Excludes both the class limits
(b) Excludes the upper class limit but includes the lower class limit
(c) Includes the upper class limit but excludes the upper class limit
(d) Either (b) or (c)

38. Mutually inclusive classification is usually meant for


(a) A discrete variable (b) A continuous variable
(c) An attribute (d) All these

39. Mutually exclusive classification is usually meant for


(a) A discrete variable (b) A continuous variable
(c) An attribute (d) Any of these

40. The LCB is


(a) An upper limit to LCL (b) A lower limit to LCL
(c) (a) and (b) (d) (a) or (b)

41. The UCB is


(a) An upper limit to UCL (b) A lower limit to LCL
(c) Both (a) and (b) (d) (a) or (b)

42. length of a class is


(a) The difference between the UCB and LCB of that class
(b) The difference between the UCL and LCL of that class
(c) (a) or (b)
(d) Both (a) and (b)

43. For a particular class boundary, the less than cumulative frequency and more than cumulative
frequency add up to
(a) Total frequency (b) Fifty per cent of the total frequency
(c) (a) or (b) (d) None of these

44. Frequency density corresponding to a class interval is the ratio of


(a) Class frequency to the total frequency
(b) Class frequency to the class length
(c) Class length to the class frequency
(d) Class frequency to the cumulative frequency

225 | P a g e
45. Mode of a distribution can be obtained from
(a) Histogram (b) Less than type ogives
(c) More than type ogives (d) Frequency polygon

46. Median of a distribution can be obtained from


(a) Frequency polygon (b) Histogram
(c) Less than type ogives (d) None of these

47. A comparison among the class frequencies is possible only in


(a) Frequency polygon (b) Histogram
(c) Ogives (d) (a) or (b)

48. Frequency curve is a limiting form of


(a) Frequency polygon (b) Histogram
(c) (a) or (b) (d) (a) and (b)

49. Most of the commonly used frequency curves are


(a) Mixed (b) Inverted J-shaped
(c) U-shaped (d) Bell-shaped

50. The distribution of profits of a company follows


(a) J-shaped frequency curve (b) U-shaped frequency curve
(c) Bell-shaped frequency curve (d) Any of these

51. Out of 1000 persons, 25 per cent were industrial workers and the rest were agricultural
workers. 300 persons enjoyed world cup matches on TV. 30 per cent of the people who had
not watched world cup matches were industrial workers. What is the number of agricultural
workers who had enjoyed world cup matches on TV?
(a) 260 (b) 240
(c) 230 (d) 250

52. A sample study of the people of an area revealed that total number of women were 40% and
the percentage of coffee drinkers were 45 as a whole and the percentage of male coffee
drinkers was 20. What was the percentage of female non-coffee drinkers?
(a) 10 (b) 15
(c) 18 (d) 20

53. Cost of sugar in a month under the heads Raw Materials, labour, direct production and others
were 12, 20, 35 and 23 units respectively. What is the difference between the central angles
for the largest and smallest components of the cost of sugar?
(a) 72o (b) 48o
o
(c) 56 (d) 92o

54. The number of accidents for seven days in a locality are given below :
No. of accidents : 0 1 2 3 4 5 6
Frequency: 15 19 22 31 9 3 2
What is the number of cases when 3 or less accidents occurred?
(a) 56 (b) 6
(c) 68 (d) 87

226 | P a g e
55. The following data relate to the incomes of 86 persons :
Income in Rs.: 500-999 1000-1499 1500-1999 2000-2499
No. of persons: 15 28 36 7
What is the percentage of persons earning more than Rs. 1500?
(a) 50 (b) 45
(c) 40 (d) 60

56. The following data relate to the marks of a group of students:


Marks: Below 10 Below 20 Below 30 Below 40 Below 50
No. of students: 15 38 65 84 100
How many students got marks more than 30?
(a) 65 (b) 50
(c) 35 (d) 43

57. Find the number of observations between 250 and 300 from the following data :
Value: More than 200 More than 250 More than 300 More than 350
No. of observations: 56 38 15 0
(a) 56 (b) 23
(c) 15 (d) 8

58. A Qualitative characteristic is known as :


(a) An attribute. (b) A variable.
(c) A discrete variable. (d) A continuous variable.

59. Methods that are employed for the collection of primary data
(a) Interview method. (b) Questionnaire method.
(c) Observation method. (d) All of these.

60. The gender of a baby is example of :


(a) A variable. (b) A discrete variable.
(c) A continuous variable. (d) An attribute.

61. The data are known to be _______ if the data, as being already collected, are used by a
different person or agency.
(a) Primary (b) Secondary
(c) Specialized (d) Subsidiary

62. The amount of non responses is likely to be maximum in _______ method of collecting data.
(a) Telephone interview method (b) Personal interview method
(c) Mailed questionnaire method (d) Observation method

63. Refer following table:


Frequency distribution of weights of 16 students
Weight in kg. No. of students
(Class interval) (Frequency)
44 – 48 4
49 – 53 5
54 – 58 7
Total 16

227 | P a g e
Find width of class interval for the second class interval.
(a) 4 (b) 5
(c) 46 (d) 48

64. This method presents data with the help of a paragraph or a number of paragraphs.
(a) Tabular presentation. (b) Textual presentation.
(c) Diagrammatic representation. (d) None of these.

65. Refer following table:


Frequency distribution of weights of 16 students
Weight in kg. No. of students
(Class interval) (Frequency)
44 – 48 4
49 – 53 5
54 – 58 7
Total 16
Find Relative frequency for the second class interval?
(a) 1/11 (b) 5/4
(c) 5/16 (d) ¼

66. To find the median graphically we use ________.


(a) Ogive (b) Frequency Polygon
(c) Histogram (d) None of these

67. The following data relates to the incomes of 90 persons.


Income in Rs.: 1500-1999 2000-2499 2500-2999 3000-3499
No. of persons: 13 32 20 25
What is the percentage of persons earning more than Rs. 2,500?
(a) 50 (b) 45
(c) 52 (d) 55

68. Relative frequency for a particular class lies between .


(a) 0 and 1 (b) 0 and 1 both inclusive
(c) -1 and1 (d) -land 1

69. Find the number of observations between 350 and 400 from the following data
Value: (More than) 200 350 400 450
No. of observations 48 25 12 0
(a) 13 (b) 15
(c) 17 (d) 19

70. When the width of all classes is same, frequency polygon has not the same area as the
Histogram .
(a) False (b) True
(c) both (d) None

71. The called graphical representation of a cumulative frequency distribution is .


(a) Histogram (b) Ogive
(c) both (d) None

228 | P a g e
72. A table has ___________ parts .
(a) four (b) two
(c) five (d) None

73. In 2000, out of total of 1,750 workers of a factory, 1,200 were members of a trade union. The
number of women employed was 200 of which 175 did not belong to a trade union. In 2004,
there were 1,800 employees who belong to a trade union and 50 who did not belong to trade
union. Of all the employees in 2004, 300 were women of whom only 8 did not belong to the
trade union. On the basis of this information, the ratio of female members of the trade union
in 2000 and 2004 is
(a) 292 : 25 (b) 8: 175
(c) 175 : 8 (d) 25 : 292

74. From the following data find the number class intervals if class length is given as 5.
73, 72, 65, 41, 54, 80, 50, 46, 49, 53.
(a) 6 (b) 5
(c) 7 (d) 8

75. The method of representing Household Expenditure is:


(a) Histogram (b) Pie Diagram
(c) Line Diagram (d) Ogive

76. Which of the following is a statistical data?


(a) Ram is 50 years old.
(b) Height of Ram is 5’6” and of Shyam and Han is 5’3” and 54” respectively.
(c) Height of Ram is 56” and weight is 90kg
(d) Sale of A was more than B and C.

77. Sales of XYZ Ltd. for 4 months is:


Months Sales
Jan. 10,000
Feb. 15,000
May 18,000
Apr. 9,000
The above data represents
(a) Discrete (b) Continuous
(c) Individual (d) None of these

78. Mid values are also called .


(a) Lower limit (b) Upper limit
(c) Class mark (d) None

79. Which of the following is not a two-dimensional figure?


(a) Line Diagram (b) Pie Diagram
(c) Square Diagram (d) Rectangle Diagram

80. Less than type and more than type gives meet at a point known as:
(a) Mean (b) Median
(c) Mode (d) None

229 | P a g e
81. Arrange the dimensions of Bar diagram, Cube diagram, Pie diagram in sequence.
(a) 1,2,3 (b) 2,1,3
(c) 1,3,2 (d) 3,2,1

82. Histogram is used to find.


(a) Mean (b) Median
(c) Mode (d) None of these

83. Using Ogive curve, we can determine .


(a) Median (b) Quartile
(c) Both (a) and (b) (d) None

84. Mode can be obtained from .


(a) Frequency polygon. (b) Histogram.
(c) Ogive (d) All of the above

85. The data obtained by the internet are .


(a) Primary Data (b) Secondary Data
(c) Both (a) and (b) (d) None of these

86.. A pie diagram used to represent the following data ………………


Source customers Excise Income Tax Wealth Tax
Revenue in millions 120 180 240 180
The central angles corresponding to income tax and wealth tax
(a) (900,1200) (b) (1200,9000)
0 0
(c) (60 ,120 ) (d) (900,600)

87. The difference between upper limit and lower limit of a class is called:
(a) Class Interval (b) Class boundaries
(c) Mid - value (d) Frequency

88. If the class intervals are


10 -14, 15 -19, 20 -24,…
Then the first class boundaries are :
(a) 9.5 – 14.5 (b) 10 – 15
(c) 9 – 15 (d) 10.5 – 15.5

89. The most appropriate diagram to represent 5 year plan outlay of India in different economic
sectors is
(a) Pie diagram (b) Histogram
(c) Line diagram (d) Frequency polygon

90. For construction of histogram the class intervals of frequency distribution is


(a) Equal (b) Unequal
(c) Either Equal or Unequal (d) None

91. 100 persons are divided into number of male/female and employed/un-employed it refers to
(a) Cardinal data (b) Ordinal Data
(c) Spatial Data (d) Temporal Data

230 | P a g e
92. If we draw a perpendicular on x-axis from the point of inter-section of both 'less than' and
'more than' frequency curves we will get the value of ___________
(a) mode (b) median
(c) arithmetic mean (d) third quartile

93. Histogram is used for the presentation of the following type of series
(a) Time series (b) Continuous frequency distribution
(c) Discrete frequency distribution (d) Individual observation

94. Curve obtained by joining the points whose x coordinates are the upper limits of the class
intervals and y coordinates are the corresponding cumulative frequencies is called.
(a) Frequency Polygon (b) Frequency curve
(c) Histogram (d) Ogive.

95. The number of observations between 150 and 200 based on the following data is
Value: More More than More More
than 150 than than
100 200 250
No. of 76 63 28 05
observations:
(a) 46 (b) 35
(c) 28 (d) 23

96. The number of car accidents more than 3 in seven days in a locality are given below:
Value: 0 1 2 3 4 5 6 7
Frequency: 12 9 11 13 8 9 6 3
(a) 32 (b) 41
(c) 26 (d) 18

97. The most common form of diagrammatic representation of a grouped frequency distribution
is:
(a) Histogram (b) Ogive
(c) Both (d) None

98. Classification is of ________ kinds.


(a) Two (b) Three
(c) One (d) Four

99. The chart that use logarithm of variable in known as:


(a) Ratio chart (b) Line chart
(c) Multiple line chart (d) Component line chart

100. Find the number of observation between 250 and 300 from the following data:
Value more than: 200 250 300 500
No. of observation 56 38 15 0
(a) 38 (b) 23
(c) 15 (d) None of the above

101 Vertical bar diagram is applicable when


(a) the data are qualitative (b) The data are quantitative
(c) When the data vary over time (d) (b) or (c)
231 | P a g e
102 In order to compare two or more related series, we consider.
(a) Multiple bar chart (b) Grouped bar chart
(c) (a) or (b) (d) (a) and (b)

103 The frequency distribution of a continuous variable is known as


(a) Grouped frequency distribution (b) Simple frequency distribution
(c) (a) or (b) (d) (a) and (b).

104 The distribution of profits of a blue-chip company relates to


(a) Discrete variable (b) Continuous variable
(c) Attributes (d) (a) or (b)

ANSWER

1. 2. 3. 4. 5. 6. 7. 8. 9. 10.
(C) (B) (D) (D) (A) (B) (B) (A) (A) (C)
11. 12. 13. 14. 15. 16. 17. 18. 19. 20.
(B) (A) (D) (C) (A) (C) (B) (A) (A) (C)
21. 22. 23. 24. 25. 26. 27. 28. 29. 30.
(D) (A) (B) (C) (D) (D) (C) (A) (A) (B)
31. 32. 33. 34. 35. 36. 37. 38. 39. 40.
(C) (B) (B) (B) (A) (A) (B) (A) (B) (B)
41. 42. 43. 44. 45. 46. 47. 48. 49. 50.
(A) (A) (A) (B) (A) (C) (D) (D) (D) (C)
51. 52. 53. 54. 55. 56. 57. 58. 59. 60.
(A) (B) (D) (D) (A) (C) (B) (A) (D) (D)
61. 62. 63. 64. 65. 66. 67. 68. 69. 70.
(B) (C) (B) (B) (C) (A) (A) (A) (A) (A)
71. 72. 73. 74. 75. 76. 77. 78. 79. 80.
(B) (C) (D) (D) (B) (B) (C) (C) (A) (B)
81. 82. 83. 84. 85. 86. 87. 88. 89. 90.
(C) (C) (C) (B) (B) (B) (A) (A) (A) (A)
91. 92. 93. 94. 95. 96. 97. 98. 99. 100.
(B) (B) (B) (D) (B) (C) (A) (D) (A) (B)
101. 102. 103. 104.
(d) (c) (a) (b)

232 | P a g e
CHAPTER – 15 - MEASURES OF CENTRAL TENDENCY AND DISPERSION

PRACTICE QUESTIONS

1. Measures of central tendency for a given set of observations measures


(a) The scatterness of the observations
(b) The central location of the observations
(c) Both (a) and (b)
(d) None of these.

2. While computing the AM from a grouped frequency distribution, we assume that


(a) The classes are of equal length
(b) The classes have equal frequency
(c) All the values of a class are equal to the mid-value of that class
(d) None of these.

3. Which of the following statements is true?


(a) Median is based on all observation (b) The mode is the mid-value
(c) The median is the second quartile (d) The mode is the fifth decile

4. Which of the following statements is true?


(a) Usually mean is the best measure of central tendency
(b) Usually median is the best measure of central tendency
(c) Usually mode is the best measure of central tendency
(d) Normally, GM is the best measure of central tendency

5. For open-end classification, which of the following is the best measure of central tendency?
(a) AM (b) GM
(c) Median (d) Mode

6. In case of an even number of observations which of the following is median?


(a) Any of the two middle-most value
(b) The simple average of these two middle values
(c) The weighted average of these two middle values
(d) Any of these

7. The most commonly used measure of central tendency is


(a) AM (b) Median
(c) Mode (d) Both GM and HM.

8. Which one of the following is not uniquely defined?


(a) Mean (b) Median
(c) Mode (d) All of these measures

9. Which of the following measure of the central tendency is difficult to compute?


(a) Mean (b) Median
(c) Mode (d) GM

10. For a moderately skewed distribution, which of he following relationship holds?


(a) Mean — Mode = 3 (Mean — Median) (b) Median — Mode = 3 (Mean — Median)
(c) Mean — Median = 3 (Mean — Mode) (d) Mean — Median = 3 (Median — Mode)

233 | P a g e
11. Which of the following results hold for a set of distinct positive observations?
(a) AM>GM>HM (b) HM>GM>AM
(c) AM>GM>HM (d) GM>AM>HM

12. When a firm registers both profits and losses, which of the following measure of central
tendency cannot be considered?
(a) AM (b) GM
(c) Median (d) Mode

13. Quartiles are the values dividing a given set of observations into
(a) Two equal parts (b) Four equal parts
(c) Five equal parts (d) None of these.

14. Quartiles can be determined graphically using


(a) Histogram (b) Frequency Polygon
(c) Ogive (d) Pie chart.

15. Which of the following measure(s) satisfies (satisfy) a linear relationship between two
variables?
(a) Mean (b) Median
(c) Mode (d) All of these

16. Which of he following measures of central tendency is based on only fifty percent of the
central values?
(a) Mean (b) Median
(c) Mode (d) Both (a) and(b)

17. If there are 3 observations 15, 20, 25 then the sum of deviation of the observations from
their AM is:
(a) 0 (b) 5
(c) —5 (d) None of these.

18. What is the median for the following observations?


5, 8, 6, 9, 11, 4.
(a) 6 (b) 7
(c) 8 (d) None of these

19. What is the modal value for the numbers 5, 8, 6, 4, 10, 15, 18, 10?
(a) 18 (b) 10
(c) 14 (d) None of these

20. What is the GM for the numbers 8, 24 and 40?


(a) 24 (b) 12
(c) 8 3 15 (d) 10

21. The harmonic mean for the numbers 2, 3, 5 is


(a) 2.00 (b) 3.33
(c) 2.90 (d) 30

234 | P a g e
22. If the AM and GM for two numbers are 6.50 and 6 respectively then the two numbers are
(a) 6 and 7 (b) 9 and 4
(c) 10 and 3 (c) 8 and 5

23. If the AM and HM for two numbers are 5 and 3.2 respectively then the GM will be
(a) 16.00 (b) 4.10
(c) 4.05 (d) 4.00

24. What is the value of the first quartile for observations 15, 18, 10, 20, 23, 28, 12, 16?
(a) 17 (b) 16
(c) 12.75 (d) 12

25. The third decile for the numbers 15, 10, 20, 25, 18, 11, 9, 12 is
(a) 13 (b) 10.7
(c) 11 (d) 11.50

26. If there are two groups containing 30 and 20 observations and having 50 and 60.as
arithmetic means, then the combined arithmetic mean is
(a) 55 (b) 56
(c) 54 (d) 52.

27. The average salary of a group of unskilled workers is Rs.10000 and that of a group of skilled
workers is Rs.15,000. If the combined salary is Rs.12000, then what is the percentage of
skilled workers?
(a) 40% (b) 50%
(c) 60% (d) none of these

28. If there are two groups with 75 and 65 as harmonic means and containing 15 and 13
observation then the combined HM is given by
(a) 65 (b) 70.36
(c) 70 (d) 71.

29. What is the HM of 1,½, 1/3 ------------1/n?


(a) n (b) 2n
2 n( n  1)
(c) (d)
(n  1) 2

30. An aeroplane flies from A to B at the rate of 500 km/hour and comes back from B to A to at
the rate of 700 km/hour. The average speed of the aeroplane is
(a) 600 km. per hour (b) 583.33 km. per hour
(c) 100 35 km. per hour (d) 620 km. per hour.

31. If a variable assumes the values 1, 2, 3...5 with frequencies as 1,2,3, ...5 then what is the
AM?
11
(a) (b) 5
3
(c) 4 (d) 4.50

235 | P a g e
32. Two variables x and y are given by y= 2x —3. If the median of x is 20, what is the median of y?
(a) 20 (b) 40
(c) 37 (d) 35

33. If the relationship between two variables u and v are given by 2u + v + 7 = 0 and if the AM
of u is 10, then the AM of v is
(a) 17 (b) —17
(c) —27 (d) 27

34. If x and y are related by x—y—10 = 0 and mode of x is known to be 23, then the
mode of y is
(a) 20 (b) 13
(c) 3 (d) 23

35. If GM of x is 10 and GM of y is 15, then the GM of x y is


(a) 150 (b) Log 10 x Log 15
(c) Log 150 (d) None of these

36. If the AM and GM for 10 observations are both 15, then the value of HM is
(a) Less than 15 (b) More than 15
(c) 15 (d) Can not be determined

37. The mean salary for a group of 40 female workers is 5200 per month and that for a group of
60 male workers is 6800 per month. What is the combined mean salary?
(a) 6500 (b) 6200
(c) 6160 (d) 6100

38. The mean salary of a group of 50 persons is Rs. 5,850. Later on it is discovered that the
salary of one employee has been wrongly taken as Rs. 8,000 instead of Rs. 7,800. The
corrected mean salary is
(a) Rs. 5,854 (b) Rs. 5,846
(c) Rs. 5,650 (d) None of the above.

39. The median of 27, 30, 26, 44, 42, 51, 37 is


(a) 30 (b) 42
(c) 44 (d) 37

40. The algebraic sum of deviations of a set of observations from their A.M. is:
(a) Negative. (b) Positive.
(c) Zero. (d) None of these.

41. For a set of observations, the sum of absolute deviations is ______, when the deviations are
taken from the median.
(a) Zero (b) Maximum
(c) Minimum (d) None of these

42. When we want to divide the given set of observations into two equal parts, we consider
______.
(a) Mean (b) Median
(c) Mode (d) None of these

236 | P a g e
43. Quartiles are values dividing a given set of observations into _______ equal parts.
(a) Two (b) Four
(c) Six (d) Ten

44. The mean salary for a group of 20 female workers is Rs. 5,000 per month and that for a
group of 30 male workers is Rs. 6,000 per month. What is combined mean salary?
(a) Rs. 5,400 (b) Rs. 5,500
(c) Rs. 5,600 (d) Rs. 5,700

45. The weight of 8 students expressed in kg. are 40, 35, 50, 45, 46, 39, 41, 42. Find median
weight?
(a) 40.5 (b) 41
(c) 41.5 (d) 42

46. If the relationship between x and y is given by 4x– 6y =13 and if the median of x is 16. Find
median of y.
(a) 7.50 (b) 8.00
(c) 8.50 (d) None of these.

47. Find Q1 for the following observations:


14, 16, 13, 15, 20, 18, 19, 22
(a) 14 (b) 14.25
(c) 15 (d) 15.25

48. _________ are the values dividing a given set of observations into ten equal parts.
(a) Quartiles (b) Deciles
(c) Centiles (d) None of these

49. What is the median for the following observations 10, 16, 12, 18, 22, 8?
(a) 12 (b) 14
(c) 16 (d) None of these

50. The variables x and y are related by 5x + 6y = 70 and median of x is 8. What is the median
of y?
(a) 4 (b) 4.5
(c) 5 (d) 5.5

51. 50% of actual values will be below & 50% of values will be above ________.
(a) Mode (b) Median
(c) Mean (d) Q1

52. What is the median for the following observations 6,10,15,16,20,5,8?


(a) 8 (b) 10
(c) 15 (d) None of these.

53. The variables x and y are related by 2x+ 3y = 6 and median of x is 2. What is the median of
y?
(a) 1/3 (b) 2/3
(c) 1 (d) None of these.

237 | P a g e
54. The variables x and y are related by 6x+ 7y = 81 and median of x is 10. What is the median
of y?
(a) 2 (b) 3
(c) 4 (d) 5

55. Find D6 for the following observations.


41, 28, 45, 25, 60, 37.5, 37.5, 40, 65, 32.5
(a) 39.60 (b) 40.60
(c) 41.60 (d) 42.60

56. For a moderately skewed distribution of marks in statistics for a group of 100 students, the
mean mark and median mark were found to be 50 and 40. What is the modal mark?
(a) 15 (b) 20
(c) 25 (d) 30

57. If y = 5+7x and mode of x is 4, what is the mode of y?


(a) 28 (b) 33
(c) 4 (d) 43

58. The value of middlemost item when they are arranged in order of magnitude is called
(a) Standard deviation (b) Mean
(c) Mode (d) Median

59. The value of deciles divides the total number of observations into _____ equal parts.
(a) 100 (b) 10
(c) 2 (d) None of these

60. The mean weight of 150 students in a class is 60 kg. The mean weight of the boys is 70 kg,
while that of girls is 55 kg, find the number of boys and number of the girls in the class.
(a) 50 boys and 100 girls (b) 100 boys and 50 girls
(c) 75 boys and 75 girls (d) None of these

61. The height of 8 boys in a class (in cumulative) are 135, 138, 160, 141, 155, 146, 158, 149.
Find 61st percentile.
(a) 139.81 (b) 151.94
(c) 153.98 (d) None of these

62. What is the modal value for the numbers 4, 3, 8, 15, 4, 3, 6, 3, 15, 3, 4.
(a) 3 (b) 4
(c) 15 (d) None of these

63. The class having maximum frequency is called ______


(a) Modal class (b) Median class
(c) Mean Class (d) None of these

64. The mode of the nos. 7, 7, 9, 7, 10, 15, 15, 15, 10 is


(a) 7 (b) 10
(c) 15 (d) 7 and 15

238 | P a g e
65. If each item is reduced by 12, A.M. is ________
(a) Reduced by 12 (b) Increased by 12
(c) Unchanged (d) None of these

66. There are 5 bags of wheat weighing on an average 102 kgs and another 8 bags weighing 98
kgs on an average. What is combined mean of 13 bags.
(a) 109.54 (b) 99.54
(c) 95.54 (d) None of these

67. Which result is true?


(a) H.M.  G.M.  A.M. (b) H.M.  G.M.  A.M.
(c) H.M. > G.M. < A.M. (d) G.M.  A.M.  H.M.

68. Median is affected by extreme values


(a) True (b) False
(c) Both (d) None of these

69. The mean height of 8 student is 152 cm. Two more students of heights 143 cm and 156 cm
join the group. New mean height is equal to
(a) 153 (b) 152.5
(c) 155 (d) 151.5

70. The average marks scored by 50 students in a class were calculated to be 38. Later it was
found, that marks of two students were wrongly copied as 34 and 23 instead of 43 and 32.
Find correct average marks?
(a) 37.36 (b) 39.00
(c) 38.36 (d) None of these

71. For open end classification, which of the following is the best measure of central tendency?
(a) A.M. (b) G.M.
(c) Median (d) Mode

72. The median of the following items, 6, 10, 4, 3, 9 11, 22, 18 is ______________
(a) 10 (b) 9
(c) 9.5 (d) 10.5

73. In a group of persons, average weight is 60 kg. If the average weight of males and females
taken separately is 80 kg and 50 kg respectively, find the ratio of the number of males to
that of females.
(a) 2:3 (b) 3:2
(c) 2:1 (d) 1:2

74. The mean annual salary of all employees in a company is Rs. 25,000. The mean salary of
male and female employees is Rs. 27,000 and Rs. 17,000 respectively. Find the percentage
of males and females employed by the company.
(a) 60% and 40% (b) 75% and 25%
(c) 70% and 30% (d) 80% and 20%

239 | P a g e
75. The mean of numbers 1, 7, 5, 3, 4, 4 is m. The numbers 3, 2, 4, 2, 3, 3, P have mean m-1.
Then mean of P and 1 is equal to _______
(a) 4.0 (b) 2.5
(c) 4.5 (d) 3.5

76. If the A.M. and G.M. of two observations are 5 and 4 respectively, then the two
observations are ________
(a) 8, 2 (b) 7, 3
(c) 6, 4 (d) 5, 5

77. If x and y are related by x - y - 10 = 0 and mode of x is known to be 23, then the mode of y is .
(a) 20 (b) 13
(c) 3 (d) 23

78. A man travels at a speed of 20 km/hr and then returns at a speed of 30 km/hr His average
speed of the whole journey is.
(a) 25 km/ hr (b) 24.5 km/hr
(c) 24 km/hr (d) None

79. The median of the data 13,8, 11,6,4, 15 ,2,18.


(a) 5 (b) 8
(c) 11 (d) 9.5

80. The G.M. of 4, 6 and 8 is:


(a) 4.77 (b) 5.32
(c) 6.14 (d) 5.77

x x x
81. The median of x, , , is 10. Find x where x > 0
2 3 5
(a) 24 (b) 32
(c) 8 (d) 16

82. The average salary of 50 men was Rs. 80 but it was found that salary of 2 of them were Rs.
46 and Rs. 28 which was wrongly taken as Rs. 64 and Rs. 82. The revised average salary is:
(a) Rs. 80 (b) Rs. 78.56
(c) Rs. 85.26 (d) Rs. 82.92

83. If A be the A.M. of two positive unequal quantities X and Y and G be their G.M., then
(a) A<G (b) A>G
(c) AG (d) AG
84. When mean is 3.57 and mode is 2.13 then the value of median is .
(a) 3.09 (b) 5.01
(c) 4.01 (d) None of these

85. The mean weight of 15 students is 110 kg. The mean weight of 5 of them is 100 kg. and of
another five students is 125 kg. then the mean weight of the remaining students is:
(a) 120 (b) 105
(c) 115 (d) None of these

240 | P a g e
86. In a class of 11 students, 3 students were failed in a test. 8 students who passed secured
10,11,20,15,12,14,26 and 24 marks respectively. What will be the median marks of the
students:
(a) 12 (b) 15
(c) 13 (c) 13.5

87. A lady travel at a speed of 20 km/h and returned at quicker speed. If her average speed of
the whole journey is 24 km/h find the speed of return journey (in km/h).
(a) 25 (b) 30
(c) 35 (d) 38

88. Let the mean of the variable 'x' be 50 then the mean of u = 10+5x will be:
(a) 250 (b) 260
(c) 265 (d) 273

89. If the difference between mean and Mode is 63, then the difference between Mean and
Median will be .
(a) 63 (b) 31.5
(c) 21 (d) None of the above.

90. If the Arithmetic mean between two numbers is 64 and the Geometric mean between them is
16. The Harmonic Mean between them is .
(a) 64 (b) 4
(c) 16 (d) 40

91. The average of 5 quantities is 6 and the average of 3 is 8. What is the average of the
remaining two.
(a) 4 (b) 5
(c) 3 (d) 3.5

92. Given the following observations arranged in ascending order with median = 25. Find the
value of n.
11, 13, 15, 19, n + 2, n + 4, 30, 35, 39, 46
(a) 22 (b) 20
(c) 15 (d) 30

93. The mean of first 3 terms is 14 and the mean of next 2 terms is 18. The mean of 5 numbers
is ____
(a) 14.5 (b) 15
(c) 14 (d) 15.6

94. G.M of three observations 40, 50, and x is 10 the value of x is


(a) 2 (b) 4
(c) 1/2 (d) None

95. Which of the following measures of central tendency cannot be shown by graphical method ?
(a) Mean (b) Median
(c) Mode (d) Quartiles

241 | P a g e
96. GM of 8, 4, 2 is ______
(a) 4 (b) 2
(c) 8 (d) None
97. The average age of 15 students is 15 years. Out of these the average age of 5 students is 14
years and that of other 9 students is 16 years, then the age of 15 th student is …………….
(a) 11 years (b) 14 years
(c) 15 years (d) None of these
98. For two numbers A.M= 10 and G.M=8, then H.M=?
(a) 9 (b) 8.9
(c) 6.4 (d) None
99. In an class of 50 students, 10 have failed and their average marks in 2.5. The total marks
secured by the entire class were 281. The average marks who have passed is:
(a) 5.32 (b) 7.25
(c) 6.40 (d) None of the above.

100. For a moderately skewed distribution of marks in statistic for a group of 200 students the
mean marks and median mark were found tgo be 55.60 and 52.40 what is the modal mark?
(a) 42
(b) 46
(c) 70
(d) 74

101. If y = 2 + 1.50x and mode of x is 15, what is the mode of y?


(a) 20
(b) 28.50
(c) 24.50
(d) 34.50

102. Find the Gm for the following distribution:


X: 2 4 8 16
F: 2 3 3 2
(a) √2
(b) 3√2
(c) 4√2
(d) 5√2

103. Find the HM for 4, 6 and 10.


(a) 6.99
(b) 5.77
(c) 1.77
(d) 3.77

104. Fin the HM for the following data:


X: 2, 4, 8, 16
F; 2, 3, 3, 2
(a) 11
(b) 12
(c) 4.44
(d) 9.44

242 | P a g e
105. Compute AM, GM, and HM for the numbers 6, 8, 12, 36.
(a) 15.5, 9.93, 12 (b) 12, 9.93, 15.05
(c) 15.5, 12, 9.93 (d) None of the

106. Find the weighted AM and weighted HM of first n natural numbers, the weights being equal
to the squares of the corresponding numbers respectively.
3n  n  1 2n  1 2n  1 2n  1
(a) , (b) ,
2  2n  1 3 3 4

3n 2n  1 2n  1 3n  n  1
(c) , (d) ,
4 3 3 2

ANSWER

1. 2. 3. 4. 5. 6. 7. 8. 9. 10.
(B) (C) (C) (A) (C) (B) (A) (C) (D) (A)
11. 12. 13. 14. 15. 16. 17. 18. 19. 20.
(C) (B) (B) (C) (D) (B) (A) (B) (B) (C)
21. 22. 23. 24. 25. 26. 27. 28. 29. 30.
(C) (B) (D) (C) (B) (C) (A) (C) (C) (B)
31. 32. 33. 34. 35. 36. 37. 38. 39. 40.
(A) (C) (C) (B) (A) (C) (C) (B) (D) (C)
41. 42. 43. 44. 45. 46. 47. 48. 49. 50.
(C) (B) (B) (C) (C) (C) (B) (B) (B) (C)
51. 52. 53. 54. 55. 56. 57. 58. 59. 60.
(B) (B) (B) (B) (B) (B) (B) (D) (B) (A)
61. 62. 63. 64. 65. 66. 67. 68. 69. 70.
(B) (A) (A) (D) (A) (B) (A) (B) (D) (C)
71. 72. 73. 74. 75. 76. 77. 78. 79. 80.
(C) (C) (D) (D) (B) (A) (B) (C) (D) (D)
81. 82. 83. 84. 85. 86. 87. 88. 89. 90.
(A) (B) (B) (A) (B) (A) (B) (B) (C) (B)
91. 92. 93. 94. 95. 96. 97. 98. 99. 100.
(C) (A) (D) (C) (A) (A) (A) (C) (C) (B)
101. 102. 103. 104. 105. 106.
(C) (C) (B) (C) (C) (A)

243 | P a g e
CHAPTER – 15 - DISPERSION
PRACTICE QUESTIONS

1. The standard deviation of 10, 16, 10, 16, 10, 10, 16, 16 is
(a) 4 (b) 6
(c) 3 (d) 0

2. If the first quartile is 142 and semi-inter quartile range is 18, then the value of median is:
(a) 151 (b) 160
(c) 178 (d) None of these

3. Which measure of dispersion is based on all the observations?


(a) Mean deviation (b) Standard deviation
(c) Quartile deviation (d) (a) and (b) but not (c)

4. What is the coefficient of range for the following wages of 8 workers?


Rs. 80, Rs. 65, Rs. 90, Rs. 60, Rs. 75, Rs. 70, Rs. 72, Rs. 85.
(a) Rs. 30 (b) Rs. 20
(c) Rs. 40 (d) 20

5. For any two numbers Standard Deviation is always


(a) Twice the range (b) Half of the range
(c) Square of the range (d) None of these

6. ______ is an absolute measure of dispersion.


(a) Range (b) Mean Deviation
(c) Standard Deviation (d) All these measures

7. If the range of x is 2, what would be the range of – 3x + 50 ?


(a) 2 (b) 6
(c) –6 (d) 44

8. What is the standard deviation of 5, 5, 9, 9, 9, 10, 5, 10, 10?


(a) 14 (b) √42/3
(c) 4.50 (d) 8

9. If the quartile deviation of a normal curve is 4.05, then its mean deviation is
(a) 5.26 (b) 6.24
(c) 4.24 (d) 4.86

10. If all the observations are increased by 10, then


(a) Standard Deviation would be increased by 10
(b) Mean deviation would be increased by 10
(c) Quartile deviation would be increased by 10
(d) All these three remain unchanged

11. The coefficient of mean deviation about mean for the first 9 natural numbers is
(a) 200/9 (b) 80
(c) 400/9 (d) 50

244 | P a g e
12. If all the observations are multiplied by 2, then
(a) New Standard Deviation would be also multiplied by 2.
(b) New Standard Deviation would be half of the previous SD.
(c) New Standard Deviation would be increased by 2.
(d) New Standard Deviation would be decreased by 2.

13. Quartiles are values dividing a given set of observations into _______ equal parts.
(a) Two (b) Four
(c) Six (d) Ten

14. Following are the marks of 10 students :


82, 79, 56, 79, 85, 95, 55, 72, 70, 66 .
Find coefficient of range.
(a) 25.66 (b) 26.67
(c) 27.66 (d) 28.67

15. Coefficient of variation of two series are 60% and 80% respectively. Their standard
deviation are 20 and 16 respectively, what are their A.M.
(a) 15 and 20 (b) 33.3 and 20
(c) 33.3 and 15 (d) 12 and 16

16. If x and y are perfectly related by 2x + 3y + 4 = 0 and SD of x is 6 then SD of y is


(a) 22 (b) 4
(c) 25 (d) 8.30

17. The standard deviation of 25, 32, 43, 53, 62, 59, 48, 31, 24, 33 is ________
(a) 13.23 (b) 12.33
(c) 11.13 (d) None of these

18. If x and y are related as 4x + 3y + 11 = 0 and mean deviation of x is 2.70. what is mean
deviation of y?
(a) 7.20 (b) 14.40
(c) 3.60 (d) None of these

19. The range of 12, 15, 17, 9, 21, 25


(a) 3 (b) 16
(c) 25 (d) 12

20. Wages of 8 workers expressed in Rs. as follows 80, 96, 51, 72, 67, 50, 70, find Coefficient
of range?
(a) 30.5 (b) 35.3
(c) 31.00 (d) 31.51

21. If all the observations are divided by 3, then


(a) New Standard Deviation would also be divided by 3
(b) New Standard Deviation would be multiplied by 3
(c) New Standard Deviation will increase by 3
(d) New Standard Deviation will decrease by 3

245 | P a g e
22. Which measures of dispersion is the quickest to compute
(a) Standard deviation (b) Mean deviation
(c) Quartile deviation (d) Range

23. If the first quartile is 104 and quartile deviation is 18, the third quartile will be
(a) 140 (b) 116
(c) 20 (d) 0

24. If the quartile deviation of x is 6 and 4x + 8y = 20, what is the quartile deviation of y
(a) 3 (b) 4
(c) 5 (d) 1

25. S.D. of first n natural numbers is


n( n  1) n(n  1)
(a) (b)
2 2
n2  1
(c) (d) None of these
12
26. For any two numbers range is always
(a) Twice the Standard Deviation (b) Half the Standard Deviation
(c) Square the Standard Deviation (d) None of these

27. The standard deviation of 100 and 150 items are 5,6 respectively; if mean of 250 item is
44, mean of 100 and 150 item are 50 and 40 respectively, then find S.D. for 250 items.
(a) 7.46 (b) 7.64
(c) 6.74 (d) 4.67

28. The marks obtained by 10 students in an examinations were as follows: 70, 65, 68, 70,
75, 73, 80, 70, 83, 86. Find mean deviation about the mean?
(a) 5.3 (b) 5.4
(c) 5.5 (d) 5.6

29. For a set of 100 observations, taking assumed mean as 4, the sum of the deviations is
–11 cm and the sum of squares of these deviations is 257 cm 2. Find the coefficient of
variation?
(a) 41.13% (b) 14.13%
(c) 25.13% (d) 52.13%

30. Find the variance given that the Arithmetic Mean = (8+4)/2
(a) 2 (b) 6
(c) 1 (d) 4

31. For the numbers 1, 2, 3, 4, 5, 6, 7 standard deviation is:


(a) 3 (b) 4
(c) 2 (d) None of these

32. If the quartile deviation of x is 8 and 3x + 6y = 20, then the quartile deviation of y is
(a) -4 (b) 3
(c) 5 (d) None of these

246 | P a g e
33. For a moderate skewed distribution, quartile deviation and the standard deviation are
related by.
(a) S.D. =
2 Q.D. (b) S.D. =
3 Q.D.
3 4
4 3
(c) S.D. = Q.D. (d) S.D. = Q.D.
3 2

34. If two variables x and y are related by 2x + 3y – 7 = 0. The mean and mean deviation
about mean of x are 1 and 0.3 respectively then the coefficient of mean deviation of y
about mean is.
(a) -5 (b) 4
(c) 12 (d) 50

35. Measures of dispersion are called averages of the order.


(a) 1st (b) 2nd
(c) 3rd (d) None

36. What is the coefficient of range for the following distribution?


Class Interval: 10-19 20-29 30-39 40-49 50-59
Frequency : 11 25 16 7 3
(a) 22 (b) 50
(c) 75.82 (d) 72.46

37. A sample of 35 observations has the mean 80 and S.D. as 4. A second sample of 65
observations from the same population has mean 70 and S.D. 3. The S.D. of the combined
sample is
(a) 5.85 (b) 6.58
(c) 10.23 (d) None of these

38. The best measure of dispersion is


(a) Q.D. (b) M.D.
(c) Range (d) S. D.
x -a
39. If the mean and S.D. of x are a and b respectively then the S.D. of is:
b
(a) a/b (b) –b
(c) 1 (d) ab

40. Suppose a population A has 100 observations 101, 102, 103……. 200 and another
population B has 100 observations 151, 152, 153……….. 250. If VA/VB represents the
variance of the two populations respectively then V A/VB=:
(a) 9/4 (b) 1
(c) 4/9 (d) 2/3

41. The Mean and S.D. for group of 100 observations are 65 and 7.03 respectively If 60 of
these observations have mean and S.D. as 70 and 3 respectively, what is the S.D. for the
group comprising 40 observations?
(a) 2.03 (b) 4.03
(c) 8.03 (d) None of these

247 | P a g e
42. The quartile deviation for the data is:
x: 2 3 4 5 6
f: 3 4 8 4 1
(a) ¼ (b) ½
(c) 1 (d) 0

( x  50)
43. Mean and S. D. of x is 50 and 5 respectively. Find mean and S.D. of .
5
(a) (1, 0) (b) (0, 1)
(c) (1, 0.1) (d) (0,5)

44. Mean and S. D. of a given set of observations is 1,500 and 400 respectively. If there is an
increment of 100 in the first year and each observation is hiked by 20% in second years
then find new mean and S.D.
(a) 1920,480 (b) 1920, 580
(c) 1600,480 (d) 1600,400

45. If 5 is subtracted from each observation of some certain item then its co efficient of
variation is 10% and if 5 is added to each item then its coefficient of variation is 6%. Find
original coefficient of variation.
(a) 8% (b) 7.5%
(c) 4% (d) None of these

46. Inter Quartile Range is ______of Quartile Deviation.


(a) Half (b) Double
(c) Triple (d) Equal

47. If L1 = highest observation and L 2 =smallest observation, then Coefficient of Range


L1  L 2 L1  L 2
(a)  100 (b)  100
L1 / L 2 L1  L 2
L1  L 2 L1 / L 2
(c)  100 (d)  100
L1  L 2 L1  L 2

48. If standard deviation of x is 5, then find the standard deviation of (2 - 3x).


(a) 10 (b) 15
(c) 5 (d) -13

49. The variance of data : 3,4,5,8 is.


(a) 4.5 (b) 3.5
(c) 5.5 (d) 6.5

50. If all observations in a distribution are increased by 6, then the variance of the series will
be .
(a) Increased (b) Decreased
(c) Unchanged (d) None of these

248 | P a g e
51. The S.D of weight of some members is 4.5 k.g. After some time it was observed that each
member was under weight by 0.5 kg then what is new S.D?
(a) < 4.5 (b) > 4.5
(c) = 4.5 (d) None

52. S.D of 1st ‘n’ natural numbers is 2 then n =


(a) 12 (b) 7
(c) 9 (d) 5

53. The S.D is independent of change of ____


(a) Origin (b) Scale
(c) Both (d) None

54. In a normal distribution Q.D is 6, then S.D is


(a) 4 (b) 9
(c) 7 (d) 6
aX  b
55. If the S.D of X is  then S.D of is
c
a b
(a)  (b) 
c c
c
(c)  (d) None
a

56. Which of the following measures of dispersion is used for finding consistency between the
series?
(a) Q.D (b) S.D
(c) Coefficient of variation (d) None of these

57. If sum of squares of the values = 3390, N=30 and standard deviation =7. Find out the
mean.
(a) 113 (b) 210
(c) 8 (d) None of these

58. If the mean of frequency distribution is 100 and coefficient of variation is 45% then
standard deviation is:
(a) 45 (b) 0.45
(c) 4.5 (d) 450

59. In a normal distribution the relation between QD and SD is _______


(a) 3 QD = 2 SD (b) 3 SD = 2 QD
(c) 4 QD = 3 SD (d) None of these

60. X 2
 3390 , n=30,  =7 then X = …………….
(a) 113 (b) 210
(c) 8 (d) None
61. If the mean of frequency distribution is 100 and coefficient of variation is 45% then
standard deviation is ……………
(a) 45 (b) 0.45
(c) 0.045 (d) None

249 | P a g e
84
62. If Arithmetic Mean = , the Variance is :
2
(a) 2 (b) 6
(c) 1 (d) 4

63. Coefficient of mean deviation about mean for the first 9 natural numbers is :
200
(a) (b) 80
9
400
(c) (d) 50
9
64. Mean = 5, S.D. = 2.6, Median = 5, Q.D. = 1.5 then Coefficient of Q.D. is :
(a) 35 (b) 39
(c) 30 (d) 32
65. Compute the mean deviation about the arithmetic mean for the following data;
X: 1 3 5 7 9
Y: 5 8 9 2 1
I so find the coefficient of the mean deviation about the Am.
(a) 55
(b) 44.33
(c) 76.33
(d) 11.33
66. Compute the SD of 9, 5, 8, 6, 2
Without any more computation, obtain the SD of
Sample I -1, -5, -2, -4, -8,
Sample II 90, 50, 80, 60, 20,
Sample III 23, 15, 21, 17, 9,
(a) 2.45, 3.45, 2.90
(b) 2.45, 12.45, 4.90
(c) 2.45, 24.50, 4.90
(d) 2.45, 2.45, 4.90
67. The mean and standard deviation of the salaries of two factories are provided below:
Factory No. of Employees Mean Salary SD of Salary
A 30 Rs. 4800 Rs. 10
B 20 Rs. 5000 Rs. 12
i) Find the combined mean salary and standard deviation of salary.
ii) Examine which factory has more consistent structure so far as satisfying its employees
are concerned.
(a) 4880, 98.58 A
(b) 4800, 48.58 A
(c) 4400, 120 A
(d) 4400, 120 B
68. The mean and variance of 5 observations are 4.80 and 6.16 respectively. If three of the
observations are 2, 3 and 6, what are the remaining observations?
(a) 5,0
(b) 4,9
(c) 11,3
(d) 6,7

250 | P a g e
69. Which one is easier to computer?
(a) Relative measures of dispersion
(b) Absolute measures of dispersion
(c) Both a) and b)
(d) Range

70. Which measure of dispersion is most useful?


(a) Standard deviation
(b) Quartile deviation
(c) Mean deviation
(d) Range

71. Which measures of dispersions is not affected by the presence of extreme observations?
(a) Range
(b) Mean deviation
(c) Standard deviation
(d) Quartile deviation

72. Which measure of dispersion is based on the absolute deviations only?


(a) Standard deviation
(b) Mean deviation
(c) Quartile deviation
(d) Range

73. Which measure of dispersion has some desirable mathematical properties?


(a) Standard deviation
(b) Mean deviation
(c) Quartile Deviation
(d) All these measures

74. Which measure of dispersion is considered for finding a pooled measure of dispersion after
combining several groups?
(a) Mean deviation
(b) Standard deviation
(c) Quartile Deviation
(d) Any of these

2
75. The mean and SD for a, b and 2 are 3 and respectively, the value of ab would be
√3
(a) 5
(b) 6
(c) 11
(d) 3

76. 1st quartile is 142, Semi-inter quartile range is 18. Then median is
(a) 151 (b) 160
(c) 178 (d) None

77. Q.D is
(a) 2/3S.D (b) 4/5S.D
(c) 5/6S.D (d) None

251 | P a g e
78. The standard deviation of a variable x is known to be 10. the standard deviation of 50+5x is
(a) 50 (b) 100
(c) 10 (d) 500

79. The harmonic mean H of two numbers is 4 and their arithmetic mean A and the geometric
mean G satisfy the equation 2A + G 2 = 27, then the numbers are
(a) (1, 3) (b) (9, 5)
(c) (6, 3) (d) (12, 7)

80. Coefficient of quartile deviation is equal to


(a) Quartile deviation x 100/median
(b) Quartile deviation x 100/mean
(c) Quartile deviation x 100/mode
(d) None

81. If all the observation are increased by 5, then


(a) S.D. would be increased by 5
(b) Mean deviation would be increased by 5
(c) Quartile deviation would be increased by 5
(d) All the three would not be increased by 5

82. What is value of mean deviation about mean from the number 5, 8, 6, 3 and 4?
(a) 5.20 (b) 7.20
(c) 1.44 (d) 2.23

83. For the observation of 6, 4, 1, 6, 5, 10, 4, 8 the range is:


(a) 10 (b) 9
(c) 8 (d) None

84. If a variance of a random variable 'x' is 23, then what is variance of 2x + 10?
(a) 56 (b) 33
(c) 46 (d) 92

85. If variance = 148.6 and x = 40, then the coefficient of variation is:
(a) 37.15 (b) 30.48
(c) 33.75 (d) None of the above

86. Quartiles can be determined graphically using:


(a) Histogram (b) Frequency polygon
(c) Ogive curve (d) Pie chart

252 | P a g e
ANSWER

1. 2. 3. 4. 5. 6. 7. 8. 9. 10.
(C) (B) (D) (D) (B) (D) (B) (B) (D) (D)
11. 12. 13. 14. 15. 16. 17. 18. 19. 20.
(C) (A) (B) (B) (B) (B) (A) (C) (B) (D)
21. 22. 23. 24. 25. 26. 27. 28. 29. 30.
(A) (D) (A) (A) (C) (A) (A) (D) (A) (D)
31. 32. 33. 34. 35. 36. 37. 38. 39. 40.
(C) (D) (D) (C) (B) (D) (A) (D) (C) (B)
41. 42. 43. 44. 45. 46. 47. 48. 49. 50.
(B) (C) (B) (A) (B) (B) (B) (B) (B) (C)
51. 52. 53. 54. 55. 56. 57. 58. 59. 60.
(C) (B) (A) (B) (A) (C) (C) (A) (A) (C)
61. 62. 63. 64. 65. 66. 67. 68. 69. 70.
(A) (B) (C) (C) (B) (C) (A) (B) (B) (A)
71. 72. 73. 74. 75. 76. 77. 78. 79. 80.
(D) (B) (D) (B) (C) (B) (A) (A) (C) (A)
81. 82. 83. 84. 85. 86.
(D) (C) (B) (D) (B) (C)

253 | P a g e
CHAPTER – 16 - CORRELATION AND REGRESSION

PRACTICE QUESTIONS – SCATTER DIAGRAM

1. Scatter diagram is considered for measuring


(a) Linear relationship between two variables
(b) Curvilinear relationship between two variables
(c) Neither (a) nor (b)
(d) Both (a) and (b)

2. If the plotted points in a scatter diagram lie from upper left to lower right, then the correlation
is
(a) Positive (b) Zero
(c) Negative (d) None of these

3. If the plotted points in a scatter diagram are evenly distributed, then the correlation is
(a) Zero (b) Negative
(c) Positive (d) (a) or (b)

4. If all the plotted points in a scatter diagram lie on a single line, then the correlation is
(a) Perfect positive (b) Perfect negative
(c) Both (a) and (b) (d) Either (a) or (b)

5. The correlation between shoe-size and intelligence is


(a) Zero (b) Positive
(c) Negative (d) None of these

6. If r = 0.8, then coefficient of determination shall be _________


(a) 0.64 (b) 0.40
(c) 0.60 (d) 0.80

7. Pearson’s correlation coefficient is used for finding


(a) Correlation for any type of relation (b) Correlation for linear relation only
(c) Correlation for curvilinear relation only
(d) Both (b) and (c)

8. In a Scatter diagram, if the plotted points lie on a single line, then the correlation is:
(a) Perfect positive (b) Perfect negative
(c) Both (a) and (b) (d) Either (a) or (b)

9. If the value of correlation coefficient is positive, then the points in a scatter diagram tend to
cluster
(a) From lower left corner to upper right corner
(b) From lower left corner to lower right corner
(c) From lower right corner to upper left corner
(d) From lower right corner to upper right corner

254 | P a g e
10. When r = 1, all the points in a scatter diagram would lie
(a) On a straight line directed from lower left to upper right
(b) On a straight line directed from upper left to lower right
(c) On a straight line
(d) Both (a) and (b)

11. Product moment correlation coefficient may be defined as the ratio of


(a) The product of standard deviations of the two variables to the covariance between
them
(b) The covariance between the variables to the product of the variances of them
(c) The covariance between the variables to the product of their standard deviations
(d) Either (b) or (c)

12. The covariance between two variables is


(a) Strictly positive (b) Strictly negative
(c) Always 0 (d) Either positive or negative or zero

13. The coefficient of correlation between two variables


(a) Can have any unit.
(b) Is expressed as the product of units of the two variables
(c) Is a unit free measure
(d) None of these

14. What are the limits of the correlation coefficient?


(a) No limit (b) –1 and 1
(c) 0 and 1, including the limits (d) –1 and 1, including the limits

15. In case the correlation coefficient between two variables is 1, the relationship between the
two variables would be
(a) y = a + bx (b) y = a + bx, b > 0
(c) y = a + bx, b < 0 (d) y = a + bx, both a and b being positive

16. Correlation Co-efficient is _______ of the units of measurements


(a) Dependent (b) Independent
(c) Both (d) None of these

17. For finding correlation between two attributes, we consider


(a) Pearson’s correlation coefficient (b) Scatter diagram
(c) Spearman’s rank correlation coefficient
(d) Coefficient of concurrent deviations

18. For finding the degree of agreement about beauty between two Judges in a Beauty Contest,
we use
(a) Scatter diagram (b) Coefficient of rank correlation
(c) Coefficient of correlation (d) Coefficient of concurrent deviation

19. If there is a perfect disagreement between the marks in Geography and Statistics, then what
would be the value of rank correlation coefficient?
(a) Any value (b) Only 1
(c) Only –1 (d) (b) or (c)

255 | P a g e
20. When we are not concerned with the magnitude of the two variables under discussion, we
consider
(a) Rank correlation coefficient (b) Product moment correlation coefficient
(c) Coefficient of concurrent deviation (d) (a) or (b) but not (c)

21. What is the quickest method to find correlation between two variables?
(a) Scatter diagram (b) Method of concurrent deviation
(c) Method of rank correlation (d) Method of product moment correlation

22. What are the limits of the coefficient of concurrent deviations?


(a) No limit
(b) Between –1 and 0, including the limiting values
(c) Between 0 and 1, including the limiting values
(d) Between –1 and 1, the limiting values inclusive

23. The two variables are known to be_______ if the movement on the part of one variable does
not produce any movement of the other variable in a particular direction.
(a) Correlated (b) Positive correlated
(c) Negative correlated (d) Uncorrelated

24. If the plotted points in a scatter diagram lie from upper left to lower right, then the
correlation is_______.
(a) Positive (b) Zero
(c) Negative (d) None of these

25. Bivariate Data are the data collected for


(a) Two variables (b) More than two variables
(c) Two variables at the same point of time
(d) Two variables at different points of time

26. For finding the degree of agreement about beauty between two Judges in a Beauty Contest,
we use______ .
(a) Scatter diagram (b) Coefficient of rank correlation
(c) Coefficient of correlation (d) Coefficient of concurrent deviation

27. When r = 0 then cov(x,y) is equal to


(a) +1 (b) –1
(c) 0 (d) None of these

28. In case ‘The ages of husbands and wives’ correlation is ______ .


(a) Positive (b) Negative
(c) Zero (d) One

29. Maximum value of Rank Correlation coefficient is


(a) –1 (b) +1
(c) 0 (d) None of these

256 | P a g e
30. The minimum value of correlation coefficient is
(a) 0 (b) –2
(c) 1 (d) –1

31. The coefficient of correlation


(a) Has no limits. (b) Can be less than one.
(c) Can be more than one. (d) Varies between ± 1. (both inclusive)

32. If the coefficient of correlation between two variables is – 0.2, then the coefficient of
determination is:
(a) 0.8 (b) 0.2
(c) 0.4 (d) 0.04

33. If x denotes height of a group of students expressed in cm. and y denotes their weight
expressed in kg. , then the correlation coefficient between height and weight
(a) Would be shown in kg. (b) Would be shown in cm.
(c) Would be shown in kg. and cm. (d) Would be free from any unit.

34. The correlation between sale of cold drinks and day temperature is _________.
(a) Zero (b) Positive
(c) Negative (d) None of these

35. In case of a _________, plotted points on a scatter diagram lie from lower left corner to
upper right corner.
(a) Zero correlation (b) Negative correlation
(c) Positive correlation (d) Simple correlation

36. The correlation between Employment and Purchasing power is ________.


(a) Positive (b) Negative
(c) Zero (d) None of these

37. If the coefficient of correlation between two variables is –0.4, then the coefficient of
determination is
(a) 0.6 (b) 0.16
(c) 0.4 (d) 0.2

38. If the coefficient of correlation between two variables is 0.7, then the percentage of variation
unaccounted for is
(a) 70% (b) 30%
(c) 51% (d) 49%

39. The correlation is said to be positive


(a) When the values of two variables move in the same direction.
(b) When the values of two variables move in the opposite direction.
(c) When the values of two variables would not change.
(d) None of these.

40. Coefficient of determination is defined as


(a) r3 (b) 1– r2
(c) 1+ r2 (d) r2

257 | P a g e
41. In case of _______, plotted points on a scatter diagram would be equally distributed without
depicting any particular pattern.
(a) Zero correlation (b) Positive correlation
(c) Negative correlation (d) Simple correlation

42. What is spurious correlation?


(a) It is a bad relation between two variables.
(b) It is very low correlation between two variables.
(c) It is the correlation between two variables having no causal relation.
(d) It is a negative correlation.

43. Pearson’s correlation coefficient is used for finding


(a) Correlation for any type of relation
(b) Correlation for linear relation only
(c) Correlation for curvilinear relation
(d) Both (b) and (c)

44. The coefficient of correlation between two variables


(a) Can have any unit.
(b) Is expressed as the product of units of the two variables
(c) Is a unit free measure
(d) None of these.

ANSWER

1. 2. 3. 4. 5. 6. 7. 8. 9. 10.
(D) (C) (A) (D) (A) (A) (B) (D) (A) (A)
11. 12. 13. 14. 15. 16. 17. 18. 19. 20.
(C) (D) (C) (D) (B) (B) (C) (B) (C) (C)
21. 22. 23. 24. 25. 26. 27. 28. 29. 30.
(B) (D) (D) (C) (C) (B) (C) (A) (B) (A)
31. 32. 33. 34. 35. 36. 37. 38. 39. 40.
(D) (D) (D) (B) (C) (A) (B) (C) (A) (D)
41. 42. 43. 44.
(A) (C) (B) (C)

PRACTICE QUESTIONS : KARL PEARSON’S METHOD (PRODUCT MOMENT CORRELATION METHOD)

1. If for two variable x and y, the covariance, variance of x and variance of y are 40, 16 and 256
respectively, what is the value of the correlation coefficient?
(a) 0.01 (b) 0.625
(c) 0.4 (d) 0.5

2. If cov(x, y) = 15, what restrictions should be put for the standard deviations of x and y?
(a) No restriction.
(b) The product of the standard deviations should be more than 15.
(c) The product of the standard deviations should be less than 15.
(d) The sum of the standard deviations should be less than 15.

258 | P a g e
3. If the covariance between two variables is 20 and the variance of one of the variables is 16,
what would be the variance of the other variable?
(a) More than 100 (b) More than 10
(c) Less than 10 (d) More than 1.25

4. If u + 5x = 6 and 3y – 7v = 20 and the correlation coefficient between x and y is 0.58 then


what would be the correlation coefficient between u and v?
(a) 0.58 (b) –0.58
(c) –0.84 (d) 0.84

6. From the following data


X: 2 3 5 4 7
Y: 4 6 7 8 10
Two coefficient of correlation was found to be 0.93. What is the correlation between u and v
as given below?
U: -3 -2 0 -1 2
V: -4 -2 -1 0 2
(a) –0.93 (b) 0.93
(c) 0.57 (d) –0.57

7. Referring to the data presented in Q. No. 6, what would be the correlation between u and v?
U: 10 15 25 20 35
V: -24 -36 -42 -48 -60
(a) –0.6 (b) 0.6
(c) –0.93 (d) 0.93

8. What is the value of correlation coefficient due to Pearson on the basis of the following data:
X: -5 -4 -3 -2 -1 0 1 2 3 4 5
Y: 27 18 11 6 3 2 3 6 11 18 27
(a) 1 (b) –1
(c) 0 (d) –0.5

9. The coefficient of correlation r between x and y when .


Cov (x, y) = -16.5, Var (x) 2.89, Var (y) =100 is:
(a) -0.97 (b) 0.97
(c) 0.89 (d) -0.89

10. Take 200 and 150 respectively as the assumed mean for X and Y series of 11 values , then
dx = X-200, dy = X-150,  dx = 13,  dx2=2667,  dy = 42,  dy2 = 6964,  dx dy =3943. The
value of r is :
(a) 0.77 (b) 0.98
(c) 0.92 (d) 0.82

11. The coefficient of correlation between X and Y is 0.6 U and V are two variables defined as U =
x 3 Y2
,V= then the coefficient of correlation between U and V is:
2 3
(a) 0.6 (b) 0.4
(c) 0.8 (d) 1

259 | P a g e
12. Assume 69 and 112 as the mean values for X and Y respectively.
 dx =47,  dx2 =1475,  dy =108,  dy2 =3468,,  dxdy =2116 and N =8. Where dx = X-69,dy
=y-112 Then the value of r is :
(a) 0.95 (b) 0.65
(c) 0.75 (d) 0.85

13. The coefficient of correlation between x and y series from-the following data:
X series Y series
Number of pairs of observation 15 15
Arithmetic Mean 25 18
Standard Deviation 3.01 3.03
Sum of the squares of deviation from mean 136 138
Sum of the product of the deviations of x and y series from their respective means = 122 is
(a) 0.89 (b) 0.99
(c) 0.69 (d) 0.91

14. If coefficient of correlation between x and y is 0.46. Find coefficient of correlation between x
y
and
2
(a) 0.46 (b) 0.92
(c) –0.46 (d) –0.92

15. Correlation coefficient between X and Y will be negative when:


(a) X and Y are decreasing (b) X is increasing, Y is decreasing
(c) X and Y are increasing (d) None of these

16. If ‘P’ is the simple correlation coefficient, the quantity P2 is known as:
(a) Coefficient of determination (b) Coefficient of Non-determination
(c) Coefficient of alienation (d) None of the above.

x5 y7
17. If the correlation coefficient between x and y is r then between U = and V = is
10 2
(a) r (b) –r
(c) (r-5)/2 (d) (r—7)/l0

18. If the sum of the product of deviation of x and y series from their mean is zero then the
coefficient of correlation will be .
(a) 1 (b) -1
(c) 0 (d) None of these

19. The covariance between two variables X and Y is 8.4 and their variances are 25 and 36
respectively. Calculate Karl Pearson’s coefficient of correlation between them.
(a) 0.82 (b) 0.28
(c) 0.01 (d) 0.09

20. Covariance of two variables x & y is 25 V(x) = 36 & V(y) = 25 then r =


(a) 0.409 (b) 0.419
(c) 0.833 (d) 0.027

260 | P a g e
21. If r=0.28, Cov (x,y) = 7.6, V (x)= 9 then  y 
(a) 8.75 (b) 9.04
(c) 6.25 (d) None

22. If r is the Karl pearson’s coefficient of correlation in a bivariate distribution, the two
regression lines are at right angles when …………..
(a) r=  1 (b) r=0
(c) r   (d) None
23. Price and Demand is example for :
(a) No correlation (b) Positive correlation
(c) Negative correlation (d) None of these
24. Determine the coefficient of correlation between x and y series :
X-Series Y-Series
Number of items 15 15
Arithmetic mean 25 18
Sum of square of deviation of mean 136 138
Sum of product deviation of x and y series from mean = 122
(a) –0.89 (b) 0.89
(c) 0.69 (d) –0.69

Answer
1. 2. 3. 4. 5. 6. 7. 8. 9. 10.
(B) (B) (A) (B) (B) (B) (C) (C) (A) (C)
11. 12. 13. 14. 15. 16. 17. 18. 19. 20.
(A) (A) (A) (A) (B) (A) (A) (C) (B) (C)
21. 22. 23. 24.
(B) (B) (C) (B)

PRACTICE QUESTIONS - SPEARMAN’S RANK CORRELATION COEFFICIENT

1. If the sum of squares of difference of ranks, given by two judges A and B, of 8 students in 21,
what is the value of rank correlation coefficient?
(a) 0.7 (b) 0.65
(c) 0.75 (d) 0.8

2. If the rank correlation coefficient between marks in management and mathematics for a
group of student in 0.6 and the sum of squares of the differences in ranks in 66, what is the
number of students in the group?
(a) 10 (b) 9
(c) 8 (d) 11
3. While computing rank correlation coefficient between profit and investment for the last 6
years of a company the difference in rank for a year was taken 3 instead of 4. What is the
rectified rank correlation coefficient if it is known that the original value of rank correlation
coefficient was 0.4?
(a) 0.3 (b) 0.2
(c) 0.25 (d) 0.28

261 | P a g e
4. For 10 pairs of observations, No. of concurrent deviations was found to be 4. What is the
value of the coefficient of concurrent deviation?
(a) 0.2 (b) – 0.2
(c) 1/3 (d) –1/3

5. If the sum of squares of the rank difference in mathematics and physics marks of 10 students
is 22 then the coefficient of rank correlation is:
(a) 0.267 (b) 0.867
(c) 0.92 (d) None
6. For the following data, the coefficient of rank correlation is:
Rank in Botany: 1 2 3 4 5
Rank in Chemistry: 2 3 1 5 4
(a) 0.93 (b) 0.4
(c) 0.6 (d) None
7. The coefficient of rank correlation of marks obtained by 10 students in English and Economics
was found to be 0.5. It was later discovered that the difference in ranks in the two subjects
obtained by one student was wrongly taken as 3 instead of 7. The correct coefficient of rank
correlation is .
(a) 0.32 (b) 0.26
(c) 0.49 (d) 0.93
8. If the sum of square of differences of rank is 50 and number of items is 8 then what is the
value of rank correlation coefficient.
(a) 0.59 (b) 0.40
(c) 0.36 (d) 0.63
9. Ranks of two characteristics by two judges are in reverse order then find the value of
Spearman rank correlation co-efficient.
(a) –1 (b) 0
(c) 1 (d) 0.75

10. The Ranks of five participants given by two judges are


PARTICIPANTS
A B C D E
Judge. 1 1 2 3 4 5
Judge. 2 5 4 3 2 1
Rank Correlation coefficient between ranks will be .
(a) 1 (b) 0
(c) -1 (d) 1/2
11. Three competitors in a contest are ranked by two judges in the order 1,2,3 and 2,3,1
respectively. Calculate the Spearman’s rank correlation coefficient.
(a) -0.5 (b) -0.8
(c) 0.5 (d) 0.8
12. In Spearman’s rank correlation coefficient the sum of difference of ranks between variables
shall be
(a) 0 (b) 1
(c) -1 (d) +1

262 | P a g e
ANSWER

1. 2. 3. 4. 5. 6. 7. 8. 9. 10.
(C) (A) (B) (D) (B) (C) (B) (B) (A) (C)
11. 12.
(A) (A)

PRACTICE QUESTIONS - COEFFICIENT OF CONCURRENT DEVIATIONS

1. The coefficient of concurrent deviation for p pairs of observations was found to be 1 / 3 . If


the number of concurrent deviations was found to be 6, then the value of p is.
(a) 10 (b) 9
(c) 8 (d) none of these

2. For 10 pairs of observations, number of concurrent deviations was found to be 4. What is the
value of the coefficient of concurrent deviation ?
(a) 0.2 (b) 1/3

(c) -1/3 (d)  0.2

3. What is coefficient of concurrent deviations for the following data;


Supply: 68 43 38 78 66 83 38 23 83 63 53
Demand:65 60 55 61 35 75 45 40 85 80 85
(a) 0.82
(b) 0.85
(c) 0.89
(d) -0.81
ANSWER

1. 2. 3.
(A) (C) (C)

PRACTICE QUESTIONS - REGRESSION ANALYSIS

1. If there are two variables x and y, then the number of regression equations could be
(a) 1 (b) 2
(c) Any number (d) 3

2. Since Blood Pressure of a person depends on age, we need consider


(a) The regression equation of Blood Pressure on age
(b) The regression equation of age on Blood Pressure
(c) Both (a) and (b)
(d) Either (a) or (b)

3. The method applied for deriving the regression equations is known as


(a) Least squares (b) Concurrent deviation
(c) Product moment (d) Normal equation

263 | P a g e
4. If 2 variables are uncorrelated then regression lines are _____
(a) Parallel (b) Perpendicular
(c) Coincident (d) Inclined at 450

5. If y = 18x+5 is the regression line of x on y then the value of b xy is ______


5
(a) (b) 18
18
1
(c) 5 (d)
18
6. For certain x and y series which are correlated, the two lines of regression are 5x – 6y + 9 =
0, 15x – 8y + 130 = 0. The correlation coefficient is:
(a) 4/5 (b) ¾
(c) 2/3 (d) ½

7. The two lines of regression become identical when


(a) r=1 (b) r = –1
(c) r=0 (d) (a) or (b)

8. What are the limits of the two regression coefficients?


(a) No limit (b) Must be positive
(c) One positive and the other negative
(d) Product of the regression coefficient must be numerically less than unity.

9. The regression coefficients remain unchanged due to a


(a) Shift of origin (b) Shift of scale
(c) Both (a) and (b) (d) (a) or (b)

10. Following are the two normal equations obtained for deriving the regression line of y and x:
5a + 10b = 40
10a + 25b = 95
The regression line of y on x is given by
(a) 2x + 3y = 5 (b) 2y + 3x = 5
(c) y = 2 + 3x (d) y = 3 + 5x

11. If the regression line of y on x and of x on y are given by 2x + 3y = –1 and 5x + 6y = –1


then the arithmetic means of x and y are given by
(a) (1, –1) (b) (–1, 1)
(c) (–1, –1) (d) (2, 3)

12. Given the regression equations as 3x+y=13 and 2x+5y=20, which one is the regression
equation of y on x?
(a) 1st equation (b) 2nd equation
(c) both (a) and (b) (d) none of these

13. Given the following equations: 2x – 3y = 10 and 3x + 4y = 15, which one is the regression
equation of x on y ?
(a) 1st equation (b) 2nd equation
(c) both the equations (d) none of these

264 | P a g e
14. If u = 2x + 5 and v = –3y – 6 and regression coefficient of y on x is 2.4, what is the
regression coefficient of v on u?
(a) 3.6 (b) –3.6
(c) 2.4 (d) –2.4

15. If 4y – 5x = 15 is the regression line of y on x and the coefficient of correlation between x


and y is 0.75, what is the value of the regression coefficient of x on y?
(a) 0.45 (b) 0.9375
(c) 0.6 (d) none of these

16. If the regression line of y on x and that of x on y are given by y = –2x + 3 and 8x = –y + 3
respectively, what is the coefficient of correlation between x and y?
(a) 0.5 (b) –1/ 2
(c) –0.5 (d) none of these

17. If the regression coefficient of y on x, the coefficient of correlation between x and y and

3
variance of y are –3/4, and 4 respectively, what is the variance of x?
2
(a) 2/ 3/2 (b) 16/3
(c) 4/3 (d) 4

18. If y = 3x + 4 is the regression line of y on x and the arithmetic mean of x is –1, what is the
arithmetic mean of y?
(a) 1 (b) –1
(c) 7 (d) none of these

19. Regression analysis is concerned with


(a) Establishing a mathematical relationship between two variables
(b) Measuring the extent of association between two variables
(c) Predicting the value of the dependent variable for a given value of the independent
variable
(d) Both (a) and (c)
20. The line x = a + by represents the regression equation of
(a) y on x (b) x on y
(c) Both of above (d) None of above

21. The method applied for deriving the regression equations is known as
(a) Least squares (b) Concurrent deviation
(c) Product moment (d) Normal equation

22. Two variables x and y are related by 10x + 9y + 8 = 0 and x = 5, then y is


(a) 6.33 (b) –6.33
(c) 6.44 (d) –6.44

23. Regression coefficient is independent of the change of


(a) Scale (b) Origin
(c) Both origin and scale (d) Neither origin nor scale

265 | P a g e
24. The regression equation are 8x – 10y + 66 = 0 and 40 x – 18y = 214, find the coefficient of
correlation
(a) 4/5 (b) –4/5
(c) 3/5 (d) –1

25. r, bxy, byx all have _____________ sign


(a) Different (b) Same
(c) Both (d) None of these

26. Equations of two lines of regression are 4x + 3y + 7 = 0 and 3x + 4y + 8 = 0, the mean of x


and y are
(a) 5/7 and 6/7 (b) – 4/7 and –11/7
(c) 2 and 4 (d) None of these

27. If one of the regression coefficient is greater than unity, then other is less than unity.
(a) True (b) False
(c) Both (d) None of these

28. If 2x + 5y – 9=0 and 3x-y-5=0 are two regression equation, then find the value of mean of x
and mean of y.
(a) 1,2 (b) 2,2
(c) 2,1 (d) None of these

29. For some bivariate data, the following results were obtained for the two variables x and y:
x = 53.2, y = 27.9, byx = -1.5, bxy = -0.2
The most probable value of y when x = 60 is .
(a) 15.6 (b) 13.4
(c) 19.7 (d) 17.7

30. The following data is given, based on 450 students for marks in Statistics and Economics at a
certain examination .
Mean marks in Statistics = 40
Mean marks in Economics = 48
S.D. of marks (Statistics) = 12
Variance of marks (Economics) = 256
Sum of the products of deviations of marks from their respective mean 42075 The marks in
Economics of candidates who obtained 50 marks in Statistics is
(a) 45 (b) 54.5
(c) 54 (d) 47.5

31. The method applied for deriving regression equations is known as:
(a) Concurrent deviation (b) Product moment
(c) Least squares (d) Normal equation

1 1
32. The correlation coefficient between x and y is  . The value of bxy =  Find b yx .
2 8
(a) –2 (b) –4
(c) 0 (d) 2

266 | P a g e
33. The coefficient of correlation between two variables x and y is the simple ____ of the
regression coefficients.
(a) A.M (b) G.M
(c) M.M (d) None

34. If the coefficient of correlation between x and y series is -0.38. The linear relations between x
& u and y & v are 3x+5u=3 and -8y-7v=44 respectively. Then the coefficient of correlation
between u & v is:
(a) 0.38 (b) -0.38
(c) 0.40 (d) None

35. Two random variables have the regression lines 3x + 2y = 26 and x + y = 31. The coefficient
of correlation between x and y is .
(a) -0.25 (b) 0.5
(c) -0.816 (d) 0.25

36. Given the following data:


bxy = 0.4 &byx = 1.6 The coefficient of determination is :
(a) 0.74 (b) 0.42
(c) 0.58 (d) 0.64

37. If the-lines of regression in a bivariate distribution are given by x + 2y = 5 and 2x + 3y = 8,


then the coefficient of correlation is
(a) 0.866 (b) -0.666
(c) 0.667 (d) -0.866

38. If the correlation coefficient between two variables is 1 then the two lines of regressions are -
(a) Parallel (b) at right angles
(c) coincident (d) None of these

39. Given the regression equations as 3x + y=13 and 2x +5y = 20. Find regression equation of y
on x.
(a) 3x+y=13 (b) 2x+y=20
(c) 3x+5y=13 (d) 2x+5y=20

40. The two regression equations are:


2x+3y+18=0
x+2y-25 =0
find the value of y if x = 9
(a) –8 (b) 8
(c) –12 (d) 0

41. Which of the following regression equations represent regression line of Y on X:


7x+2y+ 15 = 0, 2x+5y+10=0
(a) 7x+2y+15=0 (b) 2x+5y+10=0
(c) Both (a) and (b) (d) None of these

42. The two regression lines are 7x – 3y – 18=0 and 4x – y -11 = 0.Find the values of b yx and bxy
(a) 7/3, 1/4 (b) –7/3, –1/4
(c) –3/7, –1/4 (d) None of these

267 | P a g e
43. _________ of the regression Coefficients is greater than the correlation coefficient
(a) Combined mean (b) Harmonic mean
(c) Geometric mean (d) Arithmetic mean

44. If two lines of regression are x + 2y -5 = 0 and 2x + 3y -8 = 0.


The regression line of y on x is
(a) x+2y-5=0 (b) 2x+3y-8=0
(c) Any of the two lines (d) None of the two lines

45. Regression coefficient are .


(a) dependent of change of origin and of scale
(b) Independent of both change of origin and of scale
(c) Dependant of change of origin but not of scale
(d) Independent of change of origin but not of scale

46. Given: x = 16,  x = 4.8


y = 20,  y = 9.6
The coefficient of correlation between x and y is 0.6. What will be the regression coefficient of
'x' on 'y'?
(a) 0.03 (b) 0.3
(c) 0.2 (d) 0.05

47. For a bivariate data, two times of regression are 40x – 18y = 214 and 8x – 10y + 66 = 0,
then find the values of x and y.
(a) 17 and 13 (b) 13 and 17
(c) 13 and -17 (d) -13 and 17

48. Out of the following which one affects the regression coefficient:
(a) Change of origin (b) Change in scale
(c) Change of origin & scale (d) None

49. If one of regression coefficient is ______ unity, the other must be _____ unity.
(a) Less than, greater than (b) Greater than, less than
(c) (a) or (b) (d) None

50. If y is dependent variable and x is independent variable the regression coefficient y on x if r =


0.2,  x  2 and  y  4 :
(a) 0.4 (b) 0.04
(c) 0.8 (d) 0.10

51. If X & y are the A.M’s, σx & σy are the S.D’s, byx ,bxy are regression coefficients of variables x
& y respectively. The point of Intersection of regression lines x on y & y on x is _____
(a) ( X, y) (b) σx ,σy

(c) (byx ,bxy ) (d) (  x2 ,  y2 )

268 | P a g e
52. If regression lines are 8x – 10y + 66 = 0 and 40x – 18y = 214 then correlation coefficient
between x & y is
(a) -1 (b) 0.6
(c) -0.6 (d) 1

53. Two variables X and Y are related as 4x + 3y = 7 then correlation between x and y is _____
(a) Perfect positive (b) Perfect negative
(c) Zero (d) None of these

54. 8x -3y+7 = 0.14x-7y+6 =0 are two regression equations then the correlation coefficient,
r=………….
(a) 0.86 (b) -0.86
(c) 0.45 (d) -0.45

55. If r =+ 1 or –1 then the two regression lines –


(a) Have 300 angle between them (b) Have 450 angle between them
(c) Coincide (d) Perpendicular to each other

56. If mean of X and Y variable sis 20 and 40 respectively and the regression coefficient Y on X is
1.608 then the regression line of Y on X is :
(a) Y = 1.608 X + 7.84 (b) Y = 1.56 X + 4.84
(c) Y = 1.608 X + 4.84 (d) Y = 1.56 X + 7.84

57. Two regression lines are


16x-20y+132=0
80x-36y-428=0
The value of the correlation coefficient is
(a) 0.6 (b) -0.6
(c) 0.54 (d) 0.45

58. When the correlation coefficient r is equal to + 1, all the points in a scatter diagram would be
(a) On a straight line directed from upper left to lower right
(b) On a straight line directed from lower left to upper right
(c) On a straight line
(d) Both (a) and (b)

59. Out of following which is correct?


σx σy
(a) b yx  r (b) b yx  r
σy σx

(c) b yx 
. xy (d) byx 
.  xy
σx σy

60. In case of "Insurance Companies" profits and the number of claims they have to pay there is
____________.
(a) Positive (b) Negative
(c) No correlation (d) None of the above

269 | P a g e
61. The following data relate to the mean and SD of the prices of two shares in a stock
exchange:
Share Mean (in Rs.) SD (in Rs.)
Company A 44 5.60
Company B 58 6.30
Coefficient of correlation between the share prices = 0.48
Find the most likely price of share A corresponding to a price of Rs. 60 of share B and also
the most likely price of share B for a price of Rs. 50 of share A.
(a) 61.24, 44.85
(b) 50, 60
(c) 30.24, 25.20
(d) 11.24, 44.20

62. If the relationship between two variable x and u is u + 3x = 10 and between two other
variables y and v is 2y + 5v = 25, and the regression coefficient of y on x is known as 0.80,
what would be the regression coefficient of v on u?
4
(a)
75
7
(b)
25
8
(c)
75
12
(d)
25

63. The two lines of regression are given by


8x+10y = 25 and 16x + 5y = 12 respectively.
If the variance of x is 25, what is the standard deviation of y ?
(a) 16
(b) 8
(c) 64
(d) 4

ANSWER
1. 2. 3. 4. 5. 6. 7. 8. 9. 10.
(B) (A) (A) (B) (D) (C) (D) (D) (A) (C)
11. 12. 13. 14. 15. 16. 17. 18. 19. 20.
(A) (B) (D) (B) (A) (C) (B) (A) (D) (B)
21. 22. 23. 24. 25. 26. 27. 28. 29. 30.
(A) (D) (B) (C) (B) (B) (A) (D) (D) (B)
31. 32. 33. 34. 35. 36. 37. 38. 39. 40.
(C) (A) (B) (B) (C) (D) (D) (C) (D) (B)
41. 42. 43. 44. 45. 46. 47. 48. 49. 50.
(B) (A) (D) (A) (D) (B) (B) (B) (C) (A)
51. 52. 53. 54. 55. 56. 57. 58. 59. 60.
(A) (B) (B) (A) (C) (A) (A) (B) (B) (B)
61. 62. 63.
(A) (C) (B)

270 | P a g e
CHAPTER – 17 - PROBABILITY AND EXPECTED VALUE BY MATHEMATICAL EXPECTATION

SELF EXAMINATION QUESTION

1. Two broad divisions of probability are


(a) Subjective probability and objective probability
(b) Deductive probability and non–deductive probability
(c) Statistical probability and Mathematical probability
(d) None of these.

2. Subjective probability may be used in


(a) Mathematics (b) Statistics
(c) Management (d) Accountancy

3. An experiment is known to be random if the results of the experiment


(a) Can not be predicted (b) Can be predicted
(c) Can be split into further experiments (d) Can be selected at random

4. An event that can be split into further events is known as


(a) Complex event (b) Mixed event
(c) Simple event (d) Composite event

5. Which of the following pairs of events are mutually exclusive?


(a) A : The student reads in a school. B : He studies Philosophy
(b) A : Raju was born in India. B : He is a fine Engineer
(c) A : Ruma is 16 years old. B : She is a good singer
(d) A : Peter is under 15 years of age. B : Peter is a voter of Kolkata

6. If P(A  B) = 0, then the two events A and B are


(a) Mutually exclusive (b) Exhaustive
(c) Equally likely (d) Independent

7. If two events A and B, P(AUB) = 1, then A and B are


(a) Mutually exclusive events (b) Equally likely events
(c) Exhaustive events (d) Dependent events

8. If an unbiased coin is tossed once, then the two events Head and Tail are
(a) Mutually exclusive (b) Exhaustive
(c) Equally likely (d) All these (a), (b) and (c)

9. The probability of an event can assume any value between


(a) – 1 and 1 (b) 0 and 1
(c) – 1 and 0 (d) none of these

10. If P(A) = 0, then the event A


(a) will never happen (b) will always happen
(c) may happen (d) may not happen

271 | P a g e
11. If P(A) = 1, then the event A is known as
(a) symmetric event (b) dependent event
(c) improbable event (d) sure event

12. If A, B and C are mutually exclusive and exhaustive events, then P(A) + P(B) + P(C) equals to
1
(a) (b) 1
3
(c) 0 (d) any value between 0 and 1

13. Probability of getting a head when two unbiased coins are tossed simultaneously is
(a) 0.25 (b) 0.50
(c) 0.20 (d) 0.75

14. A bag contains 15 one rupee coins, 25 two rupee coins and 10 five rupee coins. If a coin is
selected at random from the bag, then the probability of not selecting a one rupee coin is
(a) 0.30 (b) 0.70
(c) 0.25 (d) 0.20

15. If two letters are taken at random from the word HOME, what is the Probability that none of
the letters would be vowels?
(a) 1/6 (b) 1/2
(c) 1/3 (d) ¼

16. If a card is drawn at random from a pack of 52 cards, what is the chance of getting a Spade
or an ace?
(a) 4/13 (b) 5/13
(c) 0.25 (d) 0.20

17. There are 10 balls numbered from 1 to 10 in a box. If one of them is selected at random,
what is the probability that the number printed on the ball would be an odd number greater
that 4?
(a) 0.50 (b) 0.40
(c) 0.60 (d) 0.30

18. Following are the wages of 8 workers in rupees:


50, 62, 40, 70, 45, 56, 32, 45
If one of the workers is selected at random, what is the probability that his wage would be
lower than the average wage?
(a) 0.625 (b) 0.500
(c) 0.375 (d) 0.450

19. If two unbiased dice are rolled together, what is the probability of getting no difference of
points?
(a) 1/2 (b) 1/3
(c) 1/5 (d) 1/6

20. Two balls are drawn from a bag containing 5 white and 7 black balls at random. What is the
probability that they would be of different colours?
(a) 35/66 (b) 30/66
(c) 12/66 (d) None of these

21. What is the chance of throwing at least sum 7 in a single cast with 2 dice?
272 | P a g e
(a) 5/12 (b) 7/12
(c) 1/4 (d) 17/36

22. A bag contains 12 balls which are numbered from 1 to 12. If a ball is selected at random,
what is the probability that the number of the ball will be a multiple of 5 or 6 ?
(a) 0.30 (b) 0.25
(c) 0.20 (d) 1/3

23. It is given that a family of 2 children has a girl, what is the probability that the other child is
also a girl ?
(a) 0.50 (b) 0.75
(c) 1/3 (d) 2/3

24. Two coins are tossed simultaneously. What is the probability that the second coin would show
a tail given that the first coin has shown a head?
(a) 0.50 (b) 0.25
(c) 0.75 (d) 0.125

25. What is the probability that a leap year selected at random would contain 53 Saturdays?
(a) 1/7 (b) 2/7
(c) 1/12 (d) 1/4

26. If an unbiased coin is tossed three times, what is the probability of getting more than one
head?
(a) 1/8 (b) 3/8
(c) 1/2 (d) 1/3

27. If two unbiased dice are rolled, what is the probability of getting sum points neither 6 nor 9?
(a) 0.25 (b) 0.50
(c) 0.75 (d) 0.80

28. What is the probability that 4 children selected at random would have different birthdays?
364  363  362 6 5 4
(a) (b)
(365) 3 73
(c) 1/365 (d) (1/7)3

29. A box contains 5 white and 7 black balls. Two successive drawn of 3 balls are made (i) with
replacement (ii) without replacement. The probability that the first draw would produce white
balls and the second draw would produce black balls are respectively
(a) 6/321 and 3/926 (b) 1/20 and 1/30
(c) 35/144 and 35/108 (d) 7/968 and 5/264

30. There are three boxes with the following composition:


Box I: 5 Red + 7 White + 6 Blue balls Box II: 4 Red + 8 White + 6 Blue balls
Box III: 3 Red + 4 White + 2 Blue balls
If one ball is drawn at random, then what is the probability that they would be of same
colour?
(a) 89/729 (b) 97/729
(c) 82/729 (d) 23/32

273 | P a g e
31. A number is selected at random from the first 1000 natural numbers. What is the probability
that the number so selected would be a multiple of 7 or 11?
(a) 0.25 (b) 0.32
(c) 0.22 (d) 0.33

32. A bag contains 8 red and 5 white balls. Two successive draws of 3 balls are made without
replacement. The probability that the first draw will produce 3 white balls and the second 3
red balls is
(a) 5/223 (b) 6/257
(c) 7/429 (d) 3/548

33. There are two boxes containing 5 white and 6 blue balls and 3 white and 7 blue balls
respectively. If one of the boxes is selected at random and a ball is drawn from it, then the
probability that the ball is blue is
(a) 115/227 (b) 83/250
(c) 137/220 (d) 127/250

34. Tom speaks truth in 30 percent cases and Dick speaks truth in 25 percent cases. What is the
probability that they would contradict each other?
(a) 0.325 (b) 0.400
(c) 0.925 (d) 0.075

35. In a packet of 500 pens 50 are defective. If a pen is drawn at random what is the chance that
it is non-defective.
8 7
(a) (b)
9 8
9 2
(c) (d)
10 3
ANSWER
1. 2. 3. 4. 5. 6. 7. 8. 9. 10.
(A) (C) (D) (D) (D) (A) (C) (D) (D) (A)
11. 12. 13. 14. 15. 16. 17. 18. 19. 20.
(D) (B) (B) (B) (A) (A) (D) (B) (D) (A)
21. 22. 23. 24. 25. 26. 27. 28. 29. 30.
(B) (D) (C) (A) (B) (C) (C) (A) (D) (A)
31. 32. 33. 34. 35.
(C) (C) (C) (B) (C)

PRACTICE QUESTIONS

1. If for two events A and B, P(A  B)  P(A)  P(B), then the two events A and B are
(a) Independent (b) Dependent
(c) Not equally likely (d) Not exhaustive.

2. If p : q are the odds in favour of an event, then the probability of that event is
P
(a) p/q (b)
Pq
q
(c) (d) none of these.
pq

274 | P a g e
3. If P(A) = 5/9, then the odds against the event A is
(a) 5:9 (b) 5:4
(c) 4:5 (d) 5 : 14

4. For two events A and B, P(A  B) = P(A) + P(B) only when


(a) A and B are equally likely events (b) A and B are exhaustive events
(c) A and B are mutually independent (d) A and B are mutually exclusive.

5. Addition Theorem of Probability states that for any two events A and B,
(a) P(A  B) = P(A) + P(B) (b) P(A  B) = P(A) + P(B) + P(A  B)
(c) P(A  B) = P(A) + P(B) – P(A  B) (d) P(A  B) = P(A) _ P(B)

6. For any two events A and B,


(a) P(A–B) = P(A) – P(B) (b) P(A–B) = P(A) – P(A  B)
(c) P(A–B) = P(A) – P(A  B) (d) P(B–A) = P(B) + P(A  B)

7. If A and B are mutually exclusive events, then


(a) P(A) = P(A–B). (b) P(B) = P(A–B).
(c) P(A) = P(A  B). (d) P(B) = P(A  B).

8. If P(A–B) = P(B–A), then the two events A and B satisfy the condition
(a) P(A) = P(B). (b) P(A) + P(B) = 1
(c) P(A  B) = 0 (d) P(A  B) = 1

9. If an unbiased coin is tossed twice, the probability of obtaining at least one tail is
(a) 0.25 (b) 0.50
(c) 0.75 (d) 1.00

10. If an unbiased die is rolled once, the odds in favour of getting a point which is a multiple of 3
is
(a) 1:2 (b) 2:1
(c) 1:3 (d) 3:1

11. A, B, C are three mutually independent with probabilities 0.3, 0.2 and 0.4 respectively. What
is P (A  B  C)?
(a) 0.400 (b) 0.240
(c) 0.024 (d) 0.500

12. What is the probability of having at least one ‘six’ from 3 throws of a perfect die?
(a) 5/6 (b) (5/6) 3
3
 5
(c) 1– (1/6) 3 (d) 1–  
 6

13. What is the chance of getting at least one defective item if 3 items are drawn randomly from
a lot containing 6 items of which 2 are defective item?
(a) 0.30 (b) 0.20
(c) 0.80 (d) 0.50

275 | P a g e
14. If A, B and C are mutually exclusive independent and exhaustive events then what is the
probability that they occur simultaneously?
(a) 1 (b) 0.50
(c) 0 (d) any value between 0 and 1

15. A, B and C are three mutually exclusive and exhaustive events such that P (A) = 2 P (B) =
3P(C). What is P (B)?
(a) 6/11 (b) 3/11
(c) 1/6 (d) 1/3

16. For two events A and B, P (B) = 0.3, P (A but not B) = 0.4 and P (not A) = 0.6. The events A
and B are
(a) exhaustive (b) independent
(c) equally likely (d) mutually exclusive

17. Given that for two events A and B, P (A) = 3/5, P (B) = 2/3 and P ( A  B ) = 3/4, what is P
(A/B)?
(a) 0.655 (b) 13/60
(c) 31/60 (d) 0.775

18. For two independent events A and B, what is P (A  B), given P(A) = 3/5 and P(B) = 2/3?
(a) 11/15 (b) 13/15
(c) 7/15 (d) 0.65

19. If P ( A  B ) = 5/6, P(A) = ½ and P ( B ) = 2/3, what is P (A  B) ?


(a) 1/3 (b) 5/6
(c) 2/3 (d) 4/9

20. If for two independent events A and B, P (A  B) = 2/3 and P (A) = 2/5, what is P (B)?
(a) 4/15 (b) 4/9
(c) 5/9 (d) 7/15

21. If P (A) = 2/3, P (B) =3/4, P (A/B) = 2/3, then what is P (B / A)?
(a) 1/3 (b) 2/3
(c) 3/4 (d) ½

22. If P (A) = a, P (B) = b and P (P (A  B) = c then the expression of P (A’  B’) in terms of a, b
and c is
(a) 1–a–b–c (b) a+b–c
(c) 1+a–b–c (d) 1–a–b+c

23. For three events A, B and C, the probability that only A occur is
(a) P (A) (b) P (A  B  C)
(c) P (A’  B  C) (d) P (A  B’  C’)

24. A problem in probability was given to three CA students A, B and C whose chances of solving
it are 1/3, 1/5 and 1/2 respectively. What is the probability that the problem would be solved?
(a) 4/15 (b) 7/8
(c) 8/15 (d) 11/15

276 | P a g e
25. There are three persons aged 60, 65 and 70 years old. The survival probabilities for these
three persons for another 5 years are 0.7, 0.4 and 0.2 respectively. What is the probability
that at least two of them would survive another five years?
(a) 0.425 (b) 0.456
(c) 0.392 (d) 0.388

26. There are two urns. The first urn contains 3 red and 5 white balls whereas the second urn
contains 4 red and 6 white balls. A ball is taken at random from the first urn and is
transferred to the second urn. Now another ball is selected at random from the second arm.
The probability that the second ball would be red is
(a) 7/20 (b) 35/88
(c) 17/52 (d) 3/20

27. The probability that a card drawn at random from the pack of playing cards may be either a
queen or an ace is
(a) 2/13 (b) 11/13
(c) 9/13 (d) None of these.

28. The chance of getting a sum of 6 in a single throw with two dice is
(a) 3/36 (b) 4/36
(c) 6/36 (d) 5/36

29. In a single throw with two dice the probability of getting a sum of five on the two dice is
(a) 1/9 (b) 5/36
(c) 5/9 (d) None of these

30. A bag contains 30 balls numbered from 1 to 30. One ball is drawn at random. The probability
that the number of the drawn balls will be multiple of 5 or 7 is
(a) ½ (b) 1/3
(c) ¼ (d) None of these.

31. A card is drawn from a pack of playing cards and then another card is drawn without the first
being replaced. What is the probability of getting two kings?
(a) 7/52 (b) 1/221
(c) 3/221 (d) None of these

32. Two cards are drawn from a well shuffled pack of 52 cards. Find the probability that they are
both kings if the first is replaced.
(a) 1/13 (b) 1/ 169
(c) 1/221 (d) None of these

33. A pair of dice is thrown and sum of the numbers on the two dice comes to be 7. What is the
probability that the number 3 has come on one of the dice?
(a) 1/9 (b) 1/3
(c) ¼ (d) None of these

34. A bag contains 5 red and 3 yellow balls. Two balls are drawn at random one after the other
without replacement. The probability that both balls drawn are yellow is
(a) 9/64 (b) 3/28
(c) 1/7 (d) None of these

277 | P a g e
35. A card is drawn from a pack of 52 cards. What is the probability that it is neither a black card
nor a king?
(a) 6/13 (b) 5/13
(c) 1/6 (d) None of these.

36. A coin is tossed two times. The toss resulted in one head and one tail. What is the probability
that the first throw resulted in tail?
(a) 1/3 (b) 1/4
(c) ½ (d) None of these

37. Two cards are drawn from a well shuffled pack of playing cards. Find the probability that both
are ace.
(a) 1:221 (b) 2:221
(c) 10:21 (d) None of these

38. A bag contains 5 white and 10 blackballs. Three balls are taken out at random. Find the
probability that all three balls drawn are black
(a) 16/91 (b) 42/91
(c) 24/91 (d) None of these

39. A card is drawn from a well shuffled pack of playing Cards. Find the probability that it is a
king or a Queen:
(a) 1/13 (b) 1/4
(c) 2/13 (d) 2/4

40. A pair of dice is rolled. If the sum on the dice is 9. Find the probability that one of dice
showed 3.
(a) 1/9 (b) 1/4
(c) ½ (d) 1

41. Probability of throwing an even number with an ordinary six faced dice is
(a) 1 (b) –1/2
(c) ½ (d) 0

42. If a pair of dice is thrown, the probability that the sum is neither 7 nor 11 is ________
(a) 7/9 (b) 5/9
(c) 11/9 (d) 2

43. A card is drawn at random from a well shuffled pack of 52 cards. Find the probability of
getting a two of heart or ace of diamond.
(a) 1/26 (b) 2/51
(c) 2/26 (d) 1/52

44. A bag contains 20 discs numbered 1 to 20. A disc is drawn from the bag. The probability that
the number on it is a multiple of 3 is _______
(a) 5/10 (b) 2/5
(c) 1/5 (d) 3/10

278 | P a g e
45. Out of numbers 1 to 120, one is related at random, what is the probability that it is divisible
by 8 or 10.
(a) 24/120 (b) 18/125
(c) 32/120 (d) None of these

46. Three horses A, B and C are in a race, A is twice as likely to win as B and B is twice as likely
to win as C. What is the possibility of C winning the race?
(a) 1/7 (b) 3/7
(c) 2/5 (d) 2/7

47. A number is selected from the set S = {1, 2, 3, 4 ….. 25}. The probability that it would be
divisible by 4 or 7 is
(a) 0.26 (b) 0.46
(c) 0.36 (d) None of these

48. Suresh is selected for three different posts. For the first post there are 2 candidates, for the
second post there are 3 candidates, for the third post there are 10 candidates. The
probability, that Suresh would be selected, is
(a) 0.7 (b) 0.5
(c) 0.6 (d) None of these

49. From a pack of cards, two are drawn, the first being replaced before the second is drawn. The
chance that the first is a diamond and the second is king is:
1 3
(a) (b)
52 2704
4 3
(c) (d)
13 52
1 3
50. The probability of getting qualified in IIT- JEE and AIEEE by a student are respectively and
5 5
The probability that the student gets qualified for at least one test is :
17 22
(a) (b)
25 25
8 3
(c) (d)
25 25

51. A, B, C are three mutually independent with probabilities 0.3, 02 and 0.4 respectively. What is
P (A ∩ B ∩ C) ?
(a) 0.400 (b) 0.240
(c) 0.024 (d) 0.500

52. A bag contains 8 red and 5 white balls. Two successive draws of 3 balls are made without
replacement. The probability that the first draw will produce 3 white balls and second 3 red
balls is:
6 5
(a) (b)
255 548
7 3
(c) (d)
429 233
53. Three identical dice are rolled. The probability that the same number will appear on each of
them is:
279 | P a g e
(a) 1/6 (b) 1/12
(c) 1/36 (d) 1

54. An article consists of two parts A and B. The manufacturing process of each part is such that
probability of defect in A is 0.08 and that B is 0.05. What is the probability that the assembled
product will not have any defect?
(a) 0.934 (b) 0.864
(c) 0.85 (d) 0.874

55. The odds are 9:5 against a person who is 50 years living till he is 70 and 8 : 6 against a
person who is 60 living till he is 80. Find the probability that at least one of them will be alive
after 20 years.
11 22
(a) (b)
14 49
31 35
(c) (d)
49 49

56. Mr. Roy is selected for three separate posts. For the first post, there are three candidates, for
the second, there are five candidates and for the thirs, there are 10 candidates. What is the
probability that Mr. Roy would be selected?
11 13
(a) (b)
25 25
20 4
(c) (d)
25 9
57. If A and B are two independent events and P(AUB)= 2/5; P(B) = 1/3. Find P(A).
(a) 2/9 (b) - 1/3
(c) 2/10 (d) 1/10

58. A random variable X has the following probability distribution.


X 0 1 2 3
P(x) 0 2K 3K K
Then, P (x<3) would be:
(a) 1/6 (b) 1/3
(c) 2/3 (d) 5/6

59. If P (A  B) =P (A) x P(B), then the events are:


(a) Independent events (b) Mutually exclusive events
(c) Exhaustive events (d) Mutually inclusive events

60. In a pack of playing cards with two jokers probability of getting king of spade is.
(a) 4/13 (b) 4/52
(c) 1/52 (d) 1/54

61. A bag contains 3 white and 5 black balls and second bag contains 4 white and 2 black balls. If
one ball is taken from each bag, the probability that both the balls are white is .
(a) ¼ (b) 1/3
(c) ½ (d) None of these
62. The odds in favour of A solving a problem is 5:7 and Odds against B solving the same problem
is 9:6. What is the probability that if both of them try, the problem will be solved?

280 | P a g e
(a) 117/180 (b) 181/200
(c) 147/180 (d) 119/180

63. Consider Urn I : 2 white balls, 3 black balls Urn II: 4 white balls, 6 black balls One ball is
randomly transferred from first to second Urn, then one ball is drawn from II Um. The
probability that drawn ball is white is
(a) 22/65 (b) 22/46
(c) 22/55 (d) 21/45

64. A bag contains 5 Red balls, 4 Blue Balls and ‘m’ Green Balls. If the random probability of
picking two green balls is 1/7. What is the no. of green Bails (m).
(a) 5 (b) 7
(c) 6 (d) None of the above.

65. The probability of Girl getting scholarship is 0.6 and the same probability for Boy is 0.8. Find
the probability that at least one of the categories getting scholarship.
(a) 0.32 (b) 0.44
(c) 0.92 (d) None of the above.

66. Rupesh is known to hit a target in 5 out of 9 shots whereas David is known to hit the same
target in 6 out of 11 shots. What is the probability that the target would be hit once they both
try?
79 11
(a) (b)
99 49
12 13
(c) (d)
49 49

67. The odds in favour of an event is 2 : 3 and the odds against another event is 3 : 7 Find the
probability that only one of the two events occurs.
20 11
(a) (b)
50 50
22 27
(c) (d)
50 50

68. One card is drawn from a pack of 52 cards. What is the probability it is a king or queen.
11 2
(a) (b)
13 13
1
(c) (d) None
13

69. Arun & Tarun appear for an interview for 2 vacancies. The Probability of Arun’s selection is
1/3 and that of Tarun’s selection is 1/5. Find the probability that only one of them will be
selected.
(a) 2/5 (b) 4/5
(c) 6/5 (d) 8/15

70. A card is drawn out of standard pack of 52 cards. What is the probability that it is a king or
red colour?

281 | P a g e
(a) 1/4 (b) 4/13
(c) 7/13 (d) ½

71. The odds against A solving a problem are 4 to 3. And the odds in favour of B solving the same
problem are 7 to 5. What is the probability that the problem will be solved if they both try?
15 16
(a) (b)
21 21
17 13
(c) (d)
21 21
72. A bag contains 6 red balls and some blue balls. If the probability of drawing a blue ball from
the bag is twice that of drawing red ball. Find the no of blue balls in the bag
(a) 10 (b) 12
(c) 14 (d) 16

73. If two dice are thrown together then the probability of getting multiple of 3 on one die and
multiple of 2 on other die is
(a) 2/3 (b) 1/6
(c) 1/3 (d) none

74. The odds that a book will be reviewed favorably by 3 independent critics are 5 to 2, 3 to 4, 4
to 3 respectively, then the probability that out of 3 critics the majority will be favourable is
______
209 209
(a) (b)
343 434
209 209
(c) (d)
443 350

75. Find the probability of drawings spade on each of 2 consecutive draws form a well shuffled
pack of cards when the draws are with out replacement.
2 3
(a) (b)
51 51
4 5
(c) (d)
51 51

76. A bag contains 2 red, 3 green and 2 blue balls. if 2 balls are drawn at random from the bag
find the probability that none of them will be blue.
11 5
(a) (b)
21 7
10 2
(c) (d)
21 7

77. A player tosses 3 coins. He wins Rs. 5 if 3 heads appear, Rs. 3 if two heads appear, Rs. 1 if
one head appear and a loss of Rs. 15 if no head appear. Find his expected gain in Rs.
__________
(a) 0.5 (b) 0.25
(c) 0.2 (d) None

78. If P (A) = 0.45, P (B) = 0.35, P (A and B) = 0.25 then P(A /B) =

282 | P a g e
(a) 1.4 (b) 1.8
(c) 0.714 (d) 0.556

79. Two coins are tossed simultaneously then the probability of getting exactly one head is
3 2
(a) (b)
4 3
1 1
(c) (d)
4 2

80. A committee of 7 members is to be formed from a group comprising 8 gentleman and 5


ladies. What is the probability that the committee would comprise 2 Ladies.
140 120
(a) (b)
429 329
100
(c) (d) None of these
429

81. Three events A, B and C are mutually exclusive, exhaustive and equally likely.
What is the probability of the complementary event of A?
(a) 1/3 (b) 2/3
(c) 4/3 (d) 1

82. A number is selected from the first 25 natural numbers. What is the probability that it would
be divisible by 4 or 7 ?
11 12
(a) (b)
25 25
9 3
(c) (d)
25 25

83. The probability that an Accountant’s job applicant has a B.Com. Degree is 0.85, that he is a
CA is 0.30 and that he is both B. Com. And CA is 0.258 out of 500 applicants how many
would be B.Com. or CA?
(a) 0.90 (b) 0
(c) 1 (d) 0.40

84. If P(A-B) = 1/5, P(A) = 1/3 and P (B) = ½, What is the probability that out of the two
events A and B, only B would occur?
10 11
(a) (b)
30 30
16 12
(c) (d)
30 30

283 | P a g e
85. There are three persons A, B and C having different ages. The probability that A survives
another 5 years is 0.80, B survives another 5 years is 0.60 and C survives another 5 years
is 0.50. the probabilities that A and B survive another 5 years is 0.46, B and C survive
another 5 years is 0.32 and A and C survive another 5 years 0.48. The probability that all
these three persons survive another 5 years is 0.26. find the probability that at least one of
them survives another 5 years.
(a) 0.90 (b) 0.80
(c) 1.40 (d) 1.20

86. The sum of numbers obtained in throw of a dice twice is S. Probability of S will be maximum
if S is
(a) 5 (b) 7
(c) 6 (d) 8

87. From 6 positive and 8 negative numbers, 4 numbers are choosen at random without
replacement and are then multiplied. The probability that the product of the choosen
numbers will be positive number is
409 70
(a) (b)
1001 1001
505 420
(c) (d)
1001 1001

88. If an unbiased die is rolled once, the odds in favour of getting a point which is multiple of 3
is:
(a) 1:2 (b) 2:1
(c) 1:3 (d) 3:1

89. A bag contains 15 one rupee coins, 25 two rupees coins and 10 five rupees coins, if a coin is
selected at random than probability for not selecting a one rupee coin is:
(a) 0.30 (b) 0.20
(c) 0.25 (d) 0.70

90. Three coins are tossed together, the probability of getting exactly two head is:
5 3
(a) (b)
8 8
1
(c) (d) None
8

91. If two letters are taken at random from the word "Home", what is the probability that none
of the letters would be vowels?
1 1
(a) (b)
6 2
1 1
(c) (d)
3 4

284 | P a g e
ANSWER

1. 2. 3. 4. 5. 6. 7. 8. 9. 10.
(B) (B) (C) (D) (C) (C) (A) (A) (C) (C)
11. 12. 13. 14. 15. 16. 17. 18. 19. 20.
(C) (D) (C) (C) (B) (D) (D) (B) (C) (B)
21. 22. 23. 24. 25. 26. 27. 28. 29. 30.
(C) (D) (D) (D) (D) (B) (A) (D) (A) (B)
31. 32. 33. 34. 35. 36. 37. 38. 39. 40.
(B) (B) (B) (B) (A) (C) (A) (C) (C) (C)
41. 42. 43. 44. 45. 46. 47. 48. 49. 50.
(C) (A) (A) (D) (A) (A) (C) (A) (A) (A)
51. 52. 53. 54. 55. 56. 57. 58. 59. 60.
(C) (C) (A) (D) (C) (B) (D) (D) (A) (D)
61. 62. 63. 64. 65. 66. 67. 68. 69. 70.
(A) (A) (C) (C) (C) (A) (D) (B) (A) (C)
71. 72. 73. 74. 75. 76. 77. 78. 79. 80.
(B) (B) (D) (A) (B) (C) (B) (C) (D) (A)
81. 82. 83. 84. 85. 86. 87. 88. 89. 90.
(B) (C) (A) (B) (A) (B) (C) (A) (D) (B)
91.
(A)

285 | P a g e
CHAPTER – 17 – EXPECTED VALUE
PRACTICE QUESTIONS

1. Values of a random variable are


(a) always positive numbers (b) always positive real numbers
(c) real numbers (d) natural numbers

2. Expected value of a random variable


(a) is always positive (b) may be positive or negative
(c) may be positive or negative or zero (d) can never be zero

3. If x and y are independent, then


(a) E(xy) = E(x)  E(y) (b) E(xy) = E(x) + E(y)
(c) E(x – y) = E(x) + E(y) (d) E(x – y) = E(x) + x E(y)

4. If a random variable x assumes the values x 1 , x2 , x3 , x4 with corresponding probabilities p1 ,


p2 , p3 , p4 then the expected value of x is
(a) p1 + p2 + p3 + p4 (b) x1 p1 + x2 p3 + x3p2 + x4 p4
(c) p1x1 + p2x2 + p3x3 + p4x4 (d) none of these

5. Variance of a random variable x is given by


(a) E (x–  )2 (b) E [x – E(x)]2
(c) 2
E (x – ) (d) (a) or (b)

6. If two random variables x and y are related by y = 2 – 3x, then the SD of y is given by
(a) –3  SD of x (b) 3  SD of x
(c) 9  SD of x (d) 2  SD of x

7. If x and y are random variables having expected values as 4.5 and 2.5 respectively, then the
expected value of (x–y) is
(a) 2 (b) 7
(c) 6 (d) 0

8. If variance of a random variable x is 23, then what is the variance of 2x+10?


(a) 56 (b) 33
(c) 46 (d) 92

9. If a random variable x assumes the values 0, 1 and 2 with probabilities 0.30, 0.50 and 0.20,
then its expected value is
(a) 1.50 (b) 3
(c) 0.90 (d) 1

10. If two random variables x and y are related as y = –3x + 4 and standard deviation of x is 2,
then the standard deviation of y is
(a) –6 (b) 6
(c) 18 (d) 3.50

286 | P a g e
11. If 2x + 3y + 4 = 0 and v(x) = 6 then v (y) is
(a) 8/3 (b) 9
(c) –9 (d) 6

12. The probability distribution of a random variable x is given below:


X: 1 2 4 5 6
P: 0.15 0.25 0.20 0.30 0.10
What is the standard deviation of x?
(a) 1.49 (b) 1.56
(c) 1.69 (d) 1.72

13. The probability that there is at least one error in an account statement prepared by 3 persons
A, B and C are 0.2, 0.3 and 0.1 respectively. If A, B and C prepare 60, 70 and 90 such
statements, then the expected number of correct statements
(a) 170 (b) 176
(c) 178 (d) 180

14. A bag contains 6 white and 4 red balls. If a person draws 2 balls and receives Rs.10 and
Rs.20 for a white and red balls respectively, then his expected amount is
(a) Rs. 25 (b) Rs. 26
(c) Rs. 29 (d) Rs. 28

15. The probability distribution of a random variable is as follows:


X: 1 2 4 6 8
P: K 2k 3k 3k K
The variance of x is
(a) 2.1 (b) 4.41
(c) 2.32 (d) 2.47

16. Moidul draws 2 balls. From a bag containing 3 white and 5 red balls. He gets Rs. 500 if he
draws a white ball and Rs. 200 if he draws a red ball. What is his expectation ? if he is asked
to pay Rs. 400 for participating in the game, would he consider it a fair game and participate?
(a) Yes
(b) No
(c) Don’t Say
(d) None

17. The probability that there is at least one error in an account statement prepared by A is 0.3
and for B and C, they are 0.4 and 0.45 respectively. A, B and C prepared 20, 10 and 40
statements respectively. The expected number of correct statements in all is
(a) 32 (b) 45
(c) 42 (d) 25

18. Daily demand for calculators is having the following probability distribution:
Demand: 1 2 3 4 5 6
Probability: 0.10 0.15 0.20 0.25 0.18 0.12
Determine the variance of the demand.
(a) 2.54 (b) 2.93
(c) 2.19 (d) 3.62

287 | P a g e
19. If:
x : -2 3 1
P(x): 1/3 1/2 1/6
Then find E(2x+5).
(a) 7 (b) 6
(c) 9 (d) 4

20. E (XY) is also known as:


(a) E(X)+E(Y) (b) E (X) E (Y)
(c) E(X)—E(Y) (d) B (X) ÷ E (Y)

21. E(13x+9)=
(a) 13x (b) 13E(x)
(c) 13E(x) + 9 (d) 9

22. A dice is thrown once. What is the mathematical expectation of the number on the dice?
(a) 16/6 (b) 13/2
(c) 3.5 (d) 4.5

23. Two unbiased dies are thrown then the excepted value of sum on the sides of the die.
(a) 3.5 (b) 7
(c) 6 (d) 12

24. If a coin is tossed twice we g et, Rs 5 i f two heads appear, Rs 2 if one head appear, Rs 1 if no
head appear. Then the expected income is
(a) 3.5 (b) 2.5
(c) 4 .5 (d) 5.5

25. Find the expected value of the following probability distribution X: -20 -10 30 75 80 P(x=x):
X: -20 -10 30 75 80
P(X=X): 3 1 1 1 1
20 5 2 10 20
(a) 20.5 (b) 22.5
(c) 21.5 (d) 24.5

26. The probability that a cricket team winning a match at Kanpur is 2/5 and loosing a match at
Delhi is 1/7. What is the probability of the team winning at least one match?
3 32
(a) (b)
35 35
18 17
(c) (d)
35 35

27. Find the expected value of the following probability distribution


X -20 -10 30 75 80
P(x) 3/20 1/5 ½ 1/10 1/20
(a) 20.5 (b) 21.5
(c) 22.5 (d) 24.5

288 | P a g e
28. In a business venture, a man can make a profit of Rs. 50,000 or incurra loss of Rs. 20,000.
The probabilities of making profit or incurring loss, from the past experience, are known to
be 0.75 and 0.25 respectively. What is his expected profit ?
(a) 11500 (b) 12700
(c) 32500 (d) 32000

29. A random variable x takes three values -1,2,3 with the respective probabilities P(-1)=1/3,
P(2)=1/3, P(3)=1/3, then E X is
(a) 3/2 (b) -5/2
(c) 2 (d) 9/2

30. An unbiased coin is tossed three times. The expected value of the number of heads is
(a) 2.5 (b) 1.0
(c) 1.5 (d) 2.0

2 3 1
31. For two events A, and A2, let P(A1) = and P(A2)= and P(A1  A2 )  , then
3 8 4
(a) mutually exclusive but not independent
(b) mutually exclusive and independent
(c) independent but not mutually
(d) None of these

ANSWER

1. 2. 3. 4. 5. 6. 7. 8. 9. 10.
(C) (C) (A) (C) (D) (B) (A) (D) (C) (B)
11. 12. 13. 14. 15. 16. 17. 18. 19. 20.
(A) (C) (C) (D) (B) (A) (C) (C) (A) (B)
21. 22. 23. 24. 25. 26. 27. 28. 29. 30.
(C) (C) (B) (B) (C) (B) (B) (C) (C) (C)
31.
(C)

289 | P a g e
CHAPTER – 18 - THEORETICAL DISTRIBUTIONS

PRACTICE QUESTIONS (Binomial Distribution)

1. Probability distribution may be


(a) discrete (b) continuous
(c) infinite (d) both (a) and (b)

2. The important characteristic(s) of Bernoulli trials


(a) each trial is associated with just two possible outcomes
(b) trials are independent
(c) trials are infinite
(d) both (a) and (b)

3. The probability mass function of binomial distribution is given by


(a) f(x) = px q n-x (b) f(x) = ncx px q n-x
n x n-x.
(c) f(x) = cxq p (d) f(x) = ncx p n-x q x

4. The mean of a binomial distribution with parameter


n and p is
(a) n (1— p) (b) np (l-p)
(c) np (d) np (1-p)

5. The variance of a binomial distribution with parameters n and p is


(a) np2 (1 — p) (b) np (1 - p).
(c) nq(1—q) (d) n p (1— p)2
2 2

6. The mean of binomial distribution is


(a) always more than its variance (b) always equal to its variance
(c) always less than its variance (d) always equal to its standard deviation

7. For a binomial distribution there may be


(a) one mode (b) two mode
(c) (a) (d) (a) or (b)

8. The maximum value of the variance of a binomial distribution with parameters n and p is
(a) n/2 (b) n/4
(c) np (1 — p) (d) 2n

9. The method usually applied for fitting a binomial distribution is known as


(a) method of least square (b) method of moments
(c) method of probability distribution (d) method of deviations

10. What is the standard deviation of the number of recoveries among 48 patients when the
probability of recovering is 0.75?
(a) 36 (b) 81
(c) 9 (d) 3

290 | P a g e
11. X is a binomial variable with n = 20. What is the mean of X if it is known that x is symmetric?
(a) 5 (b) 10
(c) 2 (d) 8

12. If X ~B (n, p), what would be the largest value of the variance of x when n = 16?
(a) 2 (b) 4
(c) 8 (d) 5
13. If x is a binomial variate with parameter 15 and 1/3, what is the value of mode of the
distribution
(a) 5 and 6 (b) 5
(c) 5.50 (d) 6

14. What is the no. of trials of a binomial distribution having mean and SD as 3 and 1.5
respectively?
(a) 2 (b) 4
(c) 8 (d) 12

15. What is the probability of getting 3 heads if 6 unbiased coins are tossed simultaneously?
(a) 0.50 (b) 0.25
(c) 0.3125 (d) 0.6875

16. If the overall percentage of success in an exam is 60, what is the probability that out of a
group of 4 students, at least one has passed?
(a) 0.6525 (b) 0.9744
(c) 0.8704 (d) 0.0256

17. What is the probability of making 3 correct guesses in 5 True — False answer type questions?
(a) 0.3125 (b) 0.5676
(c) 0.6875 (d) 0.4325

18. In Binomial distribution if n = 4 and p = 1/3, then the value of variance is


(a) 8/3 (b) 8/9
(c) 4/3 (d) None of these

19. Variance of a binomial distribution is always _________ its mean.


(a) Equal to (b) More than
(c) Less than (d) None of these

20. If in binomial distribution mean is 10 and S.D. is 2, q will be _________.


(a) 0 (b) 0.2
(c) 0.8 (d) 0.4

21. If in binomial distribution np = 6 and npq = 3, then p is equal to


(a) 1/3 (b) 1/2
(c) ¼ (d) None of these

22. If in a binomial distribution mean = 20, Standard Deviation = 4 then n is equal to


(a) 80 (b) 100
(c) 90 (d) None of these

291 | P a g e
23. In binomial distribution ‘p’ denotes probability of _______ .
(a) Success. (b) Failure.
(c) Both of these (d) None of these

24. An experiment succeeds twice as often as it fails. What is the probability that in next five
trials there will be three success.
(a) 192/243 (b) 19/243
(c) 80/243 (d) 50/243

25. If 15 days are selected at random, then the probability of getting two Fridays are
_________
(a) 0.13 (b) 0.19
(c) 0.29 (d) 0.39

26. Find the variance of binomial distribution with n = 10, p = 0.3


(a) 2.1 (b) 3
(c) 7 (d) None of these

27. The mean and variance of Binomial distribution is 12. This is


(a) True (b) False
(c) Either (a) or (b) (d) None of these

28. The overall percentage of failure in a certain examination is 0.30. What is the probability that
out of a group of 6 candidates at least 4 passed the examination?
(a) 0.74 (b) 0.71
(c) 0.59 (d) 0.67

29. Examine the validity of the following:


Mean and standard Deviation of a binomial distribution are 10 and 4 respectively
(a) Not valid (b) Valid
(c) Both (a) & (b) (d) Neither (a) nor (b)

30. An experiment succeeds twice as often as it fails. What is the probability that in next five trials
there will be at least three success?
33 46
(a) (b)
81 81
64 25
(c) (d)
81 81

31. For binomial distribution E(x) =2, V (x) = 4/3. Find the value of n.
(a) 3 (b) 4
(c) 5 (d) 6

32. What are the parameters of binomial distribution?


(a) n (b) p
(c) Both n and p (d) None of these

33. For a binomial distribution B (6,p), p(x=2) = 9p(x=4), then P is.


(a) 1/2 (b) 1/3
(c) 10/13 (d) ¼

292 | P a g e
34. In Binomial distribution n=9 and p=1/3, what is the value of variance:
(a) 8 (b) 4
(c) 2 (d) 16

35. If for a Binomial distribution B (n, p,) the means= 6 and Variance = 2 then 'p' is.
(a) 2/3 (b) 1/3
(c) 3/5 (d) ¼

36. In a Binomial Distribution, If mean is K- times the variance, then the value of ‘k’ will be.
1
(a) p (b)
p
1
(c) 1 p (d)
1p

37. The binomial distribution with mean 3 and variance 2 is.


9 9
2 1 2 1
(a)    (b)   
3 3 6 6
6 9
2 1 2 1
(c)    (d)   
3 3 5 5

38. For Binomial Distribution


(a) Variance < Mean (b) Variance > Mean
(c) Variance = Mean (d) None of these

39. If the probability of a defective bolt is 0.1, find the standard deviation for the distribution of
defective bolt in a total of 500.
(a) 7.6 (b) 7.5
(c) 6.5 (d) 6.7

40. 3% of a given lot of manufactured parts are defective, what is the probability that in a sample
of 4 items none will be defective.
(a) 0.585 (b) 0.885
(c) 0.558 (d) None of these

41. If 8 balls are distributed at random among three boxes, what is the probability that the first
box would contain 3 balls?
1792 1192
(a) (b)
6561 6561
1092 592
(c) (d)
6561 6561

293 | P a g e
ANSWER

1. 2. 3. 4. 5. 6. 7. 8. 9. 10.
(D) (D) (B) (C) (C) (A) (D) (B) (B) (D)
11. 12. 13. 14. 15. 16. 17. 18. 19. 20.
(B) (B) (B) (D) (C) (B) (A) (B) (B) (D)
21. 22. 23. 24. 25. 26. 27. 28. 29. 30.
(B) (B) (A) (C) (C) (A) (B) (A) (A) (C)
31. 32. 33. 34. 35. 36. 37. 38. 39. 40.
(D) (C) (D) (C) (A) (D) (A) (A) (D) (B)
41.
A

294 | P a g e
CHAPTER – 18 - POISSON DISTRIBUTION
PRACTICE QUESTIONS

1. Which one is uniparametric distribution?


(a) Binomial (b) Poisson
(c) Normal (d) Hyper geometric

2. For a Poisson distribution,


(a) mean and standard deviation are equal
(b) mean and variance are equal
(c) standard deviation and variance are equal
(d) both (a) and (b)

3. Poisson distribution may be


(a) unimodal (b) bimodal
(c) Multi-modal (d) (a) or (b)

4. Poisson distribution is
(a) always symmetric (b) always positively skewed
(c) always negatively skewed (d) symmetric only when m = 2

5. Number of misprints per page of a thick book follows


(a) Normal distribution (b) Poisson distribution
(c) Binomial distribution (d) Standard normal distribution

6. If the standard deviation of a Poisson variate X is 2, what is P (1.5 < X <2.9)?


(a) 0.231 (b) 0.158
(c) 0.15 (d) 0.144

7. If X~P (m) and its coefficient of variation is 50, what is the probability that X would assume
only non-zero values?
(a) 0.018 (b) 0.982
(c) 0.989 (d) 0.976

8. If 1.5 per cent of items produced by a manufacturing units are known to be defective, what is
the probability that a sample of 200 items would contain no defective item?
(a) 0.05 (b) 0.15
(c) 0.20 (d) 0.22

9. For a Poisson variate X, P (X = 1) = P (X = 2). What is the mean of X?


(a) 1.00 (b) 1.50
(c) 2.00 (d) 2.50

10. If 1 per cent of an airline’s flights suffer a minor equipment failure in an aircraft, what is the
probability that there will be exactly two such failures in the next 100 such flights?
(a) 0.50 (b) 0.184
(c) 0.265 (d) 0.256

295 | P a g e
11. If for a Poisson variable X, f(2) = 3 f(4), what is the variance of X?
(a) 2 (b) 4
(c) 2 (d) 3

12. Poisson distribution may be


(a) Unimodal (b) Bimodal
(c) Multi-modal (d) (a) or (b)

13. In ______ distribution, mean = variance.


(a) Normal (b) Binomial
(c) Poisson (d) None of above

14. If x is a Poisson variate such that P(x=2) = 9P(x=4) + 90P(x=6), find mean of x.
(a) m=2 (b) m=1
(c) m=±1 (d) m = –4

15. If X is a Poisson variate with P (X = 0) = P (X = 1), then P (X = 2)


1 e
(a) (b)
6e 6
1 e
(c) (d)
2e 3

16. In a certain manufacturing process, 5% of the tools produced turn out to be defective. Find
the probability that in a sample of 40 tools, at most 2 will be defective.
[Given : e-2 = 0.135]
(a) 0.555 (b) 0.932
(c) 0.785 (d) 0.675

17. The probability that a man aged 45 years will die within a year is 0.012. What is the
probability that of 10 men, at least 9 will reach their 46 th birthday?
[Given: e-0.12 0.88692]
(a) 0.0935 (b) 0.9934
(c) 0.9335 (d) 0.9555

18. In Poisson Distribution, probability of success is very close to.


(a) -1 (b) 0
(c) 1 (d) None

19. If the mean of a Poisson variable X is 1, what is P (x = at least one)?


(a) 0.456 (b) 0.821
(c) 0.632 (d) 0.254

20. In a poisson distribution P (x = 0) = P (X = 2). Find E (x).


(a) 2 (b) 2
(c) -1 (d) 0

21. For a poisson distribution p (x=3)=5 p(x=5), Then S.D. is.


(a) 4 (b) 2
(c) 16 (d) 2

296 | P a g e
22. If standard deviation of a poisson distribution is 2, then its.
(a) Mode is 2 (b) Mode is 4
(c) Modes are 3 and 4 (d) Modes are 4 and 5

23. If x is a Poission variate and E (x) = 1 then P(x>1) is


e 1
(a) 1- (b) 1−e-1
2
5
(c) 1− 2e−1 (d) 1  e 1
2

24. The probability that a random variable x following Poisson distribution would assume a
positive Value is (1-e-2.7). what is the mode of the distribution?
(a) 2
(b) 3
(c) 4
(d) 5

25. The standard deviation of a Poisson variety is 1.732. What is the probability that the variety
lies between -2.3 to 3.68?
(a) 0.65
(b) 0.11
(c) 0.35
(d) None of the

26. Between 9 and 10 AM, the average number of Phone calls per minute coming into the
switchboard of a company is 4. Find the probability that during one particulare minute,
there will be,
i) no phone calls ii) at most 3 phone calls (given e-4= 0.018316)
(a) 0.018316, 0.43
(b) 0.050, 0.11
(c) 0.49, 0.12
(d) None of the

ANSWER

1. 2. 3. 4. 5. 6. 7. 8. 9. 10.
(A) (B) (D) (B) (B) (D) (B) (A) (C) (B)
11. 12. 13. 14. 15. 16. 17. 18. 19. 20.
(A) (D) (C) (B) (C) (D) (B) (B) (C) (A)
21. 22. 23. 24. 25. 26.
(D) (C) (C) (A) (A) (A)

297 | P a g e
CHAPTER – 18 - NORMAL OR GAUSSIAN DISTRIBUTION

PRACTICE QUESTIONS

1. The total area of the normal curve is


(a) one (b) 50 per cent
(c) 0.50 (d) any value between 0 and 1

2. The normal curve is


(a) Bell-shaped (b) U- shaped
(c) J- shaped (d) Inverted J — shaped

3. The normal curve is


(a) positively skewed (b) negatively skewed
(c) Symmetrical (d) all these

4. Area of the normal curve is


(a) between -  to  is 0.50 (b) between  to  is 0.50
(c) between -  to  is 0.50 (d) both (a) and (b)

5. The mean and mode of a normal distribution


(a) may be equal (b) may be different
(c) are always equal (d) (a) or (b)

6. The quartile deviation of a normal distribution with mean 10 and SD 4 is


(a) 0.675 (b) 6.75
(c) 2.70 (d) 3.20

7. The interval ( -3  ,  + 3 ) covers


(a) 95% area of a normal distribution (b) 96% area of a normal distribution
(c) 99% area of a normal distribution
(d) all but not 0.27% area of a normal distribution

8. What is the first quartile of X having the following probability density function?
(x−10)2
1
function: f(x) = e− 72 for −∞ < 𝑥 < ∞
√72π
(a) 4 (b) 5
(c) 5.95 (d) 6.75

9. If the two quartiles of N (  , 2) are 14.6 and 25.4 respectively, what is the standard
deviation of the distribution?–
(a) 9 (b) 6
(c) 10 (d) 8

10. If the mean deviation of a normal variable is 16, what is its quartile deviation?
(a) 10.00 (b) 13.50
(c) 15.00 (d) 12.05

298 | P a g e
11. If the points of inflexion of a normal curve are 40 and 60 respectively, then its mean deviation
is
(a) 8 (b) 10
(c) 0.80 (d) None

12. If the quartile deviation of a normal curve is 4.05, then its mean deviation is
(a) 5.26 (b) 6.24
(c) 4.24 (d) 4.80

13. If the 1st quartile and mean deviation about median of a normal distribution are 13.25 and 8
respectively, then the mode of the distribution is
(a) 20 (b) 10
(c) 15 (d) 12

14. If the area of standard normal curve between z = 0 to z = 1 is 0.3413, then the value of
(1) is
(a) 0.5000 (b) 0.8413
(c) —0.5000 (d) 1

15. If X and Y are 2 independent normal variables with mean as 10 and 12 and SD as 3 and 4,
then (X+Y) is normally distributed with
(a) mean = 22 and SD = 7 (b) mean = 22 and SD = 25
(c) mean = 22 and SD = 5 (d) mean = 22 and SD = 49

16. The total area of the normal curve is


(a) One (b) 50 per cent
(c) 0.50 (d) Any value between 0 and 1

17. For a normal distribution with mean 150 and S.D. 45; find Q 1 and Q3
(a) 119.35 and 190.65 respectively (b) 119.65 and 180.35 respectively
(c) 180.35 and 119.65 respectively (d) 123.45 and 183.65 respectively

18. If the 1st quartile and Mean Deviation about median of a normal distribution are 13.25 and 8
respectively, then the mode of the distribution is.
(a) 20 (b) 10
(c) 15 (d) 23

19. The area under the Normal curve is..


(a) 1 (b) 0
(c) 0.5 (d) —1

20. For a normal distribution P(  -3 <x<  +3) is equal to.


(a) 0.9973 (b) 0.9546
(c) 0.9899 (d) 0.9788

21. If the inflexion points of a Normal Distribution are 6 and 14. Find its standard Deviation?
(a) 4 (b) 6
(c) 10 (d) 12

299 | P a g e
22. If x  N 3,36  and y  N 5,64  are two independent Normal variate with their standard
parameters of distribution, then if (x + y) – N (8,A) also follows normal distribution. The value
of A will be .
(a) 100 (b) 10
(c) 64 (d) 36

23. The mean and variance of a random variable x having the following P.D.F
1 2
f(x) = e−(x−4) for −∞ < 𝑥 < ∞
√π
1
(a) 4, ½ (b) 4,
2
(c) 2, 2 (d) 2,½

24. A sample of 100 dry battery cells tested to find the length of life produced the following
results: X =12 hours,  =3 hours. What percentage of battery cells are expected to have life
less than 6 hours?
[Area under the normal curve from z = 0 to z = 2 is 0.4772]
(a) 2.28% (b) 2.56%
(c) 4.56% (d) 1.93%

 x  60 
25. If x follows a normal distribution with   50 and   10 . What is the value of P :
 x  50 
[area under the nomral curve from z=0 to z=1 is 0.3413].
(a) 0.6826 (b) 0.7354
(c) 0.1983 (d) 0.5492

26. For a certain normal variate X, the mean is 12 and S.D. is 4. Find P (X >20) : [Area under the
normal curve from z=0 to z=2 is 0.4772]
(a) 0.5238 (b) 0.0472
(c) 0.7272 (d) 0.0228

27. 5,000 students were appeared in an examination. The mean of marks was 39.5 with a
Standard Deviation 12.5 marks. Assuming the distribution to be normal, find the number of
students recorded more than 60% marks.
Given: when z=1.64, Area of normal curve=0.4495
(a) 1,000 (b) 505
(c) 252 (d) 2,227

28. There are 75 students in a class and their average marks is 50 and standard derivation of
Marks is 5. Number of students who have secured more then 60 marks (given that area under
the normal curre for z=z is 0.4772 is ………………
(a) 1 (b) 2
(c) 3 (d) 4

29. Which of the following is false in case of normal distribution.


(a) it is multi model (b) mean = median = mode
(c) it is symmetric (d) Total area is 1

300 | P a g e
30. if x ~ B (5, p) and p (x = 2) = 0.4362 and p (x = 3) = 0.2181 then p =
3 1
(a) (b)
4 3
2 1
(c) (d)
3 4

31. In a Poisson distribution :


(a) Mean & SD are equal (b) Mean, variance are equal
(c) SD & variance are equal (d) both (a) and (b)

32. In Binomial Distribution,  = 4 and 2 = 3 then mode =


(a) 4 (b) 4.25
(c) 4.5 (d) 4.1

33. If the points of inflexion of a normal curve are 6 and 14 then standard deviation is
(a) 4 (b) 8
(c) 16 (d) 32

34. There are 75 students in a class and their average marks is 50 and S.D of marks is 5. Number
of students who have secured more than 60 marks (Given that area under the normal curve
for z = 2 is 0.4772) is ______
(a) 1 (b) 2
(c) 3 (d) 4

35. In normal distribution mean, median and mode are


(a) Equal (b) Not equal
(c) Zero (d) None of the above

36. If x is a binomial variable with parameters n and p, then x can assume-


(a) any value between 0 and n
(b) any value between 0 and n, both inclusive
(c) any whole number between 0 and n, both inclusive
(d) any number between 0 and infinity

37. In ________ distribution, mean = variance


(a) Normal (b) Binomial
(c) Poisson (d) None

38. Under a normal curve x ± 3 covers ___________


(a) 100% of the area (item values) (b) 99%
(c) 99.73% (d) 99.37%

1
39. If 'X' is a binomial variable with parameter 15 and , then the value of the mode of the
3
distribution.
(a) 5 (b) 5 and 6
(c) 5.50 (d) 6

301 | P a g e
40. Standard deviation of bionomial distribution is:
(a) np (b) (np) 2

(c) npq (d) (npq) 2

41. The wages of workers of factory follows:


(a) Binomial distribution (b) Poisson distribution
(c) Normal distribution (d) Chi-square distribution

ANSWER

1. 2. 3. 4. 5. 6. 7. 8. 9. 10.
(A) (A) (C) (D) (C) (C) (D) (C) (D) (B)
11. 12. 13. 14. 15. 16. 17. 18. 19. 20.
(A) (D) (A) (B) (C) (A) (B) (A) (A) (A)
21. 22. 23. 24. 25. 26. 27. 28. 29. 30.
(A) (A) (A) (A) (A) (D) (C) (B) (B) (B)
31. 32. 33. 34. 35. 36. 37. 38. 39. 40.
(B) (A) (A) (B) (A) (C) (C) (C) (A) (C)
41.
(C)

302 | P a g e
CHAPTER – 19 - INDEX NUMBERS
PRACTICE QUESTIONS

1. Index no. for the base period is always taken as


(a) 200 (b) 50
(c) 1 (d) 100

2. ________ play a very important part in the construction of index nos.


(a) weights (b) classes
(c) estimations (d) none

3. Index nos. show ________ changes rather than absolute amounts of change.
(a) relative (b) percentage
(c) both (d) none

4. Price relative is equal to


(a) Price in the given year x l00 /Price in the base year
(b) Price in the year base year x 100 / Price in the given year
(c) Price in the given year x 100
(d) Price in the base year x 100

5. Index no. is equal to


(a) sum of price relatives (b) average of the price relatives
(c) product of price relative (d) none

6. Circular Test is one of the tests of


(a) index nos. (b) hypothesis
(c) both (d) none

7. ________ is an extension of time reversal test


(a) Factor Reversal test (b) Circular test
(c) both (d) none

8. Factor Reversal test is satisfied by


(a) Fisher’s Ideal Index (b) Laspeyres Index
(c) Paasches Index (d) none

9. Laspeyre’s formula does not obey


(a) Factor Reversal Test (b) Time Reversal Test
(c) Circular Test (d) All of these

10. A ratio or an average of ratios expressed as a percentage is called


(a) a relative no. (b) an absolute no.
(c) an index no. (d) none

11. The value at the base time period serves as the standard point of comparison
(a) false (b) true
(c) both (d) none

303 | P a g e
12. An index time series is a list of _______ nos. for two or more periods of time
(a) index (b) absolute
(c) relative (d) none

13. Index nos. are often constructed from the


(a) frequency (b) class
(c) sample (d) none

14. _______ is a point of reference in comparing various data describing individual behaviour.
(a) Sample (b) Base period
(c) Estimation (d) none

15. Sum of all commodity prices in the current year x 100


Sum of all commodity prices in the base year is
(a) Relative Price Index (b) Simple Aggregative Price Index
(c) both (d) none

16. Chain index is equal to


(a) link relative of current year x chain index of the current year /100
(b) link relative of previous year x chain index of the current year /100
(c) link relative of current year x chain index of the previous year /100
(d) link relative of previous year x chain index of the previous year /100

17. P01 is the index for time


(a) 1 on 0 (b) 0 on 1
(c) 1 on 1 (d) 0 on 0

18. P10 is the index for time


(a) 1 on 0 (b) 0 on 1
(c) 1 on 1 (d) 0 on 0

19. When the product of price index and the quantity index is equal to the corresponding value
index then
(a) Unit Test (b) Time Reversal Test
(c) Factor Reversal Test (d) none holds

20. The formula should be independent of the unit in which or for which price and quantities are
quoted in
(a) Unit Test (b) Time Reversal Test
(c) Factor Reversal Test (d) none

21. Laspeyres method and Paasche’s method do not satisfy


(a) Unit Test (b) Time Reversal Test
(c) Factor Reversal Test (d) B and C

22. The index no. is a special type of average


(a) false (b) true
(c) both (d) none

304 | P a g e
23. Fisher’s Ideal Formula for calculating index nos. satisfies the _______ tests
(a) Units Test (b) Factor Reversal Test
(c) both (d) none

24. Fisher’s Ideal Formula dose not satisfy _________ test


(a) Unit test (b) Circular Test
(c) Time Reversal Test (d) none

25. _________ satisfies circular test


(a) G.M. of price relatives or the weighted aggregate with fixed weights
(b) A.M. of price relatives or the weighted aggregate with fixed weights
(c) H.M. of price relatives or the weighted aggregate with fixed weights
(d) none

26. Laspeyre’s and Paasche’s method _________ time reversal test


(a) satisfy (b) do not satisfy
(c) are (d) are not

27. Theoretically, G.M. is the best average in the construction of index nos. but in practice,
mostly the A.M. is used
(a) false (b) true
(c) both (d) none

28. Laspeyre’s or Paasche’s or the Fisher’s ideal index do not satisfy


(a) Time Reversal Test (b) Unit Test
(c) Circular Test (d) none

29. The test of shifting the base is called


(a) Unit Test (b) Time Reversal Test
(c) Circular Test (d) none

30. Shifted price Index =


Original Price x 100
Price Index of the year on which it has to be shifted
(a) True (b) false
(c) both (d) none

31. The no. of test of Adequacy is


(a) 2 (b) 5
(c) 3 (d) 4

32. Simple aggregate of quantities is a type of


(a) Quantity control (b) Quantity indices
(c) both (d) none

33. The test of shifting the base is called


(a) Unit Test. (b) Time Reversal Test.
(c) Circular Test. (d) None of these.

305 | P a g e
34. The index number of prices at a place in 1998 is 355 with 1991 as base. This means
(a) There has been on the average a 255% increase in prices
(b) There has been on the average a 355% increase in price
(c) There has been on the average a 250% increase in price
(d) None of these

35. If now the prices of all the commodities in a place have been decreased by 85% over the
base period prices, then the index number of prices for the place is now (index number of
prices of base period = 100)
(a) 100 (b) 135
(c) 65 (d) None of these

36. If the prices of all commodities in a place have increased 1.25 times in comparison to the
base period, the index number of prices of that place is now
(a) 125 (b) 150
(c) 225 (d) None of these

37. The ______ is satisfied when Pab × Pbc × Pca = 1


(a) Time reversal test (b) Factor reversal test
(c) Circular test (d) Unit test

38. ______ is an extension of time reversal test.


(a) Factor reversal test (b) Circular test
(c) Unit test (d) None of these

39. Time reversal test is satisfied when


(a) P01 × P10 = 0 (b) P01 × P10 = 1
(c) P01 × P10 <1 (d) P01 × P10 >1

40. If with a rise of 10% in prices the salaries are increased by 20%, the real salary increases by
(a) 10% (b) More than 10%
(c) 20% (d) Less than 10%

41. The total sum of the values of a given year divided by the sum of the values of the base year
is a
(a) Price index (b) Quantity index
(c) Value index (d) None of these

42. Fisher’s ideal index is


(a) Arithmetic mean of Laspeyre’s and Paasche’s index
(b) Median of Laspeyre’s and Paasche’s index
(c) Geometric mean of Laspeyre’s and Paasche’s index
(d) None of these

43. Factor reversal test is satisfied by


(a) Laspeyre’s index (b) Paasche’s index
(c) Fisher’s ideal index (d) None of these

306 | P a g e
44. Laspeyre’s index is based on
(a) Base year quantities (b) Current year quantities
(c) Average of current year and base year
(d) None of these

45. Laspeyre’s and Pasche’s method satisfy time reversal test


(a) True (b) False
(c) Both (d) None of these

46. The index number is a special type of G.M.


(a) True (b) False
(c) Both (d) None of these

47. The number of test of adequacy is ___________


(a) 2 (b) 5
(c) 3 (d) 4

48. Theoretically, A.M. is the best average in the construction of index nos. but in practice,
mostly the G.M. is used:
(a) False (b) True
(c) Both (d) None of these

49. The number of test of adequacy is ___________


(a) 2 (b) 5
(c) 3 (d) 4

50. Fisher’s ideal idex no. is equal to


(a) Laspeyse’s Index X Pasche’s Index (b) √Laspeyse’s Index X Pasche’s Index
∑P0 q1
(c) ∑P1 q1
(d) None of these

51. Fisher’s Ideal formula does not satisfy_________ test


(a) Circular test (b) Unit test
(c) Time Reversal test (d) None of these

52. If with rise of 10% in prices the wages are increased by 20%. Find the percentage of real
wage increase
(a) 109.29% (b) 9.09%
(c) 9.29% (d) None of these

53. The price level of a country in a certain year has increased 20% over the base period. The
Index number is _____________
(a) 20 (b) 120
(c) 220 (d) None of these

54. In a circular test the _____________condition must be satisfied?


(a) P01xP12xP20=1 (b) P02XP10XP20=1
(c) P10XP20XP21=1 (d) None of these

307 | P a g e
55. Bowley’s index number is expressed in terms of:
Laspeyre' s  Paasche' s Laspeyre' s  Paasche' s
(a) (b)
2 2
Laspeyre' s - Paasche' s
(c) (d) None of these
2

56. Fisher’s ideal formula for calculating index number satisfies the.
(a) Unit Test (b) Factor Reversal Test
(c) Both (a) & (b) (d) None of these

57. Circular Test is satisfied by


(a) Paasche’s Index Number.
(b) The simple geometric mean of price relatives and the weighted average with fixed
weights
(c) Laspeyre’s Index Number
(d) None of these

58. Cost of living index numbers are also used to find real wages by the process of
(a) Base shifting.. (b) Splicing of index numbers
(c) Deflating of index numbers (d) None of these

59. The prices of a commodity in the year 1975 and 1980 were 25 and 30 respectively. Taking
1980 as the base year the price relative is.
(a) 113.25 (b) 83.33
(c) 109.78 (d) None

60. Shifted Price index.


Original Price Index
  100
Price index of the year on which it has to be shifted
(a) True (b) False
(c) Partly True (d) Partly False

61. Laspeyare’s and Paasche’s Metho____ time Reversal Test:


(a) do not satisfy (b) satisfy
(c) depends on the case (d) can’t say

62. Chain index is equal to.


link relative of current year  chain index of the current year
(a)
100
link relative of current year  chain index of the Previous year
(b)
100
link relative of Previous year  chain index of the current year
(c)
100
(d) None of these

308 | P a g e
63. The Circular Test is known is.
(a) P01  P12  P20  1 (b) P12  P01 P20  1
(c) P20  P12 P01  1 (d) P02  P21 P12  1

64. In Passche’s index, weights are based on:


(a) Current year quantities (b) Base year quantities
(c) Weighted average prices (d) None of these

65. Fisher’s Ideal Index does not satisfy:


(a) Time Reversal Test (b) Factor Reversal Test
(c) Unit Test (d) Circular test

66. P01Q01 
P Q
1 1
which of following test satisfies the above?
P Q
0 0

(a) Time Reversal Test (b) Factor Reversal Test


(c) Circular Test (d) None of these.

67. Time reversal & factor reversal are:


(a) Quantity Index (b) ideal index
(c) Price Index (d) Test of Consistency

68. In Laspeyeres Index Number are used as weights?


(a) Base year price (b) Current year price
(c) Base year quantities (d) Current year quantities

69. Consumer price index is commonly known as.


(a) Chain based index (b) Ideal index
(c) wholesale price index (d) Cost of living index

70. The life expectancy, E of male is o linear function of time (year). It is given that in 1980 the
life expectancy was 70 years and in 2000 it was 75 years, Make a prediction of life
expectancy in 2012.
(a) 78 (b) 80
(c) 82 (d) 84

71. Wholesale Price Index (WP) is given by :


(a) Marshall – Edge worth Index (b) Laspere’s Index
(c) Pasche’s Index (d) None of the above.

72. Fisher’s Ideal index is obtained by :


(a) Arithmetic Mean of Laspeyer’s & Passche’s Index
(b) Geometric Mean of Laspeyre’s & Passche’s index
(c) Sum of Lasperyre’s & Passche’s Index
(d) None of the above.

73. Net monthly salary of an employee was Rs.3000. The consumer price Index in 1985 is 250
with 1980 as base year. If he has to be rightly compensated, then additional Dearness
Allowance to be paid to the employee is ____
(a) Rs.4000 (b) Rs.4800
(c) Rs.5500 (d) Rs.4500
309 | P a g e
74. The consumer price index for 2006 on the basis of 2005 from the following data is:
Commodities Quantities consumed in Price in Prices in 2006
2005 2005
A 6 5.75 6.00
B 6 5.00 8.00
C 1 6.00 9.00
D 6 8.00 10.00
E 4 2.00 1.50
F 1 20.00 15.00
(a) 128 .77 (b) 108.77
(c) 138.77 (d) 118.77

75. Suppose a business executive was earning Rs. 2,050 in the base period. What should be his
salary in the current period if his standard of living is to remain the same? Given  W = 25
and  IW = 3544
(a) Rs. 2096 (b) Rs. 2906
(c) Rs. 2106 (d) Rs. 2306

76. Calculate the Fisher ideal index from the following data.
Price(Rs.) Quantity(‘000 kg)
Commodity 2004 2005 2004 2005
Rice 9.3 4.5 100 90
Wheat 6.4 3.7 11 10
Pulse 5.1 2.7 5 3
(a) 49.13 (b) 48.13
(c) 84.13 (d) 46.12

77 .From the following data.


Group: A B C D E F
Group Index: 120 132 98 115 108 95
Weight: 6 3 4 2 1 4
The general index is given by.
(a) 113.54 (c) 115.30
(c) 117.92 (d) 111.30

78. From the following data:


Base Year Current Year
Commodity Price Quantity Price Quantity
A 7 17 13 25
B 6 23 7 25
C 11 14 13 15
D 4 10 8 8
The Marshal Edge worth index number is
(a) 144.19 (b) 143.9
(c) 4900 (d) 140.31

310 | P a g e
79. Net monthly salary of an employee was Rs. 3000 in 1980. The consumer price index
number in 1985 is 250 with 1980 as base year. If he has to be rightly compensated, then
the Dearness Allowance to be paid to the employee is.
(a) Rs. 4200 (b) Rs. 4500
(c) Rs. 4900 (d) Rs. 4300

80. An enquiry into the budgets of middle class families in a village gave the following
information
Expenses on : Food Rent Clothing Fuel Other
30% 15% 20% 10% 25%
Price in Rs.
in 1987: 100 20 70 20 40
Price in Rs.
in 2005 90 20 60 10 55
PW =10101.5, The price index number based on weighted Arithmetic Mean of price
relatives is
(a) 111.015 (b) 101.015
(c) 0.0197 (d) None

81. Given the following information.


2000 2003
Commodity Price Quantity Price Quantity
A 2 74 3 82
B 5 125 4 140
C 7 40 5 33
Which of the following is true
(a) Marshall Edge worth index for 2003 is 105.13
(b) Fisher’s index for 2003 is 90.15.
(c) Marshall Edge worth Index Number is good approximation Fisher’s Index Number
(d) None of these

82. In 2004 for working class people wheat was selling at an average price of Rs. 16 per 20 kg,
cloth at Rs. 2 per meter. house rent Rs. 30 per house and other items at Rs. 10 per unit. By
2005 cost of wheat rose by Rs. 4 per 20 Kg, house rent by Rs. 15 per house and other
items doubled in price. The working class cost of living index for the year 2005 (with 2004
as base) was 160. By how much did cloth rose in price during the period?
(a) 1.28 (b) 0.99
(c) 1.73 (d) 1.30

83. Consumer Price Index Number goes up from 100 to 200 and salary of a worker is also
raised from 300 to 500, their real wage is
(a) 300 (b) 250
(c) 600 (d) 350

84. Using following data, find Paasche’s Index Number


Base Year Current Year
Commodities Price Quantity Price Quantity
A 5 25 6 30
B 3 8 4 10
311 | P a g e
C 2 10 3 8
D 10 4 3 5
(a) 109.21 (b) 105.28
(c) 110.32 (d) 120.21

85. In the data group Bowley’s and Laspeyre’s index number is as follows. Bowley’s index
number = 150, Lspeyre’s index number = 180 then paesche’s index number is
(a) 120 (b) 30
(c) 165 (d) None of these

86. Find the paasche’s index number for prices from the following data taking 1970 as the base
year.
Commodity 1970 1975
Price Commodity Price Commodity
A 1 6 3 5
B 3 5 8 5
C 4 8 10 6
(a) 261.36 (b) 265.48
(c) 274.32 (d) 282

87. If Laspeyre's index number is 90 and Paasche's index number is 160, then Fisher's index
number will.
(a) 144 (b) 120
(c) 125 (d) None of these

88. Given the prices of 2 commodities are increased by 10% and 20% respectively and the price
of another commodity is decreased by 30%. The relative importance of 3 commodities are
in the ratio 3:3:1. Find weighted price index number.
(a) 80 (b) 109
(c) 110 (d) 108.5

89. Calculate the Fisher index number from the following data
Σp0q0 = 116 ; Σp0q1 = 140 ; Σp1q0 = 97 ; Σp1q1 = 117
(a) 83.59 (b) 184.09
(c) 120.02 (d) None of these

90. Calculate the cost of living index number for the year 1975 is
Commodity 1970 1975
price Qty Price Qty
A 1 6 3 5
B 3 5 8 5
C 4 8 10 6
(a) 260.37 (b) 265.48
(c) 274.32 (d) 282

91. If Fisher index number is 150,and Paasche’s index number is 144 then Laspeyer’s index
number is:
(a) 147.77 (b) 156.25
(c) 140.17 (d) 138.08

312 | P a g e
92. What is the formula for calculating the deflated index :
Current Value Current Value
(a)  100 (b)  100
Price Index of current year Price Index of Last Year
Current Value Current Value
(c) (d)
Price Index of Current Year Price Index of Last Year

93. The index number for the year 2012 taking 2011 as base using simple average of price
relatives method from the data given below is:
Commodity A B C D E
Price in 2011 115 108 95 80 90
Prince in 2012 125 117 108 95 95
(a) 112 (b) 117
(c) 120 (d) 111

94. An index time series is a list of ____ numbers for two or more periods of time.
(a) Index (b) absolute
(c) Relative (d) None

95. Time Reversal Test is satisfied by _______


(a) Fisher's Ideal index (b) Drobish Boweley Index
(c) Laspeyer's Index (d) None of these

96. Bowley’s index Number = 150, Laspeyer’s Index =180 then paache’s index number is ………
(a) 120 (b) 30
(c) 105 (d) None

97. In 2005 price index is 286 with base 1995 then how much price increased in 2005 with base
1995?
(a) 286% (b) 386%
(c) 86% (d) 186%

98. Factor reversal test is expressed in terms of

(a)
P Q1 1

P Q0 0

(b)
P Q x P Q
1 1 1 1

P Q P Q1
0 0 0

(c)
P Q1 1

Q P 0 1

(d)
 Q P x P Q
1 0 1 0

Q P Q P
0 0 0 1

99. If with an increase of 10% in prices, the rise in wages is 20% then the real wage has
increased by
(a) 20% (b) 10%
(c) Less than 10% (d) More than 10%

313 | P a g e
100. _________ play a very important role in the construction of index numbers.
(a) Weights
(b) Classes
(c) Estimations
(d) None

101. Consumer price index number for the year 1977, was 313, with 1960 as the base year, and
was 96 for the year 1960. The average monthly wages in 1977 of the workers into factory be
Rs.160, their real wages is:
(a) Rs. 48.40
(b) Rs. 51.12
(c) Rs. 40.30
(d) None of the above

ANSWER

1. 2. 3. 4. 5. 6. 7. 8. 9. 10.
(D) (A) (B) (A) (B) (A) (B) (A) (D) (C)
11. 12. 13. 14. 15. 16. 17. 18. 19. 20.
(B) (A) (C) (B) (B) (C) (A) (B) (C) (A)
21. 22. 23. 24. 25. 26. 27. 28. 29. 30.
(D) (B) (C) (B) (A) (B) (B) (C) (C) (A)
31. 32. 33. 34. 35. 36. 37. 38. 39. 40.
(D) (B) (C) (A) (D) (A) (C) (B) (B) (D)
41. 42. 43. 44. 45. 46. 47. 48. 49. 50.
(C) (C) (C) (A) (B) (A) (D) (A) (D) (B)
51. 52. 53. 54. 55. 56. 57. 58. 59. 60.
(A) (B) (B) (A) (A) (C) (B) (C) (B) (A)
61. 62. 63. 64. 65. 66. 67. 68. 69. 70.
(A) (B) (A) (A) (D) (B) (D) (C) (D) (A)
71. 72. 73. 74. 75. 76. 77. 78. 79. 80.
(B) (B) (D) (D) (B) (A) (D) (A) (B) (B)
81. 82. 83. 84. 85. 86. 87. 88. 89. 90.
(C) (D) (B) (B) (A) (A) (B) (D) (A) (A)
91. 92. 93. 94. 95. 96. 97. 98. 99. 100.
(B) (C) (D) (A) (A) (A) (D) (A) (C) (A)
101.
(B)

314 | P a g e
CHAPTER – 20 - TIME SERIES
Choose the most appropriate option (a) or (b) or (c) or (d).

1. An orderly set of data arranged in accordance with their time of occurrence is called:
(a) Arithmetic series (b) Harmonic series
(c) Geometric series (d) Time series

2. A time series consists of:


(a) Short-term variations (b) Long-term variations
(c) Irregular variations (d) All of the above

3. The graph of time series is called:


(a) Histogram (b) Straight line
(c) Histogram (d) Ogive

4. Secular trend can be measured by:


(a) Two methods (b) Three methods
(c) Four methods (d) Five methods

5. The secular trend is measured by the method of semi-averages when:


(a) Time series based on yearly values
(b) Trend is linear
(c) Time series consists of even number of values
(d) None of them

6. Increase in the number of patients in the hospital due to heat stroke is:
(a) Secular trend (b) Irregular variation
(c) Seasonal variation (d) Cyclical variation

7. The systematic components of time series which follow regular pattern of variations are
called:
(a) Signal (b) Noise
(c) Additive model (d) Multiplicative model

8. The unsystematic sequence which follows irregular pattern of variations are called:
(a) Noise (b) Signal
(c) Linear (d) Non-linear

9. In time series seasonal variations can occur within a period of:


(a) Four years (b) Three years
(c) One year (d) Nine years

10. Wheat crops badly damaged on account of rains is:


(a) Cyclical movement (b) Random movement
(c) Secular Trend (d) Seasonal Movement

11. The method of moving average is used to find the:


(a) Secular trend (b) Seasonal variation
(c) Cyclical variation (d) Irregular variation

315 | P a g e
12. Most frequency used mathematical model of a time series is:
(a) Additive model (b) Mixed model
(c) Multiplicative model (d) Regression

13. A time series consists of:


(a) No mathematical model (b) One mathematical model
(c) Two mathematical models (d) Three mathematical models

14. In semi-averages method, we decide the data into:


(a) Two parts (b) Two equal parts
(c) Three parts (d) Difficult to tell

15. Moving average method is used for measurement of trend when:


(a) Trend is linear (b) Trend is non-linear
(c) Trend is curvi linear (d) None of them

16. When the trend is of exponential type, the moving averages are to be computed by using:
(a) Arithmetic mean (b) Geometric mean
(c) Harmonic mean (d) Weighted mean

17. The long term trend of a time series graph appears to be:
(a) Straight-line (b) Upward
(c) Downward (d) Parabolic curve or third degree curve

18. Indicate which of the following an example of seasonal variations is:


(a) Death rate decreased due to advance in science
(b) The sale of air condition increases during summer
(c) Recovery in business
(d) Sudden causes by wars

19. The most commonly used mathematical method for measuring the trend is:
(a) Moving average method (b) Semi average method
(c) Method of least squares (d) None of them

20. A trend is the better fitted trend for which the sum of squares of residuals is:
(a) Maximum (b) Minimum
(c) Positive (d) Negative

21. Decomposition of time series is called:


(a) Historigram (b) Analysis of time series
(c) Histogram (d) Detrending

22. The fire in a factory is an example of:


(a) Secular trend (b) Seasonal movements
(c) Cyclical variations (d) Irregular variations

23. Increased demand of admission in the subject of computer in Uttar Pradesh is:
(a) Secular trend (b) Cyclical trend
(c) Seasonal trend (d) Irregular trend

24. Damages due to floods, droughts, strikes fires and political disturbances are:
(a) Trend (b) Seasonal
(c) Cyclical (d) Irregular

316 | P a g e
25. The general pattern of increase or decrease in economics or social phenomena is shown by:
(a) Seasonal trend (b) Cyclical trend
(c) Secular trend (d) Irregular trend

26. In moving average method, we cannot find the trend values of some:
(a) Middle periods (b) End periods
(c) Starting periods (d) Between extreme periods

27. Moving-averages:
(a) Give the trend in a straight line (b) Measure the seasonal variations
(c) Smooth-out the time series (d) None of them

28. The rise and fall of a time series over periods longer than one year is called:
(a) Secular trend (b) Seasonal variation
(c) Cyclical variation (d) Irregular variations

29. A time series has:


(a) Two Components (b) Three Components
(c) Four Components (d) Five Components

30. The multiplicative time series model is:


(a) Y=T+S+C+I (b) Y = TSCI
(c) Y= a + bx (d) y = a + bx + CX2

31. The additive model of Time Series


(a) Y=T+S+C+I (b) Y = TSCI
(c) Y= a + bx (d) y = a + bx + CX2

32. A pattern that is repeated throughout a time series and has a recurrence period of at most
one year is called:
(a) Cyclical variation (b) Irregular variation
(c) Seasonal variation (d) Long term variation

33. If an annual time series consisting of even number of years is coded, then each coded
interval is equal to:
(a) Half year (b) One year
(c) Both (a) and (b) (d) Two years

34. In semi averages method, if the number of values is odd then we drop:
(a) First value (b) Last value
(c) Middle value (d) Middle two values

35. The trend values in freehand curve method are obtained by:
(a) Equation of straight line (b) Graph
(c) Second degree parabola (d) All of the above

317 | P a g e
ANSWER

1. 2. 3. 4. 5. 6. 7. 8. 9. 10.

(D) (D) (B) (C) (B) (C) (A) (A) (C) (B)

11. 12. 13. 14. 15. 16. 17. 18. 19. 20.

(A) (C) (C) (B) (A) (D) (B) (b) (B) (B)

21. 22. 23. 24. 25. 26. 27. 28. 29. 30.

(D) (D) (A) (D) (D) (C) (C) (A) (C) (B)

31. 32. 33. 34. 35.

(B) (C) (C) (C) (B)

__**__

318 | P a g e
CA FOUNDATION – MAY 2019
QUESTION PAPER
BUSINESS MATHEMATICS, LOGICAL REASONING & STATISTICS

01. If the ratio of two numbers is 7 : 11. If 7 is added to each number then the new ratio will be 2 : 3 then
the numbers are.
(a) 49, 77 (b) 42, 45
(c) 43, 42 (d) 39, 40

02. log 2 2 (512) : log 3 2 324 


(a) 128 : 81 (b) 2:3
(c) 3:2 (d) None

If 2 x  3 y 12 z
2 2 2
03.
1 1 1 1 2 1
(a)   (b)  2 2
x2 y2 z 2 x 2
y z
2 1 1
(c)   (d) None
x2 y2 z 2

04. Then value of


 1  1  1  1 
log 5 1    log 5 1    log 5 1    ............ log 5 1  
 5  6  7  624 
(a) 2 (b) 3
(c) 5 (d) 0

05. Find the condition that one roots is double the other of ax2 +bx + c = 0
(a) 2b2 = 3ac (b) b2 = 3ac
(c) 2b = 9ac
2 (d) 2b2 > 9ac

x  y 4 2 3 12 7
06. If  + = then
x  y 2 1 0  5 2
(a) x = 7 y = –3 (b) x = –7, y = –3
(c) x = –7, y = 3 (d) x = 7, y = 3

07. The solution set of the in equation x +2 > 0 and 2x – 6 > 0 is


(a) (–2,) (b) (3,)
(c) (–,–2) (d) (–,–3)

319 | P a g e
08. The common region represented by the following in equalities
L1 = X1 + X2 < 4; L2 = 2X1 + X2 > 6 X2

(a) OABC (b) Outside of OAB


(c)  BCE (c)  ABE

09. A sum was invested for 3 years as per C.I and the rate of interest for first year is 9%, 2nd year is 6% and
3rd year is 3% p.a. respectively. Find the sum if the amount in three years is Rs. 550?
(a) Rs. 250 (b) Rs. 300
(c) Rs. 462.16 (d) Rs. 350

10. If pi2= Rs.96 and R = 8% compounded annually then P =____________.


(a) Rs. 14,000 (b) Rs. 15,000
(c) Rs. 16,000 (d) Rs. 17,000

PTR
11. P = Rs. 5,000 R = 15% T = 4½ using I  then I will be
100
(a) Rs. 3,375 (b) Rs. 3,300
(c) Rs. 3,735 (d) None of these

12. A sum of money amounts to Rs. 6,200 in 2 years and Rs. 7,400 in 3 years as per S.I. then the Principal is.
(a) Rs. 3,000 (b) Rs. 3,500
(c) 3,800 (d) None of these

13. The Effective Rate of interest does not depend upon


(a) Amount of Principal (b) Amount of interest
(c) Number of Conversion periods (d) None of these

14. In simple interest if the principal is Rs. 2,000 and the Rate and time are the Roots of the equation x2 –
11x + 30 = 0 then the simple interest is _________
(a) Rs. 500 (b) Rs. 600
(c) Rs. 700 (d) Rs. 800

15. The certain sum of money became Rs. 692/– in 2 yrs and Rs. 800/– in 5 years then the principle Amount
is __________
(a) Rs. 520 (b) Rs. 620
(c) Rs. 720 (d) Rs. 820

16. Determine the present value of perpetuity of Rs. 50,000 per month @ Rate of interest 12% p.a. is
_____________
(a) Rs. 45,00,000 (b) 50,00,000
(c) Rs. 55,00,000 (d) 60,00,000

320 | P a g e
17. A person wants to lease out a machine costing Rs. 5,00,000 for a 10 year period. It has fixed a rental of
Rs. 51,272 per annum payable annually starting from the end of first year. Suppose rate of interest is
10% per annum, compounded annually on which money can be invested. To whom this agreement is
favourable?
(a) Favour for lessee (b) Favour for lessor
(c) Not for both (d) Can’t be determined

18. Let a person invest a fixed sum at the end of each month in an account paying interest 12% per year
compounded monthly. It the future value of this annuity after the 12th payment is Rs. 55,000 then the
amount invested every month is?
(a) Rs. 4,8,37 (b) Rs. 4,637
(c) Rs. 4,337 (d) Rs. 3337

19. If 11C X  11C2 X  4 an d x  4 then the value of 7 C X 


(a) 20 (b) 21
(c) 22 (d) 23

20. Which of the following is not a correct statement


(a) nPn  n pn1 (b) n
pn  2.n pn  2
(c) nPn  3.n pn3 (d) nPn  n.(n  1) pn1

21. If Y = 1+x + x2 + ..................... ∞ then x =


y 1 y 1
(a) (b)
y y
y y
(c) (d)
y 1 y 1

22. If 2 + 6 + 10 + 14 + 18 + ................ + x = 882 then the value of x


(a) 78 (b) 80
(c) 82 (d) 86

23. In a G.P, If the fourth term is ‘3’ then the product of first seven terms is
(a) 35 (b) 37
(c) 36 (d) 38

24. The Ratio of sum of n terms of the two AP’s is (n +1) : (n–1) then the Ratio of their mth terms is
(a) (m + 1) : 2m (b) (m + 1) : (m –1)
(c) (2m –1 :(m + 1) (d) m : (m – 1)

25. If A = {1,2,3,4,5,6,7,8,9,}
B = {1,3,4,5,7,8};C = {2,6,8,} then find (A – B) C 
(a) {2,6,} (b) {2,6,8}
(c) {2,6,8,9} (d) None of these

321 | P a g e
26. If f (x) = x2 and g(x) x then
(a) go, f(3)= 3 (b) go f (–3) = 9
(c) go, f(9) = 3 (d) go f (–9) = 3

27. A= {1,2,3,4, ............... 10} a relation on A,R {(x, y)/x y 10, x A,yA,X  Y}then Domain of R–1 is
(a) {1,2,3,4,5} (b) {0,3,5,7,9}
(c) {1,2,4,5,6,7} (d) None of these

28. If A ={a, b, c, d}; B = {p, q, r ,s} which of the following relation is a function from A to B
(a) R1= {(a, p), (b, q),(c, s)} (b) R2 ={(p, a}, (b, r),(d, s)}
(c) R3 ={(b, p),(c, s),(b, r)} (d) R4 = {(a, p)(b, r)(c, q), (d, s)}

dy
29. If 2 x – 2 y  2 x– y then at x=y=2
dx
(a) 1 (b) 2
(c) 4 (d) 5

x3
30. If the Cost of function of a commodity is given by C  150x-5x2, , where C stands for cost and x
6
stands for output. If the average cost is equal to the marginal cost then the output x =__________
(a) 5 (b) 10
(c) 15 (d) 20

3
x
31. 
z 5 x  x
dx 

(a) 1 (b) ½
(c) 2 (d) 3/2

32. If in a certain language, MADRAS is code as NBESBT, how is BOMBAY coded in that language?
(a) CPNCBX (b) CPNCBZ
(c) CPOCBZ (d) CQOCBZ

33. Which of the following is odd one


(a) CEHL (b) KMPT
(c) OQTX (d) NPSV

34. Which of the following is odd one 4, 12, 44, 176, 890 ....................
(a) 4 (b) 12
(c) 44 (d) 176

35. 7,23, 47, 119, 167 _________


(a) 211 (b) 223
(c) 287 (d) 319

322 | P a g e
36. When a person faces north and walk 25 m and she turn left and walk 20m and again turns right and walk
25m, and turns right 25m and turns right and walks 40m in which direction is he now from his starting
point.
(a) North – West (b) North – East
(c) South – East (d) South – West

37. Madhuri moved a distance of 75 meters toward north. She then turned to the left and walking for about
25m, turned left again and walks 80m, finally she turned to the right at an angle of 45 o . In which
direction was she moving finally?
(a) South – East (b) South – West
(c) North – west (d) North – East

38. A person facing North 70o clock wise direction moving in clockwise and 300o clock wise direction. Now, in
which direction he presently facing.
(a) North-West (b) South-East
(c) North-East (d) Sought-West

39. Sangeetha leaves from her home. She first walks 30 metres in north – west direction, and then 30 m in
south west direction, next she walks 30 metres in south – east direction. Finally she turns towards her
house. In which direction is she moving
(a) North West (b) North – East
(c) South – East (d) South – West

40. Pointing to a photograph, a Man said “His Mother husband’s sister is my aunt”. Then what is relation
between a man and he?
(a) Son (b) Uncle
(c) Nephew (d) Brother

41. Pointing to old man Kailash said “his son is my son’s uncle” How is kailash is related to old man.
(a) Brother (b) Either son (or) son-in-law
(c) Father (d) Grand Father

42. Five boys A, B, C, D, E are sitting in a row A is to the right of B and E is to the left of B but to the right
of C. A is to the left of D who is second from the left end?
(a) D (b) A
(c) E (d) B

43. 5 children are sitting in a row. S is sitting next to P but not T.K is sitting next to R.K is sitting on
extreme end. T is not sitting next to K. Who are sitting adjacent to S.
(a) K & P (b) R & P
(c) Only P (d) P & T

44. Four girls are seated for a photograph. Shikha is left of Reena. Manju is to the right of Reena. Rita is
between Reena and Manju. Who is the second left in photograph.
(a) Reena (b) Manji
(c) Rita (d) Shikha

323 | P a g e
45. Statement I : Some fools are intelligent
Statement II: All intelligent is great
Conclusion I: Some fools are great
Conclusion II: All greats are intelligent
(a) Conclusion I follows (b) Conclusion II follows
(c) Neither I nor Ii follows (d) Either I nor Ii follows

46. Statement I: Sohan is good sports man


Statement II: Sports man is healthy.
Conslusion I: Sohan is healthy
Conclusion II: All sports men are good.
(a) Conclusion I follows (b) Conclusion II follows
(c) Neither I nor II follows (d) Either I nor II follows

47. Series is continuous.


(a) Open ended (b) Exclusive
(c) Close ended (d) Unequal call intervals

48. Which of the following graph is suitable for cumulative frequency distribution?
(a) Ogives (b) Histogram
(c) G.M (d) A.M

49. Histogram is used for finding


(a) Mode (b) Mean
(c) First Quartile (d) None

50. Ogive graph is used for finding


(a) Mean (b) Mode
(c) Median (d) None

51. Histogram can be shown as


(a) Ellipse (b) Rectangle
(c) Hyperbola (d) Circle

52. The AM of 15 observations is 9 and the AM of first 9 observations is 11 and then AM of remaining
observations is
(a) 11 (b) 6
(c) 5 (d) 9

53. In a moderately skewed distribution the values of mean & median are 12 & 18 respectively.
The value of mode is
(a) 6 (b) 12
(c) 15 (d) 30

54. Which of the following is positional average?


(a) Median (b) GM
(c) HM (d) AM

324 | P a g e
55. For the distribution
X 1 2 3 4 5 6
F 6 9 10 14 12 8
The value of median is
(a) 3.5 (b) 3
(c) 4 (d) 5

56. For a symmetric distribution


(a) Mean = Median = Mode (b) Mode = 3 Median –2 Mean
(c) Mode = 1/3 Median = ½ (d) None

57. The sum of mean and SD of a series is a+b, if we add 2 to each observation of the series then the sum
of mean and SD is
(a) a + b + 2 (b) 6 + a + b
(c) 4 + a – b (d) a + b + 4

58. Given that


X –3 –3/2 0 3/2 3
Y 9 9/4 0 9/4 9
The Karipeason’s coefficient of correlation is
(a) Positive (b) Zero
(c) Negative (d) None

59. If  2 100 and coefficient of variation = 20% then x mean is…………. 


(a) 60 (b) 70
(c) 80 (d) 50

60. Coefficient of quartile deviation is ¼ then Q3/Q1 is


(a) 5/3 (b) 4/3
(c) ¾ (d) 3/5

61. Standard deviation is ________ times of MD x QD


(a) 2/3 (b) 4/5
15 8
(c) (d)
8 15
62. SD of first five consecutive natural numbers is
(a) 10 (b) 8
(c) 3 (d) 2

63. The Q.D. of 6 numbers 15, 8, 36,40,38,41 is equal to


(a) 12.5 (b) 25
(c) 13.5 (d) 37

325 | P a g e
64. Given the following series:
X 10 13 12 15 8 15
Y 12 16 18 16 7 18
The rank correlation coefficient r =
2 m(m2  1)  3 m(m2  1) 
3
6 d   6  d 2   

i 1 12  i 1 12 
(a) 1  (b) 1 
m(n 2  1) n(n2  1)
2 n(n 2  1)
(c) 1 6d 2   (d) None of these
i  1 12

2
65. Find the probable error if r = and n = 36.
10
(a) 0.6745 (b) 0.06745
(c) 0.5287 (d) None

66. If the regression line of y on x is given by Y = x + 2 and Karlpearson’s coefficient of correlation is 0.5 then
y 2
__________________
x 2
(a) 3 (b) 2
(c) 4 (d) None

67. A.M. of regression coefficients is


(a) Equal to r (b) Greater then or equal to r
(c) Half of r (d) None of these

68. If a coin is Tossed 5 times then the probability of getting Tail and Head occurs alternatively is
1 1
(a) (b)
8 16
1 1
(c) (d)
32 64

69. According to bayee’s therom,


P (E ) P ( A / E )
P (E / A)  n k k
k
 P (E ) P ( A / E )
i 1 i i
(a) E1, E2 ................. are mutually exclusive (b) P(E/At), P(E/A2)……………..are equal to 1
(c) P(At/E), P(A2/E). .............. are equal to 1 (d) A & Ei ‘s are disjoint sets.

70. If mean and variance are 5 and 3 respectively then relation between p & q is
(a) p > q (b) p < q
(c) p = q (d) p is symmetric

326 | P a g e
71. 4 coins were tossed 1600 times. What is the probability that all 4 coins do not turn head upward at a
time?
(a) 1600e–100 (b) 1000e–100
(c) 100e –1600 (d) e–100

72. For a normal variable, if the first moment about 4 is 6, then the A.M is
(a) 1.5 (b) 2
(c) 10 (d) 24

73. If Y > x then mathematical expectation is


(a) E(X) > E(Y) (b) E(X) < E(Y)
(c) E(x) = E(Y) (d) E(X) . E(Y) = 1

74. The prices and quantities of 3 commodities in base and current years are as follows:
P0 P1 q0 q1
12 14 10 20
10 8 20 30
8 10 30 10
The Laspayer price index is
(a) 118.13 (b) 107.14
(c) 120.10 (d) None

75. The cost of living index numbers in years 2015 and 2018 were 97.5 and 115 respectively. The salary of
a worker in 2015 was Rs. 19500. How much additional salary was required for him in 2018 to maintain
the same statement of living as in 2015?
(a) Rs. 3000 (b) 4,000
(c) 3,500 (d) 4,500

76. Which is called an ideal index number?


(a) Laspayer’s index number (b) Pasche’s index number
(c) Fisher’s index number (d) Marshall Edge worth index number

77. Trend in semi average is


(a) Linear (b) Parabola
(c) Exponential (d) None of these

78. The most commonly used mathematical method for finding secular trend is
(a) Moving average (b) Semi averages
(c) Least squares (d) None of these

79. In Semi averages method, if the number of values is odd then we drop:
(a) First value (b) Last value
(c) Middle value (d) Middle two values

327 | P a g e
ANSWER KEY
1 A 11 A 21 A 31 C 41 B 51 B 61 C 71 D

2 C 12 C 22 C 32 B 42 C 52 B 62 D 72 C

3 C 13 A 23 B 33 D 43 D 53 D 63 C 73 B

4 B 14 B 24 D 34 C 44 C 54 A 64 B 74 B

5 C 15 B 25 C 35 C 45 A 55 C 65 B 75 C

6 D 16 B 26 A 36 B 46 A 56 A 66 C 76 C

7 B 17 A 27 A 37 C 47 B 57 A 67 B 77 A

8 D 18 C 28 D 38 C 48 A 58 B 68 B 78 B

9 C 19 B 29 A 39 B 49 A 59 D 69 A 79 C

10 B 20 D 30 C 40 D 50 C 60 A 70 B

328 | P a g e
CA FOUNDATION – NOVEMBER 2019
QUESTION PAPER
BUSINESS MATHEMATICS, LOGICAL REASONING & STATISTICS
1. The two numbers are in ratio 3 : 4. The difference between their squares is 28. Find the greater
number.
(a) 12 (b) 8
(c) 16 (d) 10
1
 9 n  2 / 4 . 3.3n n
2. If  
n
 3. 3 
(a) 1 (b) 3
(C) 9 (d) 27

 
   
1 126  1 
3. If x  3  then  x   x 
3  42   2 3
 x 
 3 
5 6
(a) (b)
6 5
2 3
(c) (d) 
3 5
1
4. If x  then the value of the expression x2– 10x + 1 is
5 2 6
(a) 0 (b) 10
(c) 26–12 2 (d) 15  3

5. log0.01 (10,000) = x; Find the value of x?


(a) 1 (b) -2
(c) -4 (d) 2

6. logxy2 – logy = log (x+y) Find the value of y in term of x


x
(a) x-1 (b)
x 1
x
(c) (d) x+1
x 1

7. Find the root of the equations. if 4x. 8y = 128 and 3x/27y = 1/3
(a) 2, 1 (b) -2, 1
(c) 2, -1 (d) 1, 2

329 | P a g e
8. The three roots of equation is. x3+9x2–x–9=0
(a) 1, -1, -9 (b) 1, -1, 9
(c) 1, 1, 9 (d) -1, -1, -9

9. Find the value of K so that x =2 is a root of the equation 3x2 – 2kx + 5 = 0


(a) 17/4 (b) 4/17
(c) –17/4 (d) –4/17

10. The solutions of the set of in equations 2x+y > 12, 5x + 8y > 74, x + 6y > 24, x > 0, y > 0 are
 126 23   126 23 
(a) (24,. 0),  ,  , ( 2, 8) (0, 12 ) (b) (0, 24), (2, 8), (0, 12),  , 
 11 11   11 11 
(c) (8, 4), (2, 8), (0, 12), (0, 24) (d) (8,4), (0, 0) (0,6) (2,0)

11. The present value of a scooter is Rs. 7290. The rate of depreciation is 10%. What was its value 3
years ago?
(a) 10,000 (b) 10010
(c) 9990 (d) 12000

12. The difference between compound interest, compounded semi annually and simple interest on Rs.
400 at 10% p.a. for one year.
(a) Rs. 1 (b) Rs. 28
(c) Rs. 35 (d) Rs. 40

13. If the interest of a money is equal to its one by nine, the rate of interest and time are equal then find
rate of interest is.
(a) % (b) 41/2 %
(c) 3% (d) 3.5%

14. 1/7 of a money is deposited at 4% per annum , ½ of a money deposited at 5% per annum and the
remaining at the rate of 6%, then total interest gained Rs. 730 find deposit amount is
(a) Rs. 14000 (b) Rs. 15500
(c) Rs. 12800 (d) Rs. 14500

15. Ram deposited Rs. 12000 in a bank at 10% per annum and remaining amount deposit in other bank
at 20% per annum. if he received interest according to 14% per annum find the Ram's amount.
(a) Rs. 20000 (b) Rs. 22000
(c) Rs. 30000 (d) Rs. 25000

16. In how much time the S.I. on a certain sum becomes 0.125 times to its principle at 10% p.a. is
(a) 1.00 yrs (b) 1.25 yrs
(c) 1.50 yrs (d) 2.00 yrs

17. If the difference between interest received by two persons A and B on the same sum of Rs. 1500
for 3 years Rs. 18. Then what is the difference between the two rates of interest.
(a) 1% (b) 2.5%
(c) % (d) 0.4%

330 | P a g e
18. In what time will a sum Rs. 800 amounts to Rs. 882 at 5% p.a. compounded annually
(a) 1 yrs (b) 2 yrs
(c) 3 yrs (d) 4 yrs

19. If the compound interest on a certain sum for 2 years at 3% p.a. is Rs. 1015. What would be the simple
interest on the sum at the same rate and same time is
(a) 1005 (b) 1010
(c) 1000 (d) 1003

20. The useful life of a machine whose cost is Rs. 10,000 is 10 years. If it depreciates at 10% p.a.
then the scrap value of the machine is.
(a) 3486.70 (b) 3158.30
(c) 3500 (d) 7033

21. Find the effective rate of interest if an amount of Rs. 30,000 deposited in a bank. For 1 year at the
rate of 10% p.a. compounded semi annually.
(a) 10.05% (b) 10.10%
(c) 10.20% (d) 10.25%

22. The present population of a town is 25,000. If it grows at the rate of 4%, 5%, 8% during 1 st year, 2nd
year, 3rd year respectively. Then find the population after 3 years.
(a) 29,484 (b) 29,844
(c) 29,448 (d) 28,944

23. An amount 35000 with the rate of interest is 7% per annum, it is compounded on a monthly basis,
then tell the effective rate of interest.
(a) 7.22% (b) 7.64%
(c) 7.0%. (d) 7.5%

24. Find the future value of annuity of Rs. 500 is made annually for 7 years interest rate of 14%
compound at annually. Given that (1.14) 7 = 2.5023
(a) Rs. 5635.35 (b) Rs. 5365.35
(c) Rs. 6535.35 (d) Rs. 6355.35

25. How many number divisible by 5 of 6 digit can be made from the digit 2, 3, 4, 5, 6, 7
(a) 120 (b) 600
(c) 240 (d) none

26. 5 boys and 3 girls are to be seated together such that no two girls are together
(a) 14,400 (b) 2400
(c) 720 (d) None of these

27. Out of 6 Boys & 4 girls, Find the number of ways for selecting 5 member committee in which there is
exactly two girls ?
(a) 120 (b) 1440
(c) 720 (d) 71

28. If np5 :np3 is 2:1 than value of n is


(a) 2 (b) -5
(c) -2 (d) 5
331 | P a g e
29. In the series 25, 5, 1,………………1/3125 which term is 1/3125?
(a) 8th term (b) 9th term
(c) 15 term
th (d) None of these

30. The sum of five terms of AP is 75 find the 3rd term is.
(a) 20 (b) 30
(c) 15 (d) None of these

31. (c+a-b)/b, (a+b-c)/c, (b+c-a)/a are in AP then a,b,c are in


(a) AP (b) GP
(c) HP (d) None of these

32. The sum of series 1/2+1/32+1/23+1/34…………………………..up to infinity is


(a) 25/24 (b) 19/24
(c) 1/12 (d) None of these

33. f(x) = x – 1 then find f -1 (1)


(a) 1 (b) 0
(c) -2 (d) 2

34. f(x) = f(x–1)+f(x–2) if f (0)= 0, f(1) = 1, x = 2, 3, 4,…………………then what is f (7)


(a) 8 (b) 13
(c) 3 (d) 5

35. f(x) = 2x3+1 then what is f–1(x) options


 x  1
1/ 3

(a) ½ (x–1)1/3 (b)  


 2 
 x  1
1/ 2

(c)   (d) None of these


 2 

36. Find the value of dy/dx if y=xx


(a) xxlogex (b) 1+logx
(c) ylogx (d) none of these

37. Find the value of  xe


x
dx
(a) ex(x-1)+c (a) ex(2x-1)+c
(b) ex(x-1) (d) None of these

38. If f(x) = a (x2 + x +1)2 and f1 (–1) = – 6 then the value of a =


(a) 1 (b) 2
(c) 3 (d) 4

39.  (4x  5)
6
Find the value of dx is equal to
(a) 1/7(4x+5)7+ c (b) 1/28(4x+5)7+ c
(c) 1/4(4x+5)7+ c (d) None of these

332 | P a g e

1
40. (2 x 3  x 3 ) dx 
1
(a) 14 (b) 104
3 4
2x x 4
(c)  (d)
3 4 3
41. Find odd one out; 1, 5, 14, 30, 51, 55, 91, ?
(a) 14 (b) 55
(c) 51 (d) 91

42. Find odd one out ; 5, 10, 17, 27, 37, 50, 65 ?
(a) 17 (b) 27
(c) 37 (d) 65

43. Find the missing figures; 4, 16, 36, 64, 100 ?


(a) 92 (b) 121
(c) 144 (d) 169

44. If "SYSTEM" be coded 131625; then 'TERMS' may be coded as :—


(a) 62251 (b) 62451
(c) 64951 (d) 62415

45. If "MADRAS" written as "NBESBT"; then "DELHI" may be coded as:—


(a) EMTIF (b) EFMIJ
(c) JEMFT (d) EEMJI

46. A man started walking West. He turned right. Then again and finally turned left. Towards which
direction was he walking now?
(a) North (b) South
(c) West (d) East

47. A starts form a point and walk 2 km north, then turns left and walk 1 km, then again turns left and
walks 2 km. Point out the direction in which he is going now?
(a) East (b) West
(c) North (d) South

48. A man is moving on cycle and move 4 km South then turns left and move 2km and turns again to
the right to move to go more. In which direction is he moving?
(a) North (b) West
(c) East (d) South

49. If Mohan travels towards north from his house then turn to left, then to south coving equal
distance. In each direction to reach Sohan's house. In which direction Mohan's house is form
Sohan's house now?
(a) East (b) South
(c) North (d) West

333 | P a g e
50. If Shyam sees the rising sun behind the tower and setting sun behind the Railway station from his
house. What is the direction of tower from the Railway station?
(a) South (b) North
(c) West (d) East

51. A man takes his dog for a walk whose house is facing East. He walks first towards west and then
walks towards south. In which direction he has to walk now to reach home?
(a) North East (b) West
(c) South (d) North West

52. 5 persons are standing in a line one of the 2 persons at the extreme ends is a professor and the
other a business man. An advocate is standing to the right of student. An author is to the left of the
business man. The student is standing between the professor and advocate. Counting from the left.
The author is at which place?
(a) 2nd (b) 3rd
(c) 4th (d) None of these

53. Parikh is sitting between narendra and babita, charu is to the left of babita, pankaj she's sitting
between charu and ashma they all sitting around a circle facing the center then who is sitting to
the right of babita?
(b) Parikh (b) Ashma
(c) Charu (d) Narendr

Direction (Q. 54-57) Read the following information carefully to answer the give questions.
Six members of a family namely A, B, C, D, E and F are travelling together. ‘B’ is the son of C but C is
not the mother of B. A and C are married couple. E is the brother of C. D is the daughter of A. F is the
brother of B.

54. How many male members are there in the family?


(a) 3 (b) 2
(c) 4 (d) 1

55. How many children does A have


(a) 1 (b) 2
(c) 3 (d) 4

56. What is the relation of E to D


(a) Uncle (b) Brother
(c) Father (d) None of these

57. Who is the mother of B?


(a) C (b) D
(c) F (d) A

Direction (Q. 58-60)


In each question below are three statements followed by four conclusions numbered I, II, III and IV.
You have to take the three given statements to be true even, if they seem to be at variance with
commonly known facts and then decide which of the given conclusions logically follows from the three
statements disregarding commonly known facts:
334 | P a g e
58. Statements:
I. Some fruits are rivers II. Some rivers are boats
III. Some rivers are fruits IV. Some flowers are fruits
(a) Only I & III follows (b) Only II & III follows
(c) Only II & IV follows (d) All follows

59. Statements:
All buildings are Rings
All papers are buildings
All dogs are papers
Conclusions :
I. All dogs are rains II. Some papers are rains
III. Some rains are buildings IV. Some rains are papers
(a) Only I & II follows (b) Only II & III follows
(c) Only I, III, IV follows (d) All follows

60. Statements:
Some flowers are rods
Some rods are doors
Some doors are houses
Conclusions :
I. Some houses are flowers II. Some doors are flowers
III. Some flowers are doors IV. No house is flower
(a) Either I or II follows (b) Either I or IV follows
(c) Only II & III follows (d) Only I & IV follows

61. Histogram is used for presentation of the following type of series.


(a) Time Services (b) Continuous Frequency Series
(c) Discrete Series (d) Individual Series

62. The graphical representation of cumulative frequency distribution is called–


(a) Histogram (b) Pie Chart
(c) Frequency Polygon (d) Ogive
63. No. of 0 1 2 3 4 5 6 7
Accidents
Frequency 36 27 33 29 24 27 18 9

In how many cases 4 or more accidents occur ?


(a) 96 (b) 133
(c) 78 (d) 54

64. The difference between upper limit and lower limit of a class is called:
(a) Class interval (b) Class boundaries
(c) Mid-value (d) Frequency

335 | P a g e
m
65.  (x  x )  ?
i 1
(a) 1 (b) 0
(c) –1 (d) None of these

66. The median of the following frequency distribution is equal to


X: 5 7 9 12 14 17 19 21
Y: 6 5 3 6 5 3 2 4
(a) 6 (b) 12
(c) 13 (d) 14

67. Find median from the following data:


Marks 0–10 10-30 30-60 60-80 80-90
No. of 5 15 30 8 2
students
(a) 8 (b) 30
(c) 40 (d) 45

68. Find the mode from the following data:


Class : 3–6 6–9 9–12 12–15 15–18 18–21 21–24
Frequency 2 5 10 23 21 12 3
(a) 23 (b) 13.3
(c) 12.6 (d) 14.6

69. Find the mode of the following distribution?


Class : 0–7 7–14 14–21 21–28 28–35 35–42 42–49
Frequency 19 25 36 72 51 43 28
(a) 24.3 (b) 25.4
(c) 72 (d) 21

70. The arithmetic mean of two numbers is 30 and geometric mean is 24 find the two number
(a) 12 and 48 (b) 14 and 46
(c) 10 and 50 (d) 16 and 44

71. Sum of the squares of deviations is minimum when deviations are taken from
(a) Mean (b) Median
(b) Mode (d) An arbitrary value

72. What will be the probable value of mean deviation when Q3 = 40 and Q1= 15?
(a) 17.50 (b) 18.75
(c) 15.00 (d) 16.00

73. Find the mean deviation about mean of 4,5,6,8,3


(a) 5.20 (b) 7.20
(c) 1.44 (d) 2.33

336 | P a g e
74. The mean and coefficiente of variance is 20 and 80 find the value of variance
(a) 16 (b) 256
(c) 36 (d) none

75. Find SD of 1, 2, 3,4, 5,6, 7, 8,9


20 81
(a) (b)
3 3
20
(c) (d) None of these
5

76. The standard deviation for the set of numbers 1,4,5,7,8, is 2.45 nearly. If 10 is added to each number
then new standard deviation is
(a) 24.45 (b) 12.45
(c) 2.45 (d) 0.245

77. If every observation is increased by 5 then:


(a) SD increase by 5 (b) MD increased by 5
(c) QD increases by 5 (d) None affected

78. For a given distribution the arithmetic mean is 15 and the standard deviation is 9 then the
coefficient of variation is equal to
15 15
(a) x 100 (b)
9 9
9 9
(c) (d) x 100
15 15

79. The mean of a distribution is 14 and the standard deviation is 5. What is the value of the coefficient
of variation?
(a) 60.4% (b) 70%
(c) 35.7% (d) 27.8%

80. If the equation of the two regression lines are 2x – 3y = 0 and 4y –5x = 8 then the correlation
coefficient between x and y is equal to
15 8
(a) (b)
8 15
6 1
(c) (d)
15 15

81. Find correlation coefficient


X54321
Y12345
(a) 1 (b) -1
(c) 0 (d) None of these

337 | P a g e
82. If scatter diagram from a line move from lower left to upper right corner then the correlation is.
(a) Perfect positive (b) Perfect negative
(c) Simple positive (d) No correlation

83. Consider to regression line 3x+2y=26, 6x+y=31 find the correlation coefficient between x and y
(a) 0.5 (b) -0.5
(c) 0.25 (d) -0.25

84. If correlation coefficient between x and y is 0.5 then Find the corlication coefficient between 2x –3
and 3–5y is
(a) 0.5 (b) –0.5
(c) 2.5 (d) –2.5

85. If two letters are taken at random from the word HOME, what is the Probability that none of the
letters would be vowels :
(a) 1/6 (b) 1/2
(c) 1/3 (d) 1/4

86. A bag contains 15 one rupee coins, 25 two rupee coins and 10 five rupee coins. If a coin is selected
at random from the bag, then the probability of not selecting a one rupee coin is :
(a) 0.30 (b) 0.70
(c) 0.25 (d) 0.20

87. The chance of getting sum 7 or 11 in a throw of 2 dice is


(a) 7/9 (b) 5/9
(c) 2/9 (d) None of these

88. In normal distribution what is the ratio of QD:MD:SD


(a) 12:10:15 (b) 15:10:12
(c) 10:15;12 (d) 10:12:15
2  2 ( x 3 ) 2
89. For a normal distribution e mean and standard deviation will be–

1
(a) 3, ½ (b) 3,
2
(c) 3, 2 (d) None of these

90. Area covered normal curve by (   3 )


(a) 68.28% (b) 95.96%
(c) 99.73% (d) 99.23%

91. If x is binomial variate with parameter 15 and 1/3 what is the value of mode of the distribution
(a) 5& 6 (b) 5.5
(c) 5 (d) 6

92. In poisson distribution which of the following is same.


(a) Mean and variance (b) Mean and SD
(c) Both (d) None of these

338 | P a g e
93. If for a binomial distribution B(n,p), ;n = 4 and also P(x=2) =3 P(x=3) then the value of P is equal to
9
(a) (b) 1
11
1 1
(c) (d)
3 9

94. Let x be a poisson random variable with parameter λ. Then p(x) is equal to
   
e e e e
(a) (b)
2 2
 
e 1 1 e
(c) (d)
2 2

95. For year 2015, price index was 267% with base year 2005. The percentage increase in price index
over base year 2005 is:
(a) 267% (b) 67%
(c) 167% (d) None of these

96. The value of the base time period serves as a standard point of comparison
(a) True (b) False
(c) Both (d) None of these

97. Fisher's ideal formula does not satisfy___________test?


(a) Unit test (b) circular test
(c) Time reversal test (d) None of these

98. In semi average method, if the no. of value is odd then we drop.
(a) First term (b) Last term
(c) Middle term (d) None of these

99. The sale of cold drink could go up in summers and go down in the winters is an example of
(a) Secular trend (b) Seasonal variation
(c) Cyclical variation (d) Irregular variation

100. Seasonal variations can occurrence within a period of


(a) 4 year (b) 3 year
(c) 1 year (d) 9 year

339 | P a g e
ANSWER KEY
1 B 21 D 41 C 61 B 81 B
2 D 22 A 42 B 62 D 82 A
3 A 23 A 43 C 63 C 83 B
4 A 24 B 44 B 64 A 84 B
5 B 25 A 45 B 65 B 85 A
6 C 26 A 46 A 66 B 86 B
7 A 27 A 47 D 67 C 87 C
8 A 28 D 48 D 68 D 88 D
9 A 29 A 49 A 69 B 89 A
10 A 30 C 50 D 70 A 90 C
11 A 31 C 51 A 71 A 91 C
12 A 32 B 52 C 72 C 92 A
13 A 33 D 53 A 73 C 93 C
14 A 34 B 54 C 74 B 94 D
15 A 35 B 55 C 75 A 95 C
16 B 36 A 56 A 76 C 96 A
17 D 37 A 57 D 77 D 97 B
18 B 38 C 58 C 78 D 98 C
19 A 39 B 59 D 79 C 99 B
20 C 40 D 60 B 80 B 100 C

340 | P a g e
CA FOUNDATION – DECEMBER 2020
QUESTION PAPER
BUSINESS MATHEMATICS, LOGICAL REASONING & STATISTICS
13
1. The average cost function of a good is 2Q + 6 + , where Q is the quantity produced. The
Q
approx cost at Q = 15 is____________.
(a) 36
(b) 42
(c) 66
(d) 130

2. Find the missing value in the series 0, 2, 3, 6, 10, 17, 28, ___?___, 75,
(a) 46
(a) 58
(c) 48
(d) 54

3. On what sum will the compound interest at 5% per annum for 2 years compounded annually be
Rs. 3,280?
(a) Rs. 32,000
(b) Rs. 16,000
(c) Rs. 48,000
(d) Rs. 64,000

4. What sum of money will produce Rs. 42,800 as an interest in 3 years and 3 months at 2.5% p.a.
simple interest?
(a) Rs. 5,26,769
(b) Rs. 3,78,000
(c) Rs. 4,22,000
(d) Rs. 2,24,000

5. Two finite sets respectively have x and y number of elements. The total number of subsets of
the first is 56 more than the total number of subsets of the second. The values of x and y are
respectively:
(a) 6 and 3
(b) 4 and 2
(c) 2 and 4
(d) 3 and 6

341 | P a g e
6. The number of items in the set A is 40; in the Set B is 32; in the Set C is 50; in both A and B is 4;
in both A and C is 5; in both B and C is 7; in all the sets is 2. How many are in only one set ?
(a) 110
(b) 65
(c) 106
(d) 84

7. A fruit basket contains 7 apples, 6 bananas and 4 mangoes. How many selections of 3 fruits can
be made so-that all 3 are apples ?
(a) 35 ways
(b) 120 ways
(c) 168 ways
(d) 70 ways

8. Out of 7 boys and 4 girls a team of a debate club of 5 is to be chosen. The number of teams
such that each team includes at least one girl is-
(a) 439
(b) 429
(c) 419
(d) 441

9. If P n4 = 20 . P n2 , where P denotes the number of permutations n = _________.


(a) 2
(b) 4
(c) 5
(d) 7

10. From a group of 8 men and 4 women, 4 persons are to be selected to form a committee so that
at least 2 women are there on the committee. In how many ways can it be done ?
(a) 168
(b) 201
(c) 202
(d) 220

11. Three numbers in G.P. with their sum 130 and their product 27,000 are-
(a) 10, 30, 90
(b) 90, 30, 10
(c) (a) & (b) both
(d) 10, 20, 30

342 | P a g e
12. The set of cubes of the natural number is –
(a) a null set
(b) a finite set
(c) an infinite set
(d) a finite set of three numbers

13. Divide 69 into three parts which are in A.P. and are such that the product of the first two parts
is 460.
(a) 21, 23, 25
(b) 20, 23, 26
(c) 19, 23, 27
(d) 22, 23, 24

14. The inverse function f-1 of f(y) =3y is –


(a) y/3
(b) 1/3y
(c) -3y
(d) 1/y

15. The 20th term of arithmetic progression whose 6th term is 38 and 10th term is 66 is_________
(a) 118
(b) 136
(c) 178
(d) 210

16. Find the missing value in 3/8, 8/19, 18/41, ?, 78/173


(a) 37/84
(b) 40/87
(c) 39/86
(d) 38/85

17. Find the wrong term in :


G4T, J10R, M20P, P43N, S90L
(a) M20P
(b) P43N
(c) J10R
(d) G4T

18. If HEALTH is written as IFBMUI, then how will NORTH be written in that code ?
(a) OPSUI
(b) GSQNM
(c) FRPML
(d) IUSPO

343 | P a g e
19. Which of the following is odd one ?
6, 9, 15, 21, 24, 26, 30
(a) 30
(b) 24
(c) 26
(d) 9

20. If y – x (x – 1)(x – 2) then dy/dx is


(a) 3x2 – 6x + 2
(b) -6x
(c) 6x + 2
(d) 3x – 6

21. One day, Ram left home and cycled 10 km southwards, turned right and cycled 5 km and turned
right and cycled 10 km and turned left and cycled 10 km. How many kilometres will he have to
cycle to reach his home straight ?
(a) 10
(b) 15
(c) 20
(d) 25

22. A main is facing west. He turns 45 degrees in the clockwise direction and then another 180
degrees in the same direction and then 270 degrees in the anticlockwise direction. Which
direction is he facing now?
(a) South – West
(b) North – West
(c) West
(d) South

23. Five girls : G, H, I, J, K are sitting in a row facing south not necessarily in the same order. H is
sitting between G and K: 1 is immediate right to K: J is immediate left to G. Which of the
following is true ?
(a) J is third to the left of K
(b) G is second to the left of I
(c) H is to the right of K
(d) H is to the left of G

24. Eight friends I, J, K, L, M, N, O, and P are sitting in a circle facing the centre. J is sitting between
O and L; P is third to the left of 3 and second to the right of I; K is sitting between I and O, and J
& M are not sitting opposite to each other. Which of the following statement is NOT correct?
(a) K is sitting third to the right of L
(b) I is sitting between K and N
(c) L and I are sitting opposite to each other
(d) M is sitting between N and L

344 | P a g e
25. A man can walk by having long, medium and short steps. Sixty meters can be covered by 100
long steps; 100 meters can be covered by 200 medium steps and 80 meters can be covered by
200 short steps. He walks taking 5000 long steps; then turn to his left and walk taking 6,000
medium steps. He turn to his right and walks by taking 2,500 short steps. How far (in meters) is
he away from his starting point?
(a) 4,000 m
(b) 5,000 m
(c) 6,000 m
(d) 7,000 m

26. Read the following information carefully and answer the questions below. In a family of six
persons. A, C, E, F, I, K, there are two married couples, G is grandmother of A and mother C, E is
wife of C and mother of K. K is the granddaughter of I. What is E to A?
(a) Daughter
(b) Mother
(c) Grandmother
(d) Aunt

27. Pointing towards a person, a man said to a woman, "His mother is the only daughter of you
father." How is the woman related to that person?
(a) Mother
(b) Daughter
(c) Sister
(d) Wife

28. Vicky introduces John as the son of the only brother of his father's wife. How is Vicky related to
John?
(a) Son
(b) Cousin
(c) Uncle
(d) Brother

29. A man said to a lady "Your mother's husband's sister is my aunt". How is the mean related to
the lady?
(a) Father
(b) Grandfather
(c) Son
(d) Brother

30. If you are facing north – east and move 10 m forward, turn left and move 7.5 m. then you are
(a) North of your initial position
(b) South of your initial position
(c) East of you initial position
(d) None of the option
345 | P a g e
31. Statements : Some cars are towns, some towns are flowers. All plants are flowers.
Conclusions :
(I) No plant is a car.
(II) Some cars are flowers.
Choose the correct answer from the following :
(a) If only conclusion (I) follows.
(b) If only conclusion (II) follows.
(c) If either (I) or (II) follows.
(d) If neither (I) nor (II) follows.

32. Statements : All huts are flats. No flat is a building. All buildings are cottages.
Conclusions :
(III) No hut is a cottage.
(IV) Some buildings are huts.
Choose the correct answer from the following :
(a) If only conclusion (I) follows.
(b) If only conclusion (II) follows.
(c) If either conclusion (I) or (II) follows.
(d) If neither conclusion (I) nor (II) follows.

33. Rahim faces towards north, Turning to his right, he walks 25 m. He then turns to his left and
walks 30 m. text, he moves 25 m to his right. He then turns to his right again and walks 55 m.
Finally, he turns to the right and moves 40 m. In which direction is he now from her starting
point ?
(a) South-West
(b) South
(c) North-West
(d) South-East

34. Pointing to a lady. A said 'that women is any nephew's maternal grandmother". How is that
woman related to A's sister who has no sister?
(a) Cousin
(b) Son-in-law
(c) Mother
(d) Mother-in-law

35. Pointing out to a lady, Sahil said, "She is the daughter of the woman who is the mother of the
husband of my mother." Who is the lady to Sahil?
(a) Aunt
(b) Sister
(c) Daughter
(d) Sister-in-law

346 | P a g e
36. Sweetness of a sweet dish is –
(a) An attribute
(b) A discrete variable
(c) A continuous variable
(d) A variable

37. Five auditors of your firm had reported their incomes. You computed their average and
obtained Rs. 67,000 per month. You now state that the average income per month of all the
auditors of your firm is Rs. 67,000. This is an example of__________ statistics.
(a) Descriptive
(b) Inferential
(c) Detailed
(d) Non-detailed

38. The harmonic mean of A and B is 1/3 and harmonic mean of C and D is 1/5. The harmonic mean
of A, B, C and D is
(a) 8/15
(b) 1/4
(c) 1/15
(d) 5/3

39. Statistics cannot deal with__________data.


(a) Quantitative
(b) Qualitative
(c) Textual
(d) Attribute

40. Given that for a distribution the mean, median and mode are 23, 24 and 25.5. It is most likely
that the distribution is________skewed.
(a) Positively;
(b) Symmetrically
(c) Asymptotically
(d) Negatively

41. Choose the one from the following which is not a measure of scatter in data.
(a) Half range
(b) Average of first and third quartiles
(c) Average of squared deviations
(d) Average of absolute deviations

347 | P a g e
42. Two years ago, a term of four persons had an average age of 14. Now, a new member is added
to the team and the average age of the team is 17. What is the age of the new member ?
(a) 17
(b) 19
(c) 21
(d) 23

43. The number of types of cumulative frequency is –


(a) 1
(b) 2
(c) 3
(d) 4

44. For open – end classification, which of the following is the best measure of central tendency –
(a) AM
(b) GM
(c) Median
(d) Mode

45. The numbers of times city had mild, medium and heavy rains, respectively are 17, 10 and 5.
Which of the following represent it ?
(a) (17, 10, 5)
(b) Quantitative
(c) Continuous
(d) Average

46. A fire engine rush to a place of fire at a speed of 70 kmph and after the work, it returned to the
base at a speed of 35 kmph. The average speed per hour per direction is obtained
as_______speeds.
(a) Average of
(b) Harmonic means of
(c) Geometric mean of
(d) Half of harmonic mean of

47. The__________are used when one wants to visually examine the relationship between two
variables.
(a) Bar graphs
(b) Pie charts
(c) Line charts
(d) Scatter plots

348 | P a g e
48. When data are classified according to one criterion, then it is called___________ classification.
(a) Quantitative
(b) Qualitative
(c) Simple
(d) Factored

49. Which measure of dispersion is based on the absolute deviations only ?


(a) Range
(b) Standard deviation
(c) Mean deviation
(d) Quartile deviation

50. A cricketer's run scores of last ten test matches are available. Statistics cannot be used to find
the
(a) Least score
(b) Largest score
(c) Best score
(d) Median score

51. The person A speaks times and B in 80% of times. In what percentage of times are they likely to
contradict each other in narrating the same incident ?
(a) 0.60
(b) 0.6
(c) 0.65
(d) 0.35

52. A basket contains 15 apples, 24 mangoes and 10 bananas. If a fruit is selected at random from
the basket, then the probability of not selecting as apple is-
(a) 0.20
(b) 0.25
(c) 0.30
(d) 0.70

53. If an unbiased coins is tossed twice, the probability of obtaining at least one fail is-
(a) 1
(b) 0.5
(c) 0.75
(d) 0.25

349 | P a g e
54. When three fair dice are rolled simultaneously, what is the probability of getting a number on
third die greater then the sum of numbers appeared on the first two dice?
(a) 12/216
(b) 24/216
(c) 36/216
(d) 48/216

55. Two fair dice are rolled simultaneously. What is the probability of getting a sum of the
outcomes from the dice is a multiple of 3 ?
(a) 4/36
(b) 12/36
(c) 6/36
(d) 9/36

56. If we change the parameter(s) of____________distribution, the shape of the probability curve
does not change.
(a) Normal
(b) Binomial
(c) Poisson
(d) Non-Gaussian

57. Which one of the following is uniparametric distribution?


(a) Poisson
(b) Normal
(c) Binomial
(d) Hyper geometric

58. For a Poisson distributed variable X, we have P[X = 7] = 8 . P[X = 9], the mean of the distribution
is
(a) 3
(b) 4
(c) 7
(d) 9

59. The quartile deviation of a normal distribution, mean 10 and standard deviation 4, is-
(a) 23.20
(b) 54.24
(c) 0.275
(d) 2.70

350 | P a g e
60. If the probability for success in a binomial distribution is less than one-half, then the binomial
distribution
(a) is skewed to right
(b) is skewed to left
(c) has two modes
(d) has median at a point > mean + ½

61. Scatter diagram does not help us to


(a) Identify whether variables are correlated or not
(b) find the type of correlation
(c) determine the linear or nonlinear type
(d) find the numerical value of the correlation coefficient

62. Which of the following is spurious correlation ?


(a) Negative correlation
(b) Correlation between 2 variables having no causal relation
(c) Bad relation between 2 variables
(d) Very low correlation between 2 variables

63. The weight for numbers 1, 2, ……… n is respectively 12, 22, 32, …… n2. The weighted harmonic
mean is_________.
(a) (2n + 1) / 6
(b) (2n + 1) / 4
(c) (2n + 1) / 3
(d) (2n + 1) / 2

64. Index numbers are expressed as_________.


(a) Ratios
(b) Squares
(c) Percentages
(d) Combinations

65. Census reports used as a source of data is___________data.


(a) Primary
(b) Secondary
(c) Organized
(d) Confidential

66. Two values yielded an arithmetic mean of 24 and a harmonic mean of 6. The geometric mean of
these values is__________.
(a) 8
(b) 12
(c) 14
(d) 16
351 | P a g e
67. Decomposition of time series is known as –
(a) Histogram
(b) Deterending
(c) Analysis of time series
(d) Historiagram

68. In time series seasonal variations can occur within a period of:
(a) One year
(b) Three years
(c) Nine years
(d) Five years

69. Fisher’s ideal index number does not satisfy__________test.


(a) Time reversal
(b) Circular
(c) Factor reversal
(d) Unit

70. If Laspeyre’s index number = 110, Fisher’s ideal index number is 109, then Paasche’s index
number is___________
(a) 118
(b) 110
(c) 109
(d) 108

71. A plotted time series shows a periodic variation such that the recurrence period is more than
one year. Hence the time series has__________variation.
(a) Irregular
(b) Seasonal
(c) Cyclical
(d) Long term

72. The covariance between two variable in


(a) Strictly positive
(b) Strictly negative
(c) Always zero
(d) Either positive or negative or zero

73. Which one of the following has Poisson distribution?


(a) The number of days to get a complete cure.
(b) The number of defects per meter on long roll of coated polythene sheet.
(c) The errors obtained in repeated measuring of the length of a rod.
(d) The number of claims rejected by an insurance agency.

352 | P a g e
74. A partially legible working sheet for the calculation of rank correlation coefficient revealed that
the coefficient of Rank correlation is 1/3 and the sum of squared differences of ranks is 80.
What is the number of observations ?
(a) 9
(b) 8
(c) 7
(d) 6

75. The fiftieth percentile can be computed from


(a) Mode
(b) Interquartile range
(c) Median
(d) Average of first and third quartiles

76. If a : b = 3 : 7, then 3a + 2b : 4a + 5b = ?
(a) 23 : 47
(b) 27 : 43
(c) 24 : 51
(d) 29 : 53

77. If loga 3 = 1/6, find the value of a


(a) 9
(b) 81
(c) 27
(d) 3

78. log 9 + log 5 is expressed as –


(a) log 4
(b) log 9/5
(c) log 5/9
(d) log 45

a b
79. If a : b = 9 : 4, then  ?
b a
3
(a)
2
2
(b)
3
6
(c)
13
13
(d)
6

353 | P a g e
80. The ratio of number of boys and the number of girls in a school is found to be 15 : 32. How
many boys and equal number of girls should be added to bring the ratio to 2/3?
(a) 19
(b) 20
(c) 23
(d) 27

81. The rational root of the equation 0 = 2p3 – p2 -4p + 2 is


(a) 2
(a) -2
(c) ½
(d) -1/2
82. Transpose of a row matrix is
(a) Column matrix
(b) Zero matrix
(c) Row matrix
(d) Diagonal matrix

83. If 2x2 – (a + 6)2x + 12a = 0, then the roots are


(a) 6 and a
(b) 4 and a2
(c) 3 and 2a
(d) 6 and 3a

 0  i 1 0 
84. If A3 =  4
 and A =
2 2
0 1 , where i = -1, then A =______________
  i 0   
 1 0 
(a)  0  1
 
 i 0 
(b)  0  i
 
0  1
(c)  i
 0 
0 i
(d) i
 0

85. Find the value of a from the following :


( 9 ) 5  ( 3 ) 7  ( 3 )  a
(a) 11
(b) 13
(c) 15
(d) 17

354 | P a g e
86. Solving equation m + m = 6/25 the value of m works out to
(a) 1/25
(b) 2/25
(c) 3/25
(d) 1

87. The ratio of principal and the compound interest value for three years (compounded annually)
is 216 : 127. The rate of interest is
(a) 0.1777
(b) 0.1567
(c) 0.1666
(d) 0.1588

88. An amount P becomes Rs, 5,100.5 and Rs. 5,203 after second and fourth years respectively, at
r% of interest per annum compounded annually. Thus, values of P and r are
(a) Rs. 4,000 and 1.5
(b) Rs. 5,000 and 1
(c) Rs. 6,000 and 2
(d) Rs. 5,500 and 3

89. A certain sum invested at 4% per annum compounded semi-annually amounts to Rs. 1,20,000
at the end of one year. Find the sum.
(a) 1,15,340
(b) 1,10,120
(c) 1,12,812
(d) 1,13,113

90. Solving equation 3g2 – 14g + 16 = 0, we get roots as-


(a) +5
(b) 0
(c) 8 and 2/3
(d) 2 and 8/3

91. Find the future value of annuity of Rs. 1,000 made annually for 7 years at interest rate of 14%
compounded annually. Given that 1.147 – 2.5023
(a) 10,730.7
(b) 5,365.35
(c) 8,756
(d) 9,892.34

355 | P a g e
92. Find the present value of Rs. 1,00,000 to be required after 5 years if the interest rate be 9%.
Given that 1.095 – 1.5386
(a) 78,995.98
(b) 64,994.20
(c) 88,992.43
(d) 93,902.12

93. A five-year annuity due has periodic cash flow of Rs. 100 each year. If the interest rate is 8%,
the future value of this annuity is given by
(a) (Rs. 100) x (Future value at rate 8% for 5 years) x (0.08)
(b) (Rs. 100) x (Future value at rate 8% for 5 years) x (1 - .08)
(c) (Rs. 100) x (Future value at rate 8% for 5 years) x (1 + .08)
(d) (Rs. 100) x (Future value at rate 8% for 5 years) x (1 / 0.08)

94. A person decides to invest Rs. 1,25,000 per year for the next five years in an annuity which gives
5% per annum compounded annually. What is the approx. future value? (use 1.055 = 1.2762, if
needed)
(a) 1,59,535
(b) 6,90,704
(c) 5,90,704
(d) 3,59,535
95. Find the compound interest if an amount of Rs. 50,000 is deposited in a bank for one year at
the rate of 8% per annum compounded semi annually
(a) Rs. 3080
(b) Rs. 4080
(c) Rs. 5456
(d) Rs. 7856
96. Which of the following statements is TRUE ? (Assume that the yearly cash flows are identical for
both annuities)
(a) The present value of an annuity due is greater than the present value of an ordinary
annuity.
(b) The present value of an ordinary annuity is greater than the present value of an annuity
due.
(c) The future value of an ordinary annuity is greater than the future value of an annuity
due.
(d) The future value of an annuity due is equal to future value of an ordinary annuity.

97. Rs. 2,500 is paid every year for 10 years to pay off a loan. What is the loan amount if interest
rate be 14% per annum compounded annually?
(a) 15,847.90
(b) 13,040.27
(c) 14,674.21
(d) 16,345.11
356 | P a g e
98. Suppose you deposit Rs. 900 per month into an account that pays 4.8% interest, compounded
monthly. How much money (rounded to nearest Rupee) will you get after 9 months ? (Use, if
needed : 1.00044 – 1.0008)
(a) Rs. 9,000
(b) Rs. 8,113
(c) Rs. 9,200
(d) Rs. 1,000

99. An amount is lent at a nominal rate of 4.5% per annum compounded quarterly. What would he
the gain in rupees over when compounded annually ?
(a) 0.56
(b) 0.45
(c) 0.76
(d) 0.85

100. A stock pays annually an amount of Rs. 10 from 6 th year onwards. What is the present value of
the perpetuity, if the rate of return is 20% ?
(a) 20.1
(b) 19.1
(c) 21.1
(d) 22.1

ANSWER KEY
1 A 21 B 41 B 61 D 81 C
2 B 22 A 42 C 62 B 82 A
3 A 23 A 43 B 63 C 83 A
4 A 24 D 44 C 64 A 84 A
5 A 25 B 45 B 65 B 85 D
6 96 26 B 46 B 66 B 86 A
7 A 27 A 47 A 67 B 87 C
8 D 28 B 48 C 68 A 88 B
9 D 29 D 49 C 69 B 89 C
10 B 30 D 50 C 70 D 90 D
11 C 31 C 51 D 71 C 91 A
12 C 32 D 52 D 72 D 92 B
13 B 33 D 53 C 73 A 93 C
14 A 34 C 54 20/216 74 A 94 B
15 B 35 A 55 B 75 C 95 B
16 D 36 A 56 A 76 A 96 B
17 C 37 B 57 A 77 C 97 B
18 A 38 B 58 A 78 D 98 8,230
19 C 39 B 59 D 79 D 99 0.076
20 A 40 D 60 A 80 A 100 A

357 | P a g e

You might also like